Calculus
Calculus
Copyright © 2024 Md. Shouvik Iqbal. This book is licensed under the Creative
Commons Attribution-NonCommercial-ShareAlike 4.0 International License
(CC BY-NC-SA 4.0)
Copyright © 2024 Md. Shouvik Iqbal. This book is licensed under a Creative Commons Attribution-NonCommercial-ShareAlike 4.0 International License (CC BY-NC-SA 4.0)
Download for free at https://md-shouvik-iqbal.github.io/calculus
Copyright © 2024 by Md. Shouvik Iqbal
All Rights Reserved.
Edition: First
Version: 1.0
Calculus © 2024 by Md. Shouvik Iqbal is licensed under the Creative Commons
Attribution-NonCommercial-ShareAlike 4.0 International License (CC BY-NC-
SA 4.0). To view a copy of this license, visit https://creativecommons.org/
licenses/by-nc-sa/4.0/. Under this license, one can share, remix, and build
upon the contents as long as (1) for noncommercial purposes only, (2) proper
attributions are provided, regardless of whether the content is used in whole or
in part, and (3) distributed under the same license. When sharing the original
or adapted material in any format, the following attribution must appear on
every physical or digital page/view:
This book was set in Computer Modern and EB Garamond, and was composed
using the free-to-use LATEX editors: TeXstudio and Overleaf; and all the fig-
ures/illustrations were custom-made using the free-to-use application: Figma;
by Md. Shouvik Iqbal.
Note. Readers are invited to provide feedback and ratings on this book at the
link: https://forms.gle/oNTeZjXrqVubCthr7.
Copyright © 2024 Md. Shouvik Iqbal. This book is licensed under a Creative Commons Attribution-NonCommercial-ShareAlike 4.0 International License (CC BY-NC-SA 4.0)
Download for free at https://md-shouvik-iqbal.github.io/calculus
A Message from the Author
Over the course of three years, more than 1,000 hours have
been meticulously dedicated to authoring, researching, edit-
ing, reviewing, designing, illustrating, typesetting, format-
ting, indexing, proofreading, troubleshooting, and supervis-
ing every other aspect of this book’s creation by a one-man
army—the author—with the hope that it will provide help
to someone in need, that even being entirely free of any cost
whatsoever. If this book proves beneficial to you in any way
at any point, and if circumstances allow, a generous donation
to support this one-man army would be greatly appreciated.
You may do so at the link: https://md-shouvik-iqbal.
github.io/calculus or by scanning the following QR code.
Copyright © 2024 Md. Shouvik Iqbal. This book is licensed under a Creative Commons Attribution-NonCommercial-ShareAlike 4.0 International License (CC BY-NC-SA 4.0)
Download for free at https://md-shouvik-iqbal.github.io/calculus
This page intentionally left blank
Thus, this page intentionally left not blank!1
1
Due to the double-sided format, this page had to be left blank intentionally, with the
statement clarifying that it is not the result of any printing errors. However, if this page itself
contains the statement that it was blank, then the page can no longer be considered blank!
Copyright © 2024 Md. Shouvik Iqbal. This book is licensed under a Creative Commons Attribution-NonCommercial-ShareAlike 4.0 International License (CC BY-NC-SA 4.0)
Download for free at https://md-shouvik-iqbal.github.io/calculus
Contents
Preface 13
Copyright © 2024 Md. Shouvik Iqbal. This book is licensed under a Creative Commons Attribution-NonCommercial-ShareAlike 4.0 International License (CC BY-NC-SA 4.0)
Download for free at https://md-shouvik-iqbal.github.io/calculus
1.6.2 Continuity over an Interval · · · · · · · · · · · · · · · · · · · · 97
1.6.3 Properties of Continuity · · · · · · · · · · · · · · · · · · · · · · 100
1.6.3.1 Continuity of Combined Functions · · · · · · · · · · · 100
1.6.3.2 Continuity of Composite Functions · · · · · · · · · · · 102
1.6.4 The Intermediate Value Theorem (IVT) · · · · · · · · · · · · · 102
1.7 (ε − δ) Definition of Limit ································································ 104
II Differential Calculus
2 An Introduction to Differential Calculus 115
2.1 The Tangent Line Problem ······························································· 117
2.2 Definition of the Derivative of a function············································ 127
2.3 Notation for Derivatives ··································································· 134
2.4 Differentiability and Continuity························································· 136
2.5 Higher-Order Derivatives·································································· 139
Copyright © 2024 Md. Shouvik Iqbal. This book is licensed under a Creative Commons Attribution-NonCommercial-ShareAlike 4.0 International License (CC BY-NC-SA 4.0)
Download for free at https://md-shouvik-iqbal.github.io/calculus
3.6.1 Differentiation of ln(x) · · · · · · · · · · · · · · · · · · · · · · 195
3.6.2 Differentiation of loga (x) · · · · · · · · · · · · · · · · · · · · · 198
3.7 Logarithmic Differentiation······························································· 200
3.8 Differentiation Rules for Inverse Trigonometric Functions····················· 206
3.8.1 Differentiation of sin−1 (x) · · · · · · · · · · · · · · · · · · · · · 206
3.8.2 Differentiation of cos−1 (x) · · · · · · · · · · · · · · · · · · · · 208
3.8.3 Differentiation of tan−1 (x) · · · · · · · · · · · · · · · · · · · · 210
3.8.4 Differentiation of cot−1 (x) · · · · · · · · · · · · · · · · · · · · · 212
3.8.5 Differentiation of sec−1 (x) · · · · · · · · · · · · · · · · · · · · · 214
3.8.6 Differentiation of csc−1 (x) · · · · · · · · · · · · · · · · · · · · · 216
3.9 Differentiation of General Inverse Functions ······································· 218
3.10 Summary ······················································································· 221
Copyright © 2024 Md. Shouvik Iqbal. This book is licensed under a Creative Commons Attribution-NonCommercial-ShareAlike 4.0 International License (CC BY-NC-SA 4.0)
Download for free at https://md-shouvik-iqbal.github.io/calculus
III Integral Calculus
5 An Introduction to Integral Calculus 295
5.1 The Area Under a Curve·································································· 297
5.1.1 The Summation ( ) Notation · · · · · · · · · · · · · · · · · · 298
P
Copyright © 2024 Md. Shouvik Iqbal. This book is licensed under a Creative Commons Attribution-NonCommercial-ShareAlike 4.0 International License (CC BY-NC-SA 4.0)
Download for free at https://md-shouvik-iqbal.github.io/calculus
7.3.3 Integrals Involving Powers of tan(x) and sec(x) · · · · · · · · · 409
7.3.4 Integrals Involving Product of Powers of tan(x) and sec(x) · · · 411
7.4 Trigonometric Substitution······························································· 412
√
7.4.1 Integrals Involving a2 − x2 · · · · · · · · · · · · · · · · · · · 413
√
7.4.2 Integrals Involving a2 + x2 · · · · · · · · · · · · · · · · · · · 419
√
7.4.3 Integrals Involving x2 − a2 · · · · · · · · · · · · · · · · · · · 425
7.5 Partial Fractions ············································································· 431
7.5.1 The denominator Q(x) is a product of linear factors · · · · · · 433
7.5.2 The denominator Q(x) contains repeated product of linear factors435
7.5.3 The denominator Q(x) contains irreducible quadratic factors · · 438
7.5.4 The denominator Q(x) contains repeated irreducible quadratic
factors · · · · · · · · · · · · · · · · · · · · · · · · · · · · · · · 441
7.6 Improper Integrals··········································································· 443
7.6.1 Infinite Intervals · · · · · · · · · · · · · · · · · · · · · · · · · · 443
7.6.2 Discontinuous Integrands · · · · · · · · · · · · · · · · · · · · · 446
7.7 Table of Integrals ············································································ 450
7.7.1 Elementary Integrals · · · · · · · · · · · · · · · · · · · · · · · · 451
7.7.2 Trigonometric Integrals · · · · · · · · · · · · · · · · · · · · · · 452
7.7.3 Inverse Trigonometric Integrals · · · · · · · · · · · · · · · · · · 453
7.7.4 Reduction Formula for Trigonometric Functions · · · · · · · · · 453
7.7.5 Integrals of the Form a2 − x2 , a > 0 · · · · · · · · · · · · · · · 454
7.7.6 Integrals of the Form x2 − a2 , a > 0 · · · · · · · · · · · · · · · 455
7.7.7 Integrals of the Form a2 + x2 , a > 0 · · · · · · · · · · · · · · · 455
7.7.8 Integrals of the Form ax ± b, a 6= 0, b > 0 · · · · · · · · · · · 456
7.7.9 Integrals with Exponential and Trigonometric Functions · · · · 457
7.7.10 Integrals with Exponential and Logarithmic Functions · · · · · 457
7.7.11 Miscellaneous Integral Forms · · · · · · · · · · · · · · · · · · · 457
Copyright © 2024 Md. Shouvik Iqbal. This book is licensed under a Creative Commons Attribution-NonCommercial-ShareAlike 4.0 International License (CC BY-NC-SA 4.0)
Download for free at https://md-shouvik-iqbal.github.io/calculus
8.6 The Area of a Surface of Revolution ·················································· 491
8.6.1 The Generalized Area Formula for a Surface of Revolution · · · 495
8.7 Work ····························································································· 501
Index 507
License 513
Copyright © 2024 Md. Shouvik Iqbal. This book is licensed under a Creative Commons Attribution-NonCommercial-ShareAlike 4.0 International License (CC BY-NC-SA 4.0)
Download for free at https://md-shouvik-iqbal.github.io/calculus
Copyright © 2024 Md. Shouvik Iqbal. This book is licensed under a Creative Commons Attribution-NonCommercial-ShareAlike 4.0 International License (CC BY-NC-SA 4.0)
Download for free at https://md-shouvik-iqbal.github.io/calculus
Preface
Indeed! Everything in life is changing except for the state of being changed itself.
From the smallest particles to the largest of galaxies and beyond, everything is
in a state of flux—a continuous change. Calculus—as we know it today—is the
mathematics of this very change. More precisely, it is the mathematics to deal
with variables that are continuously changing and can thus be used to under-
stand the changing nature around us and to answer the fundamental questions
within our minds.
Calculus—at least, in my eyes—is not just all about that. I view it as a pro-
found inspiration where philosophical ideas intersect with that of mathematical
reasoning to produce something nothing short of elegant. What inspired this
book was my desire to share this elegance I found within calculus. The origin
13
Copyright © 2024 Md. Shouvik Iqbal. This book is licensed under a Creative Commons Attribution-NonCommercial-ShareAlike 4.0 International License (CC BY-NC-SA 4.0)
Download for free at https://md-shouvik-iqbal.github.io/calculus
of which can be traced back to the collection of notes I made during my own
journey of learning the subject years ago. This, I believe, ensures that the topics
that fascinated me as a learner will resonate just as deeply with readers. I’ve
made every effort to present the material in a clear and accessible language,
without sacrificing the eloquence of tone. Likewise, I tried my best to explain
the reasoning behind each concept without excluding any contents covered in
any other formal textbook. That is, this book covers all the topics typically in-
cluded in any introductory calculus text, but it distinguishes itself in how those
ideas are explained. Therefore, the reader may find it comfortable to use it as a
main text, especially when learning the concepts.
The book is divided into three parts to discuss two major branches of calculus:
differential calculus and integral calculus. The first part serves as a foundation
for the contents that follow and consists of the opening chapter. The second
part focuses on differential calculus and includes chapters two, three, and four.
The third part focuses on integral calculus, covering chapters five through eight.
A detailed purpose and direction of each chapter is discussed below.
Copyright © 2024 Md. Shouvik Iqbal. This book is licensed under a Creative Commons Attribution-NonCommercial-ShareAlike 4.0 International License (CC BY-NC-SA 4.0)
Download for free at https://md-shouvik-iqbal.github.io/calculus
• Chapter 3. The Derivation of Derivatives.
The purpose of this chapter is to derive the derivatives of various functions in order to avoid
reapplying the definition of the derivative repeatedly. The chapter begins with the section
“Elementary Rules of Differentiation,” which covers several fundamental rules: Constant
Rule, Constant Multiple Rule, Power Rule, Summation Rule, Difference Rule, Product Rule,
and Quotient Rule. These rules serve as the foundation for differentiating a wide range of
functions. The next section, “Differentiation Rules for Trigonometric Functions,” focuses
on the derivatives of the six trigonometric functions along with their examples. Following
this, the section “Differentiation of Composite Functions” introduces the Chain Rule and
explores its application through numerous examples. Understanding the Chain Rule is cru-
cial for comprehending the following topics. Next, in the section “Implicit Differentiation,”
the process of differentiating implicitly defined functions is discussed, along with multi-
ple examples to illustrate the method. The section “Differentiation Rules for Exponential
Functions” derives and discusses the derivatives of exponential functions with various ex-
amples. The subsequent section “Differentiation Rules for Logarithmic Functions,” derives
and discusses the derivatives of logarithmic functions with various examples. Following this,
the section on “Logarithmic Differentiation” explains a technique for simplifying functions
using logarithmic properties before differentiating, which proves beneficial in various cases.
The next section, “Differentiation Rules for Inverse Trigonometric Functions,” derives the
derivatives of the six inverse trigonometric functions and provides various examples. Follow-
ing that, the section “Differentiation of General Inverse Functions,” extends the discussion
to cover general inverse functions. Finally, the chapter concludes with a summary of the
key derivatives for quick reference.
Copyright © 2024 Md. Shouvik Iqbal. This book is licensed under a Creative Commons Attribution-NonCommercial-ShareAlike 4.0 International License (CC BY-NC-SA 4.0)
Download for free at https://md-shouvik-iqbal.github.io/calculus
signed area. The properties of definite integrals are then discussed through various exam-
ples. Finally, the chapter concludes with a discussion of numerical integration, specifically
the Midpoint Rule and the Trapezoidal Rule, which are methods for approximating definite
integrals.
Prerequisite.
Copyright © 2024 Md. Shouvik Iqbal. This book is licensed under a Creative Commons Attribution-NonCommercial-ShareAlike 4.0 International License (CC BY-NC-SA 4.0)
Download for free at https://md-shouvik-iqbal.github.io/calculus
are not covered in detail within this book, except when necessary for clarity.
While prior exposure to introductory calculus concepts, such as limits, may be
beneficial, it is not required in this text. What is required is a solid grasp of the
precalculus contents to follow along comfortably.
From my experience, there are three category of potential readers: (1) those
who have a strong grasp of the precalculus topics, (2) those with a basic under-
standing of the precalculus topics, and (3) those with little to no knowledge of
the precalculus topics and may struggle comprehending statements such as “Let
a be a real number defined over the domain of f (x)” or “for all x, there exist y
such that this and that is true” or something along those lines.
Readers in the first category can proceed through the book without additional
resources. Those in the second category are encouraged to keep a relevant math-
ematical textbook that covers the precalculus topics or use online resources as
needed. However, the third category, lacking a foundation in precalculus, may
find the book difficult to follow and are generally advised not to begin without
first mastering the necessary background, though this choice is ultimately up to
them.
So, the bottom line is that a firm command of precalculus topics is crucial to
fully engage with the material, as these topics are not explicitly covered in the
book.
Copyright © 2024 Md. Shouvik Iqbal. This book is licensed under a Creative Commons Attribution-NonCommercial-ShareAlike 4.0 International License (CC BY-NC-SA 4.0)
Download for free at https://md-shouvik-iqbal.github.io/calculus
levels.
Dhaka, Bangladesh
30 October 2024
Copyright © 2024 Md. Shouvik Iqbal. This book is licensed under a Creative Commons Attribution-NonCommercial-ShareAlike 4.0 International License (CC BY-NC-SA 4.0)
Download for free at https://md-shouvik-iqbal.github.io/calculus
About the Cover
The figure in the cover of this book was taken from the following photograph of
the statue of Issac Newton, located in the chapel of Trinity College, Cambridge.
Copyright © 2024 Md. Shouvik Iqbal. This book is licensed under a Creative Commons Attribution-NonCommercial-ShareAlike 4.0 International License (CC BY-NC-SA 4.0)
Download for free at https://md-shouvik-iqbal.github.io/calculus
Copyright © 2024 Md. Shouvik Iqbal. This book is licensed under a Creative Commons Attribution-NonCommercial-ShareAlike 4.0 International License (CC BY-NC-SA 4.0)
Download for free at https://md-shouvik-iqbal.github.io/calculus
Part I
21
Copyright © 2024 Md. Shouvik Iqbal. This book is licensed under a Creative Commons Attribution-NonCommercial-ShareAlike 4.0 International License (CC BY-NC-SA 4.0)
Download for free at https://md-shouvik-iqbal.github.io/calculus
Copyright © 2024 Md. Shouvik Iqbal. This book is licensed under a Creative Commons Attribution-NonCommercial-ShareAlike 4.0 International License (CC BY-NC-SA 4.0)
Download for free at https://md-shouvik-iqbal.github.io/calculus
Chapter 1
In order to travel a distance and reach its end, one must first have
to cover half of that distance. From there, to reach the end, the re-
maining half must be covered, and then again the remaining half, and
again the half of what remains, and the process continues endlessly.
Each step brings closer and closer to the endpoint, but never exactly
reaches it, as there’s always another halfway point to be traveled to
reach the endpoint. Thus, the conclusion left to be drawn is that the
endpoint can never be exactly reached.
This seemingly absurd observation above appears to have been first made by
Zeno of Elea, a pre-Socratic Greek philosopher. It is a paradox suggesting that
a moving object can certainly approach the endpoint but never exactly reach it.
The underlying idea is similar to that of the mathematical concept referred to
as the “limit.” The notion of such in mathematics stands as such an ineffable
construct that, without its existence, the entire field itself might not have devel-
oped in the form we see it today. For example, calculus is a fundamental branch
of mathematics, and among the significant topics in calculus lie derivatives and
integrals, both of which can be thought of as a unique representation of limits.
In this chapter, we aim to focus on exploring limits, their definitions, properties,
associated theorems, and tackling problems related to limits. Thus, we start
with the introduction of the concept of limit.
23
Copyright © 2024 Md. Shouvik Iqbal. This book is licensed under a Creative Commons Attribution-NonCommercial-ShareAlike 4.0 International License (CC BY-NC-SA 4.0)
Download for free at https://md-shouvik-iqbal.github.io/calculus
Page 24 An Introduction to the Limit Chapter 1
Consider a hypothetical bouncing ball “b” with the following property “P.”
So, we use the symbol “b(P)” to denote a bouncing ball with the property P.
Say, we drop the ball b(P) from a height of 1 unit. When it is dropped from
that height, it bounces back up 0.5 units on the first bounce, then 0.25 units,
0.125 units, and so on. So, the following question arises naturally,
Question: How many times will the ball bounce before it comes to rest on the
ground, if it ever does?
Let us logically think about this problem. The ball was released from a height
of 1 unit. So, the initial height of that ball is 1 unit. Then, after each bounce,
1
the new height of the ball is half, i.e., times the previous height due to the
2
property P. This follows that,
1 1
after bounce = 1, the new height = 1 × = units
|{z} 2 2
previous height
1
with no loss of physical properties such as energy in a way that if dropped from a height
x
of x units, it rebounds back up to units after 1 bounce.
2
Copyright © 2024 Md. Shouvik Iqbal. This book is licensed under a Creative Commons Attribution-NonCommercial-ShareAlike 4.0 International License (CC BY-NC-SA 4.0)
Download for free at https://md-shouvik-iqbal.github.io/calculus
Chapter 1 An Introduction to the Limit Page 25
1 1 1
after bounce = 2, the new height = × = 2 units
2
|{z} 2 2
previous height
1 1 1 1
after bounce = 3, the new height = × × = 3 units
2 2 2 2
| {z }
previous height
1 1 1 1 1
after bounce = 4, the new height = × × × = 4 units
2 2 2 2 2
| {z }
previous height
..
.
previous height
}| { z
1 1 1 1 1 1
after bounce = n, the new height = × × × ··· × × = n units
2 2 2 2 2 2
| {z }
n
1
Notice that as the number of bounces n increases, the height n tends to ap-
2
proach a value of 0, as shown in the table below.
Table 1.1: Table
n 0 1 2 3 4 5 ··· ∞
1
1 0.5 0.25 0.125 0.0625 0.03125 ··· 0
2n
Therefore, we say,
1
as n approaches ∞, approaches 0
2n
1
where, n is the number of bounces and n is the height of the ball b(P) after
2
each bounce. This can symbolically be written as,
1
as n → ∞, →0
2n
by letting the symbol “→” to mean “approaching,” just like the symbol “=” is
used to mean “equal.”
1
Since, the height n gets closer and closer to a value 0 as the number of bounces
2
n increases without a bound, therefore, the value 0 can be thought of as the
1
ultimate destination—the limit—for the height n to ultimately reach, as n
2
approaches infinity. We express this idea symbolically as follows,
1
lim n = 0
n→∞ 2
Copyright © 2024 Md. Shouvik Iqbal. This book is licensed under a Creative Commons Attribution-NonCommercial-ShareAlike 4.0 International License (CC BY-NC-SA 4.0)
Download for free at https://md-shouvik-iqbal.github.io/calculus
Page 26 An Introduction to the Limit Chapter 1
1
In other words, n → 0, as n → ∞. Thus, we have found our answer to the
2
question above that—hypothetically—the ball b(P) will increasingly approach
the ground to settle down as the number of bounces grows without a bound.
Now, notice in the table above that for each bounce n, there exists a corre-
1
sponding new height n . Therefore, the table above can be modified into a
2
1
height function f of each bounce n, that is, f (n) = n . Notice that n in the
2
1
function f (n) = n is restricted to be a positive integer as the number of bounces
2
can’t be negative or fractional. The function, however, maps these positive in-
tegers to real numbers as an output.
1.5
0.5
x
0.5 1 1.5 2 2.5
Figure 1.1
1
Notice that the domain (simply, the input) of f (n) = n is a subset of the
2
1 1
domain of f (x) = x , therefore if the limit of f (x) = x is 0 as the input ap-
2 2
1
proaches infinity, then the limit of f (n) = n automatically becomes 0 for the
2
Copyright © 2024 Md. Shouvik Iqbal. This book is licensed under a Creative Commons Attribution-NonCommercial-ShareAlike 4.0 International License (CC BY-NC-SA 4.0)
Download for free at https://md-shouvik-iqbal.github.io/calculus
Chapter 1 An Introduction to the Limit Page 27
same.
Now, what if the input x approaches a finite value, say x = 1? What value does
1
the function f (x) = x approach? It can be observed—with no effort—from the
2
graph shown below that when x → 1, f (x) → 0.5.
1.5
0.5
x
0.5 1 1.5 2 2.5
Figure 1.2
It may seem absurd to even consider the concept of approaching a value in this
1
case. Because anyone can evaluate the function f (x) = x at the point x = 1
2
1
and get the answer that f (1) = 1 = 0.5. However, the idea of limits really
2
becomes necessary when a function is not defined at a point. For example,
consider the following function,
x2 − 42
f (x) =
x−4
What is the value of the function at the point x = 4? Evaluating the function
at x = 4, results in the following,
x2 − 42
f (x) =
x−4
42 − 42
=
4−4
0
=
0
This can be interpreted as the measure of nothing is broken down into the
measure of nothing, which is nothing but absurdity. However, by examining
the behavior of the function f (x) in the vicinity of x = 4, we can predict by
Copyright © 2024 Md. Shouvik Iqbal. This book is licensed under a Creative Commons Attribution-NonCommercial-ShareAlike 4.0 International License (CC BY-NC-SA 4.0)
Download for free at https://md-shouvik-iqbal.github.io/calculus
Page 28 An Introduction to the Limit Chapter 1
x2 − 42
x<4 f (x) =
x−4
3.9 7.9
3.99 7.99
3.999 7.999
3.9999 7.9999
The table above indicates that as the values of x are getting closer and closer
to the point x = 4, the values of f (x) are getting closer and closer to the point
f (x) = 8. Now, we estimate f (x) using values of x greater than 4.
x2 − 42
x>4 f (x) =
x−4
4.1 8.1
4.01 8.01
4.001 8.001
4.0001 8.0001
Similar to the previous table, this table above also indicates that as x is ap-
proaching 4, f (x) is approaching the value 8.
Copyright © 2024 Md. Shouvik Iqbal. This book is licensed under a Creative Commons Attribution-NonCommercial-ShareAlike 4.0 International License (CC BY-NC-SA 4.0)
Download for free at https://md-shouvik-iqbal.github.io/calculus
Chapter 1 An Introduction to the Limit Page 29
2 4 6 8
Figure 1.3
Thus, it may be sufficiently logical to conclude that the limit of the function
x2 − 42
f (x) = , as x gets closer and closer to the value 4, is the value 8.
x−4
We can generalize the idea behind the limit and define the limit of a function as
follows. Note that this definition is informal and the formal definition is given
at the end of this chapter.
lim f (x) = L
x→a
Note that x does not necessarily have to be defined at the point a. In fact, when
examining the limit of f (x) as x approaches a, the value of x at a is not even
taken into account. The sole concern is how f (x) behaves in the vicinity of a.
Copyright © 2024 Md. Shouvik Iqbal. This book is licensed under a Creative Commons Attribution-NonCommercial-ShareAlike 4.0 International License (CC BY-NC-SA 4.0)
Download for free at https://md-shouvik-iqbal.github.io/calculus
Page 30 One-Sided Limits Chapter 1
x
a
Figure 1.4
Copyright © 2024 Md. Shouvik Iqbal. This book is licensed under a Creative Commons Attribution-NonCommercial-ShareAlike 4.0 International License (CC BY-NC-SA 4.0)
Download for free at https://md-shouvik-iqbal.github.io/calculus
Chapter 1 One-Sided Limits Page 31
Thus, the informal definition of this one-sided limit can be given as,
1. Limit from the left: Let f (x) be a function defined over an open in-
terval (b, a) and let L be a real number. If f (x) → L as x → a, where
x < a, then the limit of f (x) is L as x approaches a from the left.
Symbolically,
lim− f (x) = L
x→a
2. Limit from the right: Let f (x) be a function defined over an open
interval (a, b) and let L be a real number. If f (x) → L as x → a,
where a < x, then the limit of f (x) is L as x approaches a from the
right. Symbolically,
lim+ f (x) = L
x→a
At first glance, defining one-sided limits might seem trivial. Yet, it turns out
to be a crucial clarification. This is because not every function approaches the
same limit as its input approaches a single value. For example, consider the
x
graph of the function f (x) = . As x → 0, what value does y = f (x) get
|x|
closer to?
y
x
0
Figure 1.5
The answer is not obvious because if x approaches 0 from the left side of 0, it
seems like f (x) approaches the value −1. However, if x approaches 0 from the
right side of 0, it seems like f (x) approaches the value 1. Therefore, the overall
limit of f (x) as x → 0, does not agree on any single limit and thus does not
Copyright © 2024 Md. Shouvik Iqbal. This book is licensed under a Creative Commons Attribution-NonCommercial-ShareAlike 4.0 International License (CC BY-NC-SA 4.0)
Download for free at https://md-shouvik-iqbal.github.io/calculus
Page 32 Existence of a Limit Chapter 1
x x
lim− = −1 and lim+ =1
x→0 |x| x→0 |x|
.
Therefore, the situation at hand gives rise to the study of the existence of a limit
of a function, as outlined below.
Theorem 1.3.1. (Existence of a Limit) Let f (x) be a function and let a and
L be real numbers. Then,
While violating Theorem 1.3.1 indeed leads to the absence of a function’s limit,
it’s important to note that this isn’t the sole factor causing the existence of
a limit to fail. Various other reasons can also contribute to the failure of a
function’s limit to exist. For example, consider the function below,
1
f (x) =
(x − 4)2
What value does f (x) approach as x → 4? To determine, we look at the graph
1
of f (x) = , as shown below.
(x − 4)2
Copyright © 2024 Md. Shouvik Iqbal. This book is licensed under a Creative Commons Attribution-NonCommercial-ShareAlike 4.0 International License (CC BY-NC-SA 4.0)
Download for free at https://md-shouvik-iqbal.github.io/calculus
Chapter 1 Existence of a Limit Page 33
x
2 4 6 8
Figure 1.6
1
It is evident that as x → 4 from both sides, the function f (x) = grows
(x − 4)2
1
with a bound. That is, f (x) = → ∞. Since ∞ is not a number,
(x − 4)2
1
therefore the limit of f (x) = as x → 4, does not exists.
(x − 4)2
One other reason for which the limit of a function fails to exist is oscillating
behavior. For example, consider the following function,
1
f (x) = sin
x
1
What value does f (x) = sin approach as x → 0?
x
1 1
As x → 0, → ∞ and sin oscillate between -1 and 1 infinitely often.
x x
1
Since f (x) = sin does not approach a fixed value, therefore the limit of
x
1
f (x) = sin as x → 0, does not exist.
x
In general, the following are the most common reasons for which the limit of a
function fails to exist.
Remark 1.3.1.
1. The limit of f (x) differs between the left and right sides, as x → a.
Copyright © 2024 Md. Shouvik Iqbal. This book is licensed under a Creative Commons Attribution-NonCommercial-ShareAlike 4.0 International License (CC BY-NC-SA 4.0)
Download for free at https://md-shouvik-iqbal.github.io/calculus
Page 34 Analytic Evaluation of Limit of a Function Chapter 1
2. f (x) → ±∞, as x → a.
then,
1. lim c = c
x→a
2. lim x = a
x→a
Copyright © 2024 Md. Shouvik Iqbal. This book is licensed under a Creative Commons Attribution-NonCommercial-ShareAlike 4.0 International License (CC BY-NC-SA 4.0)
Download for free at https://md-shouvik-iqbal.github.io/calculus
Chapter 1 Analytic Evaluation of Limit of a Function Page 35
3. lim c · f (x) = c · lim f (x) = c · L
x→a x→a
4. lim f (x) + g (x) = lim f (x) + lim g (x) = L + M
x→a x→a x→a
5. lim f (x) − g (x) = lim f (x) − lim g (x) = L − M
x→a x→a x→a
6. lim f (x) · g (x) = lim f (x) · lim g (x) = L · M
x→a x→a x→a
f (x)
lim f (x) L
7. lim = x→a
= where, g(x) 6= 0
x→a g (x) lim g (x) M
x→a
n n
8. lim f (x) = lim f (x) = Ln
x→a x→a
√
9. lim lim f (x) = where, n is a positive integer.
p q n
n
f (x) = n L,
x→a x→a
Acknowledging these limit laws, we now begin to use them to evaluate the limit
of a function.
lim c = c
x→a
This implies,
lim 33 = 33
x→2
lim x = a
x→a
This implies,
lim x = 2
x→2
Copyright © 2024 Md. Shouvik Iqbal. This book is licensed under a Creative Commons Attribution-NonCommercial-ShareAlike 4.0 International License (CC BY-NC-SA 4.0)
Download for free at https://md-shouvik-iqbal.github.io/calculus
Page 36 Analytic Evaluation of Limit of a Function Chapter 1
=π·4
= 4π
π
I Example 1.4.4. If lim f (x) = and lim g (x) = 4, then evaluate
x→e 2 x→e
πf (x) + g (x)
lim
x→e f (x) − f (x) g (x)
Copyright © 2024 Md. Shouvik Iqbal. This book is licensed under a Creative Commons Attribution-NonCommercial-ShareAlike 4.0 International License (CC BY-NC-SA 4.0)
Download for free at https://md-shouvik-iqbal.github.io/calculus
Chapter 1 Analytic Evaluation of Limit of a Function Page 37
π2 + 8
=−
3π
π2 + 8
πf (x) + g (x) π
Therefore, lim =− , given that lim f (x) = and
x→e f (x) − f (x) g (x) 3π x→e 2
lim g (x) = 4.
x→e
5
I Example 1.4.5. If lim Z (X) = 5, then evaluate lim Z (X)
X→4 X→4
Copyright © 2024 Md. Shouvik Iqbal. This book is licensed under a Creative Commons Attribution-NonCommercial-ShareAlike 4.0 International License (CC BY-NC-SA 4.0)
Download for free at https://md-shouvik-iqbal.github.io/calculus
Page 38 Analytic Evaluation of Limit of a Function Chapter 1
What value does P (x) approach as x → 5? To determine, we can use the limit
laws to evaluate the limit of P (x), as follows.
From the example above, it can be seen that the limit of the polynomial function
P (x) is easily found by direct substitution. That is, if P (x) is a polynomial
function, then the limit of it, as x → a, is P (a). This is due to the fact that
polynomial functions are one of the simplest types of functions and are defined
for all real numbers. Thus, lim P (x) in the example above, can be evaluated
x→5
directly as,
= 5 + 2 · 54 + 4 · 53 + 7 · 5 + 9
5
= 4919
The theorem above further suggests that if P (x) and Q(x) are polynomial func-
P (x) P (a)
tions such that Q(x) 6= 0, then the limit of , as x → a, is . Thus, we
Q(x) Q(a)
have the following theorem.
P (x)
Theorem 1.4.3. (The Limit of Rational Functions) If R(x) = is a
Q(x)
rational function, and a is a real number such that Q(x) 6= 0, then,
P (x) P (a)
lim R(x) = lim =
x→a x→a Q(x) Q(a)
Copyright © 2024 Md. Shouvik Iqbal. This book is licensed under a Creative Commons Attribution-NonCommercial-ShareAlike 4.0 International License (CC BY-NC-SA 4.0)
Download for free at https://md-shouvik-iqbal.github.io/calculus
Chapter 1 Analytic Evaluation of Limit of a Function Page 39
x3 + 3x
I Example 1.4.6. Evaluate lim .
x→1 x2 + 1
x3 + 3x 13 + 3 · 1
lim =
x→1 x2 + 1 12 + 1
1+3
=
1+1
4
=
2
=2
x3 + 3x
Therefore, lim =2
x→1 x2 + 1
x1 + x2 + x3 + x4 + x5
I Example 1.4.7. If → L as x → 1, then what is the
x6 + x7 + x8 + x9 + x10
value of L?
x1 + x2 + x3 + x4 + x5
Therefore, 6 → 1 as x → 1.
x + x7 + x8 + x9 + x10
Copyright © 2024 Md. Shouvik Iqbal. This book is licensed under a Creative Commons Attribution-NonCommercial-ShareAlike 4.0 International License (CC BY-NC-SA 4.0)
Download for free at https://md-shouvik-iqbal.github.io/calculus
Page 40 Analytic Evaluation of Limit of a Function Chapter 1
1
I Example 1.4.8. Evaluate lim .
x→2 2x
Solution. From the theorem above, we get,
1 1
lim x = 2
x→2 2 2
1
=
4
1 1
Therefore, lim x =
x→2 2 4
1
I Example 1.4.9. If f (x) = x3 , then what value does approaches as x →
|x3 |
−π?
Solution. From the theorem above, we get,
1 1
lim 3
=
x→−π |x | |−π 3 |
1
= 3
π
1 1
Therefore, 3 approaches 3 , as x → −π.
|x | π
Copyright © 2024 Md. Shouvik Iqbal. This book is licensed under a Creative Commons Attribution-NonCommercial-ShareAlike 4.0 International License (CC BY-NC-SA 4.0)
Download for free at https://md-shouvik-iqbal.github.io/calculus
Chapter 1 Analytic Evaluation of Limit of a Function Page 41
1. lim bx = ba ,
x→a
1. lim sin (x) = sin (a) 4. lim cot (x) = cot (a)
x→a x→a
2. lim cos (x) = cos (a) 5. lim sec (x) = sec (a)
x→a x→a
3. lim tan (x) = tan (a) 6. lim csc (x) = csc (a)
x→a x→a
1. lim sin−1 (x) = sin−1 (a) 4. lim cot−1 (x) = cot−1 (a)
x→a x→a
lim ex = e1
x→1
=0
Therefore, lim ex = 0
x→1
Copyright © 2024 Md. Shouvik Iqbal. This book is licensed under a Creative Commons Attribution-NonCommercial-ShareAlike 4.0 International License (CC BY-NC-SA 4.0)
Download for free at https://md-shouvik-iqbal.github.io/calculus
Page 42 Analytic Evaluation of Limit of a Function Chapter 1
Theorem 1.4.6. (The Limit of a Composite Function) If f (x) and g(x) are
functions such that lim g (x) = L and lim f (x) = f (L), then,
x→a x→L
lim f g (x) = f lim g (x) = f (L)
x→a x→a
x→∞
3
discussed later in this chapter
Copyright © 2024 Md. Shouvik Iqbal. This book is licensed under a Creative Commons Attribution-NonCommercial-ShareAlike 4.0 International License (CC BY-NC-SA 4.0)
Download for free at https://md-shouvik-iqbal.github.io/calculus
Chapter 1 Analytic Evaluation of Limit of a Function Page 43
Solution. Here,
1
f (x) = ex and g(x) =
x
1
First, find the limit of the inner function g(x) = as x → ∞, as shown below.
x
1
=0 lim
x→∞ x
Now, apply the outer function f (x) = ex to the limit of the inner function, that
1
is, lim = 0, as shown below.
x→∞ x
e0 = 1
lim 1
Therefore, lim e x = ex→∞ x = 1.
1
x→∞
√
I Example 1.4.14. Evaluate lim x2 + π
x→1
Solution. Here,
√
f (x) = x and g(x) = x2 + π
First, find the limit of the inner function g(x) = x2 + π as x → 1,
lim x2 + π = 1 + π
x→1
√
Now, apply the outer function f (x) = x to the limit of the inner function.
That is,
√
1 + π ≈ 2.035
√
Therefore, lim x2 + π = lim (x2 + π) ≈ 2.035.
q
x→1 x→1
Now, apply the outer function f (x) = ln(x) to the limit of the inner fnction.
That is,
ln(1) = 0
Therefore, lim ln(x2 ) = ln(lim x2 ) = 0.
x→1 x→1
Copyright © 2024 Md. Shouvik Iqbal. This book is licensed under a Creative Commons Attribution-NonCommercial-ShareAlike 4.0 International License (CC BY-NC-SA 4.0)
Download for free at https://md-shouvik-iqbal.github.io/calculus
Page 44 Analytic Evaluation of Limit of a Function Chapter 1
2 2
1 1
1 2 3 1 2 3
(a) (b)
Figure 1.7
The only difference between these two functions is that f (x) is not defined at
the point x = 1, whereas g(x) is defined at that point. Thus, f (x) = g(x) for
Copyright © 2024 Md. Shouvik Iqbal. This book is licensed under a Creative Commons Attribution-NonCommercial-ShareAlike 4.0 International License (CC BY-NC-SA 4.0)
Download for free at https://md-shouvik-iqbal.github.io/calculus
Chapter 1 Analytic Evaluation of Limit of a Function Page 45
all points except x = 1. Because the concept of a limit focuses solely on the
neighboring points of a given point, ignoring the specific behavior at that point
itself, the limits of both functions are equal, as the functions happen to be equal
everywhere except at x = 1. This allows us to write,
√
x−1 1
lim = lim √
x→1 x − 1 x→1 x+1
1
=√
1+1
1
=
1+1
1
=
2
√
x−1 1
Therefore, lim = . The following theorem presents the generalization
x→1 x − 1 2
of this method to evaluate the limit of a function.
Theorem 1.4.7. Let a be a real number and f (x) = g(x) be functions defined
on an open interval containing a, except at the point x = a. If
exists, then
lim f (x) = lim g (x) = g (a)
x→a x→a
1
Note that in the previous example, we found the function g(x) = √ as a
√ x + 1
x−1
function that equals f (x) = for all x except x = 1. Yet, the method by
x−1
which we accomplished this remains undiscussed. Relying on guesswork alone
1
is insufficient. In fact, the expression √ was actually derived from the ex-
√ x+1
x−1
pression algebraically. Thus, it becomes necessary to be aware of certain
x−1
algebraic techniques associated with finding the limit of a function analytically.
We will talk about two such important techniques here. One is referred com-
monly as the “dividing out technique” and the other one is as the “rationalizing
technique.”
Copyright © 2024 Md. Shouvik Iqbal. This book is licensed under a Creative Commons Attribution-NonCommercial-ShareAlike 4.0 International License (CC BY-NC-SA 4.0)
Download for free at https://md-shouvik-iqbal.github.io/calculus
Page 46 Analytic Evaluation of Limit of a Function Chapter 1
x2 − 42
For that, first notice that the numerator of the expression can be fac-
x−4
(x + 4) · (x − 4)
tored into (x + 4) · (x − 4) resulting . Now, notice that the
x−4
numerator and the denominator have a common factor of (x − 4), thus they get
canceled out and we’re left with a new function g (x) = x + 4, that is equivalent
x2 − 42
to f (x) = for all x but x = 4. Thus, by the theorem above, we write,
x−4
x2 − 42
lim = lim (x + 4)
x→4 x − 4 x→4
=4+4
=8
x2 − 42
Which matches the previous answer that the limit of f (x) = , as x → 4,
x−4
is 8.
The dividing out technique thus can be generalized as follows.
Remark 1.4.1.
Copyright © 2024 Md. Shouvik Iqbal. This book is licensed under a Creative Commons Attribution-NonCommercial-ShareAlike 4.0 International License (CC BY-NC-SA 4.0)
Download for free at https://md-shouvik-iqbal.github.io/calculus
Chapter 1 Analytic Evaluation of Limit of a Function Page 47
x2 − 9
I Example 1.4.16. Evaluate lim
x→3 x − 3
x2 − 9 (x−3)
(x + 3)
=
x−3 (x−3)
=x+3 for x 6= 3
x2 − 9
lim = lim (x + 3) = 6
x→3 x − 3 x→3
x3 − 1
I Example 1.4.17. Evaluate lim
x→1 x − 1
x3 − 1 2
(x−1)(x + x + 1)
lim = lim
x→1 x − 1 (x− 1)
x→1
= lim (x2 + x + 1)
x→1
=3
x3 − 1
Therefore, lim =3
x→1 x − 1
x3 − 6x2 + 11x − 6
I Example 1.4.18. Evaluate lim
x→1 x−1
Copyright © 2024 Md. Shouvik Iqbal. This book is licensed under a Creative Commons Attribution-NonCommercial-ShareAlike 4.0 International License (CC BY-NC-SA 4.0)
Download for free at https://md-shouvik-iqbal.github.io/calculus
Page 48 Analytic Evaluation of Limit of a Function Chapter 1
= lim (x − 2)(x − 3)
x→1
=2
x3 − 6x2 + 11x − 6
Therefore, lim =2
x→1 x−1
Copyright © 2024 Md. Shouvik Iqbal. This book is licensed under a Creative Commons Attribution-NonCommercial-ShareAlike 4.0 International License (CC BY-NC-SA 4.0)
Download for free at https://md-shouvik-iqbal.github.io/calculus
Chapter 1 Analytic Evaluation of Limit of a Function Page 49
Therefore we get, √
x−1 (x − 1)
= √
x−1 (x − 1) ( x + 1)
Notice that we now have a common factor in the numerator and denominator
1
which can be canceled out and be left with an expression √ , that is defined
x+1
1
as g(x) = √ . This is precisely the function g(x) that we’d previously used
x+1 √
x−1
to express its equivalence to f (x) = for all x but x = 1.
x−1
Now taking the limit as x → 1, yields the same result as before. That is,
√
x−1 1
lim = lim √
x→1 x − 1 x→1 x+1
1
=√
1+1
1
=
1+1
1
=
2
The rationalizing technique now can be generalized as follows.
Remark 1.4.2.
√ √
1+x− 1−x
I Example 1.4.19. Evaluate lim
x→0 x
Solution. Substituting x = 0 directly into the function yields an indeterminate
form, √ √
1+0− 1−0 0
=
0 0
Now, to solve the problem we multiply the numerator and denominator by the
√ √
conjugate of 1 + x − 1 − x, and simplify as follows.
√ √ √ √ √ √
1+x− 1−x 1+x− 1−x 1+x+ 1−x
lim = lim √ √
x→0 x x→0 x 1+x+ 1−x
Copyright © 2024 Md. Shouvik Iqbal. This book is licensed under a Creative Commons Attribution-NonCommercial-ShareAlike 4.0 International License (CC BY-NC-SA 4.0)
Download for free at https://md-shouvik-iqbal.github.io/calculus
Page 50 Analytic Evaluation of Limit of a Function Chapter 1
(1 + x) − (1 − x)
= lim √ √
x→0 x 1+x+ 1−x
2x
= lim √ √
x 1+x+ 1−x
x→0
2
= lim √ √
x→0 1+x+ 1−x
2
= √ √
1+0+ 1−0
=1
√ √
1+x− 1−x
Therefore, lim =1
x→0 x
√ √
3 + 7x − 3 + 5x
I Example 1.4.20. Evaluate lim
x→0 x
Solution. Substituting x = 0 directly into the function yields an indeterminate
form, √ √
3+7·0− 3+5·0 0
=
0 0
Now, to solve the problem we multiply the numerator and denominator by the
√ √
conjugate of 3 + 7x − 3 + 5x, and simplify as follows.
√ √ √ √ √ √
3 + 7x − 3 + 5x 3 + 7x − 3 + 5x 3 + 7x + 3 + 5x
lim = lim √ √
x→0 x x→0 x 3 + 7x + 3 + 5x
(3 + 7x) − (3 + 5x)
= lim √ √
x→0 x 3 + 7x + 3 + 5x
7x − 5x
= lim √ √
x→0 x 3 + 7x + 3 + 5x
2
x
= lim √ √
x 3 + 7x + 3 + 5x
x→0
2
= lim √ √
x→0 3 + 7x + 3 + 5x
2
=√ √
3+7·0+ 3+5·0
2
= √
2 3
1
=√
3
√ √
3 + 7x − 3 + 5x 1
Therefore, lim =√
x→0 x 3
Copyright © 2024 Md. Shouvik Iqbal. This book is licensed under a Creative Commons Attribution-NonCommercial-ShareAlike 4.0 International License (CC BY-NC-SA 4.0)
Download for free at https://md-shouvik-iqbal.github.io/calculus
Chapter 1 Analytic Evaluation of Limit of a Function Page 51
x
I Example 1.4.21. Evaluate lim √
x→0 1+x−1
Solution. Substituting x = 0 directly into the function yields an indeterminate
form,
0 0
√ =
1+0−1 0
Now, to solve the problem we multiply the numerator and denominator by the
√ √
conjugate of 1 + x − 1, which is 1 + x + 1, and simplify as follows.
√
x x 1+x+1
lim √ = lim √ √
x→0 1 + x − 1 x→0 1+x−1 1+x+1
√
x 1+x+1
= lim
x→0
√1+x−1
x 1+x+1
= lim
x→0
√ x
= lim 1+x+1
x→0
√
= 1+0+1
=2
x
Therefore, lim √ =2
x→0 1+x−1
√ √
a + bx − a + dx
I Example 1.4.22. Evaluate lim
x→0 x
Solution. Substituting x = 0 directly into the function yields an indeterminate
form, √ √
a+b·0− a+d·0 0
=
0 0
Now, to solve the problem we multiply the numerator and denominator by the
√ √ √ √
conjugate of a + bx − a + dx, which is a + bx + a + dx, and simplify as
follows.
√ √ √ √ √ √
a + bx − a + dx a + bx − a + dx a + bx + a + dx
lim = lim √ √
x→0 x x→0 x a + bx + a + dx
(a + bx) − (a + dx)
= lim √ √
x→0 x a + bx + a + dx
bx − dx
= lim √ √
x→0 x a + bx + a + dx
Copyright © 2024 Md. Shouvik Iqbal. This book is licensed under a Creative Commons Attribution-NonCommercial-ShareAlike 4.0 International License (CC BY-NC-SA 4.0)
Download for free at https://md-shouvik-iqbal.github.io/calculus
Page 52 Analytic Evaluation of Limit of a Function Chapter 1
x (b − d)
√ = lim √
a + bx + a + dx x
x→0
b−d
= lim √ √
x→0 a + bx + a + dx
b−d
=√ √
a+0+ a+0
b−d
= √
2 a
√ √
a + bx − a + dx b−d
Therefore, lim = √
x→0 x 2 a
Remark 1.4.3.
√ √
a + bx − a + dx b−d
lim = √
x→0 x 2 a
√ √
a + bx − a − dx b+d
lim = √
x→0 x 2 a
The squeeze theorem might seem convoluted, however, the idea behind is quite
simple and is synonymous with the following analogy.
Imagine a race competition among three cat breeds: Bengal, Turkish Angora,
and Ragdoll, each completing to showcase their speed. While they share a
comparable running pace on average, however the Bengal cat stands out as
particularly athletic due to its natural traits. Therefore, the premise is that the
Bengal cat outpaces the Turkish Angora, which in turn outpaces the Ragdoll.
Thus, the hierarchical speed order among them can be shown by the following
Copyright © 2024 Md. Shouvik Iqbal. This book is licensed under a Creative Commons Attribution-NonCommercial-ShareAlike 4.0 International License (CC BY-NC-SA 4.0)
Download for free at https://md-shouvik-iqbal.github.io/calculus
Chapter 1 Analytic Evaluation of Limit of a Function Page 53
inequality.
Similar to the analogy above, the Squeeze Theorem is based on the idea that
a function f (x) is squeezed between two other functions h(x) and g(x) near a
point x = a. If the functions h(x) and g(x) have the same limit L at the point
x = a, then the function f (x) being trapped between h(x) and g(x) must also
have the same limit L, at the point x = a. This can be better understood by
looking at the figure below.
Figure 1.8
Now, this is interesting. Because, no matter how complicated f (x) is, if h(x) and
Copyright © 2024 Md. Shouvik Iqbal. This book is licensed under a Creative Commons Attribution-NonCommercial-ShareAlike 4.0 International License (CC BY-NC-SA 4.0)
Download for free at https://md-shouvik-iqbal.github.io/calculus
Page 54 Analytic Evaluation of Limit of a Function Chapter 1
g(x) are relatively simple functions and their limits are equal at a point x = a
where f (x) is squeezed between them near that point, then evaluating the limit
of f (x) at that point becomes trivial, as it doesn’t require explicit evaluation of
the limit of f (x).
If h(x) ≤ f (x) ≤ g(x) for all x over an open interval containing a, except
possibly at a itself, and if the following is true,
then,
lim f (x) = L
x→a
This theorem allows us to analytically evaluate functions that were not supposed
to be evaluated by other methods discussed before. For example, consider the
following function,
sin (x)
f (x) =
x
What is the limit of f (x), as x → 0?
1
the area of triangle = (base × height)
2
1
the area of a sector = r2 θ
2
where, r is the radius and θ is the angle.
Copyright © 2024 Md. Shouvik Iqbal. This book is licensed under a Creative Commons Attribution-NonCommercial-ShareAlike 4.0 International License (CC BY-NC-SA 4.0)
Download for free at https://md-shouvik-iqbal.github.io/calculus
Chapter 1 Analytic Evaluation of Limit of a Function Page 55
With that, we draw the following figure of a unit circle to evaluate the limit
sin (x)
lim . Note that, since it is a unit circle, therefore the radius r = 1.
x→0 x
y
B’
1
x
x
O A A’
1
Figure 1.9
For ∆A0 OB 0 ,
opposite side
tan (x) =
adjacent side
opposite side
=
1
Copyright © 2024 Md. Shouvik Iqbal. This book is licensed under a Creative Commons Attribution-NonCommercial-ShareAlike 4.0 International License (CC BY-NC-SA 4.0)
Download for free at https://md-shouvik-iqbal.github.io/calculus
Page 56 Analytic Evaluation of Limit of a Function Chapter 1
= opposite side
∴ opposite side = tan (x)
where,
1 1 sin (x)
area of ∆OA0 B =
(base × height) = · 1 · sin (x) =
2 2 2
1 1 x
area of sector OA0 B = r2 · θ = (1)2 · x =
2 2 2
1 1 tan (x)
area of ∆OA0 B 0 = (base × height) = · 1 · tan (x) =
2 2 2
Thus, inequality (1) can be written as,
sin (x)
x tan (x)
2 2 2
≤ ≤
sin (x)
sin (x) sin (x)
2
2 2
x tan (x)
1≤ ≤
sin (x) sin (x)
sin (x)
x cos (x)
1≤ ≤
sin (x) sin (x)
x 1
1≤ ≤
sin (x) cos (x)
Now, by taking the reciprocals, we get,
sin (x)
1≥ ≥ cos (x)
x
Now, taking the limit as x → 0, we get,
Copyright © 2024 Md. Shouvik Iqbal. This book is licensed under a Creative Commons Attribution-NonCommercial-ShareAlike 4.0 International License (CC BY-NC-SA 4.0)
Download for free at https://md-shouvik-iqbal.github.io/calculus
Chapter 1 Analytic Evaluation of Limit of a Function Page 57
sin (x)
lim 1 ≥ lim ≥ lim cos (x)
x→0 x→0 x x→0
sin (x)
1 ≥ lim ≥1
x→0 x
Therefore, by the Squeeze Theorem, we get,
sin (x)
=1 lim
x→0 x
The limit above is crucial because it frequently aids in evaluating other limits of
functions using the Squeeze Theorem. Therefore, we have the following remark.
sin (x)
lim =1
x→0 x
x
I Example 1.4.23. Evaluate lim
x→0 sin(x)
sin (x)
Solution. We know that lim = 1, therefore,
x→0 x
x 1
lim = lim
x→0 sin(x) x→0 sin(x)
x
1
=
sin(x)
lim
x→0 x
1
=
1
x
Therefore, lim =1
x→0 sin(x)
Remark 1.4.5.
sin (x) x
lim = lim =1
x→0 x x→0 sin(x)
tan(x)
I Example 1.4.24. Evaluate lim
x→0 x
Copyright © 2024 Md. Shouvik Iqbal. This book is licensed under a Creative Commons Attribution-NonCommercial-ShareAlike 4.0 International License (CC BY-NC-SA 4.0)
Download for free at https://md-shouvik-iqbal.github.io/calculus
Page 58 Analytic Evaluation of Limit of a Function Chapter 1
sin(x)
Solution. We know that lim = 1 and lim cos(x) = 1, therefore,
x→0 x x→0
sin(x)
tan(x) cos(x)
lim = lim
x→0 x x→0 x
sin(x) 1
= lim ·
x→0 cos(x) x
1 sin(x)
= lim ·
x→0 cos(x) x
1 sin(x)
= lim · lim
x→0 cos(x) x→0 x
=1
tan(x)
Therefore, lim =1
x→0 x
x
I Example 1.4.25. Evaluate lim
x→0 tan(x)
tan(x)
Solution. We know that lim = 1, therefore,
x→0 x
x 1
lim = lim
x→0 tan(x) x→0 tan(x)
x
1
=
tan(x)
lim
x→0 x
=1
x
Therefore, lim =1
x→0 tan(x)
Remark 1.4.6.
tan(x) x
lim = lim =1
x→0 x x→0 tan(x)
cos(x) − 1
I Example 1.4.26. Evaluate lim
x→0 x
sin(x)
Solution. Since lim = 1, therefore,
x→0 x
cos(x) − 1 cos(x) − 1 cos(x) + 1
lim = lim ·
x→0 x x→0 x cos(x) + 1
Copyright © 2024 Md. Shouvik Iqbal. This book is licensed under a Creative Commons Attribution-NonCommercial-ShareAlike 4.0 International License (CC BY-NC-SA 4.0)
Download for free at https://md-shouvik-iqbal.github.io/calculus
Chapter 1 Analytic Evaluation of Limit of a Function Page 59
cos2 (x) − 1
= lim
x→0 x · (cos(x) + 1)
−sin2 (x)
= lim
x→0 x · (cos(x) + 1)
sin(x) sin(x)
= − lim ·
x→0 x cos(x) + 1
0
= −1 ·
1+1
=0
cos(x) − 1
Therefore, lim .
x→0 x
Remark 1.4.7.
cos(x) − 1
lim =0
x→0 x
sin(x)
I Example 1.4.27. Evaluate lim
x→∞ x
Solution. We know that for all x, the sine function oscillates between −1 to 1.
Therefore,
−1 ≤ sin(x) ≤ 1
1 sin(x) 1
≤ ≤ −
x x x
1 sin(x) 1
− lim ≤ lim ≤ lim
x→∞ x x→∞ x x→∞ x
sin(x)
0 ≤ lim ≤0
x→∞ x
As a result, by the Squeeze Theorem,
sin(x)
lim =0
x→∞ x
cos(x)
I Example 1.4.28. Evaluate lim
x→∞ x
Solution. We know that for all x, the sine function oscillates between −1 to 1.
Therefore,
−1 ≤ cos(x) ≤ 1
Copyright © 2024 Md. Shouvik Iqbal. This book is licensed under a Creative Commons Attribution-NonCommercial-ShareAlike 4.0 International License (CC BY-NC-SA 4.0)
Download for free at https://md-shouvik-iqbal.github.io/calculus
Page 60 Analytic Evaluation of Limit of a Function Chapter 1
1 cos(x) 1
≤ − ≤
x x x
1 cos(x) 1
− lim ≤ lim ≤ lim
x→∞ x x→∞ x x→∞ x
cos(x)
0 ≤ lim ≤0
x→∞ x
As a result, by the Squeeze Theorem,
cos(x)
lim =0
x→∞ x
Remark 1.4.8.
sin(x) cos(x)
lim = lim =0
x→∞ x x→∞ x
1. Reciprocal Identities
1 1
sin(x) = csc(x) =
csc(x) sin(x)
1 1
cos(x) = sec(x) =
sec(x) cos(x)
1 1
tan(x) = cot(x) =
cot(x) tan(x)
Copyright © 2024 Md. Shouvik Iqbal. This book is licensed under a Creative Commons Attribution-NonCommercial-ShareAlike 4.0 International License (CC BY-NC-SA 4.0)
Download for free at https://md-shouvik-iqbal.github.io/calculus
Chapter 1 Analytic Evaluation of Limit of a Function Page 61
sin(x) cos(x)
tan(x) = cot(x) =
cos(x) sin(x)
3. Pythagorean Identities
4. Reduction Formulas
5. Cofunction Identities
π π
sin − x = cos(x) csc − x = sec(x)
2 2
π π
cos − x = sin(x) sec − x = csc(x)
2 2
π π
tan − x = cot(x) cot − x = tan(x)
2 2
Copyright © 2024 Md. Shouvik Iqbal. This book is licensed under a Creative Commons Attribution-NonCommercial-ShareAlike 4.0 International License (CC BY-NC-SA 4.0)
Download for free at https://md-shouvik-iqbal.github.io/calculus
Page 62 Analytic Evaluation of Limit of a Function Chapter 1
1 + cos(2x)
cos2 (x) =
2
1 − cos(2x)
tan2 (x) =
1 + cos(2x)
9. Sum to Product Formulas
x+y x−y
sin(x) + sin(y) = 2 sin cos
2 2
x+y x−y
sin(x) − sin(y) = 2 cos sin
2 2
x+y x−y
cos(x) + cos(y) = 2 cos cos
2 2
x+y x−y
cos(x) − cos(y) = −2 sin sin
2 2
10. Product to Sum Formulas
1
sin(x) sin(y) = [cos(x − y) − cos(x + y)]
2
1
cos(x) cos(y) = [cos(x − y) + cos(x + y)]
2
1
sin(x) cos(y) = [sin(x + y) + sin(x − y)]
2
1
cos(x) sin(y) = [sin(x + y) − sin(x − y)]
2
A few examples using the identities and formulas above are provided below.
1 − sin(x)
I Example 1.4.29. Evaluate limπ
x→ 2 cos(x)
Solution. In order to evaluate the limit, we first simplify it as follows.
1 − sin(x) 1 − sin(x) 1 + sin(x)
limπ = limπ ·
x→ 2 cos(x) x→ 2 cos(x) 1 + sin(x)
(1 + sin(x))(1 − sin(x))
= limπ
x→ 2 (1 + sin(x)) cos(x)
1 − sin2 (x)
= limπ
x→ 2 (1 + sin(x)) cos(x)
Copyright © 2024 Md. Shouvik Iqbal. This book is licensed under a Creative Commons Attribution-NonCommercial-ShareAlike 4.0 International License (CC BY-NC-SA 4.0)
Download for free at https://md-shouvik-iqbal.github.io/calculus
Chapter 1 Analytic Evaluation of Limit of a Function Page 63
cos(x)
= limπ
x→ 2 1 + sin(x)
π
cos
= 2
π
1 + sin
2
0
=
1+1
=0
1 − sin(x)
Therefore, limπ =0
x→ 2 cos(x)
Copyright © 2024 Md. Shouvik Iqbal. This book is licensed under a Creative Commons Attribution-NonCommercial-ShareAlike 4.0 International License (CC BY-NC-SA 4.0)
Download for free at https://md-shouvik-iqbal.github.io/calculus
Page 64 Analytic Evaluation of Limit of a Function Chapter 1
x
= lim tan
x→0
2
0
= tan
2
=0
1 1
I Example 1.4.31. Evaluate lim −
x→0 sin(x) tan(x)
Solution. In order to evaluate the limit, we first simplify it as follows.
cos(x)
1 1 1
lim − = lim −
x→0 sin(x) tan(x) x→0 sin(x) sin(x)
1 − cos(x)
= lim
x→0 sin(x)
x
Now, since cos(2x) = 1 − 2 sin2 (x), therefore, cos(x) = 1 − 2 sin2 . Likewise,
x x2
since sin(2x) = 2 sin(x) cos(x), therefore, sin(x) = 2 sin cos . Thus,
2 2
2 x
1 − cos(x) 1 − 1 − 2sin
lim = lim x2
sin(x)
x
x→0 x→0
2 sin cos
x 2
x
2sin2
= lim x 2 x
x→0
2 sin
cos
xx 2
sin
= lim 2x
x→0
cos
x 2
= lim tan
x→0
2
0
= tan
2
=0
1 1
Therefore, lim − =0
x→0 sin(x) tan(x)
Copyright © 2024 Md. Shouvik Iqbal. This book is licensed under a Creative Commons Attribution-NonCommercial-ShareAlike 4.0 International License (CC BY-NC-SA 4.0)
Download for free at https://md-shouvik-iqbal.github.io/calculus
Chapter 1 Analytic Evaluation of Limit of a Function Page 65
1 − cos(x)
I Example 1.4.32. Evaluate lim
x→0 x
Solution. Since it is known, from the double angle identity, that cos(2x) = 1 −
2sin2 (x), therefore,
2 x
cos(x) = 1 − 2 sin
2
2 x
cos(x) − 1 = −2 sin
2
2 x
1 − cos(x) = 2 sin
2
This implies that,
x
2
1 − cos(x) 2sin
lim = lim 2
x→0 x x→0 x
sin(x)
Now, we do the following simplification to apply the standard limit lim =
x→0 x
1. That is,
2 x
1 − cos(x) 2sin
lim = lim 2
x→0 x x→0 x
x
2sin2
= lim 2 ·x
x→0 x x
x
2sin2
= lim 2 ·x
x→0 x2
x
2sin2
= lim 2 ·x
x→0 x2
·4
4
x
2sin2
= lim 22 · x
x→0 x2
·4
2
x
2sin2 x
= lim 22 ·
x→0 x2 4
2
x
sin2 x
= lim
2 · 22 ·
x→0 x2 4
2
Copyright © 2024 Md. Shouvik Iqbal. This book is licensed under a Creative Commons Attribution-NonCommercial-ShareAlike 4.0 International License (CC BY-NC-SA 4.0)
Download for free at https://md-shouvik-iqbal.github.io/calculus
Page 66 Analytic Evaluation of Limit of a Function Chapter 1
x 2
sin 2 x
= lim 2 ·2
x→0 x
2
2
x
sin 2 x
= lim 2 · lim
x→0 x x→0 2
2
x 1
Now, as x → 0, → 0, since x is multiplied by a constant of . Therefore, by
2 2
sin (x)
the the standard limit lim = 1, we get the following,
x→0 x
2 2
x x
sin 2 x sin 2 x
lim 2 · lim = xlim 2 · xlim
x→0 x x→0 2 2
→0 x →0 2
|2 {z }
2 2 0
| {z }
1
=1·0
=0
1 − cos(x)
Therefore, lim =0
x→0 x
cos(2x) − cos(3x)
I Example 1.4.33. Evaluate lim
x→0 x2
Solution. Using the Sum to Product
Formula
x+y x−y
cos(x) − cos(y) = −2 sin sin , we write the following,
2 2
2x + 3x 2x − 3x
−2 sin sin
cos(2x) − cos(3x) 2 2
lim = lim
x→0 x2 x→0
x2
5x x
2 sin sin
2 2
= lim
x→0 x2
sin (x)
Now, we do the following simplification to apply the standard limit lim =
x→0 x
Copyright © 2024 Md. Shouvik Iqbal. This book is licensed under a Creative Commons Attribution-NonCommercial-ShareAlike 4.0 International License (CC BY-NC-SA 4.0)
Download for free at https://md-shouvik-iqbal.github.io/calculus
Chapter 1 Analytic Evaluation of Limit of a Function Page 67
1.
5x
sin 2
x
5x sin 2
x
2 · · x ·
5x x 5x 2 2
2 sin sin
2 2 2 2
lim 2
= lim
x→0 x x2 x→0
5x
sin
x
sin
2
2
· 2 · 5x · x
5x x 2 2
2
= lim 2
2
x→0
x
5x
sin 2
x
sin 2
= lim 2 · 2 · 5x · 1
x
x→0 5x 4 x2
2 2
5x
sin 2
x
sin 2
= lim · 2 · 5x · 1
x
x→0 5x 2 x2
2 2
5x x
sin 2 sin
2 · 5
= lim
· x
x→0 5x 2
2 2
5x
sin
x
sin
= lim
2
· lim 2 ·5
x→0 5x x→0 x 2
2 2
5x x
Now, as x → 0, both → 0 and → 0, since they are multiplied by a constant
2 2
5 1 sin (x)
of and , respectively. Therefore, by the the standard limit lim = 1,
2 2 x→0 x
we write,
5x 5x
sin sin
x x
sin sin
lim
2
· lim 2 · 5 = lim 2
· lim 2 ·5
x→0 5x x→0 x 2 5x
→0 5x x
→0 x 2
2 2
2 2 2 2
5
=1·1·
2
5
=
2
cos(2x) − cos(3x) 5
Therefore, lim 2
=
x→0 x 2
Copyright © 2024 Md. Shouvik Iqbal. This book is licensed under a Creative Commons Attribution-NonCommercial-ShareAlike 4.0 International License (CC BY-NC-SA 4.0)
Download for free at https://md-shouvik-iqbal.github.io/calculus
Page 68 Limits Involving ∞ Chapter 1
1
f (x) =
(x − 4)2
The graph of f (x) is shown below.
y
x
2 4 6 8
Figure 1.10
1 1
1. lim =∞ 2. lim =0
x→4 (x − 4)2 x→∞ (x − 4)2
In the first limit, when the input x approaches a finite value of 4 from both
sides, the function f (x) increases without a bound and thus the limit does not
exist.
Copyright © 2024 Md. Shouvik Iqbal. This book is licensed under a Creative Commons Attribution-NonCommercial-ShareAlike 4.0 International License (CC BY-NC-SA 4.0)
Download for free at https://md-shouvik-iqbal.github.io/calculus
Chapter 1 Limits Involving ∞ Page 69
On the other hand, in the second limit, as x increases without a bound, f (x)
approaches a finite value of 0, and thus the limit, in this case, does exist.
The first limit is an example of an “infinite limit,” whereas the second one is of
“limit at infinity.”
Definition 1.5.1. (Infinite Limits) Let f (x) be a function defined for all x in
an open interval containing a. Then,
Note that, since the symbol ∞ does not represents a number, therefore the limits
lim f (x) = +∞ and lim f (x) = −∞ does not imply that the limit exists, but
x→a x→a
rather denotes the behavior of the limit (whether the function is growing in
a positive or negative direction). Definition above can be elaborated in detail
through the use of one-sided limits as follows.
1. Infinite limits from the left: Let f (x) be a function defined for all x
near a. Then,
Copyright © 2024 Md. Shouvik Iqbal. This book is licensed under a Creative Commons Attribution-NonCommercial-ShareAlike 4.0 International License (CC BY-NC-SA 4.0)
Download for free at https://md-shouvik-iqbal.github.io/calculus
Page 70 Limits Involving ∞ Chapter 1
lim f (x) = +∞
x→a−
lim f (x) = −∞
x→a−
2. Infinite limits from the right: Let f (x) be a function defined for all
x near a. Then,
lim f (x) = +∞
x→a+
lim f (x) = −∞
x→a+
Copyright © 2024 Md. Shouvik Iqbal. This book is licensed under a Creative Commons Attribution-NonCommercial-ShareAlike 4.0 International License (CC BY-NC-SA 4.0)
Download for free at https://md-shouvik-iqbal.github.io/calculus
Chapter 1 Limits Involving ∞ Page 71
y y
x
a
x
a
(a) (b)
y y
x
a
x
a
(c) (d)
Figure 1.11
Figure 1.11(a) shows the limit lim− f (x) = +∞, Figure 1.11(b) shows the limit
x→a
lim− f (x) = −∞, Figure 1.11(c) shows the limit lim+ f (x) = +∞, lastly, Fig-
x→a x→a
ure 1.11(d) shows the limit lim+ f (x) = −∞.
x→a
We now began to evaluate functions that have an infinite limit. However, before
a
that, notice that, if we have a fraction, , where a 6= 0 and b 6= 0, then
b
a
1. if b → 0, then → ∞.
b
Copyright © 2024 Md. Shouvik Iqbal. This book is licensed under a Creative Commons Attribution-NonCommercial-ShareAlike 4.0 International License (CC BY-NC-SA 4.0)
Download for free at https://md-shouvik-iqbal.github.io/calculus
Page 72 Limits Involving ∞ Chapter 1
a
2. if b → ∞, then → 0.
b
x2 + 1
I Example 1.5.1. Evaluate lim+
x→0 x
x2 + 1
Solution. Notice that as x → 0+ , the denominator of the function ap-
x
proaches 0, whereas the numerator (x2 + 1) approaches the value 1. Therefore,
x2 + 1
lim+ =∞
x→0 x
2
x +1
Therefore, lim+ =∞
x→0 x
Copyright © 2024 Md. Shouvik Iqbal. This book is licensed under a Creative Commons Attribution-NonCommercial-ShareAlike 4.0 International License (CC BY-NC-SA 4.0)
Download for free at https://md-shouvik-iqbal.github.io/calculus
Chapter 1 Limits Involving ∞ Page 73
=π+0
=π
1 1
Similarly, if x → −∞, the term → 0, that is, lim = 0. Thus, we write
x x→−∞ x
the following,
1 1
lim π+ = lim π + lim
x→−∞ x x→−∞ x→−∞ x
=π+0
=π
-4 -2 2 4
Figure 1.12
Copyright © 2024 Md. Shouvik Iqbal. This book is licensed under a Creative Commons Attribution-NonCommercial-ShareAlike 4.0 International License (CC BY-NC-SA 4.0)
Download for free at https://md-shouvik-iqbal.github.io/calculus
Page 74 Limits Involving ∞ Chapter 1
lim f (x) = L
x→+∞
If f (x) becomes arbitrarily close to L for all sufficiently large x < 0, then
lim f (x) = L
x→−∞
Notice that if f (x) → L, as x → ±∞, the graph of f (x) approaches the hori-
zontal line y = L. The horizontal line y = L is called the horizontal asymptote
of f (x).
ex + 1
I Example 1.5.4. Evaluate lim
x→∞ x
Solution. In order to evaluate the limit, we do the following simplification,
ex + 1 ex 1
lim = lim +
x→∞ x x→∞ x x
1
= lim e +
x→∞ x
1
Now, as x → ∞, the term → 0. This implies that,
x
1 1
lim e + = lim e + lim
x→∞ x x→∞ x→∞ x
=e+0
=e
ex + 1
Therefore, lim =e
x→∞ x
Copyright © 2024 Md. Shouvik Iqbal. This book is licensed under a Creative Commons Attribution-NonCommercial-ShareAlike 4.0 International License (CC BY-NC-SA 4.0)
Download for free at https://md-shouvik-iqbal.github.io/calculus
Chapter 1 Limits Involving ∞ Page 75
x + x2 + x3 + x4 + x5
I Example 1.5.5. Evaluate lim
x→∞ x5
Solution. In order to evaluate the limit, we do the following simplification,
x + x2 + x3 + x4 + x5 x x2 x3 x4 x5
lim = lim + + + +
x→∞ x5 x→∞ x5 x5 x5 x5 x5
1 1 1 1
= lim 4 + 3 + 2 + + 1
x→∞ x x x x
Now, as x → ∞, the all term except the last one approach 0 in the limit above,
therefore, we write,
1 1 1 1 1 1 1 1
lim + + + + 1 = lim + lim + lim + lim + lim 1
x→∞ x4 x3 x2 x x→∞ x4 x→∞ x3 x→∞ x2 x→∞ x x→∞
=0+0+0+0+1
=1
x + x2 + x3 + x4 + x5
Therefore, lim =1
x→∞ x5
Chap. 1 / Sec. 1.5 / Subsec. 1.5.3 : Infinite Limits at Infinity and End Behav-
iors
If a function f (x) increases or decreases without a bound as its input x increases
or decreases without a bound, then we have an infinite limit at infinity. For
example, consider the following function,
f (x) = x3
Copyright © 2024 Md. Shouvik Iqbal. This book is licensed under a Creative Commons Attribution-NonCommercial-ShareAlike 4.0 International License (CC BY-NC-SA 4.0)
Download for free at https://md-shouvik-iqbal.github.io/calculus
Page 76 Limits Involving ∞ Chapter 1
-2 -1 1 2
-1
-2
Figure 1.13
Having discussed the idea of the infinite limits at infinite, it is now crucial to
discuss the end behavior of a function. End behavior refers to how a function
f (x) behaves as its input x increases or decreases indefinitely. The function
can show various behaviors at its endpoints. The following are among the most
common ones.
2. f (x) → +∞ or f (x) → −∞
3. Neither 1 nor 2.
Copyright © 2024 Md. Shouvik Iqbal. This book is licensed under a Creative Commons Attribution-NonCommercial-ShareAlike 4.0 International License (CC BY-NC-SA 4.0)
Download for free at https://md-shouvik-iqbal.github.io/calculus
Chapter 1 Limits Involving ∞ Page 77
f (x) = xn
Remark 1.5.1.
(
−∞, n = 1, 3, 5, · · ·
lim xn = ∞ n = 1, 2, 3, · · · and lim xn =
x→∞ x→−∞ +∞, n = 2, 4, 6, · · ·
Now consider the reciprocal of the power function, where n is a positive integer.
1
f (x) = = x−n
xn
For the reciprocal, as x → ±∞, f (x) → 0, regardless of whether n is even or
odd.
1. As x → +∞, f (x) → +∞
2. As x → −∞, f (x) → −∞
1. As x → +∞, f (x) → +∞
2. As x → −∞, f (x) → +∞
Copyright © 2024 Md. Shouvik Iqbal. This book is licensed under a Creative Commons Attribution-NonCommercial-ShareAlike 4.0 International License (CC BY-NC-SA 4.0)
Download for free at https://md-shouvik-iqbal.github.io/calculus
Page 78 Limits Involving ∞ Chapter 1
1
I Example 1.5.8. Analyze the end behavior of f (x) = .
x3
1
Solution. Here, f (x) = is a reciprocal of x3 . That’s why,
x3
1. As x → +∞, f (x) → 0
2. As x → −∞, f (x) → 0
This implies that as x → ±∞, the behavior of the polynomial P (x) is determined
by the term with the highest power of x, that is, an xn . Therefore, we have the
following remark.
Copyright © 2024 Md. Shouvik Iqbal. This book is licensed under a Creative Commons Attribution-NonCommercial-ShareAlike 4.0 International License (CC BY-NC-SA 4.0)
Download for free at https://md-shouvik-iqbal.github.io/calculus
Chapter 1 Limits Involving ∞ Page 79
I Example 1.5.9. If P (x) = 2x4 − 3x3 + 5x2 − 7x + 11, evaluate lim P (x).
x→±∞
=∞
=∞
since the degree is even.
I Example 1.5.10. If P (x) = −3x5 + 5x4 − 7x3 + 11x2 − 13x + 17, evaluate
lim P (x)
x→±∞
= −∞
=∞
since the degree is odd and the leading coefficient is negative.
Copyright © 2024 Md. Shouvik Iqbal. This book is licensed under a Creative Commons Attribution-NonCommercial-ShareAlike 4.0 International License (CC BY-NC-SA 4.0)
Download for free at https://md-shouvik-iqbal.github.io/calculus
Page 80 Limits Involving ∞ Chapter 1
Since the degree of the numerator is less than that of the denominator, therefore
we modify the expression by dividing every term in both the numerator and
denominator by the greatest power of x in the denominator. By doing so, we’ll
be able to identify the term in the overall expression that dominates the behavior
of the function for large values of x. That is,
x2 − 1
2
x −1 3
lim 3 = lim 3x
x→∞ x − 1 x→∞ x − 1
x3
x2 1
3
− 3
= lim x3 x
x→∞ x 1
−
x3 x3
1 1
− 3
= lim x x
x→∞ 1
1− 3
x
1 1
lim − lim
x→∞ x x→∞ x3
=
1
1 − lim
x→∞ x3
0−0
=
1−0
=0
x2 − 1
This can be verified by looking at the graph of f (x) = 3 as x → ∞ as
x −1
follows.
Copyright © 2024 Md. Shouvik Iqbal. This book is licensed under a Creative Commons Attribution-NonCommercial-ShareAlike 4.0 International License (CC BY-NC-SA 4.0)
Download for free at https://md-shouvik-iqbal.github.io/calculus
Chapter 1 Limits Involving ∞ Page 81
-2 -1 1 2
-1
-2
Figure 1.14
x2 − 1
Notice that, since lim 3 = 0, therefore f (x) has an horizontal asymptote
x→∞ x − 1
at y = 0.
Now, we shall consider a function where the degree of the numerator is equal to
that of the denominator and x → ∞. That is,
9x + π
f (x) =
3x + e
Since π and e are constants, therefore as x → ∞, both the numerator and de-
nominator approach ∞. To deal with this absurdity, we divide each term in both
the numerator and denominator by the highest power of x in the denominator
as before. That is,
9x + π
f (x) =
3x + e
9x + π
= x
3x + e
x
9x π
+
= x x
3x e
+
x x
π
9+
= x
e
3+
x
Copyright © 2024 Md. Shouvik Iqbal. This book is licensed under a Creative Commons Attribution-NonCommercial-ShareAlike 4.0 International License (CC BY-NC-SA 4.0)
Download for free at https://md-shouvik-iqbal.github.io/calculus
Page 82 Limits Involving ∞ Chapter 1
π
9+
∴ f (x) = x
e
3+
x
Now, taking the limit of f (x) as x → ∞, we get,
π
9+
lim x = 9+0
x→∞ e 3+0
3+
x
9
=
3
=3
9x + π
Note that, since lim = 3, therefore f (x) has a horizontal asymptote at
3x + e x→∞
y = 3. This can be seen in the following figure.
5 3
-10 -5 5 10
Figure 1.15
Copyright © 2024 Md. Shouvik Iqbal. This book is licensed under a Creative Commons Attribution-NonCommercial-ShareAlike 4.0 International License (CC BY-NC-SA 4.0)
Download for free at https://md-shouvik-iqbal.github.io/calculus
Chapter 1 Limits Involving ∞ Page 83
x
=
1
1+
x
x
∴ f (x) =
1
1+
x
Now, if we take the limit of f (x) as x → ±∞, then the denominator of f (x)
approaches the value 1, and the numerator approaches +∞ as x → +∞ and
−∞ as x → −∞. Thus, the limit is,
x
lim = ±∞
x→±∞ 1
1+
x
This presents a special case. Previously, we observed that rational functions
tend to approach a horizontal asymptote as x → ±∞. However, in this case,
the limit doesn’t approach a horizontal asymptote as x → ±∞. Nevertheless, it
does approach a linear asymptote, which becomes apparent when seen visually
as follows.
-10 -5 5 10
-5
-10
Figure 1.16
Looking at the figure above, it appears that the function f (x) resembles a linear
function as x → ±∞. To determine the linear function that f (x) is approaching,
we do the long division on f (x) and obtain the following.
x2
f (x) =
1+x
1
by long division
=x−1+
x+1
Now, we take the limit as x → ±∞,
1 1
lim x − 1 + = lim (x − 1) + lim
x→±∞ x+1 x→±∞ x→±∞ x + 1
Copyright © 2024 Md. Shouvik Iqbal. This book is licensed under a Creative Commons Attribution-NonCommercial-ShareAlike 4.0 International License (CC BY-NC-SA 4.0)
Download for free at https://md-shouvik-iqbal.github.io/calculus
Page 84 Limits Involving ∞ Chapter 1
1
Here, as x → ±∞, the term → 0, therefore,
x+1
1
lim x − 1 + = lim (x − 1)
x→±∞ x+1 x→±∞
This suggests that the function f (x) behaves similarly to the linear function
x − 1, as x → ±∞. For this reason, the graph of f (x) approaches the graph
of linear function x − 1, as x → ±∞. Thus, the line x − 1 is an asymptote of
the function f (x) as x → ±∞. Such an asymptote is called slant asymptote or
oblique asymptote. Oblique asymptotes occur when the numerator has a degree
that is exactly one more than that of the denominator.
Remark 1.5.3.
Let P (x) and Q(x) be polynomial functions such that
P (x)
R (x) =
Q (x)
an xn + an−1 xn−1 + · · · + a1 x + a0
=
bm xm + bm−1 xm−1 + · · · + b1 x + b0
where, an 6= 0, bm 6= 0. Now,
Copyright © 2024 Md. Shouvik Iqbal. This book is licensed under a Creative Commons Attribution-NonCommercial-ShareAlike 4.0 International License (CC BY-NC-SA 4.0)
Download for free at https://md-shouvik-iqbal.github.io/calculus
Chapter 1 Limits Involving ∞ Page 85
2x3 + 3x2 + 5x + 7
I Example 1.5.11. Evaluate lim
x→±∞ 11x2 + 13x + 17
Solution. Since the degree of the numerator (that is, 3) is greater than the degree
of the denominator (that is, 2), we divide the numerator and denominator by
xn , where n is the greatest power in the denominator (that is, 2).
2x3 + 3x2 + 5x + 7
2x3 + 3x2 + 5x + 7 x2
lim 2
= lim 2
x→±∞ 11x + 13x + 17 x→±∞ 11x + 13x + 17
x2
2x3 3x2 5x 7
2
+ 2 + 2+ 2
= lim x x x x
x→±∞ 11x2 13x 17
+ 2 + 2
x2 x x
5 7
2x + 3 + + 2
= lim x x
x→±∞ 13 17
11 + + 2
x x
= ±∞
2x3 + 3x2 + 5x + 7
Therefore, as x → +∞, the function f (x) = → +∞. Simi-
11x2 + 13x + 17
2x3 + 3x2 + 5x + 7
larly, as x → −∞, the function f (x) = → −∞.
11x2 + 13x + 17
πx3 + e
I Example 1.5.12. Evaluate lim
x→±∞ ex2 + π
Solution. Since the degree of the numerator (that is, 3) is greater than the degree
of the denominator (that is, 2), we divide the numerator and denominator by
xn , where n is the greatest power in the denominator.
πx3 + e
3
πx + e x2
lim = lim
x→±∞ ex2 + π x→±∞ ex2 + π
x2
πx3 e
2
+ 2
= lim x 2 x
x→±∞ ex π
+
x2 x2
Copyright © 2024 Md. Shouvik Iqbal. This book is licensed under a Creative Commons Attribution-NonCommercial-ShareAlike 4.0 International License (CC BY-NC-SA 4.0)
Download for free at https://md-shouvik-iqbal.github.io/calculus
Page 86 Limits Involving ∞ Chapter 1
e
πx +
= lim x2
x→±∞ π
e+ 2
x
= ±∞
πx3 + e
Therefore, as x → +∞, the function f (x) = → +∞. Similarly, as
ex2 + π
πx3 + e
x → −∞, the function f (x) = → −∞.
ex2 + π
3x3 + 2x2
I Example 1.5.13. Evaluate lim
x→±∞ 9x3 + 6
Solution. Since the degree of the numerator (that is, 3) is equal to the degree of
the denominator (that is also 3), we divide the numerator and denominator by
xn , where n is the greatest power in the denominator.
3x3 + 2x2
3x3 + 2x2 x3
lim 3
= lim 3
x→±∞ 9x + 6 x→±∞ 9x + 6
x3
3x3 2x2
3
+ 3
= lim x 3 x
x→±∞ 9x 6
3
+ 3
x x
2
3+
= lim x
x→±∞ 6
9+ 3
x
3
=
9
3x3 + 2x2 3
Therefore, as x → +∞, the function f (x) = → . Similarly, as
9x3 + 6 9
3x3 + 2x2 3
x → −∞, the function f (x) = 3
→ .
9x + 6 9
2eπx2 + x
I Example 1.5.14. Evaluate lim
x→±∞ x2 + 2x
Solution. Since the degree of the numerator (2) is equal to the degree of the
denominator (2), divide the numerator and denominator by xn , where n is the
Copyright © 2024 Md. Shouvik Iqbal. This book is licensed under a Creative Commons Attribution-NonCommercial-ShareAlike 4.0 International License (CC BY-NC-SA 4.0)
Download for free at https://md-shouvik-iqbal.github.io/calculus
Chapter 1 Limits Involving ∞ Page 87
2eπx2 + x
2
2eπx + x x2
lim = lim 2
x→±∞ x2 + 2x x→±∞ x + 2x
x2
2eπx2 x
+
= lim x2 x2
2
x→±∞ x 2x
2
+ 2
x x
1
2eπ +
= lim x
x→±∞ 2
1+
x
= 2eπ
2eπx2 + x
Therefore, as x → +∞, the function f (x) = → 2eπ. Similarly, as
x2 + 2x
2eπx2 + x
x → −∞, the function f (x) = → 2eπ.
x2 + 2x
3x3 + x
I Example 1.5.15. Evaluate lim
x→±∞ 5x5 + 2x2
Solution. Since the degree of the numerator (3) is less than the degree of the
denominator (5), we divide the numerator and denominator by xn , where n is
the greatest power in the denominator. That is,
3x3 + x
3
3x + x x5
lim = lim
5
x→±∞ 5x + 2x 2 x→±∞ 5x + 2x2
5
x5
3
3x x
5
+ 5
= lim x5 x
x→±∞ 5x 2x2
+
x5 x5
3 1
2
+ 4
= lim x x
x→±∞ 2
5+ 3
x
0
=
5
=0
Copyright © 2024 Md. Shouvik Iqbal. This book is licensed under a Creative Commons Attribution-NonCommercial-ShareAlike 4.0 International License (CC BY-NC-SA 4.0)
Download for free at https://md-shouvik-iqbal.github.io/calculus
Page 88 Limits Involving ∞ Chapter 1
3x3 + x
Therefore, as x → +∞, the function f (x) = → 0. Similarly, as
5x5 + 2x2
3x3 + x
x → −∞, the function f (x) = → 0.
5x5 + 2x2
x3 + 2x2 + 3x + 4
I Example 1.5.16. Evaluate lim 5
x→±∞ x + 2x4 + 3x3 + 4x2 + 5x + 6
Solution. Since the degree of the numerator (3) is less than the degree of the
denominator (5), we divide the numerator and denominator by xn , where n is
the greatest power in the denominator. That is,
x3 + 2x2 + 3x + 4
x3 + 2x2 + 3x + 4 x5
lim 5 = lim 5
4 3 2
x→±∞ x + 2x + 3x + 4x + 5x + 6 x→±∞ x + 2x + 3x + 4x2 + 5x + 6
4 3
x5
3
x 2x2 3x 4
5
+ 5
+ 5+ 5
= lim 5 x
4
x
3
x
2
x
x→±∞ x 2x 3x 4x 5x 6
5
+ 5 + 5 + 5 + 5+ 5
x x x x x x
1 2 3 4
+ + +
= lim x2 x3 x4 x5
x→±∞ 2 3 4 5 6
1+ + 2 + 3 + 4 + 5
x x x x x
0
=
1
=0
x3 + 2x2 + 3x + 4
Therefore, as x → +∞, the function f (x) = →
x5 + 2x4 + 3x3 + 4x2 + 5x + 6
x3 + 2x2 + 3x + 4
0. Similarly, as x → −∞, the function f (x) = 5 →
x + 2x4 + 3x3 + 4x2 + 5x + 6
0.
Copyright © 2024 Md. Shouvik Iqbal. This book is licensed under a Creative Commons Attribution-NonCommercial-ShareAlike 4.0 International License (CC BY-NC-SA 4.0)
Download for free at https://md-shouvik-iqbal.github.io/calculus
Chapter 1 Limits Involving ∞ Page 89
√ x
9x + π
= lim x
e
x→∞
3+
x
Now, a problem arises due to the presence of a radical in the numerator. To
address this issue, notice that x ≥ 0 as x → +∞, therefore
√
x = |x| = x2
This same strategy can be used to evaluate the limit of the following function.
Consider the function below.
10x2 + x
f (x) = √
2x2 + 8
Copyright © 2024 Md. Shouvik Iqbal. This book is licensed under a Creative Commons Attribution-NonCommercial-ShareAlike 4.0 International License (CC BY-NC-SA 4.0)
Download for free at https://md-shouvik-iqbal.github.io/calculus
Page 90 Limits Involving ∞ Chapter 1
We will use the previous strategy. Divide the numerator and denominator by
the highest power of x in the denominator. The highest power of x in the
denominator of f (x) is x2 . However, since x2 is under a square root, therefore
√
we need to divide both the numerator and denominator by x2 . That is,
10x2 + x
√
10x2 + x 2
lim √ = lim √ x
x→+∞ 2x2 + 8 x→+∞ 2x2 + 8
√
x2
10x2 + x
√
2
= lim r x
x→+∞ 2x2 + 8
x2
2
10x x
√ +√
2 x2
= lim rx
x→+∞ 2x2 8
+
x2 x2
√
Since x2 = |x| = x, for all x ≥ 0, therefore,
10x2 x 10x2 x
√ +√ +
2 x2 = lim r x
lim rx x
x→+∞ 2 x→+∞ 2
2x 8 2x 8
2
+ 2 2
+ 2
x x x x
10x + 1
= lim r
x→+∞ 8
2+ 2
x
+∞ + 1
= √
2+0
= +∞
√
Let us now consider x → −∞, where x2 = |x| = −x, for all x < 0,
10x2 + x
√
10x2 + x 2
lim √ = lim √ x
x→−∞ 2x2 + 8 x→−∞ 2x2 + 8
√
x2
Copyright © 2024 Md. Shouvik Iqbal. This book is licensed under a Creative Commons Attribution-NonCommercial-ShareAlike 4.0 International License (CC BY-NC-SA 4.0)
Download for free at https://md-shouvik-iqbal.github.io/calculus
Chapter 1 Limits Involving ∞ Page 91
10x2 + x
= lim r −x
x→−∞ 2x2 + 8
x2
2
10x x
+
= lim r −x −x
x→−∞ 2
2x 8
2
+ 2
x x
−10x − 1
= lim r
x→−∞ 8
2+ 2
x
− (−∞) − 1
= √
2+0
= +∞
lim ex = ∞
x→∞
Copyright © 2024 Md. Shouvik Iqbal. This book is licensed under a Creative Commons Attribution-NonCommercial-ShareAlike 4.0 International License (CC BY-NC-SA 4.0)
Download for free at https://md-shouvik-iqbal.github.io/calculus
Page 92 Limits Involving ∞ Chapter 1
lim π x = 0
x→−∞
1 1
lim =
x→∞ ex lim ex
x→∞
=0
Remark 1.5.5. (Logarithmic Functions) Let f (x) = loga (x), where a > 1,
then,
lim loga (x) = ∞ and lim+ loga (x) = ∞
x→∞ x→0
Solution. Since ln(x) = loge (x) and e > 1, therefore, the natural logarithmic
function grows slowly but unbounded as x → ∞. Therefore,
lim ln(x) = ∞
x→∞
Copyright © 2024 Md. Shouvik Iqbal. This book is licensed under a Creative Commons Attribution-NonCommercial-ShareAlike 4.0 International License (CC BY-NC-SA 4.0)
Download for free at https://md-shouvik-iqbal.github.io/calculus
Chapter 1 Limits Involving ∞ Page 93
Solution. Since ln(x) = loge (x) and e > 1, therefore, the natural logarithmic
function approaches negative infinity as x approaches 0 from the positive side.
Thus,
lim+ ln(x) = −∞
x→0
Copyright © 2024 Md. Shouvik Iqbal. This book is licensed under a Creative Commons Attribution-NonCommercial-ShareAlike 4.0 International License (CC BY-NC-SA 4.0)
Download for free at https://md-shouvik-iqbal.github.io/calculus
Page 94 The Concept of Continuity Chapter 1
Copyright © 2024 Md. Shouvik Iqbal. This book is licensed under a Creative Commons Attribution-NonCommercial-ShareAlike 4.0 International License (CC BY-NC-SA 4.0)
Download for free at https://md-shouvik-iqbal.github.io/calculus
Chapter 1 The Concept of Continuity Page 95
Figure 1.17
Here, the function f (x) is not defined at the point x = a, therefore f (x) is
discontinuous at the point x = a. A similarity can be seen in the following
figure.
Figure 1.18
Here, the value of the function f (x) at the point x = a differ from the limit
of f (x) as x → a, therefore f (x) is discontinuous at the point x = a.
Copyright © 2024 Md. Shouvik Iqbal. This book is licensed under a Creative Commons Attribution-NonCommercial-ShareAlike 4.0 International License (CC BY-NC-SA 4.0)
Download for free at https://md-shouvik-iqbal.github.io/calculus
Page 96 The Concept of Continuity Chapter 1
sudden “jump” in the value of the function at a certain point, causing the
left-sided and the right-sided limit to differ. The function’s value jumps
from one level to another without passing through any intermediate values.
For example, consider the figure below.
Figure 1.19
Here, lim− f (x) 6= lim+ f (x), therefore the limit of f (x) as x → a does
x→a x→a
not exist.
Figure 1.20
Copyright © 2024 Md. Shouvik Iqbal. This book is licensed under a Creative Commons Attribution-NonCommercial-ShareAlike 4.0 International License (CC BY-NC-SA 4.0)
Download for free at https://md-shouvik-iqbal.github.io/calculus
Chapter 1 The Concept of Continuity Page 97
1. f (a) is defined.
If a function does not satisfy any of the specified criteria at the point a, then
the function is not continuous at that point.
Copyright © 2024 Md. Shouvik Iqbal. This book is licensed under a Creative Commons Attribution-NonCommercial-ShareAlike 4.0 International License (CC BY-NC-SA 4.0)
Download for free at https://md-shouvik-iqbal.github.io/calculus
Page 98 The Concept of Continuity Chapter 1
a b
Figure 1.21
In the figure above, the function f (x) is continuous over the open interval (a, b)
and at the left endpoint over the closed interval [a, b], because
However, the function f (x) is not continuous at the right endpoint over the
closed interval [a, b], because
Copyright © 2024 Md. Shouvik Iqbal. This book is licensed under a Creative Commons Attribution-NonCommercial-ShareAlike 4.0 International License (CC BY-NC-SA 4.0)
Download for free at https://md-shouvik-iqbal.github.io/calculus
Chapter 1 The Concept of Continuity Page 99
Now, based on definition above, the continuity of a function over a closed interval
[a, b] is defined as follows.
To fully grasp the concept of continuity over an interval, consider the following
example.
I Example 1.6.1. Consider the figure below to identify points where f (x) has
discontinuities within the interval [0, 8].
f(x)
x
1 2 3 4 5 6 7 8
Figure 1.22
Copyright © 2024 Md. Shouvik Iqbal. This book is licensed under a Creative Commons Attribution-NonCommercial-ShareAlike 4.0 International License (CC BY-NC-SA 4.0)
Download for free at https://md-shouvik-iqbal.github.io/calculus
Page 100 The Concept of Continuity Chapter 1
Copyright © 2024 Md. Shouvik Iqbal. This book is licensed under a Creative Commons Attribution-NonCommercial-ShareAlike 4.0 International License (CC BY-NC-SA 4.0)
Download for free at https://md-shouvik-iqbal.github.io/calculus
Chapter 1 The Concept of Continuity Page 101
1. c · f (x)
2. f (x) + g(x)
3. f (x) − g(x)
4. f (x) · g(x)
f (x)
5. , where, g(x) 6= 0
g(x)
are also continuous at the point a.
The following types of functions are continuous at every point within their re-
spective domains.
5. Inverse Trigonometric Functions, that is, sin−1 (x), cos−1 (x), tan−1 (x),
cot−1 (x), sec−1 (x), csc−1 (x).
Copyright © 2024 Md. Shouvik Iqbal. This book is licensed under a Creative Commons Attribution-NonCommercial-ShareAlike 4.0 International License (CC BY-NC-SA 4.0)
Download for free at https://md-shouvik-iqbal.github.io/calculus
Page 102 The Concept of Continuity Chapter 1
The theorem above suggests that for f (g(x)) to be continuous at a point a, both
the inner function g(x) must be continuous at a, as well as the outer function f
must be continuous at the value that g(x) takes at a, that is, g(a). For example,
consider f (g(x)) = sin(x2 ), where f (x) = sin(x) and g(x) = x2 . The function
g(x) = x2 is continuous on its entire domain, and the function f (x) = sin(x)
is also continuous everywhere. Therefore, the function f (g(x)) = sin(x2 ) is also
continuous everywhere.
Chap. 1 / Sec. 1.6 / Subsec. 1.6.4 : The Intermediate Value Theorem (IVT)
The Intermediate Value Theorem (IVT) is a fundamental result in Calculus. It
formalizes the intuitive idea that if a function is continuous on a closed interval,
and if it takes on two distinct values at the endpoints of that interval, then it
must take on every value between those two endpoints at some point within
the interval. The idea is indeed intuitive and to get a gist of it, consider the
following analogy.
Suppose we’re hiking up a hill, starting at point A on the ground and heading
upwards to point B, which is at a higher elevation. The path we’re taking is
smooth, without any sudden drops or jumps. As we walk up, we inevitably pass
through every height between A and B. For instance, if we start at 0 meters
and end at 100 meters, then there must be a point where we have been at 50
meters. This is because the path is continuous, so we can’t skip any heights or
jump straight from 0 to 100 meters. This is the core idea of the Intermediate
Value Theorem.
Suppose now the hill’s height is represented by a continuous function, f (x), and
Copyright © 2024 Md. Shouvik Iqbal. This book is licensed under a Creative Commons Attribution-NonCommercial-ShareAlike 4.0 International License (CC BY-NC-SA 4.0)
Download for free at https://md-shouvik-iqbal.github.io/calculus
Chapter 1 The Concept of Continuity Page 103
let two points a and b on the x-axis represent the start and end of the walk,
respectively. If f (a) = 0 and f (b) = 100, then there must be a point c between
a and b such that the function’s value f (c) = 50. This is guaranteed by the
continuity of the function.
Following the same logic as in the analogy above, we can state the following
theorem.
The Intermediate Value Theorem (IVT) can be used in many clever ways to
ensure the existence of solutions within a certain interval. A few of them are
shown below.
I Example 1.6.3. Prove that the function f (x) = x3 − 10 has a solution on the
interval [2, 2.2].
Copyright © 2024 Md. Shouvik Iqbal. This book is licensed under a Creative Commons Attribution-NonCommercial-ShareAlike 4.0 International License (CC BY-NC-SA 4.0)
Download for free at https://md-shouvik-iqbal.github.io/calculus
Page 104 (ε − δ) Definition of Limit Chapter 1
f (2) = 23 − 10 = −2
f (2.2) = 2.23 − 10 = 0.648
Since f (2) = −2 and f (2.2) = 0.648, therefore, by the Intermediate Value The-
orem, there must exist a point c ∈ (2, 2.2) such that f (c) = 0, because f (x) is
continuous and crosses the x-axis between x = 2 and x = 2.2.
To understand why, suppose you are the mathematician who devised the concept
of limits. Intuitively, what you’ve written in the following is pretty clear.
lim f (x) = L
x→a
Now, the question arises: how do you define what it means to be “close,” as
you have used this term in your definition of limits? Specifically, what does it
mathematically mean for some variable x to be “close” to a number a? After
pondering, you may figure out that the distance between x and a is simply x−a.
Since distance cannot be negative, therefore you use the absolute value of x − a,
Copyright © 2024 Md. Shouvik Iqbal. This book is licensed under a Creative Commons Attribution-NonCommercial-ShareAlike 4.0 International License (CC BY-NC-SA 4.0)
Download for free at https://md-shouvik-iqbal.github.io/calculus
Chapter 1 (ε − δ) Definition of Limit Page 105
that is, |x−a|. So, “closeness” is, in this case, if the distance |x−a| is kept small.
So, this implies that the function f (x) is close to a value L if the distance be-
tween f (x) and L, that is, |f (x) − L| is kept small by keeping the distance
between x and a sufficiently small.
The issue, however, is not resolved. Because, you may have clearly defined what
“closeness” implies in this case, but it was done so by leveraging the vagueness
of the term “smallness.” That is, “smallness” is a relative term, so “how small
is really small?” is a question to be answered now.
To address the issue, you can assign a fixed real number to indicate the exact
amount of “smallness.” Say, ε ∈ R is such a real number. So, |f (x) − L| < ε
implies that the distance between f (x) and L is less than the chosen “small”
real number ε, no matter how small. In other words, f (x) is within ε-distance
of L.
You can similarly assign a fixed value to clearly define the “smallness” for the
distance between x and a. Let δ ∈ R be such a real number. So, |x − a| < δ
implies that the distance between x and a is less than the chosen “small” real
number δ, no matter how small.
So, after precisely defining the ambiguous words, you can now ensure that if
|x − a| < δ, then |f (x) − L| < ε, where ε is the chosen value. However, you soon
realize that since x 6= a for the reason that the function may not even be defined
at x = a, therefore, the distance between x and a can never be zero, that is,
|x − a| > 0. So, this implies that
In other words, to keep the distance between f (x) and L less than ε, i.e.,
|f (x) − L| < ε; you would have to keep the distance between x and a less
than δ and greater than 0, i.e., 0 < |x − a| < δ. Notice that the magnitude of
δ depends on the function f (x) itself. For very steep functions, you would need
a much smaller δ to keep f (x) within a given ε. So, this implies that for every
chosen real number ε > 0 (no matter how small), there exists a corresponding
δ > 0, but δ will vary based on the behavior of the function.
Copyright © 2024 Md. Shouvik Iqbal. This book is licensed under a Creative Commons Attribution-NonCommercial-ShareAlike 4.0 International License (CC BY-NC-SA 4.0)
Download for free at https://md-shouvik-iqbal.github.io/calculus
Page 106 (ε − δ) Definition of Limit Chapter 1
lim f (x) = L
x→a
for every real number ε > 0, there exist a corresponding real number δ > 0
such that for all real number x, if 0 < |x − a| < δ, then |f (x) − L| < ε.
All the details of the (ε − δ) definition might seem overwhelming at once. There-
fore, to better illustrate this concept, we will consider a visual approach. For
that, consider the following figure.
x
a
Figure 1.23
From the figure above, it is evident that f (x) → L as x → a from both sides.
Thus, the limit lim f (x) = L may informally be defined as “the limit of f (x),
x→a
as x approaches the value a, is the value L.”
This definition, however, is vague. For example, what is meant by “f (x) ap-
proaches the value L” or “x approaches the value a”? If this implies that the
value of f (x) gets closer and closer to L whenever x gets closer and closer to a,
then how close is really “close”? How close is close enough to conclude that a
limit is indeed approaching a certain value?
We deal with the idea of closeness by introducing the idea of error tolerance.
That is, if we assign a value of error tolerance below which the distance be-
tween the function f (x) and the value L must be kept, then we can keep x close
Copyright © 2024 Md. Shouvik Iqbal. This book is licensed under a Creative Commons Attribution-NonCommercial-ShareAlike 4.0 International License (CC BY-NC-SA 4.0)
Download for free at https://md-shouvik-iqbal.github.io/calculus
Chapter 1 (ε − δ) Definition of Limit Page 107
enough to a in order for the distance between f (x) and L to be less than that
given error tolerance. The key idea is that the distance between f (x) and L can
be minimized to be smaller than any specified level of error tolerance by letting
x sufficiently close to a.
We let ε be a small positive real number as a value of error tolerance. Since the
error tolerance ε is the value less than which the distance between f (x) and L
must be, therefore the symbolic form of this is |f (x) − L| < ε. The absolute
value is used to show that distance cannot be a negative quantity. Now, we
derive the range within which f (x) must lie as follows,
|f (x) − L| < ε
−ε < f (x) − L < ε
−ε + L < f (x) − L + L < ε + L
L − ε < f (x) < L + ε
This implies that f (x) must lie inside an open interval (L − ε, L + ε), as shown
in the Figure 1.24.
Now, we let δ be the value less than which the distance between x and a must
be kept in order for |f (x) − L| to be less than the given error tolerance ε. The
symbolic form of this is |x − a| < δ. Based on that, we derive the range for x as
follows,
|x − a| < δ
−δ < x − a < δ
−δ + a < x − a + a < δ + a
a−δ <x<a+δ
This implies that in order for the distance between f (x) and L be less than a
given error tolerance ε, x must be kept within an open interval (a − δ, a + δ),
where, x 6= a, as shown in the following figure.
Copyright © 2024 Md. Shouvik Iqbal. This book is licensed under a Creative Commons Attribution-NonCommercial-ShareAlike 4.0 International License (CC BY-NC-SA 4.0)
Download for free at https://md-shouvik-iqbal.github.io/calculus
Page 108 (ε − δ) Definition of Limit Chapter 1
f(x)
L
x
a
x
Figure 1.24
is that for all every chosen ε > 0, there exists a corresponding δ > 0 such
that
lim (2x − 5) = 1
x→3
Copyright © 2024 Md. Shouvik Iqbal. This book is licensed under a Creative Commons Attribution-NonCommercial-ShareAlike 4.0 International License (CC BY-NC-SA 4.0)
Download for free at https://md-shouvik-iqbal.github.io/calculus
Chapter 1 (ε − δ) Definition of Limit Page 109
Now, what is the value less than which the distance between x and 3 must be
kept in order for |f (x) − 1| to be less than ε = 0.0001?
Solution. The value less than which the distance between x and 3 must be kept
in order for |f (x) − 1| to be less than 0.0001, is the value δ. That is, |x − 3| < δ.
We know that, for each ε > 0, there is a δ > 0 such that if 0 < |x − a| < δ, then
|f (x) − L| < ε.
Solution. In example 1.7.1, it was found that δ = 0.00005. This implies that
|x − 3| < 0.00005. Based on that, we derive,
|f (x) − L| = |(2x − 5) − 1|
= |2x − 6|
= 2 |x − 3|
< 2 (0.00005) = 0.0001 = ε
This matches the error tolerance ε given in Example 1.7.1. This completes the
verification.
Copyright © 2024 Md. Shouvik Iqbal. This book is licensed under a Creative Commons Attribution-NonCommercial-ShareAlike 4.0 International License (CC BY-NC-SA 4.0)
Download for free at https://md-shouvik-iqbal.github.io/calculus
Page 110 (ε − δ) Definition of Limit Chapter 1
Solution. We can prove that lim (3x + 1) = 7, by showing that for any chosen
x→2
ε > 0, there exists a δ > 0 such that if 0 < |x − 2| < δ, then |(3x + 1) − 7| < ε.
Solution. We can prove that lim x2 = 1, by showing that for any chosen ε > 0,
x→1
there exists a δ > 0 such that if 0 < |x − 1| < δ, then |x2 − 1| < ε.
|x − a| < δ = |x − 1| < δ
= −δ < x − 1 < δ
=1−δ <x<1+δ
or,
√
1+δ = 1+ε
Copyright © 2024 Md. Shouvik Iqbal. This book is licensed under a Creative Commons Attribution-NonCommercial-ShareAlike 4.0 International License (CC BY-NC-SA 4.0)
Download for free at https://md-shouvik-iqbal.github.io/calculus
Chapter 1 (ε − δ) Definition of Limit Page 111
√
δ= 1+ε−1
√ √
Thus, we choose the smallest distance δ = min{1 − 1 − ε, 1 + ε − 1}.
√ √
This implies that if 0 < |x − 1| < min{1 − 1 − ε, 1 + ε − 1}, then |x2 − 1| <
ε.
√
I Example 1.7.5. Prove that lim x=2
x→4
√
Solution. We can prove that lim x = 2, by showing that for any chosen ε > 0,
√ x→4
there exists a δ > 0 such that if 0 < |x − 4| < δ, then | x − 2| < ε.
√
Since f (x) = x and L = 2, therefore,
√
|f (x) − L| < ε = x − 2 < ε
√
= −ε < x − 2 < ε
√
=2−ε< x<2+ε
= (2 − ε)2 < x < (2 + ε)2
|x − a| < δ = |x − 4| < δ
= −δ < x − 4 < δ
=4−δ <x<4+δ
4 − δ = (2 − ε)2
δ = 4 − (2 − ε)2
or,
4 + δ = (2 + ε)2
δ = (2 + ε)2 − 4
√
This implies that if 0 < |x − 4| < min{4 − (2 − ε)2 , (2 + ε)2 − 4}, then | x − 2| <
ε.
Copyright © 2024 Md. Shouvik Iqbal. This book is licensed under a Creative Commons Attribution-NonCommercial-ShareAlike 4.0 International License (CC BY-NC-SA 4.0)
Download for free at https://md-shouvik-iqbal.github.io/calculus
Page 112 (ε − δ) Definition of Limit Chapter 1
Copyright © 2024 Md. Shouvik Iqbal. This book is licensed under a Creative Commons Attribution-NonCommercial-ShareAlike 4.0 International License (CC BY-NC-SA 4.0)
Download for free at https://md-shouvik-iqbal.github.io/calculus
Part II
Differential Calculus
113
Copyright © 2024 Md. Shouvik Iqbal. This book is licensed under a Creative Commons Attribution-NonCommercial-ShareAlike 4.0 International License (CC BY-NC-SA 4.0)
Download for free at https://md-shouvik-iqbal.github.io/calculus
Copyright © 2024 Md. Shouvik Iqbal. This book is licensed under a Creative Commons Attribution-NonCommercial-ShareAlike 4.0 International License (CC BY-NC-SA 4.0)
Download for free at https://md-shouvik-iqbal.github.io/calculus
Chapter 2
An Introduction to Differential
Calculus
Differential calculus is a subfield of calculus that studies the rates at which quan-
tities change at any given instant. In modern terms, it primarily addresses the
question: how does a function react to a change in its input variable? For exam-
ple, consider the function f (x). We know that when the independent variable
x changes, the dependent variable f (x) also changes correspondingly, but the
question that differential calculus essentially addresses is more about the “how”
than the “how much.” That is, it explores the nature of this change—to what
sensitivity does f (x) react to a change in x? What’s the “rate1 ” at which f (x)
changes reacting to a change in x, no matter how small?
Finding the rate of change of f (x) may seem straightforward: measure the
amount x changes and compare it with the corresponding change in f (x). For
example, if x changed from the point x = a to x = a + h, where h > 0 is
an increment, then the change in x is the difference (a + h) − a = h and the
corresponding change in f (x) for an increment h is the difference between f (a+h)
and f (a), that is, f (a + h) − f (a). Now, by comparing these two changes, we
get,
f (a + h) − f (a)
(1)
h
1
A rate is a specific type of ratio that compares two quantities of different units. It describes
how one quantity changes relative to another.
115
Copyright © 2024 Md. Shouvik Iqbal. This book is licensed under a Creative Commons Attribution-NonCommercial-ShareAlike 4.0 International License (CC BY-NC-SA 4.0)
Download for free at https://md-shouvik-iqbal.github.io/calculus
Page 116 Chapter 2
This quotient above represents the “rate” at which f (x) changes as x changes
from x = a to x = a+h, and is called the difference quotient . So, for any change
f (x + h) − f (x)
h in x in general, f (x) changes at a rate of . For example, if
h
f (x) = x, then as x changes from x = a to x = a + h, the change in f (x) is,
f (a + h) − f (a) = (a + h) − a
=h
f (a + h) − f (a) h
=
h h
=1
This is because the function is f (x) = x, therefore the rate of change of f (x)
matches with the rate of change of x, resulting in a ratio of 1. It is also noticeable
when the function is graphed, as shown below.
Figure 2.1
It is worth mentioning here is that, when graphed, the rate of change given by
the difference quotient is essentially the same as the slope of the straight line.
This is because we know that
vertical change
slope, m =
horizontal change
f (a + h) − f (a)
=
(a + h) − a
Copyright © 2024 Md. Shouvik Iqbal. This book is licensed under a Creative Commons Attribution-NonCommercial-ShareAlike 4.0 International License (CC BY-NC-SA 4.0)
Download for free at https://md-shouvik-iqbal.github.io/calculus
Chapter 2 The Tangent Line Problem Page 117
f (a + h) − f (a)
=
h
Therefore, in general, the slope of a line on a graph represents the rate of change
between two variables being graphed. In other words, it represents the rate at
which the dependent variable changes reacting to a change in the independent
variable.
With that in mind, let us now pose a “philosophical question” with the purpose
to understand the tool through which differential calculus examines the rate of
change of a function—namely, the derivative.
Philosophical Question: At what rate does f (x) change when there’s no change
in x whatsoever?
This question is absurd! Why wouldn’t it be? The dependent variable f (x) only
changes when the independent variable x changes. If the change in x is 0, then
0
the change in f (x) is also 0, and thus the rate at which f (x) changes is... ?
0
2 0
Can there be such a real thing as ?
0
This question can best be answered through problems that have perplexed
solvers for centuries. One such problem is the tangent line problem. Under-
standing which can allow us to define the aforementioned heart of differential
calculus: the derivative.
Copyright © 2024 Md. Shouvik Iqbal. This book is licensed under a Creative Commons Attribution-NonCommercial-ShareAlike 4.0 International License (CC BY-NC-SA 4.0)
Download for free at https://md-shouvik-iqbal.github.io/calculus
Page 118 The Tangent Line Problem Chapter 2
Figure 2.2
First of all, what does it mean for a line to be “tangent” to the graph of a function
at a point? The term “tangent” originated from the Latin word “tangere,”
meaning “to touch”; so the term “tangent” is indicative of touching something.
However, defining this precisely isn’t as straightforward as it might appear. The
definition of a tangent line is simple for common curves such as a circle. For
example, a line is tangent to a circle if it meets the circle at exactly one and
only point, known as the point of tangency, as shown below.
Figure 2.3
This intuitive definition might suggest that a line is tangent to a curve at a point
if it “meets” the curve at that point. However, this definition only applies to
simple curves like the one shown above and falls short for more complex curves.
To understand why, consider the following figure.
Copyright © 2024 Md. Shouvik Iqbal. This book is licensed under a Creative Commons Attribution-NonCommercial-ShareAlike 4.0 International License (CC BY-NC-SA 4.0)
Download for free at https://md-shouvik-iqbal.github.io/calculus
Chapter 2 The Tangent Line Problem Page 119
Figure 2.4
Here, the line “meets” the curve at exactly one point, but it goes without saying
that it does not align with our previous understanding of a tangent line. There-
fore, a revised definition would be that a line is tangent to a curve at a point if
it “just touches” the curve at that point. This revised definition emphasizes the
idea of a line gradually nearing the curve until it makes contact.
Sadly, however, this revised definition is also not sufficient for the reason depicted
below.
Figure 2.5
Here, the tangent line “just touches” the curve at only one point P , but it also
crosses by intersecting the curve at that point. That is, a part of the curve lies
above the tangent line while another part falls beneath it. Thus, it violates the
current definition of a tangent line. Therefore, a revised definition would be that
Copyright © 2024 Md. Shouvik Iqbal. This book is licensed under a Creative Commons Attribution-NonCommercial-ShareAlike 4.0 International License (CC BY-NC-SA 4.0)
Download for free at https://md-shouvik-iqbal.github.io/calculus
Page 120 The Tangent Line Problem Chapter 2
Figure 2.6
Here, the tangent line not only intersects the curve at point P , but also at an-
other point ahead of P . This is again a violation of the current revised definition
as it requires the point of intersection to be exactly one. Therefore, the defini-
tion can again be revised such that a line is tangent to a curve at a point if it,
“up close,” just touches or intersects the curve at that point. But unfortunately,
even this definition eventually falls short.
That is, in order to define a line, at least two points are required. But if two
points are used, then the line can no longer be defined as a tangent line—it
becomes a secant line! A secant line to a curve is one that intersects the curve
at two different points. It “cuts” the curve at least at two distinct points, as the
term “secant” itself originated from the Latin word “secare,” meaning “to cut.”
It is at this point we may come to realize why the absurd philosophical question
was posed earlier.
Copyright © 2024 Md. Shouvik Iqbal. This book is licensed under a Creative Commons Attribution-NonCommercial-ShareAlike 4.0 International License (CC BY-NC-SA 4.0)
Download for free at https://md-shouvik-iqbal.github.io/calculus
Chapter 2 The Tangent Line Problem Page 121
To understand why, first recall that in Analytic Geometry a line can be defined
using a single point if its slope is known. For example, recall the point-slope
form of a line for a given point (x1 , y1 ) and a slope m,
y − y1 = m(x − x1 ) (1)
Now, if the slope m of the tangent line can somehow be determined, it then
becomes possible to define the tangent line using only a single point by the
equation (1). However, the core problem remains as the slope or the rate of
0
change of two quantities at a single point becomes , as previously explained.
0
Historically, efforts were made to calculate the slope of the tangent line to the
curve at a single point by introducing a new quantity known as the “infinitesi-
mal.” To oversimplify, it was a quantity that was infinitely small but not exactly
equal to 0. More specifically, it is a positive real number less than every positive
real number. For example, consider the figure below,
Figure 2.7
In order to calculate the slope of the tangent line to the curve of f (x) at the
point P = (a, f (a)), a small increment dx is made to x = a, resulting in a point
Q = (a + dx, f (a + dx)) and then to draw a secant line P Q. Now, let dx be
an infinitesimal increment. Since dx is an infinitesimal increment and infinitely
small, therefore the point Q is infinitely closer to the point P in a way that
they merge into one single point, becoming indistinguishable from each other,
and thus, the slope of the secant line P Q is infinitely closer to the slope of the
Copyright © 2024 Md. Shouvik Iqbal. This book is licensed under a Creative Commons Attribution-NonCommercial-ShareAlike 4.0 International License (CC BY-NC-SA 4.0)
Download for free at https://md-shouvik-iqbal.github.io/calculus
Page 122 The Tangent Line Problem Chapter 2
tangent line to the curve at the point P in a way that it is indistinguishably the
same.
In modern calculus, we can simply calculate the slope of the tangent line by a
limiting process where the point Q = (a + dx, f (a + dx)) approaches the point
P = (a, f (a)) along the curve, i.e., the difference between Q = (a+dx, f (a+dx))
and P = (a, f (a)) approaches 0 as a function as dx → 0, given that dx 6= 0.
More specifically, consider calculating the slope of the tangent line to the curve
at point P , as shown in the figure below.
Copyright © 2024 Md. Shouvik Iqbal. This book is licensed under a Creative Commons Attribution-NonCommercial-ShareAlike 4.0 International License (CC BY-NC-SA 4.0)
Download for free at https://md-shouvik-iqbal.github.io/calculus
Chapter 2 The Tangent Line Problem Page 123
Figure 2.8
The slope can be determined by a limiting process using the slope of the secant
line. To do this, let Q = (a + h, f (a + h)) be a second point on the curve that
is close to P = (a, f (a)), where h > 0. Now, we draw the secant line P Q that
intersects the points P and Q, as follows.
Figure 2.9
Copyright © 2024 Md. Shouvik Iqbal. This book is licensed under a Creative Commons Attribution-NonCommercial-ShareAlike 4.0 International License (CC BY-NC-SA 4.0)
Download for free at https://md-shouvik-iqbal.github.io/calculus
Page 124 The Tangent Line Problem Chapter 2
Figure 2.10
Definition 2.1.1. (The Slope of a Tangent Line and The Instantaneous Rate
of Change)
The slope of the curve y = f (x) at a single point a equals the slope of the
tangent line or the instantaneous rate of change of the curve at that point.
Which is defined as,
f (a + h) − f (a)
mtan = lim
h→0 h
provided that the limit exists, where h > 0 is a real number.
Copyright © 2024 Md. Shouvik Iqbal. This book is licensed under a Creative Commons Attribution-NonCommercial-ShareAlike 4.0 International License (CC BY-NC-SA 4.0)
Download for free at https://md-shouvik-iqbal.github.io/calculus
Chapter 2 The Tangent Line Problem Page 125
Figure 2.11
Definition 2.1.2. (The Slope of a Tangent Line and The Instantaneous Rate
of Change)
The slope of the curve y = f (x) at a single point a equals the slope of the
tangent line or the instantaneous rate of change of the curve at that point.
Which is defined as,
f (a) − f (x)
mtan = lim
x→a x−a
provided that the limit exists, where x 6= a.
Since we can now determine the slope of the tangent line mtan at the point
P = (a, f (a)) on the curve, we can define the tangent line using the point-slope
form of a line as given by the equation 1 above. That is,
y − y1 = m(x − x1 )
y − f (a) = mtan (x − a)
= mtan (x − a) + f (a)
Copyright © 2024 Md. Shouvik Iqbal. This book is licensed under a Creative Commons Attribution-NonCommercial-ShareAlike 4.0 International License (CC BY-NC-SA 4.0)
Download for free at https://md-shouvik-iqbal.github.io/calculus
Page 126 The Tangent Line Problem Chapter 2
f (a + h) − f (a)
= lim (x − a) + f (a)
h→0 h
Summarizing our discussion, we finally have the long-awaited definition of a line
that is tangent to a curve at a single point as follows.
Definition 2.1.3. (Tangent Line) A line that is tangent to the graph of f (x)
at the point P = (a, f (a)) is given by,
y = mtan (x − a) + f (a)
where,
f (a + h) − f (a)
mtan = lim
h→0 h
provided that the limit exists.
This is the generalized equation of a line that is tangent to the graph of f (x)
at the point a. Using this definition, the tangent line can be determined to the
graph of any function at any point. For demonstration, consider the following
example.
I Example 2.1.1. What is the tangent line to the graph of f (x) = x2 at the point
(2, 4).
Solution. Since we’ve to find the tangent line at the point (2, 4), therefore we
first have to determine its slope at that point. From Definition 2.1.1, we know
that the slope of the tangent line is given by,
f (a + h) − f (a)
mtan = lim
h→0 h
Therefore, the slope of the tangent line at the point (2, 4) yields,
f (2 + h) − f (2)
mtan = lim
h→0 h
2
(2 + h) − 22
= lim
h→0 h
2 + 2 · 2 · h + h2 − 22
2
= lim
h→0 h
2
4h + h
= lim
h→0 h
4h h2
= lim +
h→0 h h
Copyright © 2024 Md. Shouvik Iqbal. This book is licensed under a Creative Commons Attribution-NonCommercial-ShareAlike 4.0 International License (CC BY-NC-SA 4.0)
Download for free at https://md-shouvik-iqbal.github.io/calculus
Chapter 2 Definition of the Derivative of a function Page 127
= lim 4 + h
h→0
=4
Since mtan = 4, therefore the tangent line from Definition 2.1.3 yields,
y = mtan (x − a) + f (a)
= 4(x − 2) + 4
= 4x − 8 + 4
= 4x − 4
Having discussed the tangent line problem and its resolution, we now move to
discuss the derivative of a function in the following section and see how it is
connected to the tangent line problem.
The derived function or the derivative can be expressed symbolically. For ex-
ample, if f (x) represents a function, then we let f 0 (x) represent its derivative.
Although many other notations exist and we will discuss them later on, but for
now, we let f 0 (x) (read as f prime of x) denote the derivative of a function f (x).
Copyright © 2024 Md. Shouvik Iqbal. This book is licensed under a Creative Commons Attribution-NonCommercial-ShareAlike 4.0 International License (CC BY-NC-SA 4.0)
Download for free at https://md-shouvik-iqbal.github.io/calculus
Page 128 Definition of the Derivative of a function Chapter 2
f (a + h) − f (a)
Since lim represents the slope of the tangent line at a single
h→0 h
point a, therefore,
f (x + h) − f (x)
lim
h→0 h
represents the slope of the tangent line at a varying point x, which is precisely
said to be the derivative and thus denoted by f 0 (x). Notice how the limit
f (x + h) − f (x)
f 0 (x) = lim can only be evaluated to find the slope of the tan-
h→0 h
gent line when x is specified, and thus the derivative f 0 (x) is indeed a function
of x.
Summarizing the facts discussed, we have the following definition of the deriva-
tive of a function.
f (x + h) − f (x)
f 0 (x) = lim
h→0 h
provided that the limit exists.
The derivative of a function can be thought of as a tool that helps us find the
slope of the tangent line, i.e., the instantaneous rate of change of a function f (x)
at any given point. So, the derivative of a function may informally be said as
follows.
Think of it in this way: a curve does not maintain a constant slope at every
point—it changes as you move along it. To determine the slope of the curve at
a specific point, a tangent line is drawn to the curve at that point through a
limiting process. The slope of this tangent line represents the curve’s slope at
that exact location. By calculating the slope at various points along the curve,
using a variable x that moves across all points on the curve, we generate a new
function called the “derivative.” This derivative describes how the slope varies
along the curve, representing the curve’s instantaneous rate of change at each
point.
Copyright © 2024 Md. Shouvik Iqbal. This book is licensed under a Creative Commons Attribution-NonCommercial-ShareAlike 4.0 International License (CC BY-NC-SA 4.0)
Download for free at https://md-shouvik-iqbal.github.io/calculus
Chapter 2 Definition of the Derivative of a function Page 129
f (x) − f (a)
f 0 (x) = lim
x→a x−a
provided that the limit exists.
for x =⇒ f (x) = x2
for x+h =⇒ f (x + h) = (x + h)2
= x2 + 2xh + h2
(x + h) − x =⇒ h
f (x + h) − f (x) 2xh + h2
=
h h
2xh h2
= +
h h
= 2x + h
f (x + h) − f (x)
Since the derivative is defined as lim , therefore
h→0 h
f (x + h) − f (x)
lim = lim (2x + h)
h→0 h h→0
= 2x + lim h
h→0
= 2x
Copyright © 2024 Md. Shouvik Iqbal. This book is licensed under a Creative Commons Attribution-NonCommercial-ShareAlike 4.0 International License (CC BY-NC-SA 4.0)
Download for free at https://md-shouvik-iqbal.github.io/calculus
Page 130 Definition of the Derivative of a function Chapter 2
This implies that the derivative of f (x) = x2 is f 0 (x) = 2x—but what does it
really mean? The function f (x) = x2 defines a specific relationship between
the input x and the output f (x), resulting in a parabola visually. However, its
derivative, f 0 (x) = 2x, explains how this relationship changes at every point
along the curve. Specifically, it shows how the slope (i.e., the rate of change)
of the parabola varies at different values of x. Basically, while f (x) = x2 repre-
sents a parabolic curve that changes its slope continuously at different points,
its derivative, f 0 (x) = 2x, tells us the exact rate of this change at those points.
For example, at x = 1, the slope of f (x) = x2 is f 0 (1) = 2 · 1 = 2. At x = 2,
the slope of f (x) = x2 is f 0 (2) = 2 · 2 = 4. At x = 3, the slope of f (x) = x2 is
f 0 (2) = 2 · 3 = 6.
Copyright © 2024 Md. Shouvik Iqbal. This book is licensed under a Creative Commons Attribution-NonCommercial-ShareAlike 4.0 International License (CC BY-NC-SA 4.0)
Download for free at https://md-shouvik-iqbal.github.io/calculus
Chapter 2 Definition of the Derivative of a function Page 131
f(x)
x
1 2 3 4
f’(x)
x
1 2 3 4
Figure 2.12
The figures above illustrate that the graph of f (x) = x2 becomes progressively
steeper as x increases. This suggests that the slope, or rate of change, also grows
with every increase of x. As a result, the graph of the derivative f 0 (x) = 2x of
Copyright © 2024 Md. Shouvik Iqbal. This book is licensed under a Creative Commons Attribution-NonCommercial-ShareAlike 4.0 International License (CC BY-NC-SA 4.0)
Download for free at https://md-shouvik-iqbal.github.io/calculus
Page 132 Definition of the Derivative of a function Chapter 2
=5
√
I Example 2.2.2. If f (x) = x, then derive the derivative of f (x).
1
= lim √ √
h→0 x+h+ x
1
=√ √
x+0+ x
1
= √
2 x
√ 1
Therefore, the derivative of f (x) = x is f 0 (x) = √ .
2 x
Copyright © 2024 Md. Shouvik Iqbal. This book is licensed under a Creative Commons Attribution-NonCommercial-ShareAlike 4.0 International License (CC BY-NC-SA 4.0)
Download for free at https://md-shouvik-iqbal.github.io/calculus
Chapter 2 Definition of the Derivative of a function Page 133
f (x + h) − f (x) (x + h)3 − x3
lim = lim
h→0 h h→0 h
x + 3x2 h + 3xh2 + h3 − x3
3
= lim
h→0 h
3x + 3xh + h2
h 2
= lim
h→0 h
= lim 3x + 3xh + h2
2
h→0
1
I Example 2.2.4. If f (x) = , then derive the derivative of f (x).
x2
Solution. By the definition of derivative, we get,
1 1
2 − x2
f (x + h) − f (x) (x + h)
lim = lim
h→0 h h→0
h
1 1 1
= lim · −
h→0 h (x + h)2 x2
!
1 x2 − (x + h)2
= lim ·
h→0 h x2 (x + h)2
!
1 x2 − x2 − 2xh − h2
= lim ·
h→0 h x2 (x + h)2
−2xh − h2
1
= lim ·
h→0 h x2 (x + h)2
1 h (−2x − h)
= lim ·
h x2 (x + h)2
h→0
−2x − h
= lim
h→0 x2 (x + h)2
Copyright © 2024 Md. Shouvik Iqbal. This book is licensed under a Creative Commons Attribution-NonCommercial-ShareAlike 4.0 International License (CC BY-NC-SA 4.0)
Download for free at https://md-shouvik-iqbal.github.io/calculus
Page 134 Notation for Derivatives Chapter 2
−2x − 0
=
x2 (x + 0)2
−2x
= 2 2
x ·x
2
=− 3
x
1 2
Therefore, the derivative of f (x) = 2
is f 0 (x) = − 3 .
x x
Copyright © 2024 Md. Shouvik Iqbal. This book is licensed under a Creative Commons Attribution-NonCommercial-ShareAlike 4.0 International License (CC BY-NC-SA 4.0)
Download for free at https://md-shouvik-iqbal.github.io/calculus
Chapter 2 Notation for Derivatives Page 135
Figure 2.13
The slope of the secant line that passes through the points P and Q (the yellow
colored line) is given by the difference quotient,
f (x + ∆x) − f (x)
∆x
Since the change in y is often denoted as ∆y = f (x + ∆x) − f (x), therefore, the
slope of the secant line yields,
f (x + ∆x) − f (x) ∆y
=
∆x ∆x
Since the slope of the tangent line at a movable point x (i.e., the derivative) is
f (x + ∆x) − f (x)
defined as lim , therefore
∆x→0 ∆x
f (x + ∆x) − f (x)
derivative, f 0 (x) = lim
∆x→0 ∆x
∆y
= lim
∆x→0 ∆x
dy
=
dx
dy ∆y
Here, is simply a single notation to represent the limit lim is NOT a
dx ∆→0 ∆x
dy
ratio of dy over dx. The notation is read as “the derivative of y with respect
dx
to x,” or simply, “dee y dee x.” Note that it is never read as “dee y divided by
dy
dee x” or “the ratio of dee y and dee x.” The notation is not yet a quotient
dx
Copyright © 2024 Md. Shouvik Iqbal. This book is licensed under a Creative Commons Attribution-NonCommercial-ShareAlike 4.0 International License (CC BY-NC-SA 4.0)
Download for free at https://md-shouvik-iqbal.github.io/calculus
Page 136 Differentiability and Continuity Chapter 2
of two quantities dy and dx because the meaning of dy and dx have not yet
dy ∆y
been defined other than the fact that represents the limit lim . We will
dx ∆x→0 ∆x
later consider giving dy and dx a meaning to set up a real quotient between them.
dy d
f 0 (a) f (x)
dx x=a dx x=a
Considering all the different notations for derivatives at once may be overwhelm-
ing. However, it’s important to note that in most cases, the Leibniz notation
with its variants and the f 0 (x) notation introduced earlier are the ones most
commonly used. The notation f 0 (x) is named after the Italian mathematician
Joseph-Louis Lagrange.
Copyright © 2024 Md. Shouvik Iqbal. This book is licensed under a Creative Commons Attribution-NonCommercial-ShareAlike 4.0 International License (CC BY-NC-SA 4.0)
Download for free at https://md-shouvik-iqbal.github.io/calculus
Chapter 2 Differentiability and Continuity Page 137
Not every continuous function has a derivative at every point. When a function
f (x) has a derivative f 0 (x) at any given point x, the function is said to be dif-
ferentiable at that point. Similarly, when f (x) is differentiable at every point
over an open interval I 4 , it is said to be differentiable over that interval I.
f (a + h) − f (a)
f 0 (a) = lim
h→0 h
This limit, if it exists, is called the derivative of f (x) at the point a.
A crucial result in the derivative of a function f (x) lies in the fact that the
differentiability of f (x) implies its continuity. That is, if a function f (x) is
differentiable at the point x = a, i.e., if it has a derivative at the point x = a,
then that function is necessarily continuous at that point x = a.
f (x) = f (x)
0
z }| {
= f (x) − f (a) + f (a)
x−a
= f (x) − f (a) · + f (a)
x−a
4
finite or infinite
Copyright © 2024 Md. Shouvik Iqbal. This book is licensed under a Creative Commons Attribution-NonCommercial-ShareAlike 4.0 International License (CC BY-NC-SA 4.0)
Download for free at https://md-shouvik-iqbal.github.io/calculus
Page 138 Differentiability and Continuity Chapter 2
f (x) − f (a)
∴ f (x) = · (x − a) + f (a)
x−a
Now, we take the limit as x → a on both sides of the equation above and simplify
as follows.
f (x) − f (a)
lim f (x) = lim · (x − a) + f (a)
x→a x→a x−a
f (x) − f (a)
= lim · lim (x − a) + lim f (a)
x→a x−a x→a
| {z } x→a | {z }
0 f (a)
f (x) − f (a)
= lim +f (a)
x→a
| x−a }
{z
0
= f (a)
∴ lim f (x) = f (a)
x→a
This implies that f (x) is also continuous at the point x = a. This completes the
proof.
The theorem above states that if f (x) is differentiable at a point x = a, then it
necessarily is continuous at that point x = a. This logically follows that if f (x)
is not continuous at x = a, then it is necessarily not differentiable at x = a and
the following theorem states just that.
Notice carefully that if f (x) is continuous at a given point, then it may or may
not be differentiable at that point. Conversely, if f (x) is differentiable at a given
point, then it is necessarily continuous at that point.
Remark 2.4.1.
Copyright © 2024 Md. Shouvik Iqbal. This book is licensed under a Creative Commons Attribution-NonCommercial-ShareAlike 4.0 International License (CC BY-NC-SA 4.0)
Download for free at https://md-shouvik-iqbal.github.io/calculus
Chapter 2 Higher-Order Derivatives Page 139
Copyright © 2024 Md. Shouvik Iqbal. This book is licensed under a Creative Commons Attribution-NonCommercial-ShareAlike 4.0 International License (CC BY-NC-SA 4.0)
Download for free at https://md-shouvik-iqbal.github.io/calculus
Page 140 Higher-Order Derivatives Chapter 2
d2 y
The second-order Leibniz notation can be viewed in terms of the first no-
dx2
tation, that is,
d2 y d dy
=
dx2 dx dx
to mean that the operation of differentiation is applied to the first derivative of
the function y = f (x).
dy d2 y
I Example 2.5.1. If f (x) = x3 and = 3x2 , then what is 2 ?
dx dx
dy
Solution. The derivative of = 3x2 and the second-order derivative of f (x) =
dx
x3 is,
= 6x + lim 3h
h→0
= 6x
d2 y
This implies that if f (x) = x , then 2 = 6x. 3
dx
dy d2 y d3 y
I Example 2.5.2. If f (x) = x3 , = 3x2 , and 2 = 6x, then what is 3 ?
dx dx dx
d2 y
Solution. The derivative of 2 = 6x and the third-order derivative of f (x) = x3
dx
is,
d3 y 6(x + h) − 6x
= lim
dx3 h→0 h
6x + 6h −6x
= lim
h→0 h
Copyright © 2024 Md. Shouvik Iqbal. This book is licensed under a Creative Commons Attribution-NonCommercial-ShareAlike 4.0 International License (CC BY-NC-SA 4.0)
Download for free at https://md-shouvik-iqbal.github.io/calculus
Chapter 2 Higher-Order Derivatives Page 141
6h
= lim
h→0 h
= lim 6
h→0
=6
d3 y
This implies that if f (x) = x3 , then = 6.
dx3
2
dy 2 d y d3 y
I Example 2.5.3. If f (x) = x , = 3x , 2 = 6x, and 3 = 6, then what is
3
dx dx dx
4
d y
?
dx4
d3 y
Solution. The derivative of 3 = 6 and the fourth-order derivative of f (x) = x3
dx
is,
d4 y 6−6
4
= lim
dx h→0 h
0
= lim
h→0 h
= lim 0
h→0
=0
d4 y
This implies that if f (x) = x3 , then = 0.
dx4
Copyright © 2024 Md. Shouvik Iqbal. This book is licensed under a Creative Commons Attribution-NonCommercial-ShareAlike 4.0 International License (CC BY-NC-SA 4.0)
Download for free at https://md-shouvik-iqbal.github.io/calculus
Copyright © 2024 Md. Shouvik Iqbal. This book is licensed under a Creative Commons Attribution-NonCommercial-ShareAlike 4.0 International License (CC BY-NC-SA 4.0)
Download for free at https://md-shouvik-iqbal.github.io/calculus
Chapter 3
The key idea is that once we derive the rules from the limit definition for common
types of functions, we no longer need to perform complex limit calculations
repeatedly, over and over again. Thus, we now begin to derive the differentiation
rules for the most common types of functions in the following sections.
143
Copyright © 2024 Md. Shouvik Iqbal. This book is licensed under a Creative Commons Attribution-NonCommercial-ShareAlike 4.0 International License (CC BY-NC-SA 4.0)
Download for free at https://md-shouvik-iqbal.github.io/calculus
Page 144 Elementary Rules of Differentiation Chapter 3
Figure 3.1
d
(c) = 0
dx
= lim 0
h→0
=0
Copyright © 2024 Md. Shouvik Iqbal. This book is licensed under a Creative Commons Attribution-NonCommercial-ShareAlike 4.0 International License (CC BY-NC-SA 4.0)
Download for free at https://md-shouvik-iqbal.github.io/calculus
Chapter 3 Elementary Rules of Differentiation Page 145
Copyright © 2024 Md. Shouvik Iqbal. This book is licensed under a Creative Commons Attribution-NonCommercial-ShareAlike 4.0 International License (CC BY-NC-SA 4.0)
Download for free at https://md-shouvik-iqbal.github.io/calculus
Page 146 Elementary Rules of Differentiation Chapter 3
d f (x + h) − f (x)
(c · f (x)) = lim
dx h→0 h
c · f (x + h) − c · f (x)
= lim
h→0
h
f (x + h) − f (x)
= lim c ·
h→0 h
f (x + h) − f (x)
= lim (c) · lim
h→0 h→0 h
f (x + h) − f (x)
= c · lim
h→0 h
0
= c · f (x)
x2
I Example 3.1.4. If f (x) = x2 , then what is the derivative of , where e is a
e
d
constant and x2 = 2x?
dx
Solution. Applying Theorem 3.1.2, we get,
d x2 d 1 2
= ·x
dx e dx e
1 d 2
= · x
e dx
1
= · 2x
e
2x
=
e
x2 2x
Therefore, the derivative of the function f (x) = is f 0 (x) = .
e e
Copyright © 2024 Md. Shouvik Iqbal. This book is licensed under a Creative Commons Attribution-NonCommercial-ShareAlike 4.0 International License (CC BY-NC-SA 4.0)
Download for free at https://md-shouvik-iqbal.github.io/calculus
Chapter 3 Elementary Rules of Differentiation Page 147
Figure 3.2
However, for n > 1, there isn’t a specific general case such as the one men-
tioned above. For example, how are the derivatives calculated for functions such
as f (x) = x2 or f (x) = x3 ? Currently, we can only apply the definition of
derivatives to these functions to determine their derivatives. For example, to
determine the derivative of f (x) = x2 , we proceed with the following approach.
(a + b)2 = a2 + 2ab + b2
d f (x + h) − f (x)
f (x) = lim
dx h→0 h
Copyright © 2024 Md. Shouvik Iqbal. This book is licensed under a Creative Commons Attribution-NonCommercial-ShareAlike 4.0 International License (CC BY-NC-SA 4.0)
Download for free at https://md-shouvik-iqbal.github.io/calculus
Page 148 Elementary Rules of Differentiation Chapter 3
(x + h)2 − x2
= lim
h→0 h
x + 2xh + h2 − x2
2
= lim
h→0 h
h(2x + h)
= lim
h→0 h
= lim (2x + h)
h→0
= 2x
A pattern may be visible. That is, if f (x) = x2 , then f 0 (x) = 2x. Similarly, if
f (x) = x3 , then f 0 (x) = 3x2 . Indeed, this pattern suggests that if we have a
function f (x) defined as x raised to the power of n, where n is a positive integer,
then its derivative equals n multiplied by x raised to the power of n − 1. This
observation can be proven.
Before presenting the proof of the theorem above, we first need to review the
Binomial Theorem.
Copyright © 2024 Md. Shouvik Iqbal. This book is licensed under a Creative Commons Attribution-NonCommercial-ShareAlike 4.0 International License (CC BY-NC-SA 4.0)
Download for free at https://md-shouvik-iqbal.github.io/calculus
Chapter 3 Elementary Rules of Differentiation Page 149
(x + h)2 = x2 + 2xh + h2
(x + h)3 = x3 + 3x3 h + 3xh2 + h3
(x + h)4 = x4 + 4x3 h + 6x2 h2 + 4xh3 + h4
(x + h)5 = x5 + 5x4 h + 10x3 h2 + 10x2 h3 + 5xh4 + h5
Copyright © 2024 Md. Shouvik Iqbal. This book is licensed under a Creative Commons Attribution-NonCommercial-ShareAlike 4.0 International License (CC BY-NC-SA 4.0)
Download for free at https://md-shouvik-iqbal.github.io/calculus
Page 150 Elementary Rules of Differentiation Chapter 3
= 999x998
dy
I Example 3.1.6. If f (x) = x1+2+3+4+5+6+7+8+9 , then what is ?
dx
Solution. Since 1 + 2 + 3 + 4 + 5 + 6 + 7 + 8 + 9 = 45, therefore, applying the
Power Rule,
dy d
x45
=
dx dx
= 45x45−1
= 45x44
dy
Therefore, if f (x) = x1+2+3+4+5+6+7+8+9 , = 45x44 .
dx
Theorem 3.1.5. (The Summation Rule) If f (x) and g(x) are both differen-
tiable functions, then
d d d
f (x) + g(x) = f (x) + g(x)
dx dx dx
Copyright © 2024 Md. Shouvik Iqbal. This book is licensed under a Creative Commons Attribution-NonCommercial-ShareAlike 4.0 International License (CC BY-NC-SA 4.0)
Download for free at https://md-shouvik-iqbal.github.io/calculus
Chapter 3 Elementary Rules of Differentiation Page 151
f (x + h) − f (x) g(x + h) − g(x)
= lim +
h→0 h h
f (x + h) − f (x) g(x + h) − g(x)
= lim + lim
h→0 h h→0 h
0 0
= f (x) + g (x)
d d
= f (x) + g(x)
dx dx
This completes the proof.
This summation rule can be extended for more than two differentiable func-
tions such as f1 (x), f2 (x), f3 (x), · · · , fn−1 , fn (x), yielding a generalized summa-
tion rule as follows.
d d d d
f1 (x) + f2 (x) + · · · + fn (x) = f1 (x) + f2 (x) + · · · + fn (x)
dx dx dx dx
I Example 3.1.7. What is the derivative of f (x) = 2x2 + 3x3 + 5x5 + 7x7 ?
Solution. Since the derivatives of 2x2 , 3x3 , 5x5 , 7x7 are 4x, 9x2 , 25x4 , 49x6 ,
respectively, by the Power Rule. Therefore, the derivative of f (x) = 2x2 + 3x3 +
5x5 + 7x7 yields,
d d 2 d d d 7
2x2 + 3x3 + 5x5 + 7x7 = 2x + 3x3 + 5x5 +
7x
dx dx dx dx dx
d 2 d 3 d d 7
= 2x + 3x + 5x5 + 7x
dx dx dx dx
= 4x + 9x + 25x + 49x6
2 4
d
Therefore,
2x2 + 3x3 + 5x5 + 7x7 = 4x + 9x2 + 25x4 + 49x6
dx
d 2 d 3 d
I Example 3.1.8. If x = 2x and x = 3x2 , then what is x(x + x2 )?
dx dx dx
d
Solution. First, notice that, since x(x + x2 ) = x2 + x3 , therefore x(x + x2 ) =
dx
d 2
x + x3 . Thus,
dx
d 2 d 2 d 3
x + x3 = x + x
dx dx dx
Copyright © 2024 Md. Shouvik Iqbal. This book is licensed under a Creative Commons Attribution-NonCommercial-ShareAlike 4.0 International License (CC BY-NC-SA 4.0)
Download for free at https://md-shouvik-iqbal.github.io/calculus
Page 152 Elementary Rules of Differentiation Chapter 3
= 2x + 3x2
= x(2 + 3x)
Theorem 3.1.6. (The Difference Rule) If f (x) and g(x) are both differen-
tiable functions, then
d d d
f (x) − g(x) = f (x) − g(x)
dx dx dx
Copyright © 2024 Md. Shouvik Iqbal. This book is licensed under a Creative Commons Attribution-NonCommercial-ShareAlike 4.0 International License (CC BY-NC-SA 4.0)
Download for free at https://md-shouvik-iqbal.github.io/calculus
Chapter 3 Elementary Rules of Differentiation Page 153
d
Therefore,
x1 − x2 − x3 − x4 − x5 = 1 − 2x1 − 3x2 − 4x3 − 5x4
dx
where, e and π are constant. Now, by the Difference Rule, Constant Multiple
Rule, and Power Rule; we get,
d d 1 d d d d
ex1 − πx3 − ex5 − πx7 − ex9 = ex − πx3 − ex5 − πx7 − ex9
dx dx dx dx dx dx
d 1 d 3 d 5 d 7 d
= e x − π x − e x − π x − e x9
dx dx dx dx dx
2 4 6 8
= e − 3πx − 5ex − 7πx − 9ex
Copyright © 2024 Md. Shouvik Iqbal. This book is licensed under a Creative Commons Attribution-NonCommercial-ShareAlike 4.0 International License (CC BY-NC-SA 4.0)
Download for free at https://md-shouvik-iqbal.github.io/calculus
Page 154 Elementary Rules of Differentiation Chapter 3
To understand the logic behind it, we can take a visual approach to explore the
underlying factors as follows.
Assume that f (x) and g(x) are both differentiable functions. Then the product
of these functions, f (x) · g(x) can be interpreted as an area of a rectangle as
shown below.
Figure 3.3
Copyright © 2024 Md. Shouvik Iqbal. This book is licensed under a Creative Commons Attribution-NonCommercial-ShareAlike 4.0 International License (CC BY-NC-SA 4.0)
Download for free at https://md-shouvik-iqbal.github.io/calculus
Chapter 3 Elementary Rules of Differentiation Page 155
Figure 3.4
Now, the product f (x) + ∆f · g(x) + ∆g represents the area of the large
rectangle. And the change in the area f (x) · g(x) of the initial rectangle, that is,
Figure 3.5
is the following,
∆ f (x) · g(x) = f (x) + ∆f · g(x) + ∆g − f (x) · g(x)
= f (x) · g(x) + f (x) · ∆g + ∆f · g(x) + ∆f · ∆g − f (x) · g(x)
= ·
f (x) g(x) + f (x) · ∆g + ∆f · g(x) + ∆f · ∆g − ·
f (x) g(x)
Copyright © 2024 Md. Shouvik Iqbal. This book is licensed under a Creative Commons Attribution-NonCommercial-ShareAlike 4.0 International License (CC BY-NC-SA 4.0)
Download for free at https://md-shouvik-iqbal.github.io/calculus
Page 156 Elementary Rules of Differentiation Chapter 3
f (x) · ∆g ∆f · g(x) ∆f · ∆g
= + +
∆x ∆x ∆x
∆g ∆f ∆g
= f (x) · + g(x) · + ∆f ·
∆x ∆x ∆x
dy ∆y
Now, since the derivative is defined as = lim , there we write the
dx ∆x→0 ∆x
following to get the instantaneous rate of change (i.e., the derivative) of the
product f (x) · g(x),
d
= (f (x) · g(x))
dx
∆ f (x) · g(x)
= lim
∆x→0
∆x
∆g ∆f ∆g
= lim f (x) · + g(x) · + ∆f ·
∆x→0 ∆x ∆x ∆x
∆g ∆f ∆g
= f (x) · lim + g(x) · lim + lim ∆f · lim
∆x→0 ∆x ∆x→0 ∆x ∆x→0 ∆x→0 ∆x
∆g ∆f ∆g
= f (x) · lim + g(x) · lim + lim f (x + ∆x) − f (x) · lim
∆x→0 ∆x ∆x→0 ∆x ∆x→0 ∆x→0 ∆x
d d d
= f (x) · g(x) + g(x) · f (x) + 0 · g(x)
dx dx dx
d d
= f (x) · g(x) + g(x) · f (x)
dx dx
This implies that the derivative of the product of two functions f (x) and g(x)
d
is not merely the product of their derivatives, that is, f (x) · g(x) 6=
dx
d d d d d
f (x) · g(x), but rather, f (x) · g(x) = f (x) · g(x) + g(x) · f (x).
dx dx dx dx dx
Theorem 3.1.7. (The Product Rule) If f (x) and g(x) are both differentiable
functions, then
d d d
f (x) · g(x) = f (x) · g(x) + g(x) · f (x)
dx dx dx
Copyright © 2024 Md. Shouvik Iqbal. This book is licensed under a Creative Commons Attribution-NonCommercial-ShareAlike 4.0 International License (CC BY-NC-SA 4.0)
Download for free at https://md-shouvik-iqbal.github.io/calculus
Chapter 3 Elementary Rules of Differentiation Page 157
0
z }| {
f (x + h) · g(x + h) −f (x + h) · g(x) + f (x + h) · g(x) −f (x) · g(x)
= lim
h→0
h
f (x + h) · g(x + h) − f (x + h) · g(x) f (x + h) · g(x) − f (x) · g(x)
= lim +
h→0 h h
g(x + h) − g(x) f (x + h) − f (x)
= lim f (x + h) · + g(x) ·
h→0 h h
g(x + h) − g(x) f (x + h) − f (x)
= lim f (x + h) · + lim g(x) ·
h→0 h h→0 h
g(x + h) − g(x) f (x + h) − f (x)
= lim f (x + h) · lim + lim g(x) · lim
h→0 h→0 h h→0 h→0 h
0 0
= f (x) · g (x) + g(x) · f (x)
This completes the proof.
Compared to the theorems previously discussed, this theorem seems to be quite
convoluted. Therefore, it is often helpful to simplify it in the following equivalent
manner.
d
I Example 3.1.12. Verify ex(x + 1) = e(2x + 1) in the previous example
dx
without using the Product Rule.
Copyright © 2024 Md. Shouvik Iqbal. This book is licensed under a Creative Commons Attribution-NonCommercial-ShareAlike 4.0 International License (CC BY-NC-SA 4.0)
Download for free at https://md-shouvik-iqbal.github.io/calculus
Page 158 Elementary Rules of Differentiation Chapter 3
Solution. Notice that ex(x + 1) = ex2 + ex, therefore, by the Power Rule, we get,
d d 2 d
ex2 + ex =
ex + ex
dx dx dx
d 2 d
=e x +e x
dx dx
= 2xe + e
= e(2x + 1)
d
This confirms that ex(x + 1) = e(2x + 1)
dx
Copyright © 2024 Md. Shouvik Iqbal. This book is licensed under a Creative Commons Attribution-NonCommercial-ShareAlike 4.0 International License (CC BY-NC-SA 4.0)
Download for free at https://md-shouvik-iqbal.github.io/calculus
Chapter 3 Elementary Rules of Differentiation Page 159
d d f (x)
f (x) − g(x) ·
dx dx g(x)
=
g(x)
d d f (x)
f (x) − g(x) ·
dx dx g(x) g(x)
= ·
g(x) g(x)
| {z }
1
d d f (x)
g(x) · f (x) − g(x) ·
dx dx g(x)
=
g(x) · g(x)
d d f (x)
g(x) f (x) − g(x) · g(x)
dx dx g(x)
=
(g(x))2
d d
g(x) f (x) − f (x) · g(x)
= dx dx
2
(g(x))
d d
d f (x) g(x) f (x) − f (x) · g(x)
∴ = dx dx
dx g(x) 2
(g(x))
f (x)
This implies that the derivative of the quotient function q(x) = is
g(x)
d d
·g(x) f (x) − f (x) · g(x)
dx
2
dx . As a result, we have the following theorem.
(g(x))
Theorem 3.1.8. (The Quotient Rule) If f (x) and g(x) are both differentiable
functions, then
d d
d
f (x)
g(x) · f (x) − f (x) · g(x)
= dx dx
dx g(x)
2
g(x)
Copyright © 2024 Md. Shouvik Iqbal. This book is licensed under a Creative Commons Attribution-NonCommercial-ShareAlike 4.0 International License (CC BY-NC-SA 4.0)
Download for free at https://md-shouvik-iqbal.github.io/calculus
Page 160 Elementary Rules of Differentiation Chapter 3
0
z }| {
f (x + h) · g(x) −f (x) · g(x) + f (x) · g(x) −f (x) · g(x + h)
= lim
h→0 h · g(x) · g(x + h)
f (x + h) · g(x) − f (x) · g(x) + f (x) · g(x) − f (x) · g(x + h)
= lim h
h→0 g(x) · g(x + h)
f (x + h) · g(x) − f (x) · g(x) f (x) · g(x) − f (x) · g(x + h)
+
= lim h h
h→0 g(x) · g(x + h)
f (x + h) − f (x) g(x + h) − g(x)
g(x) · − f (x) ·
h h
= lim
h→0 g(x) · g(x + h)
f (x + h) − f (x) g(x + h) − g(x)
lim g(x) · − lim f (x) ·
h→0 h h→0 h
=
lim g(x) · g(x + h)
h→0
f (x + h) − f (x) g(x + h) − g(x)
lim g(x) · lim − lim f (x) · lim
= h→0 h→0 h h→0 h→0 h
lim g(x) · lim g(x + h)
h→0 h→0
d d
g(x) · f (x) − f (x) · g(x)
= dx dx
2
(g(x))
This completes the proof.
Similar to the Product Rule, the Quotient Rule also may appear convoluted,
thus it is often helpful to simplify it in the following equivalent manner.
Remark 3.1.2. (The Quotient Rule) If f (x) = u and g(x) = v are both
differentiable functions, then
d d
d u v · dx (u) − u · dx (v)
=
dx v (v)2
ex
I Example 3.1.13. What is the derivative of f (x) = ?
x
Solution. Let u = ex and v = x, then by the Quotient Rule,
d d
d ex x · dx ex − ex · dx x
=
dx x x2
Copyright © 2024 Md. Shouvik Iqbal. This book is licensed under a Creative Commons Attribution-NonCommercial-ShareAlike 4.0 International License (CC BY-NC-SA 4.0)
Download for free at https://md-shouvik-iqbal.github.io/calculus
Chapter 3 Differentiation Rules for Trigonometric Functions Page 161
dx dx
x·e − ex ·
= dx dx
x2
ex − ex
=
x2
0
= 2
x
=0
ex
Therefore, the derivative of f (x) = is 0
x
I Example 3.1.14. Verify the result in the preceding example without using the
Quotient Rule.
ex
Solution. Notice that f (x) = = e. This suggests, by the constant rule, that
x
d
(e) = 0
dx
ex
Therefore, the derivative of f (x) = is 0. This completes the verification.
x
Copyright © 2024 Md. Shouvik Iqbal. This book is licensed under a Creative Commons Attribution-NonCommercial-ShareAlike 4.0 International License (CC BY-NC-SA 4.0)
Download for free at https://md-shouvik-iqbal.github.io/calculus
Page 162 Differentiation Rules for Trigonometric Functions Chapter 3
Theorem 3.2.1. (The Derivative of the Sine Function) If f (x) = sin(x), then
d
sin(x) = cos(x)
dx
Prior to proving the theorem above, we first recall that if f (x) = sin(x), then
f (x + h) is given by the angle sum identity1 , that is,
f (x + h) = sin(x + h)
= sin(x) · cos(h) + cos(x) · sin(h)
and consider the following two limits that we had previously evaluated.
f (x + h) − f (x)
f 0 (x) = lim
h→0 h
sin(x + h) − sin(x)
= lim
h→0
h
sin(x) · cos(h) + cos(x) · sin(h) − sin(x)
= lim
h→0 h
sin(x) · cos(h) − sin(x) + cos(x) · sin(h)
= lim
h→0
h hi
sin(x) · (cos(h) − 1) + cos(x) · sin(h)
= lim
h→0 h
h i
sin(x) · (cos(h) − 1)
cos(x) · sin(h)
= lim +
h→0 h h
cos(h) − 1 sin(h)
= lim sin(x) · + lim cos(x) ·
h→0 h h→0 h
cos(h) − 1 sin(h)
= sin(x) · lim + cos(x) · lim
h→0 h h→0 h
| {z } | {z }
the limit is 0 the limit is 1
= sin(x) · 0 + cos(x) · 1
= cos(x)
Copyright © 2024 Md. Shouvik Iqbal. This book is licensed under a Creative Commons Attribution-NonCommercial-ShareAlike 4.0 International License (CC BY-NC-SA 4.0)
Download for free at https://md-shouvik-iqbal.github.io/calculus
Chapter 3 Differentiation Rules for Trigonometric Functions Page 163
Solution. Since f (x) is a product of two functions, namely x and sin(x), therefore
the Product Rule will be used as outlined below.
d d
f (x) = x sin(x)
dx dx
d d
=x· sin(x) + sin(x) x
dx dx
= x · cos(x) + sin(x) · 1
= x cos(x) + sin(x)
sin(x) + 1
I Example 3.2.2. Differentiate f (x) =
sin(x) − 1
Solution. Since f (x) is a quotient of two functions sin(x) + 1 and sin(x) − 1,
therefore the Quotient Rule will be used as outlined below.
d sin(x) + 1
d
f (x) =
dx dx sin(x) − 1
d d
(sin (x) − 1) · (sin (x) + 1) − (sin (x) + 1) · (sin (x) − 1)
= dx dx
(sin (x) − 1)2
d d d d
(sin (x) − 1) · sin (x) + (1) − (sin (x) + 1) · sin (x) − (1)
= dx dx dx dx
(sin (x) − 1)2
(sin (x) − 1) · cos (x) + 0 − (sin (x) + 1) · cos (x) − 0
=
(sin (x) − 1)2
(sin (x) − 1) · cos (x) − (sin (x) + 1) · cos (x)
=
(sin (x) − 1)2
sin (x) cos (x) − cos (x) − sin (x) cos (x) − cos (x)
=
(sin (x) − 1)2
− cos (x) − cos (x)
=
(sin (x) − 1)2
2 cos (x)
=−
(sin (x) − 1)2
Copyright © 2024 Md. Shouvik Iqbal. This book is licensed under a Creative Commons Attribution-NonCommercial-ShareAlike 4.0 International License (CC BY-NC-SA 4.0)
Download for free at https://md-shouvik-iqbal.github.io/calculus
Page 164 Differentiation Rules for Trigonometric Functions Chapter 3
Now that the derivative of sin(x) is determined, we move to find the derivative
of cos(x).
f (x + h) = cos(x + h)
= cos(x) · cos(h) − sin(x) · sin(h)
d cos(x + h) − cos(x)
cos(x) = lim
dx h→0
h
cos(x) · cos(h) − sin(x) · sin(h) − cos(x)
= lim
h→0 h
cos(x) · cos(h) − cos(x) − sin(x) · sin(h)
= lim
h→0
h h i
cos(x) · cos(h) − 1 − sin(x) · sin(h)
= lim
h→0 h
[cos(x) · (cos(h) − 1)] sin(x) · sin(h)
= lim −
h→0 h h
cos(h) − 1 sin(h)
= lim cos(x) · − lim sin(x) ·
h→0 h h→0 h
2
cos (α + β) = cos (α) cos (β) − sin (α) sin (β)
Copyright © 2024 Md. Shouvik Iqbal. This book is licensed under a Creative Commons Attribution-NonCommercial-ShareAlike 4.0 International License (CC BY-NC-SA 4.0)
Download for free at https://md-shouvik-iqbal.github.io/calculus
Chapter 3 Differentiation Rules for Trigonometric Functions Page 165
cos(h) − 1 sin(h)
= cos(x) · lim − sin(x) · lim
h→0 h h→0 h
| {z } | {z }
the limit is 0 the limit is 1
= cos(x) · 0 − sin(x) · 1
= − sin(x)
Solution. Since f (x) is a product of two functions x2 and cos(x), thus the Product
Rule will be used as follows.
d d 2
f (x) = x cos(x)
dx dx
d d 2
2
=x · cos(x) + cos(x) · x
dx
dx
= x2 · − sin(x) + cos(x) · 2x
= −x2 sin(x) + 2x cos(x)
= x 2 cos(x) − x sin(x)
Therefore, the derivative of f (x) = x2 cos(x) is f 0 (x) = x 2 cos(x)−x sin(x) .
Copyright © 2024 Md. Shouvik Iqbal. This book is licensed under a Creative Commons Attribution-NonCommercial-ShareAlike 4.0 International License (CC BY-NC-SA 4.0)
Download for free at https://md-shouvik-iqbal.github.io/calculus
Page 166 Differentiation Rules for Trigonometric Functions Chapter 3
sin(x)
tan(x) =
cos(x)
sin(x)
d d
tan(x) =
dx dx cos(x)
d d
cos(x) · sin(x) − sin(x) · cos(x)
= dx dx
cos2 (x)
cos(x) · cos(x) − sin(x) · (− sin(x))
=
cos2 (x)
cos2 (x) + sin2 (x)
=
cos2 (x)
1
=
cos2 (x)
= sec2 (x)
This implies that the derivative of the tangent function is the secant function
squared.
Copyright © 2024 Md. Shouvik Iqbal. This book is licensed under a Creative Commons Attribution-NonCommercial-ShareAlike 4.0 International License (CC BY-NC-SA 4.0)
Download for free at https://md-shouvik-iqbal.github.io/calculus
Chapter 3 Differentiation Rules for Trigonometric Functions Page 167
sin2 (x)
I Example 3.2.6. Differentiate f (x) =
cos(x)
sin2 (x)
Solution. Since can be simplified as follows,
cos(x)
sin2 (x) sin(x)
= · sin(x)
cos(x) cos(x)
= tan(x) · sin(x)
sin2 (x)
Therefore, the derivative of f (x) = is f 0 (x) = sin(x)(1 + sec2 (x)).
cos(x)
Copyright © 2024 Md. Shouvik Iqbal. This book is licensed under a Creative Commons Attribution-NonCommercial-ShareAlike 4.0 International License (CC BY-NC-SA 4.0)
Download for free at https://md-shouvik-iqbal.github.io/calculus
Page 168 Differentiation Rules for Trigonometric Functions Chapter 3
(−1) · (1)
=
sin2 (x)
1
=− 2
sin (x)
= − csc2 (x)
This implies that the derivative of the cotangent function is the negative of the
cosecant function squared. That is,
cos2 (x)
I Example 3.2.8. Differentiate f (x) =
e sin(x)
cos2 (x)
Solution. Since can be simplified as follows,
e sin(x)
Copyright © 2024 Md. Shouvik Iqbal. This book is licensed under a Creative Commons Attribution-NonCommercial-ShareAlike 4.0 International License (CC BY-NC-SA 4.0)
Download for free at https://md-shouvik-iqbal.github.io/calculus
Chapter 3 Differentiation Rules for Trigonometric Functions Page 169
1 d d
= cot(x) · cos(x) + cos(x) · cot(x)
e dx dx
1
cot(x) · (− sin(x)) + cos(x) · (−csc2 (x))
=
e
1
− sin(x) cot(x) − cos(x)csc2 (x)
=
e
− cos(x) − cos(x)csc2 (x)
sin(x)
sin(x)
=
e
− cos(x) − cos(x)csc2 (x)
=
e
cos2 (x) − cos(x) − cos(x)csc2 (x)
Therefore, the derivative of f (x) = is f 0 (x) =
e sin(x) e
This implies that the derivative of the secant function is the secant function
multiplied by the tangent function. That is,
Copyright © 2024 Md. Shouvik Iqbal. This book is licensed under a Creative Commons Attribution-NonCommercial-ShareAlike 4.0 International License (CC BY-NC-SA 4.0)
Download for free at https://md-shouvik-iqbal.github.io/calculus
Page 170 Differentiation Rules for Trigonometric Functions Chapter 3
then
d
sec(x) = sec(x) tan(x)
dx
Therefore,
d d 2 d
x2 − 2 sec(x) = x − 2 sec(x)
dx dx dx
= 2x − 2 sec(x) tan(x)
Therefore, the derivative of f (x) = x2 cos(0) − 2 cos(0) · sec(x) is 2x−2 sec(x) tan(x).
Copyright © 2024 Md. Shouvik Iqbal. This book is licensed under a Creative Commons Attribution-NonCommercial-ShareAlike 4.0 International License (CC BY-NC-SA 4.0)
Download for free at https://md-shouvik-iqbal.github.io/calculus
Chapter 3 Differentiation Rules for Trigonometric Functions Page 171
This implies that the derivative of the cosecant function is the negative of the
cosecant function multiplied by the cotangent function. That is,
Copyright © 2024 Md. Shouvik Iqbal. This book is licensed under a Creative Commons Attribution-NonCommercial-ShareAlike 4.0 International License (CC BY-NC-SA 4.0)
Download for free at https://md-shouvik-iqbal.github.io/calculus
Differentiation of Composite Functions
Page 172 (The Chain Rule) Chapter 3
Therefore, the derivative of f (x) = csc2 (x) is f 0 (x) = −2csc2 (x) cot(x).
(1)
3
y = x4 + 14
To differentiate this function using the rules discussed so far, we must first need
to expand it and then apply the appropriate differentiation rules. For example,
since,
We can now differentiate (3) using the Power and Summation Rule. That is,
d d
x12 + 42x8 + 588x4 + 2744
(y) =
dx dx
Copyright © 2024 Md. Shouvik Iqbal. This book is licensed under a Creative Commons Attribution-NonCommercial-ShareAlike 4.0 International License (CC BY-NC-SA 4.0)
Download for free at https://md-shouvik-iqbal.github.io/calculus
Chapter 3 The Chain Rule Page 173
d
x12 + 42x8 + 588x4 + 2744
=
dx
d d d d
x12 + 42x8 + 588x4 +
= (2744)
dx dx dx dx
= 12x11 + 336x7 + 2352x3 (4)
2352x3 . Now, can the same approach be taken to find the derivative of y =
? The answer is quite obviously no. Therefore, let us begin
235711131719
x4 + 14
to develop a new method to tackle such situations.
Now, since u in turn is a function of x, therefore the derivative (i.e., the rate of
change) of u with respect to x is,
du d
x4 + 14
=
dx dx
d d
x4 +
= (14)
dx dx
= 4x3 + 0
= 4x3
And, if the output u changes 4x3 times as fast as the input x, since y = x4 + 14
Copyright © 2024 Md. Shouvik Iqbal. This book is licensed under a Creative Commons Attribution-NonCommercial-ShareAlike 4.0 International License (CC BY-NC-SA 4.0)
Download for free at https://md-shouvik-iqbal.github.io/calculus
Page 174 The Chain Rule Chapter 3
In notations,
dy
= 3u2 if
du
du
and if = 4x3
dx
dy dy du
then = 3u2 × 4x3
= ×
dx du dx
2
3u2 · 4x3 = 3 x4 + 14 · 4x3
2
= 12x3 x4 + 14
using the method described above. For that, let u = x4 + 14, then y =
u235711131719 . Since y is a function of u, therefore, it is differentiated with respect
to u,
dy d
u235711131719
=
du du
= 235711131719u235711131719−1
= 235711131719u235711131718
Copyright © 2024 Md. Shouvik Iqbal. This book is licensed under a Creative Commons Attribution-NonCommercial-ShareAlike 4.0 International License (CC BY-NC-SA 4.0)
Download for free at https://md-shouvik-iqbal.github.io/calculus
Chapter 3 The Chain Rule Page 175
235711131718
= 942844526876x3 x4 + 14
dy dy du
= ·
dx du dx
or, equivalently,
We here provide an intuitive argument of the Chain Rule. Note that it isn’t
sufficiently rigorous to be accepted as a complete proof of the theorem.
Intuitive Argument. Since u is a function of x, therefore any change ∆x in x pro-
duces a corresponding change ∆u in u. This, in turn, produces a corresponding
change ∆y in y, as y is a function of u. Now, consider the following,
dy ∆y
= lim by the definition of derivative
dx ∆x→0 ∆x
∆y
= lim ·1
∆x→0 ∆x
∆y ∆u
= lim ·
∆x→0 ∆x ∆u
∆y ∆u
= lim ·
∆x→0 ∆u ∆x
∆y ∆u
= lim · lim
∆x→0 ∆u ∆x→0 ∆x
Copyright © 2024 Md. Shouvik Iqbal. This book is licensed under a Creative Commons Attribution-NonCommercial-ShareAlike 4.0 International License (CC BY-NC-SA 4.0)
Download for free at https://md-shouvik-iqbal.github.io/calculus
Page 176 The Chain Rule Chapter 3
To better understand the inner and outer function analogy, consider the following
functions.
sin(x2 ) and sin2 (x)
Although the inner and outer parts of these functions may resemble one another,
they are fundamentally different. The inner part of the first function is u = x2 ,
and the outer part of it is y = sin(u). Whereas the inner part of the second
function is u = sin(x), and the outer part of it is y = u2 , since sin2 (x) =
2
sin(x) .
Copyright © 2024 Md. Shouvik Iqbal. This book is licensed under a Creative Commons Attribution-NonCommercial-ShareAlike 4.0 International License (CC BY-NC-SA 4.0)
Download for free at https://md-shouvik-iqbal.github.io/calculus
Chapter 3 The Chain Rule Page 177
= cos(u) · 2x
= cos(x2 ) · 2x
= 2x cos(x2 )
To demonstrate this, we apply the Chain Rule directly to the function shown in
Example 3.3.1, as follows,
dy d
sin x2
=
dx dx
Copyright © 2024 Md. Shouvik Iqbal. This book is licensed under a Creative Commons Attribution-NonCommercial-ShareAlike 4.0 International License (CC BY-NC-SA 4.0)
Download for free at https://md-shouvik-iqbal.github.io/calculus
Page 178 The Chain Rule Chapter 3
d
= cos x2 · x2
dx
= cos x · 2x
2
= 2x cos x2
That is, the outermost function is first differentiated and then simply multiplied
by the derivative of the inner function.
√
I Example 3.3.3. Differentiate y = x2 + x
√
Solution. For the function y = x2 + x, the inner and outer parts are u = x2 + x
√
and y = u, respectively. Therefore, by the Chain Rule,
dy dy du
= ·
dx du dx
d √ d 2
= u · x +x
du dx
1
= √ · 2x + 1
2 u
2x + 1
= √
2 u
2x + 1
= √
2 x2 + x
√ 2x + 1
Therefore, the derivative of f (x) = x2 + x is f 0 (x) = √ .
2 x2 + x
2
x2
I Example 3.3.4. Differentiate y =
x+1
2 2
x
Solution. For the function y = , the inner and outer parts are u =
x+1
x2
and u2 , respectively. Therefore, by the Chain Rule,
x+1
dy dy du
= ·
dx du dx 2
d 2
d x
= u ·
du dx x + 1
d d
(x + 1) · x2 − x2 · (x + 1)
= 2u · dx dx
(x + 1)2
Copyright © 2024 Md. Shouvik Iqbal. This book is licensed under a Creative Commons Attribution-NonCommercial-ShareAlike 4.0 International License (CC BY-NC-SA 4.0)
Download for free at https://md-shouvik-iqbal.github.io/calculus
Chapter 3 The Chain Rule Page 179
(x + 1) · 2x − x2 · 1
= 2u ·
(x + 1)2
2x (x + 1) − x2
= 2u ·
(x + 1)2
2
x 2x (x + 1) − x2
=2 ·
x+1 (x + 1)2
2x2 2x2 + 2x − x2
= ·
x+1 (x + 1)2
2x2 2x2 + 2x − x2
=
(x + 1)3
4x4 + 4x3 − 2x4
=
(x + 1)3
2x3 (2x + 2 − x)
=
(x + 1)3
2x3 (x + 2)
=
(x + 1)3
2 2
x 2x3 (x + 2)
Therefore, the derivative of f (x) = is f 0 (x) = .
x+1 (x + 1)3
2 2
x x4
Note that if the equation were to be simplified as = prior to
x+1 (x + 1)2
differentiating it solely using the Quotient Rule, the result would’ve remained
the same.
Since the Chain Rule is one of the most important and frequently encountered
rule among the differentiation rules, therefore a few additional examples are
given below.
dp
f 0 (x) = sin (x)
dx
d
sin (x) 2
1
=
dx
1 d
= sin (x)− 2 · sin (x)
1
2 dx
1 1
= · · cos (x)
2 sin (x) 12
Copyright © 2024 Md. Shouvik Iqbal. This book is licensed under a Creative Commons Attribution-NonCommercial-ShareAlike 4.0 International License (CC BY-NC-SA 4.0)
Download for free at https://md-shouvik-iqbal.github.io/calculus
Page 180 The Chain Rule Chapter 3
cos (x)
= p
2 sin (x)
cos (x)
Therefore, the derivative of f (x) = sin (x) is f 0 (x) = p .
p
2 sin (x)
Therefore, the derivative of f (x) = sin cos x2 is f 0 (x) = −2x cos x2 sin x2 .
Therefore, the derivative of f (x) = sin cos tan x2 is f 0 (x) = −2x·cos cos tan x2
·
sin tan x2 · sec2 x2 .
Copyright © 2024 Md. Shouvik Iqbal. This book is licensed under a Creative Commons Attribution-NonCommercial-ShareAlike 4.0 International License (CC BY-NC-SA 4.0)
Download for free at https://md-shouvik-iqbal.github.io/calculus
Chapter 3 The Chain Rule Page 181
d
tan cot sec csc x2
·
dx
= cos cos tan cot sec csc x2 · − sin tan cot sec csc x2
d
· sec2 cot sec csc x2 cot sec csc x2
·
dx
= cos cos tan cot sec csc x2 · − sin tan cot sec csc x2
d
· sec2 cot sec csc x2 · −csc2 sec csc x2 sec csc x2
·
dx
= cos cos tan cot sec csc x 2
· − sin tan cot sec csc x2
· sec2 cot sec csc x2 · −csc2 sec csc x2 · sec csc x2 tan csc x2
d
csc x2
·
dx
= cos cos tan cot sec csc x2 · − sin tan cot sec csc x2
· sec2 cot sec csc x2 · −csc2 sec csc x2 · sec csc x2 tan csc x2
d
· − csc x2 cot x2 · x2
dx
= cos cos tan cot sec csc x2 · − sin tan cot sec csc x2
· sec2 cot sec csc x2 · −csc2 sec csc x2 · sec csc x2 tan csc x2
· − csc x2 cot x2 · 2x
= − 2x cos cos tan cot sec csc x2 sin tan cot sec csc x2
sec2 cot sec csc x2 csc sec csc x2 sec csc x2 tan csc x2
2
csc x2 cot x2
Therefore, the derivative of f (x) = sin cos tan cot sec csc x2 is f 0 (x) =
−2x cos cos tan cot sec csc x2 · sin tan cot sec csc x2
· sec2 cot sec csc x2 · csc2 sec csc x2 · sec csc x2 · tan csc x2 ·
csc x2 cot x2 .
Copyright © 2024 Md. Shouvik Iqbal. This book is licensed under a Creative Commons Attribution-NonCommercial-ShareAlike 4.0 International License (CC BY-NC-SA 4.0)
Download for free at https:// md-shouvik-iqbal.github.io/ calculus
Page 182 Implicit Differentiation Chapter 3
y = f (x)
xy − 1 = 0
xy − 1 = 0
xy = 1
1
y=
x
It is now evident—after rearrangement of the equation—that y is indeed a
function of x, even though it was initially defined implicitly by the equation
xy − 1 = 0. Such a function, where y is defined indirectly by an equation, is
called an implicit function of x.
F (x, y) = 0 (1)
In contrast to our intuition, an implicit equation may not always define a single
function. When the graph of an implicit equation fails the vertical line test, the
equation defines more than one function. For example, consider the following
equation,
x2 + y 2 = 1
Copyright © 2024 Md. Shouvik Iqbal. This book is licensed under a Creative Commons Attribution-NonCommercial-ShareAlike 4.0 International License (CC BY-NC-SA 4.0)
Download for free at https://md-shouvik-iqbal.github.io/calculus
Chapter 3 Implicit Differentiation Page 183
The graph of this equation is the graph of a unit circle that fails the vertical line
test. Solving this for y yields,
p
y = ± 1 − x2
√ √
where, y = + 1 − x2 represents the upper semicircle and y = − 1 − x2 repre-
sents the lower semicircle of the circle x2 + y 2 = 1.
With that in mind, notice now that some equations defining implicit functions
y are notoriously difficult—if not impossible—to express in terms of x explicitly.
And, so far, we have seen that in order to differentiate a function, it needs to be
in the form of y = f (x). So, how such implicitly defined functions be differenti-
ated? Is it even possible?
xy − 1 = 0
xy = 1 (2)
The typical approach to differentiating this equation is to first rewrite the equa-
1
tion as y = and then to differentiate y with respect to x, as shown below.
x
dy d 1
=
dx dx x
d −1
= x
dx
= −1 · x−1−1
= −x−2
1
=− 2
x
Now, instead of this typical method, if we consider y as a differentiable function
of x, we can differentiate equation (2) directly as follows.
d d
xy = (1)
dx dx
d d
x · (y) + y · (x) = 0
dx dx
dy
x· +y =0 (3)
dx
Copyright © 2024 Md. Shouvik Iqbal. This book is licensed under a Creative Commons Attribution-NonCommercial-ShareAlike 4.0 International License (CC BY-NC-SA 4.0)
Download for free at https://md-shouvik-iqbal.github.io/calculus
Page 184 Implicit Differentiation Chapter 3
dy
Now, solving for algebraically yields,
dx
dy
x· = −y
dx
dy y
=−
dx x
y
Thus, the derivative of the implicit function xy = 1 is − . Notice that the
x
derivative, in this case, is expressed in terms of both x and y, instead of just
x. So, to evaluate the derivative at a specific point, both values of x and y are
required.
Additionally, notice how the Chain Rule is used in equation (3). That is, when
differentiating terms with respect to x that contain only the variable x, the pro-
cess follows the usual method. However, when we encounter terms involving
variables other than x, the Chain Rule is applied there.
Remark 3.4.1.
Copyright © 2024 Md. Shouvik Iqbal. This book is licensed under a Creative Commons Attribution-NonCommercial-ShareAlike 4.0 International License (CC BY-NC-SA 4.0)
Download for free at https://md-shouvik-iqbal.github.io/calculus
Chapter 3 Implicit Differentiation Page 185
With that in mind, we will now consider a few examples to understand the
implicit differentiation.
x2 + y 3 = 5
d d
x2 + y 3 =
(5)
dx dx
d 2 d
x + y3 = 0
dx dx
2 dy
2x + 3y · =0
dx
dy
3y 2 · = −2x
dx
dy 2x
=− 2
dx 3y
dy 2x
Therefore, the derivative of x2 + y 3 = 5 is = − 2.
dx 3y
Copyright © 2024 Md. Shouvik Iqbal. This book is licensed under a Creative Commons Attribution-NonCommercial-ShareAlike 4.0 International License (CC BY-NC-SA 4.0)
Download for free at https://md-shouvik-iqbal.github.io/calculus
Page 186 Implicit Differentiation Chapter 3
dy
x + 3y 2 = 1 − 2x − y
dx
dy 1 − 2x − y
=
dx x + 3y 2
dy 1 − 2x − y
Therefore, the derivative of x2 + xy + y 3 = x is = .
dx x + 3y 2
sin(x) = tan(y)
dy dy
sin(x) = tan(y)
dx dx
dy
cos(x) = sec2 (y) ·
dx
dy cos(x)
=
dx sec2 (y)
dy cos(x)
Therefore, the derivative of sin(x) = tan(y) is = .
dx sec2 (y)
Copyright © 2024 Md. Shouvik Iqbal. This book is licensed under a Creative Commons Attribution-NonCommercial-ShareAlike 4.0 International License (CC BY-NC-SA 4.0)
Download for free at https://md-shouvik-iqbal.github.io/calculus
Chapter 3 Implicit Differentiation Page 187
x−1
= x
1−y
y
x−1 y
= ·
x 1−y
y(x − 1)
=
x(1 − y)
dy y(x − 1)
Therefore, the derivative of ln(x) + ln(y) = x + y is = .
dx x(1 − y)
r
√
q
I Example 3.4.5. Differentiate y =
p
x+ x+ x + x + ···
r
√
q
I Example 3.4.6. Since y = x + x + · · ·, therefore, it can be
p
x+ x+
simplified as follows,
s r 2
√
q
y2 = x+ x + x + x + · · ·
r
√
q
2
y = x + x + x + x + ···
| {z }
y
y2 = x + y
Copyright © 2024 Md. Shouvik Iqbal. This book is licensed under a Creative Commons Attribution-NonCommercial-ShareAlike 4.0 International License (CC BY-NC-SA 4.0)
Download for free at https://md-shouvik-iqbal.github.io/calculus
Page 188 Differentiation Rules for Exponential Functions Chapter 3
r
√
q
Therefore, the derivative of y = x + x + x + x + · · · is
p
dy 1
= r .
dx q p √
2 x + x + x + x + ··· − 1
Instances such as the one above are indeed interesting, especially, when consider-
ing the connection between the rates of change of the function and its derivative.
However, what’s even more interesting is the existence of a function where the
rate at which it changes at a point is equivalent to the value of the function itself
at that point!
Copyright © 2024 Md. Shouvik Iqbal. This book is licensed under a Creative Commons Attribution-NonCommercial-ShareAlike 4.0 International License (CC BY-NC-SA 4.0)
Download for free at https://md-shouvik-iqbal.github.io/calculus
Chapter 3 Differentiation Rules for Exponential Functions Page 189
2h − 1
= lim 2 · x
h→0 h
h
2 − 1
= 2x · lim
h→0 h
h
2 −1
Now, we evaluate the limit lim numerically by the following table as
h→0 h
h approaches to 0,
Table 3.1: Table
2h − 1
h
h
−0.1 0.66967...
−0.01 0.69075...
−0.001 0.69290...
+0.001 0.69338...
+0.01 0.69555...
+0.1 0.71773...
The table above suggests that as h → 0 from both sides of 0, the expression
2h − 1
→ k such that 0.69290... < k < 0.69338.... This implies that
h
d x
(2 ) = k · 2x
dx
This further implies,
d x
(2 ) ∝ 2x
dx
This is indeed fascinating that there exists a function f (x) = 2x where its
derivative is proportional to itself. However, this does not answer our search
for a function whose derivative is equal to itself. Therefore, let us generalize
the case by letting f (x) = ax this time, where a ∈ R. If f (x) = ax , then the
d x
derivative (a ) is,
dx
d x ax+h − ax
(a ) = lim
dx h→0 h
a · ah − ax
x
= lim
h→0 h
h
a −1
= lim a ·
x
h→0 h
h
a −1
= a · lim
x
h→0 h
Copyright © 2024 Md. Shouvik Iqbal. This book is licensed under a Creative Commons Attribution-NonCommercial-ShareAlike 4.0 International License (CC BY-NC-SA 4.0)
Download for free at https://md-shouvik-iqbal.github.io/calculus
Page 190 Differentiation Rules for Exponential Functions Chapter 3
ah − 1
To evaluate the limit lim , we create the following table for different
h→0 h
values of a, where h ∈ R is a small increment less than 1 and greater than 0.
Table 3.2: Table
ah − 1
a
h
2 0.69314…
3 1.09861…
4 1.38630…
5 1.60945…
Now, a clue is found! From the table above, it is visible that between a = 2 and
ah − 1
a = 3, there exists a value of a for which the limit lim yields 1. And why
h→0 h
ah − 1
are we interested lim resulting in 1? Because then the derivative of ax
h→0 h
(whatever the value of a might be), would be equal to itself. Thus, let us first
ah − 1 ah − 1
search the value of a for which lim = 1. To do that, let ≈ 1, for
h→0 h h
a small h ∈ R. This implies, for a small h ∈ R,
ah − 1
≈1
h
ah − 1 ≈ h
ah ≈ h + 1
√
h
a≈ 1+h
1
a ≈ (1 + h) h
lim (1 + h) h = a
1
h→0
table,
Table 3.3: Table
1
h (1 + h) h
−0.1 2.86796 · · ·
−0.01 2.73198 · · ·
Copyright © 2024 Md. Shouvik Iqbal. This book is licensed under a Creative Commons Attribution-NonCommercial-ShareAlike 4.0 International License (CC BY-NC-SA 4.0)
Download for free at https://md-shouvik-iqbal.github.io/calculus
Chapter 3 Differentiation Rules for Exponential Functions Page 191
−0.001 2.71963 · · ·
+0.001 2.71691 · · ·
+0.01 2.70480 · · ·
+0.1 2.59373 · · ·
d x
e = ex
dx
The result of the example above is of such frequency that it should have its own
remark, as given below.
Copyright © 2024 Md. Shouvik Iqbal. This book is licensed under a Creative Commons Attribution-NonCommercial-ShareAlike 4.0 International License (CC BY-NC-SA 4.0)
Download for free at https://md-shouvik-iqbal.github.io/calculus
Page 192 Differentiation Rules for Exponential Functions Chapter 3
d −x
e = −e−x
dx
eln(x) = x
Copyright © 2024 Md. Shouvik Iqbal. This book is licensed under a Creative Commons Attribution-NonCommercial-ShareAlike 4.0 International License (CC BY-NC-SA 4.0)
Download for free at https://md-shouvik-iqbal.github.io/calculus
Chapter 3 Differentiation Rules for Exponential Functions Page 193
= 1 · ln(a)
h
a −1
Thus, it can be shown that lim = ln(a) and therefore,
h→0 h
h
d x a −1
(a ) = a · lim
x
dx h→0 h
= ax · ln(a)
d x
(a ) = ax · ln(a)
dx
Copyright © 2024 Md. Shouvik Iqbal. This book is licensed under a Creative Commons Attribution-NonCommercial-ShareAlike 4.0 International License (CC BY-NC-SA 4.0)
Download for free at https://md-shouvik-iqbal.github.io/calculus
Page 194 Differentiation Rules for Exponential Functions Chapter 3
= π x ln2 (π)
Copyright © 2024 Md. Shouvik Iqbal. This book is licensed under a Creative Commons Attribution-NonCommercial-ShareAlike 4.0 International License (CC BY-NC-SA 4.0)
Download for free at https://md-shouvik-iqbal.github.io/calculus
Chapter 3 Differentiation Rules for Logarithmic Functions Page 195
axa
= x · a ln (a) + a ·
a x x
x
a
= x a ln (a) +
a x
x
a
Therefore, the derivative of f (x) = xa ax is f 0 (x) = xa ax ln (a) + .
x
x = eln(x)
Copyright © 2024 Md. Shouvik Iqbal. This book is licensed under a Creative Commons Attribution-NonCommercial-ShareAlike 4.0 International License (CC BY-NC-SA 4.0)
Download for free at https://md-shouvik-iqbal.github.io/calculus
Page 196 Differentiation Rules for Logarithmic Functions Chapter 3
1
This implies that the derivative of f (x) = ln(x) is f 0 (x) = , for all x > 0.
x
Therefore, we have the following theorem.
d 1
Note that since the domain of ln(x) is (0, ∞), therefore ln(x) = is true for
dx x
all values of x > 0. These values can be extended by considering the function
ln |x|, instead of ln(x). Because, by the definition of absolute value, we know,
ln(x) if x > 0
ln |x| =
ln(−x) if x < 0
This implies that ln |x| is defined for all x 6= 0. Thus, for x > 0, we have
d d
ln |x| = ln(x)
dx dx
1
=
x
And, for x < 0, we have,
d d
ln |x| = ln (−x)
dx dx
1 d
= · (−x)
−x dx
1 d
= · − (x)
−x dx
1
= · −1
−x
1
=
x
Thus, we have the following theorem.
Copyright © 2024 Md. Shouvik Iqbal. This book is licensed under a Creative Commons Attribution-NonCommercial-ShareAlike 4.0 International License (CC BY-NC-SA 4.0)
Download for free at https://md-shouvik-iqbal.github.io/calculus
Chapter 3 Differentiation Rules for Logarithmic Functions Page 197
1
= · f 0 (x)
f (x)
f 0 (x)
=
f (x)
d
Theorem 3.6.3. If ln |f (x)|, where f (x) 6= 0, then
dx
d f 0 (x)
ln |f (x)| =
dx f (x)
Solution.
d d d
x ln |x| = x · ln |x| + ln |x| · (x)
dx dx dx
1
= x · + ln |x| · 1
x
= 1 + ln |x|
Solution.
d 1 d
ln |sec (x)| = · sec (x)
dx sec (x) dx
1
= · sec (x) tan (x)
sec (x)
= tan (x)
Copyright © 2024 Md. Shouvik Iqbal. This book is licensed under a Creative Commons Attribution-NonCommercial-ShareAlike 4.0 International License (CC BY-NC-SA 4.0)
Download for free at https://md-shouvik-iqbal.github.io/calculus
Page 198 Differentiation Rules for Logarithmic Functions Chapter 3
y = loga (x) ⇔ x = ay
dy 1
= y
dx a ln (a)
1
=
x ln (a)
Therefore, we have the following theorem.
d 1
loga (x) =
dx x ln (a)
Similar to the natural logarithm, the values of the domain of loga (x) can be
extended by considering loga |x|. As a result, we have the following theorem.
d 1
loga |x| =
dx x ln (a)
For all x 6= 0.
Similar to the previous subsection, if there exists a function f (x) 6= 0, then the
derivative of loga |f (x)| happens to be the following,
d 1 d
loga |f (x)| = · f (x)
dx f (x) ln(a) dx
Copyright © 2024 Md. Shouvik Iqbal. This book is licensed under a Creative Commons Attribution-NonCommercial-ShareAlike 4.0 International License (CC BY-NC-SA 4.0)
Download for free at https://md-shouvik-iqbal.github.io/calculus
Chapter 3 Differentiation Rules for Logarithmic Functions Page 199
1
= · f 0 (x)
f (x) ln(a)
f 0 (x)
=
f (x) ln(a)
Thus, we have the following theorem.
d
Theorem 3.6.6. If log |f (x)|, where f (x) 6= 0, then
dx a
d f 0 (x)
loga |f (x)| =
dx f (x) ln(a)
Copyright © 2024 Md. Shouvik Iqbal. This book is licensed under a Creative Commons Attribution-NonCommercial-ShareAlike 4.0 International License (CC BY-NC-SA 4.0)
Download for free at https://md-shouvik-iqbal.github.io/calculus
Page 200 Logarithmic Differentiation Chapter 3
1
=
x ln(π) ln(x)
1
Therefore, the derivative of f (x) = logπ (ln(x)) is f 0 (x) = .
x ln(π) ln(x)
1
4. ln = − ln(a)
a
2. ln = ln(a) − ln(b) a
b
Now, using the properties of natural logarithms shown above, the function can
be rewritten as follows:
x+1
e x
ln f (x) = ln
x+1
= ln ex+1 x − ln (x + 1)
= ln ex+1 + ln (x) − ln (x + 1)
Copyright © 2024 Md. Shouvik Iqbal. This book is licensed under a Creative Commons Attribution-NonCommercial-ShareAlike 4.0 International License (CC BY-NC-SA 4.0)
Download for free at https://md-shouvik-iqbal.github.io/calculus
Chapter 3 Logarithmic Differentiation Page 201
= (x + 1) ln (e) + ln (x) − ln (x + 1)
= (x + 1) + ln (x) − ln (x + 1)
Now, differentiating both sides of the equation above yields the following,
d d
ln f (x) = (x + 1) + ln (x) − ln (x + 1)
dx dx
d d d
= (x + 1) + ln (x) − ln (x + 1)
dx dx dx
f 0 (x) 1 1
=1+ −
f (x) x x+1
0 1 1
f (x) = 1 + − · f (x)
x x+1
x+1
1 1 e x
= 1+ − ·
x x+1 x+1
x+1
x (x + 1) + (x + 1) − x e x
= ·
x (x + 1) x+1
2 x+1
x +x+1 e x
= ·
x (x + 1) x+1
2
x+1
x +x+1 e
=
(x + 1)2
0 ex+1 x2 + ex+1 x + ex+1
∴ f (x) =
(x + 1)2
ex+1 x
Therefore, the derivative f 0 (x) of the function f (x) = is
x+1
ex+1 x2 + ex+1 x + ex+1
f 0 (x) = .
(x + 1)2
Using the logarithmic differentiation, the proof for the general Power Rule can
be given as follows.
d r
x = rxr−1
dx
y = xr
ln (y) = ln (xr )
Copyright © 2024 Md. Shouvik Iqbal. This book is licensed under a Creative Commons Attribution-NonCommercial-ShareAlike 4.0 International License (CC BY-NC-SA 4.0)
Download for free at https://md-shouvik-iqbal.github.io/calculus
Page 202 Logarithmic Differentiation Chapter 3
ln (y) = r · ln (x)
1 dy 1
· =r·
y dx x
dy y
=r·
dx x
Since y = xr , therefore,
dy xr
=r·
dx x
= r · xr−1
√
I Example 3.7.1. If f (x) = x = x 2 , then
1
d √ d 1
( x) = (x 2 )
dx dx
1 1
= x 2 −1
2
1 1
= x− 2
2
1
= √
2 x
√ 1
Therefore, the derivative of f (x) = x = x 2 is f 0 (x) = √ .
1
2 x
Remark 3.7.1.
d √ 1
( x) = √
dx 2 x
xr+1
I Example 3.7.2. If f (x) = , where r ∈ R, then,
r+1
r+1
d x 1 d
xr+1
= ·
dx r + 1 r + 1 dx
1
· r + 1 · xr+1−1
=
r+1
1
= · r+ 1 · xr
r+1
Copyright © 2024 Md. Shouvik Iqbal. This book is licensed under a Creative Commons Attribution-NonCommercial-ShareAlike 4.0 International License (CC BY-NC-SA 4.0)
Download for free at https://md-shouvik-iqbal.github.io/calculus
Chapter 3 Logarithmic Differentiation Page 203
= xr
xr+1
Therefore, the derivative of f (x) = is f 0 (x) = xr
r+1
Remark 3.7.2.
xr+1
d
= xr
dx r+1
where, r ∈ R.
A few more examples are provided below to demonstrate the logarithmic differ-
entiation.
ln f (x) = ln (xx )
= x · ln (x)
√ √
I Example 3.7.4. Differentiate f (x) =
x
x
Copyright © 2024 Md. Shouvik Iqbal. This book is licensed under a Creative Commons Attribution-NonCommercial-ShareAlike 4.0 International License (CC BY-NC-SA 4.0)
Download for free at https://md-shouvik-iqbal.github.io/calculus
Page 204 Logarithmic Differentiation Chapter 3
xy = y x
ln (xy ) = ln (y x )
y · ln(x) = x · ln(y)
Copyright © 2024 Md. Shouvik Iqbal. This book is licensed under a Creative Commons Attribution-NonCommercial-ShareAlike 4.0 International License (CC BY-NC-SA 4.0)
Download for free at https://md-shouvik-iqbal.github.io/calculus
Chapter 3 Logarithmic Differentiation Page 205
1 dy 1 dy
y· + ln(x) · =x· + ln(y) · 1
x dx y dx
y dy x dy
+ ln(x) = + ln(y)
x dx y dx
dy x dy y
ln(x) − = ln(y) −
dx y dx x
dy x y
ln(x) − = ln(y) −
dx y x
y
dy ln(y) −
= x
x
dx ln(x) −
y
x ln(y) − y
= x
y ln(x) − x
y
x ln(y) − y y
= ·
x y ln(x) − x
y (x ln(y) − y)
=
x (y ln(x) − x)
y (x ln(y) − y)
Therefore, differentiating xy = y x , we get f 0 (x) = .
x (y ln(x) − x)
ln(xy) = x + y
ln(x) + ln(y) = x + y
Copyright © 2024 Md. Shouvik Iqbal. This book is licensed under a Creative Commons Attribution-NonCommercial-ShareAlike 4.0 International License (CC BY-NC-SA 4.0)
Download for free at https://md-shouvik-iqbal.github.io/calculus
Page 206 Differentiation Rules for Inverse Trigonometric Functions Chapter 3
1
dy 1−
= x
dx 1
−1
y
x−1
= x
1−y
y
x−1 y
= ·
x 1−y
y (x − 1)
=
x (1 − y)
y (x − 1)
Therefore, differentiating ln(xy) = x + y, we get f 0 (x) = .
x (1 − y)
x = sin (y)
d d
(x) = sin (y)
dx dx
dy
1 = cos (y) ·
dx
cos · dy
(y)
1
dx
=
cos (y) cos
(y)
dy 1
= (1)
dx cos (y)
Copyright © 2024 Md. Shouvik Iqbal. This book is licensed under a Creative Commons Attribution-NonCommercial-ShareAlike 4.0 International License (CC BY-NC-SA 4.0)
Download for free at https://md-shouvik-iqbal.github.io/calculus
Chapter 3 Differentiation Rules for Inverse Trigonometric Functions Page 207
Now, from the Pythagorean identity, it is known that sin2 (θ) + cos2 (θ) = 1.
Letting θ = y yields,
sin2 (y) + cos2 (y) = 1
cos2 (y) = 1 − sin2 (y)
q
cos(y) = 1 − sin2 (y)
Now, continuing with equation (1),
d 1
(y) =
dx cos (y)
1
=p
1 − sin2 (y)
1
=√
1 − x2
d 1
sin−1 (x) = √
dx 1 − x2
1
Therefore, the derivative of f (x) = sin−1 (x) is f 0 (x) = √ .
1 − x2
Copyright © 2024 Md. Shouvik Iqbal. This book is licensed under a Creative Commons Attribution-NonCommercial-ShareAlike 4.0 International License (CC BY-NC-SA 4.0)
Download for free at https://md-shouvik-iqbal.github.io/calculus
Page 208 Differentiation Rules for Inverse Trigonometric Functions Chapter 3
sin (x)
I Example 3.8.2. Differentiate f (x) =
sin−1 (x)
Solution. Applying the Quotient Rule, we write,
sin (x)
dy d
=
dx dx sin−1 (x)
d d
sin−1 (x) · sin (x) − sin (x) · sin−1 (x)
= dx dx
sin (x)
−1
2
1
sin−1 (x) · cos (x) − sin (x) · √
1 − x2
=
sin−1 (x)
2
sin (x)
cos (x) sin−1 (x) − √
1 − x2
=
sin−1 (x)
2
√
1 − x2 cos (x) sin−1 (x) sin (x)
√ −√
1 − x2 1 − x2
=
sin−1 (x)
2
√
1 − x2 cos (x) sin−1 (x) − sin (x)
√
1 − x2
=
sin−1 (x)
2
√
1 − x2 cos (x) sin−1 (x) − sin (x)
= √
1 − x2 sin−1 (x)
2
√
1 − x2 cos (x) sin−1 (x) − sin (x)
Therefore, f (x) = 0
√ is the derivative of
1 − x2 sin−1 (x)
2
sin (x)
f (x) =
sin−1 (x)
x = cos (y)
dx d
= cos (y)
dx dx
Copyright © 2024 Md. Shouvik Iqbal. This book is licensed under a Creative Commons Attribution-NonCommercial-ShareAlike 4.0 International License (CC BY-NC-SA 4.0)
Download for free at https://md-shouvik-iqbal.github.io/calculus
Chapter 3 Differentiation Rules for Inverse Trigonometric Functions Page 209
dy
1 = − sin (y) ·
dx
−sin · dy
(y)
1
dx
=
− sin (y) − sin (y)
dy 1
= (2)
dx − sin (y)
Now, from the Pythagorean identity, it is known that sin2 (θ) + cos2 (θ) = 1.
Letting θ = y yields,
Theorem 3.8.2. (The Derivative of the Cosine Function) If f (x) = cos−1 (x),
then
d 1
cos−1 (x) = − √
dx 1 − x2
for −1 < x < 1
Copyright © 2024 Md. Shouvik Iqbal. This book is licensed under a Creative Commons Attribution-NonCommercial-ShareAlike 4.0 International License (CC BY-NC-SA 4.0)
Download for free at https://md-shouvik-iqbal.github.io/calculus
Page 210 Differentiation Rules for Inverse Trigonometric Functions Chapter 3
1
= −√ · 2x
1 − x4
2x
= −√
1 − x4
2x
Therefore, the derivative of f (x) = cos−1 x2 + 1 is f 0 (x) = − √ .
1 − x4
= −1
x = tan (y)
d d
(x) = tan (y)
dx dx
dy
1 = sec2 (y) ·
dx
dy
1 sec
2(y) ·
dx
=
sec2 (y) sec
2(y)
Copyright © 2024 Md. Shouvik Iqbal. This book is licensed under a Creative Commons Attribution-NonCommercial-ShareAlike 4.0 International License (CC BY-NC-SA 4.0)
Download for free at https://md-shouvik-iqbal.github.io/calculus
Chapter 3 Differentiation Rules for Inverse Trigonometric Functions Page 211
dy 1
= (3)
dx sec (y)
2
Now, from the Pythagorean identity, we know that tan2 (θ) + 1 = sec2 (θ).
Letting θ = y yields,
sec2 (y) = 1 + tan2 (y)
d 1
(y) =
dx sec (y)
2
1
=
1 + tan2 (y)
d 1
tan−1 (x) =
dx 1 + x2
1
Therefore, the derivative of f (x) = tan−1 (x) is f 0 (x) =
1 + x2
Copyright © 2024 Md. Shouvik Iqbal. This book is licensed under a Creative Commons Attribution-NonCommercial-ShareAlike 4.0 International License (CC BY-NC-SA 4.0)
Download for free at https://md-shouvik-iqbal.github.io/calculus
Page 212 Differentiation Rules for Inverse Trigonometric Functions Chapter 3
1
= · sec2 (x)
1 + tan2 (x)
sec2 (x)
=
1 + tan2 (x)
sec
2 (x)
=
sec
2 (x)
=1
d d
(x) = cot (y)
dx dx
dy
1 = −csc2 (y) ·
dx
dy
2
−csc (y) ·
1
dx
2
= 2
−csc (y) −csc
(y)
dy 1
=− 2 (4)
dx csc (y)
Now, from the Pythagorean identity, we know that 1+cot2 (θ) = csc2 (θ). Letting
θ = y yields,
d 1
(y) = − 2
dx csc (y)
Copyright © 2024 Md. Shouvik Iqbal. This book is licensed under a Creative Commons Attribution-NonCommercial-ShareAlike 4.0 International License (CC BY-NC-SA 4.0)
Download for free at https://md-shouvik-iqbal.github.io/calculus
Chapter 3 Differentiation Rules for Inverse Trigonometric Functions Page 213
d 1
cot−1 (x) = −
dx 1 + x2
1
Therefore, the derivative of f (x) = cot−1 (x) is f 0 (x) = −
1 + x2
d 1
cot−1 (x) = −
dx 1 + x2
for −∞ < x < ∞
Copyright © 2024 Md. Shouvik Iqbal. This book is licensed under a Creative Commons Attribution-NonCommercial-ShareAlike 4.0 International License (CC BY-NC-SA 4.0)
Download for free at https://md-shouvik-iqbal.github.io/calculus
Page 214 Differentiation Rules for Inverse Trigonometric Functions Chapter 3
tan (y) = ± x2 − 1
p
Copyright © 2024 Md. Shouvik Iqbal. This book is licensed under a Creative Commons Attribution-NonCommercial-ShareAlike 4.0 International License (CC BY-NC-SA 4.0)
Download for free at https://md-shouvik-iqbal.github.io/calculus
Chapter 3 Differentiation Rules for Inverse Trigonometric Functions Page 215
The derivative of y = sec−1 (x) above is quite confusing. However, we can simply
get around this confusion by using the absolute value function. That is, if we let
|x| > 1, then we can simplify the derivative of inverse secant as the following.
d 1
sec−1 (x) = √
dx |x| x2 − 1
1
Therefore, the derivative of f (x) = sec−1 (x) is f 0 (x) = √ , where
|x| x2 − 1
|x| > 1.
exe−1
=q √
(xe )2 x2e − 1
Copyright © 2024 Md. Shouvik Iqbal. This book is licensed under a Creative Commons Attribution-NonCommercial-ShareAlike 4.0 International License (CC BY-NC-SA 4.0)
Download for free at https://md-shouvik-iqbal.github.io/calculus
Page 216 Differentiation Rules for Inverse Trigonometric Functions Chapter 3
e
= √
2e
x x −1
e
Therefore, the derivative of f (x) = sec−1 (xe ) is f 0 (x) = √ .
x x2e − 1
csc(y) = x
1
=x
sin(y)
1
sin(y) =
x
Now, differentiating both sides of the equation above implicitly, we get,
d d 1
sin(y) =
dx dx x
dy 1
cos(y) · =− 2
dx x
dy 1
=− 2 (1)
dx x cos(y)
Now, from the Pythagorean identity, we know that sin2 (θ)+cos2 (θ) = 1. Letting
1
θ = y and sin(y) = yields,
x
sin2 (y) + cos2 (y) = 1
2
1
+ cos2 (y) = 1
x
1
cos2 (y) = 1 −
x2
2
x −1
= 2
rx
x2 − 1
cos(y) = 2
√ x
x2 − 1
= √
2
√ x
x2 − 1
=
|x|
Copyright © 2024 Md. Shouvik Iqbal. This book is licensed under a Creative Commons Attribution-NonCommercial-ShareAlike 4.0 International License (CC BY-NC-SA 4.0)
Download for free at https://md-shouvik-iqbal.github.io/calculus
Chapter 3 Differentiation Rules for Inverse Trigonometric Functions Page 217
√
x2 − 1
∴ cos(y) =
|x|
√
x2 − 1
Now, substituting cos(y) = into equation (1) yields the following,
|x|
dy 1
=− √
dx x2 − 1
x2 ·
|x|
1
= − 2√
x x2 − 1
|x|
|x|
=− √
x2 x2 − 1
|x| 1
Since = , therefore,
x2 |x|
dy |x|
=− √
dx x2 x2 − 1
1
=− √
|x| x2 − 1
1
Therefore, the derivative of f (x) = csc−1 (x) is f 0 (x) = − √
|x| x2 − 1
Copyright © 2024 Md. Shouvik Iqbal. This book is licensed under a Creative Commons Attribution-NonCommercial-ShareAlike 4.0 International License (CC BY-NC-SA 4.0)
Download for free at https://md-shouvik-iqbal.github.io/calculus
Page 218 Differentiation of General Inverse Functions Chapter 3
1
= ex + √
x x2 − 1
1
Therefore, the derivative of f (x) = ex − csc−1 (x) is f 0 (x) = ex + √ .
x x2 − 1
1
=√ q √ q
x (csc (x)) x − 1 (csc−1 (x))2 − 1
2 −1 2 2
1
= √ q
xcsc (x) x − 1 (csc−1 (x))2 − 1
−1 2
y = f (x)
= f f −1 (f (x))
| {z }
x
Copyright © 2024 Md. Shouvik Iqbal. This book is licensed under a Creative Commons Attribution-NonCommercial-ShareAlike 4.0 International License (CC BY-NC-SA 4.0)
Download for free at https://md-shouvik-iqbal.github.io/calculus
Chapter 3 Differentiation of General Inverse Functions Page 219
= f f −1 (f (x))
Now, differentiating both sides of the equation above, by the Chain Rule, with
respect to y; we get,
dy d
= f (f −1 (f (x)))
dy dy
d −1
1 = f 0 (f −1 (f (x))) · f (f (x))
dy
1 d −1
= f (f (x))
f 0 (f −1 (f (x))) dy
Since x = f −1 (f (x)), therefore,
d −1 1
f (f (x)) = 0 −1
dy f (f (f (x)))
dx 1
∴ = 0
dy f (x)
1
=
dy
dx
dx 1 dy
This implies that = , given that 6= 0. Therefore, we have the following
dy dy dx
dx
theorem.
dx 1
=
dy dy
dx
dy
given that = f 0 (x) 6= 0
dx
Copyright © 2024 Md. Shouvik Iqbal. This book is licensed under a Creative Commons Attribution-NonCommercial-ShareAlike 4.0 International License (CC BY-NC-SA 4.0)
Download for free at https://md-shouvik-iqbal.github.io/calculus
Page 220 Differentiation of General Inverse Functions Chapter 3
q
= 1 − sin2 (y)
p
= 1 − x2
Since,
dx 1
=
dy dy
dx
p 1
1 − x2 =
dy
dx
dy 1
∴ =√
dx 1 − x2
1
Therefore, sin−1 (x) = √ .
1 − x2
d
cos(y) = − sin(y)
dy
= − 1 − cos2 (y)
p
p
= − 1 − x2
Since,
dx 1
=
dy dy
dx
p 1
− 1 − x2 =
dy
dx
dy 1
∴ = −√
dx 1 − x2
1
Therefore, cos−1 (x) = − √ .
1 − x2
Copyright © 2024 Md. Shouvik Iqbal. This book is licensed under a Creative Commons Attribution-NonCommercial-ShareAlike 4.0 International License (CC BY-NC-SA 4.0)
Download for free at https://md-shouvik-iqbal.github.io/calculus
Chapter 3 Summary Page 221
§ 3.10 Summary
Having covered the derivative of necessary functions, a few of them are sum-
marized for a quick reference. Note that the derivatives summarized below are
absolutely crucial to have on your mind for a smooth understanding of the chap-
ters that follow. However, mindless memorization is strongly discouraged, even
though arguments for being helpful may be made. It is suggested to rather
play 3 with them until they’re unforgettable. Take it as learning to type on a
keyboard. While many of us with decent typing speeds may not have memorized
each individual location of each key, enough mistakes have been made over time
to recall where they are efficiently.
3
for example, ask why the derivative of sin(x) cannot be equal to itself, while the derivative
of e can be; or notice the pattern that the derivative of sin−1 (x) and cos−1 (x), tan−1 (x) and
x
cot−1 (x), sec−1 (x) and csc−1 (x) are almost the same except for a sign in front of them; etc.
Copyright © 2024 Md. Shouvik Iqbal. This book is licensed under a Creative Commons Attribution-NonCommercial-ShareAlike 4.0 International License (CC BY-NC-SA 4.0)
Download for free at https://md-shouvik-iqbal.github.io/calculus
Page 222 Summary Chapter 3
d d
1. (c) = 0 12. cos(x) = − sin(x)
dx dx
dx d
2. =1 13. tan(x) = sec2 (x)
dx dx
d n d
3. x = nxn−1 14. cot(x) = − csc2 (x)
dx dx
n+1 d
4.
d x
= xn 15. sec(x) = sec(x) tan(x)
dx n + 1 dx
d
d√ 1 16. csc(x) = − csc(x) cot(x)
5. x= √ dx
dx 2 x
d 1
d x 17. sin−1 (x) = √
6. e = ex dx 1 − x2
dx
d 1
d −x 18. cos−1 (x) = − √
7. e = −e−x dx 1 − x2
dx
d 1
8.
d x
a = ax · ln(a) 19. tan−1 (x) =
dx dx 1 + x2
d 1
9.
d
ln(x) =
d
loge (x) =
1 20. cot−1 (x) = −
dx dx x dx 1 + x2
d 1
10.
d
loga (x) =
1 21. sec−1 (x) = √
dx x ln (a) dx |x| x2 − 1
d 1
11.
d
sin(x) = cos(x) 22. csc−1 (x) = − √
dx dx |x| x2 − 1
Copyright © 2024 Md. Shouvik Iqbal. This book is licensed under a Creative Commons Attribution-NonCommercial-ShareAlike 4.0 International License (CC BY-NC-SA 4.0)
Download for free at https://md-shouvik-iqbal.github.io/calculus
Chapter 4
Applications of Derivatives
In the previous chapters, we discussed the concept of derivatives and their deriva-
tion. Now, the focus turns to discussing how these ideas can be applied in
practice. The aim is to explore how derivatives can serve as more than just a
computational tool and to reveal themselves as the key to solve and understand
a wide range of mathematical questions.
223
Copyright © 2024 Md. Shouvik Iqbal. This book is licensed under a Creative Commons Attribution-NonCommercial-ShareAlike 4.0 International License (CC BY-NC-SA 4.0)
Download for free at https://md-shouvik-iqbal.github.io/calculus
Page 224 Extreme Values of a Function Chapter 4
Copyright © 2024 Md. Shouvik Iqbal. This book is licensed under a Creative Commons Attribution-NonCommercial-ShareAlike 4.0 International License (CC BY-NC-SA 4.0)
Download for free at https://md-shouvik-iqbal.github.io/calculus
Chapter 4 Extreme Values of a Function Page 225
f(x)
Figure 4.1
However, the same cannot be said for the interval (0, a]. Over the interval (0, a],
the function f (x) = x2 has an absolute maximum value of f (a) at the point
x = a, but no absolute minimum value, as shown below.
f(x)
x
a
Figure 4.2
Similarly, over the closed interval [0, a], the function f (x) = x2 has both absolute
maximum and minimum values of f (0) and f (a) at the points x = 0 and x = a,
respectively. This is shown below.
Copyright © 2024 Md. Shouvik Iqbal. This book is licensed under a Creative Commons Attribution-NonCommercial-ShareAlike 4.0 International License (CC BY-NC-SA 4.0)
Download for free at https://md-shouvik-iqbal.github.io/calculus
Page 226 Extreme Values of a Function Chapter 4
f(x)
x
a
Figure 4.3
Likewise, over the open interval (0, a), the function f (x) = x2 neither has an
absolute maximum nor minimum value, as shown below.
f(x)
x
a
Figure 4.4
This suggests that in order for a function to have any absolute maximum or min-
imum values over an interval I, the interval I must be bounded, provided that
the function is continuous over I. As a result, we have the following theorem.
Copyright © 2024 Md. Shouvik Iqbal. This book is licensed under a Creative Commons Attribution-NonCommercial-ShareAlike 4.0 International License (CC BY-NC-SA 4.0)
Download for free at https://md-shouvik-iqbal.github.io/calculus
Chapter 4 Extreme Values of a Function Page 227
Then, there exists an absolute maximum and minimum value of the func-
tion f (x) over the interval I.
The theorem above is the existence theorem, which provides conditions for abso-
lute maximum or minimum values of a function. However, it does not provide a
method for finding these values. To determine how to find them, it is necessary
to understand the concept of “critical points,” which in turn requires knowledge
of the relative extreme values of a function. As a result, we now discuss the
relative extreme values.
a b c d e
Figure 4.5
At which points does the function attain its absolute maximum and minimum
values? Clearly, the function attains its absolute maximum at x = b and its
absolute minimum at x = e. However, at the points c and d, the function also
shows some characteristics of being a maximum or minimum. Specifically, at
these points, the function is a maximum or minimum relative to the neighboring
points. Thus, at the point d, the function has a relative maximum value. Simi-
larly, at the point c, the function has a relative minimum value.
Copyright © 2024 Md. Shouvik Iqbal. This book is licensed under a Creative Commons Attribution-NonCommercial-ShareAlike 4.0 International License (CC BY-NC-SA 4.0)
Download for free at https://md-shouvik-iqbal.github.io/calculus
Page 228 Extreme Values of a Function Chapter 4
1. If f (c) ≥ f (x) for all x near c, then f (c) is the relative maximum of
f (x) over I.
2. If f (c) ≤ f (x) for all x near c, then f (c) is the relative minimum of
f (x) over I.
Now that we have discussed the relative extreme values, let’s now move on to
critical points. These points are essential for finding the absolute extreme values
of a function.
a b c d e
Figure 4.6
At which points does the function attain its relative maximum and minimum
values? As before, it is visible that the function has a relative maximum value
at the points x = b and x = d. Likewise, it has a relative minimum value at
the points x = c. Now, notice how the function only has its relative maximum
and minimum values at points where the derivative is either zero (at the point
x = b) or fails to exist (at the point x = d).
Copyright © 2024 Md. Shouvik Iqbal. This book is licensed under a Creative Commons Attribution-NonCommercial-ShareAlike 4.0 International License (CC BY-NC-SA 4.0)
Download for free at https://md-shouvik-iqbal.github.io/calculus
Chapter 4 Extreme Values of a Function Page 229
This implies, from the definition of relative maximum, that for x near c, f (c) ≥
f (x) or f (x) − f (c) ≤ 0. Now,
Copyright © 2024 Md. Shouvik Iqbal. This book is licensed under a Creative Commons Attribution-NonCommercial-ShareAlike 4.0 International License (CC BY-NC-SA 4.0)
Download for free at https://md-shouvik-iqbal.github.io/calculus
Page 230 Extreme Values of a Function Chapter 4
Therefore,
negative or zero
z }| {
f (x) − f (c)
≥0
x − c
| {z }
negative
negative or zero
z }| {
f (x) − f (c)
lim ≥0
x→c− x − c
| {z }
negative
f (x) − f (c)
This implies that the limit f 0 (c) = lim must be 0.
x→c x−c
This implies, from the definition of relative minimum, that for x near c, f (c) ≤
f (x) or 0 ≤ f (x) − f (c). Now,
Copyright © 2024 Md. Shouvik Iqbal. This book is licensed under a Creative Commons Attribution-NonCommercial-ShareAlike 4.0 International License (CC BY-NC-SA 4.0)
Download for free at https://md-shouvik-iqbal.github.io/calculus
Chapter 4 Extreme Values of a Function Page 231
f (x) − f (c)
This again implies that the limit f 0 (c) = lim must be 0. This
x→c x−c
completes the proof.
The relative maximum or minimum values of a function can occur not only at
points where f 0 (c) = 0, but also at points where the derivative does not exist.
Taking these conditions into account, we define such points as “Critical Points.”
1. f 0 (c) = 0
2. f 0 (c) = undefined
The definition above implies that if a function has a relative maximum or min-
imum value at a point c, then c must be a Critical Point. However, a Critical
Point at c does not guarantee a relative maximum or minimum value at that
point.
This is because it is possible that f 0 (c) = 0, but there are no relative maximum
or minimum values of the function at that point, as shown below.
f(x)
x
a
Figure 4.7
Similarly, it is possible that f 0 (c) = undefined, yet there are no relative maximum
or minimum values of the function at that point, as shown below.
Copyright © 2024 Md. Shouvik Iqbal. This book is licensed under a Creative Commons Attribution-NonCommercial-ShareAlike 4.0 International License (CC BY-NC-SA 4.0)
Download for free at https://md-shouvik-iqbal.github.io/calculus
Page 232 Extreme Values of a Function Chapter 4
f(x)
x
a
Figure 4.8
This observation indicates that Critical Points are not necessarily where the rel-
ative maximum or minimum values of a function occur; rather, they are potential
locations—a candidate—where these extrema might be found.
dy d
10x2 − 5x2
=
dx dx
= 20x − 5
f 0 (x) = 0
20x − 5 = 0
5
x=
20
5
Therefore, the only Critical Point of the function f (x) = 10x2 −5x2 is x =
20
Copyright © 2024 Md. Shouvik Iqbal. This book is licensed under a Creative Commons Attribution-NonCommercial-ShareAlike 4.0 International License (CC BY-NC-SA 4.0)
Download for free at https://md-shouvik-iqbal.github.io/calculus
Chapter 4 Extreme Values of a Function Page 233
f 0 (x) = 0
3x2 − 1 = 0
r
1
x=±
3
r
1
Therefore, the Critical Points of the function f (x) = x − x are x = + 3
and
r 3
1
x=±− , respectively.
3
Having discussed Critical Points and their identification, we now return to the
topic that was left unexplored: locating absolute extreme values.
Chap. 4 / Sec. 4.1 / Subsec. 4.1.4 : Location of Absolute (Global) Extreme Val-
ues
The Extreme Value Theorem guarantees the existence of absolute maximum or
minimum values under certain conditions. However, it does not provide any
method to determine them. Nevertheless, with an understanding of relative
extreme values, the following assumptions can be made.
Recall that the relative extreme values occur at the Critical Points. Thus, with
the knowledge of locating such Critical Points, the absolute extreme values can
now be located with almost no effort.
As a result, the assumption above can be turned into a general guideline for
locating absolute extreme values as follows.
Copyright © 2024 Md. Shouvik Iqbal. This book is licensed under a Creative Commons Attribution-NonCommercial-ShareAlike 4.0 International License (CC BY-NC-SA 4.0)
Download for free at https://md-shouvik-iqbal.github.io/calculus
Page 234 Extreme Values of a Function Chapter 4
Let f (x) be a function defined over a closed interval [a, b]. Then, to locate
the absolute maximum or minimum values of f (x), consider the following
steps:
4. The greatest of the evaluated values is the maximum and the smallest
of the evaluated values is the minimum value of f (x).
I Example 4.1.3. Find the absolute maximum and minimum values of the func-
tion f (x) = x4 − 2x over [−2, 2]
Solution. In order to find the absolute maximum and minimum values of the
function f (x) = x4 −2x over [−2, 2], the Critical Points must first be determined.
In order to determine the Critical Points, the function f (x) = x4 − 2x needs to
be differentiated as follows.
d
f 0 (x) = x4 − 2x
dx
= 4x3 − 2
Now, the derivative f 0 (x) = 4x3 − 2 is defined for all x ∈ R. Therefore, to find
the Critical Points, let f 0 (x) = 0 as follows,
f 0 (x) = 0
4x3 − 2 = 0
r
3 1
x=
2
r
1
Therefore, the Critical Point of the function f (x) = x4 − 2x is x = 3
.
2
This Critical Point, along with the endpoints [−2, 2] are the possible points
where the function has its absolute maximum and minimum values. Therefore,
solving the function at those points yields,
Copyright © 2024 Md. Shouvik Iqbal. This book is licensed under a Creative Commons Attribution-NonCommercial-ShareAlike 4.0 International License (CC BY-NC-SA 4.0)
Download for free at https://md-shouvik-iqbal.github.io/calculus
Chapter 4 Extreme Values of a Function Page 235
Comparing these values, it is evident that −1.191 < 12 < 20. Thus, the func-
tion attains its maximum value at x = 20 and minimum value at x ≈ −1.191.
I Example 4.1.4. Find the absolute maximum and minimum values of the func-
tion f (x) = 2x 3 − 4x over [−1, 1]
2
0 d 2
f (x) = 2x − 4x
3
dx
2 1
= 2 · x− 3 − 4
3
4 1
= · 1 −4
3 x3
4
= 1 − 4
3x 3 !
1
1 − 3x 3
=4 1
3x 3
Now, when x = 0.037, the function f (x) has a Critical Point. Because at
x = 0.037 the derivative of f (x) yields f 0 (0.037) = 0. Similarly, when x = 0,
the function f (x) has a Critical Point. Because at x = 0, the derivative of f (x)
yields f 0 (0) = division by zero = undefined. Therefore, the Critical Points of
the function f (x) = 2x 3 − 4x are x = 0, 0.037.
2
The Critical Points along with the endpoints [−1, 1] are the possible points for
maximum and minimum values. Therefore, solving the function at those points
yields,
Copyright © 2024 Md. Shouvik Iqbal. This book is licensed under a Creative Commons Attribution-NonCommercial-ShareAlike 4.0 International License (CC BY-NC-SA 4.0)
Download for free at https://md-shouvik-iqbal.github.io/calculus
Page 236 Increasing and Decreasing Functions Chapter 4
By comparing these values, it is evident that −2 < 0 < 0.074 < 6. There-
fore, the function attains its maximum value at x = −1 and minimum value at
x = 1.
a b c d
Figure 4.9
In the figure above, the function is increasing over the interval [a, b], constant
over [b, c], and then decreasing over [c, d]. That is, when x increases over the
interval [a, b], the function moves up. Over the interval [b, c], the function re-
mains the same. And lastly, it moves down, as x increases over the interval [c, d].
Copyright © 2024 Md. Shouvik Iqbal. This book is licensed under a Creative Commons Attribution-NonCommercial-ShareAlike 4.0 International License (CC BY-NC-SA 4.0)
Download for free at https://md-shouvik-iqbal.github.io/calculus
Chapter 4 Increasing and Decreasing Functions Page 237
Figure 4.10
Copyright © 2024 Md. Shouvik Iqbal. This book is licensed under a Creative Commons Attribution-NonCommercial-ShareAlike 4.0 International License (CC BY-NC-SA 4.0)
Download for free at https://md-shouvik-iqbal.github.io/calculus
Page 238 Increasing and Decreasing Functions Chapter 4
Figure 4.11
Figure 4.12
Likewise, when the derivative is zero, the tangent line on that interval represents
a horizontal line, suggesting the function is constant on that interval.
With this intuition, we have the following theorem to identify intervals on which
a function is increasing, decreasing, or constant.
Copyright © 2024 Md. Shouvik Iqbal. This book is licensed under a Creative Commons Attribution-NonCommercial-ShareAlike 4.0 International License (CC BY-NC-SA 4.0)
Download for free at https://md-shouvik-iqbal.github.io/calculus
Chapter 4 Increasing and Decreasing Functions Page 239
1. If f 0 (x) > 0 for all x ∈ (a, b), then f (x) is increasing over [a, b].
2. If f 0 (x) < 0 for all x ∈ (a, b), then f (x) is decreasing over [a, b].
3. If f 0 (x) = 0 for all x ∈ (a, b), then f (x) is constant over [a, b].
10
I Example 4.2.1. Find the intervals on which the function f (x) = x3 − 10x is
3
increasing and decreasing.
Solution. It is known from the theorem above that a function f (x) is increasing
if its derivative f 0 (x) is greater than 0, and decreasing if its derivative f 0 (x)
is less than 0. Therefore, the first task would be to differentiate the function
10
f (x) = x3 − 10x, as follows.
3
0 d 10 3
f (x) = x − 10x
dx 3
d 10 3 d
= x − (10x)
dx 3 dx
10
= ·3x2 − 10
3
= 10x2 − 10
= 10 x2 − 1
f 0 (x) = 0
10 x2 − 1 = 0
x2 − 1 = 0
x2 = 1
√
x=± 1
x = ±1
The Critical Points are x = 1 and x = −1. The Critical Points divide the real
line into the following intervals:
Copyright © 2024 Md. Shouvik Iqbal. This book is licensed under a Creative Commons Attribution-NonCommercial-ShareAlike 4.0 International License (CC BY-NC-SA 4.0)
Download for free at https://md-shouvik-iqbal.github.io/calculus
Page 240 Increasing and Decreasing Functions Chapter 4
Now, for the first interval, choose a value of x such that it lies within that
interval. Say, x = −2 ∈ (−∞, −1) is chosen as a test value. Now, evaluate
f 0 (x) = 10 x2 − 1 for x = −2. If the result yields a positive value, then f (x)
Since f 0 (x) > 0 over the interval (−∞, −1), therefore f (x) is increasing over
(−∞, −1).
Similarly, let x = 0 ∈ (−1, 1) as a test value for the second interval. Then,
f 0 (0) = 10 02 − 1
= −10 < 0
Since f 0 (x) < 0 over the interval (−1, 1), therefore f (x) is decreasing over (−1, 1).
Likewise, let x = 2 ∈ (1, ∞) as a test value for the third interval. Then,
f 0 (2) = 10 22 − 1
= 30 > 0
Since f 0 (x) > 0 over the interval (1, ∞), therefore f (x) is increasing over (1, ∞).
In conclusion, the function f (x) is increasing on (−∞, −1) and (1, ∞), and de-
creasing on (−1, 1).
Copyright © 2024 Md. Shouvik Iqbal. This book is licensed under a Creative Commons Attribution-NonCommercial-ShareAlike 4.0 International License (CC BY-NC-SA 4.0)
Download for free at https://md-shouvik-iqbal.github.io/calculus
Chapter 4 Increasing and Decreasing Functions Page 241
f(x)
f(c)
x
a c b
Figure 4.13
In the graph above, the function f (x) has a Critical Point at x = c, as f 0 (c) = 0.
Now, observe that f (x) is increasing over the interval (a, c) and decreasing over
the interval (c, b). Which suggests that f 0 (x) > 0 over the interval (a, c) and
f 0 (x) < 0 over (c, b). The derivative f 0 (x) changes its sign from positive to
negative at the Critical Point x = c, where f 0 (c) = 0. As a result, the function
f 0 (x) has a relative maximum at the Critical Point x = c.
Similarly, for the relative minimum values, consider the following graph of a
function.
Copyright © 2024 Md. Shouvik Iqbal. This book is licensed under a Creative Commons Attribution-NonCommercial-ShareAlike 4.0 International License (CC BY-NC-SA 4.0)
Download for free at https://md-shouvik-iqbal.github.io/calculus
Page 242 Increasing and Decreasing Functions Chapter 4
f(x)
f(c)
x
a c b
Figure 4.14
Notice that f (x) is decreasing over the interval (a, c) and increasing over the
interval (c, b). This suggests that f 0 (x) < 0 over the interval (a, c) and f 0 (x) > 0
over (c, b). The derivative f 0 (x) changes its sign from negative to positive at
the Critical Point x = c, where f 0 (c) = 0. As a result, the function f 0 (x) has a
relative minimum at the Critical Point x = c.
From the observation above, it can be noted that when a function f (x) has a
relative maximum or minimum value at a point, the derivative f 0 (x) changes its
sign at that point. This idea is crucial for identifying the relative maximum or
minimum values of a function.
Recall that Critical Points are points where a function might have a relative
maximum or minimum, but is not guaranteed. The derivative f 0 (x) does not
always change sign at these points. Therefore, examining whether the derivative
changes sign at Critical Points can serve as a test to identify if they correspond
to any relative maximum or minimum values of the function. Thus, we have the
following theorem to identify relative extreme values of a function.
Copyright © 2024 Md. Shouvik Iqbal. This book is licensed under a Creative Commons Attribution-NonCommercial-ShareAlike 4.0 International License (CC BY-NC-SA 4.0)
Download for free at https://md-shouvik-iqbal.github.io/calculus
Chapter 4 Increasing and Decreasing Functions Page 243
3. If the sign of f 0 (x) remains the same from both sides of c, then f (c)
is neither a relative maximum nor a relative minimum.
I Example 4.2.2. Identify the relative maximum and minimum values of the
10
function f (x) = x3 − 10x.
3
Solution. To identify the relative extreme values of the function, we need to know
the intervals on which the function is increasing and decreasing. From Exam-
10 3
ple 4.2.1, it was known that the function f (x) = x − 10x is increasing on
3
(−∞, −1), decreasing on (−1, 1), and then again increasing on (1, ∞).
In Example 4.2.1, it has been observed through the test points that f 0 (x) changes
its sign from positive to negative as x goes through the Critical Point x = −1.
Thus, f (x) has a relative maximum value at x = −1. Similarly, since f 0 (x)
changes its sign from negative to positive as x goes through the Critical Point
x = 1, therefore f (x) has a relative minimum value at x = 1.
I Example 4.2.3. Identify the relative maximum and minimum values of the
x2 + 4x x2 + 4x + 6
function f (x) =
4
Solution. To identify the relative extreme values of the function, the first step is
to find its derivative to locate the Critical Points. Then, by analyzing whether
the derivative changes sign at these Critical Points, the relative extreme values
can be determined. Thus, the derivative of f (x) is as follows,
2
2 !
d x + 4x x + 4x + 6
f 0 (x) =
dx 4
1 2
d 2
2
d 2
= · x + 4x · x + 4x + 6 + x + 4x + 6 · x + 4x
4 dx dx
1
= · x2 + 4x · (2x + 4) + x2 + 4x + 6 · (2x + 4)
4
Copyright © 2024 Md. Shouvik Iqbal. This book is licensed under a Creative Commons Attribution-NonCommercial-ShareAlike 4.0 International License (CC BY-NC-SA 4.0)
Download for free at https://md-shouvik-iqbal.github.io/calculus
Page 244 Increasing and Decreasing Functions Chapter 4
1
· 2x3 + 12x2 + 16x + 2x3 + 12x2 + 28x + 24
=
4
1
= · 4x3 + 24x2 + 44x + 24
4
= x3 + 6x2 + 11x + 6
= (x + 1) (x + 2) (x + 3)
Now, since f (x) is continuous at every point, therefore the f 0 (x) = 0 are the
only Critical Points. Letting f 0 (x) = 0 yields,
(x + 1) (x + 2) (x + 3) = 0
x = −1, −2, −3
These Critical Points divide the real number line into the following intervals.
Now, by the first derivative test, it is concludable that the function f (x) has a
relative minimum at x = −3, −1, as the derivative changed its sign from negative
to positive and relative maximum at x = −1, as the derivative changed its sign
from negative to positive.
Copyright © 2024 Md. Shouvik Iqbal. This book is licensed under a Creative Commons Attribution-NonCommercial-ShareAlike 4.0 International License (CC BY-NC-SA 4.0)
Download for free at https://md-shouvik-iqbal.github.io/calculus
Chapter 4 Concavity and the Second Derivative Test Page 245
Increasing
Concave Down
Increasing
Concave Up
a b a b
The graph of the function on the left is increasing with an increasing rate,
whereas the graph on the right is increasing with a decreasing rate. This is the
core idea behind what is referred to as the concavity of a function.
The concavity of a function is the direction in which the function curves. It can
curve upwards or downwards. For example, the graph in the left figure shown
above curves upwards, suggesting that the tangent lines to the curve get steeper
and steeper as x increases. This implies that the first derivative of the function
is increasing over that interval. When a function’s first derivative increases on
an interval, the function is said to be concave up within that interval.
Similarly, the graph in the right figure shown above curves downwards, suggest-
ing that the tangent lines to the curve get steeper and steeper as x increases.
This implies that the first derivative of the function is decreasing over that in-
terval. When a function’s first derivative decreases on an interval, the function
is said to be concave down within that interval.
Copyright © 2024 Md. Shouvik Iqbal. This book is licensed under a Creative Commons Attribution-NonCommercial-ShareAlike 4.0 International License (CC BY-NC-SA 4.0)
Download for free at https://md-shouvik-iqbal.github.io/calculus
Page 246 Concavity and the Second Derivative Test Chapter 4
a b a b
(a) (b)
a b a b
(c) (d)
Figure 4.17
In Figure 4.17(a), the function f (x) is increasing and concave up because its
first derivative, f 0 (x), is also increasing. In contrast, in Figure 4.17(b), although
the function f (x) is still increasing, but it is concave down, as the derivative
f 0 (x) is decreasing.
Copyright © 2024 Md. Shouvik Iqbal. This book is licensed under a Creative Commons Attribution-NonCommercial-ShareAlike 4.0 International License (CC BY-NC-SA 4.0)
Download for free at https://md-shouvik-iqbal.github.io/calculus
Chapter 4 Concavity and the Second Derivative Test Page 247
Similarly, in Figure 4.17(c), the function f (x) is decreasing and concave up since
f 0 (x) is increasing. In contrast, in Figure 4.17(d), the function f (x) is decreasing
and concave down because f 0 (x) is decreasing.
Based on the discussion above, we have the following test for the concavity of a
function as follows.
Theorem 4.3.1. (Concavity Test) Let f (x) be a function such that its second
derivative exists over an interval I. Then,
Copyright © 2024 Md. Shouvik Iqbal. This book is licensed under a Creative Commons Attribution-NonCommercial-ShareAlike 4.0 International License (CC BY-NC-SA 4.0)
Download for free at https://md-shouvik-iqbal.github.io/calculus
Page 248 Concavity and the Second Derivative Test Chapter 4
Figure 4.18
Inflection points are such points where a curve changes its concavity. That is, at
those points, the second derivative changes f 00 (x) of the function f (x) changes
its sign. As a result, we have the following definition.
Recall that to find the relative extreme values of a function, the first step was to
identify its Critical Points. Critical Points are points where the derivative f 0 (x)
equals zero or is undefined. These points act as potential candidates for relative
extreme values.
Copyright © 2024 Md. Shouvik Iqbal. This book is licensed under a Creative Commons Attribution-NonCommercial-ShareAlike 4.0 International License (CC BY-NC-SA 4.0)
Download for free at https://md-shouvik-iqbal.github.io/calculus
Chapter 4 Concavity and the Second Derivative Test Page 249
I Example 4.3.1. Identify the point of inflection of the function f (x) = sin−1 (x).
Solution. To identify the point of inflection of the function f (x) = sin−1 (x),
the first step is to compute the second derivative f 00 (x) of the function f (x).
Therefore,
d
f 0 (x) = sin−1 (x)
dx
1
=√
1 − x2
d 1
f 00 (x) = √
dx 1 − x2
d 1
=
dx (1 − x2 ) 12
d − 1
= 1 − x2 2
dx
1 − 3 d
= − 1 − x2 2 · 1 − x2
2 dx
1 − 3
= − 1 − x2 2 · (−2x)
2
1 − 2x
=−
2 (1 − x2 ) 32
x
= 3
(1 − x2 ) 2
Now, to find the inflection points, let f 00 (x) = 0. That is,
f 00 (x) = 0
x
3 = 0
(1 − x2 ) 2
32
x = 0 · 1 − x2
x=0
This suggests that f 00 (x) changes its sign from negative to positive at x = 0.
x
This is because denominator of f 00 (x) = 3 is positive on the entire inter-
(1 − x2 ) 2
val (−1, 1), however, the numerator is negative on (−1, 0) and positive on (0, 1),
Copyright © 2024 Md. Shouvik Iqbal. This book is licensed under a Creative Commons Attribution-NonCommercial-ShareAlike 4.0 International License (CC BY-NC-SA 4.0)
Download for free at https://md-shouvik-iqbal.github.io/calculus
Page 250 Rolle's Theorem and The Mean Value Theorem Chapter 4
x
making f 00 (x) = 3 negative on (−1, 0) and positive on (0, 1). Therefore,
(1 − x2 ) 2
f (x) is concave down on (−1, 0) and concave up on (0, 1), making x = 0 is a
inflection point of f (x).
Copyright © 2024 Md. Shouvik Iqbal. This book is licensed under a Creative Commons Attribution-NonCommercial-ShareAlike 4.0 International License (CC BY-NC-SA 4.0)
Download for free at https://md-shouvik-iqbal.github.io/calculus
Chapter 4 Rolle's Theorem and The Mean Value Theorem Page 251
interval (a, b) with f (a) = f (b). Then there exists a point c between a and b
such that the derivative at c is 0. That is, the slope of the tangent line at that
point is 0, as shown below.
f(x)
f(c)
x
a c b
Figure 4.19
The basic idea is that if a smooth curve starts and ends at the same height (that
is, if f (a) = f (b)), it must have at least one point where the slope of the curve
flattens out (i.e., the derivative becomes zero). This occurs because, intuitively,
the function has to “turn around” somewhere between a and b to return to the
same value at both endpoints. Therefore, we have the following theorem.
Theorem 4.4.1. (Rolle’s Theorem) Let f (x) be a function meeting the fol-
lowing conditions,
3. f (a) = f (b)
then there must exist at least one point c ∈ (a, b) such that f 0 (c) = 0.
Copyright © 2024 Md. Shouvik Iqbal. This book is licensed under a Creative Commons Attribution-NonCommercial-ShareAlike 4.0 International License (CC BY-NC-SA 4.0)
Download for free at https://md-shouvik-iqbal.github.io/calculus
Page 252 Rolle's Theorem and The Mean Value Theorem Chapter 4
Case 1: Suppose, f (x) = k, for all x ∈ (a, b), then f 0 (x) = 0 for all x ∈ (a, b), as
f (x) is a constant function.
Case 2: Suppose, f (x) > k, for some x ∈ (a, b). Since f (x) is continuous over
[a, b], therefore, by the Extreme Value Theorem, it has an absolute maximum
over [a, b]. Given that f (a) = f (b), the absolute maximum cannot occur at the
endpoints a or b. Thus, the absolute maximum must occur at some interior
point c in (a, b).
Since all absolute maximums are also relative maximums, therefore f (x) must
have a relative maximum at some point c ∈ (a, b). Now, according to the Rela-
tive/Local Extreme Value Theorem, if f (x) has a relative maximum at c and is
differentiable at c, then f 0 (c) = 0.
Case 3: Suppose, f (x) < k, for some x ∈ (a, b). Since f (x) is continuous over
[a, b], therefore, by the Extreme Value Theorem, it has an absolute minimum
over [a, b]. Given that f (a) = f (b), the absolute minimum cannot occur at the
endpoints a or b. Thus, the absolute maximum must occur at some interior
point c in (a, b).
Since all absolute minimums are also relative minimums, therefore f (x) must
have a relative minimum at some point c ∈ (a, b). Now, according to the Rela-
tive/Local Extreme Value Theorem, if f (x) has a relative minimum at c and is
differentiable at c, then f 0 (c) = 0.
The theorem above can be applied in certain situations very cleverly. For in-
stance, consider the following example.
I Example 4.4.1. Show that f (x) = x3 + x + 1 has exactly one real solution.
Solution. Notice that the value of the function f (x) = x3 + x + 1 is −5 and 1
for x = −2 and x = 0, respectively. Therefore, it follows from the Intermediate
Value Theorem (IVT) that there exists a point k between x = −2 and x = 0
such that f (k) = 0.
Now, besides of that single point k, if there were two points x = a and x = b
for which f (a) = f (b) = 0, then the Rolle’s theorem would guarantee that there
exists a point x = c between the points x = a and x = b for which f 0 (c) = 0.
However, this can never happen as the derivative of f (x) = x3 + x + 1 is,
d d
x3 + x + 1
f (x) =
dx dx
Copyright © 2024 Md. Shouvik Iqbal. This book is licensed under a Creative Commons Attribution-NonCommercial-ShareAlike 4.0 International License (CC BY-NC-SA 4.0)
Download for free at https://md-shouvik-iqbal.github.io/calculus
Chapter 4 Rolle's Theorem and The Mean Value Theorem Page 253
= 3x2 + 1 > 0
Which is always greater than 0 for all x. Therefore, Rolle’s theorem cannot be
met and there doesn’t exist any point other than one single x = k, for which
f (k) = 0.
Chap. 4 / Sec. 4.4 / Subsec. 4.4.2 : The Mean Value Theorem (MVT)
The Mean Value Theorem can best be understood visually. For example, con-
sider the figure below.
f(x)
f(c)
f(b)
f(a)
x
a c b
Figure 4.20
In the figure above, the slope of the secant line that passes through the point
(a, f (a)) and (b, f (b)) is
f (b) − f (a)
b−a
Now, the Mean Value Theorem essentially states that there exists at least one
point c ∈ (a, b) such that the slope of the tangent line at c is equal to the slope
of the secant line that passes through the points (a, f (a)) and (b, f (b)). That is,
f (b) − f (a)
f 0 (c) =
b−a
This implies that there is a point c where the tangent line at that point c
is parallel to the secant line that passes through the (a, f (a)) and (b, f (b)).
Therefore, we have the following theorem.
Copyright © 2024 Md. Shouvik Iqbal. This book is licensed under a Creative Commons Attribution-NonCommercial-ShareAlike 4.0 International License (CC BY-NC-SA 4.0)
Download for free at https://md-shouvik-iqbal.github.io/calculus
Page 254 Rolle's Theorem and The Mean Value Theorem Chapter 4
f (b) − f (a)
f 0 (c) =
b−a
or, equivalently,
f (b) − f (b) = f 0 (c)(b − a)
Proof. Consider a straight line that goes through the points (a, f (a)) and (b, f (b)).
The slope of this line is,
f (b) − f (a)
m=
b−a
f (b) − f (a)
Since, the line goes through the point (a, f (a)) and has a slope m = ,
b−a
therefore the point-slope equation of the line yields,
y − f (a) = m(x − a)
or,
f (b) − f (a)
(x − a) + f (a)
y=
b−a
Now, let a new function g(x) be the difference between the function f (x) and
the line y as follows,
g(x) = f (x) − y
f (b) − f (a)
= f (x) − (x − a) + f (a)
b−a
f (b) − f (a)
= f (x) − (x − a) − f (a)
b−a
Now, if x = a, then g(x) yields,
f (b) − f (a)
g(x) = f (x) − (x − a) − f (a)
b−a
f (b) − f (a) :0
= f (a) − (a−a) − f (a)
b−a
= f (a) − f (a)
Copyright © 2024 Md. Shouvik Iqbal. This book is licensed under a Creative Commons Attribution-NonCommercial-ShareAlike 4.0 International License (CC BY-NC-SA 4.0)
Download for free at https://md-shouvik-iqbal.github.io/calculus
Chapter 4 Rolle's Theorem and The Mean Value Theorem Page 255
=0
This implies that g(a) = 0 = g(b). Now, since f (x) and the straight line y
are a differentiable function over (a, b), therefore g(x) is also a differentiable
function over (a, b). Furthermore, since f (x) and y are a continuous function
over [a, b], therefore g(x) is also a continuous function over [a, b]. Thus, g(x)
satisfies Rolle’s Theorem implying that there exists a point c ∈ (a, b) for which
g 0 (c) = 0. Therefore, let us differentiate the function g(x),
d
g 0 (x) = g (x)
dx
d f (b) − f (a)
= f (x) − (x − a) − f (a)
dx b−a
d d f (b) − f (a) d
= f (x) − (x − a) − f (a)
dx dx b−a dx
f (b) − f (a) d
= f 0 (x) − · (x − a)
b−a dx
f (b) − f (a)
= f 0 (x) − ·1
b−a
f (b) − f (a)
∴ g 0 (x) = f 0 (x) −
b−a
Since, g(x) satisfies Rolle’s Theorem suggesting that there exists a point c ∈
(a, b) for which g 0 (c) = 0, therefore we let,
g 0 (c) = 0
f (b) − f (a)
f 0 (c) − =0
b−a
f (b) − f (a)
∴ f 0 (c) =
b−a
This completes the proof.
A few examples to demonstrate the theorem above are provided below.
Copyright © 2024 Md. Shouvik Iqbal. This book is licensed under a Creative Commons Attribution-NonCommercial-ShareAlike 4.0 International License (CC BY-NC-SA 4.0)
Download for free at https://md-shouvik-iqbal.github.io/calculus
Page 256 Rolle's Theorem and The Mean Value Theorem Chapter 4
I Example 4.4.2. Test the Mean Value Theorem for f (x) = x2 on the interval
[1, 4].
Now, the slope of the secant line that passes through the points x = 1 and x = 4
is,
f (4) − f (1) 16 − 1
=
4−1 4−1
15
=
3
=5
Now, to test the Mean Value Theorem, we need to find a point c such that
f 0 (c) = 5. Therefore, we differentiate f (x) = x2 as follows.
d 2
x = 2x
dx
Setting f 0 (c) = 5, we get,
2c = 5
5
∴c=
2
= 2.5
Thus, at x = 2.5, the function f (x) = x2 has a tangent line where its slope
matches the secant line that passes through the point x = 1 and x = 4.
I Example 4.4.3. Test the Mean Value Theorem for f (x) = sin(x) on the interval
[0, π].
The slope of the secant line passing through the points x = 0 and x = π is,
sin(π) − sin(0) 0−0
=
π−0 π
Copyright © 2024 Md. Shouvik Iqbal. This book is licensed under a Creative Commons Attribution-NonCommercial-ShareAlike 4.0 International License (CC BY-NC-SA 4.0)
Download for free at https://md-shouvik-iqbal.github.io/calculus
Chapter 4 Related Rates Page 257
=0
Now, to test the Mean Value Theorem, we need to find a point c such that
f 0 (c) = 0. Therefore,
d
sin(x) = cos(x)
dx
Settings f 0 (c) = 0, we get,
cos(c) = 0
∴ c = cos−1 (0)
π
=
2
π
Thus, at x = , the function f (x) = sin(x) has a tangent line where its slope
2
matches the secant line that passes through the points x = 0 and x = π.
y = 2x3
d d
2x3
(y) =
dt dt
dy dx
= 6x2 ·
dt dt
Now, since the rate of change of x with respect to t, when x = a, is n; therefore
dx
= n. Thus, the rate of change of y with respect to t, when x = a, is,
dt
dy
= 6a2 n
dt
Copyright © 2024 Md. Shouvik Iqbal. This book is licensed under a Creative Commons Attribution-NonCommercial-ShareAlike 4.0 International License (CC BY-NC-SA 4.0)
Download for free at https://md-shouvik-iqbal.github.io/calculus
Page 258 Related Rates Chapter 4
In the real world, there are numerous examples of related variables that changes
over time. For example, consider inflating a balloon. As the balloon gets inflated
with respect to time, it becomes evident that its size expands, indicating an
increase in its volume V . Now, assuming the shape of the inflated balloon is
4
spherical and knowing the formula for the volume of a sphere, that is, V = πr3 ,
3
it can be said that as the volume increases over time, so does the radius r. This
dV
implies that the rate of change in the volume, that is is related to the rate
dt
dr 4
of change in the radius since V is related to r by the equation V = πr3 .
dt 3
We shall use these relations to solve the following problem.
Solution. Let r be the radius of the circle and A be the area of the circle at a
dr cm dA
time t, then we’re given =2 . We need to find , when r = 100 m.
dt s dt
In order to do that notice first that the area of the circle A and the radius r of
the circle is related by the following equation.
A = πr2
Copyright © 2024 Md. Shouvik Iqbal. This book is licensed under a Creative Commons Attribution-NonCommercial-ShareAlike 4.0 International License (CC BY-NC-SA 4.0)
Download for free at https://md-shouvik-iqbal.github.io/calculus
Chapter 4 Related Rates Page 259
= 2π (100) (2)
cm2
≈ 1256.64
s
cm2
Therefore, the area of the circle is increasing at a rate of approximately 1256.64 ,
s
when the radius is 100 cm.
Solution. Let h be the height of the water, and V the volume of water in the
dV m3
tank. We are given = −3 (“−3” is due to the reason that the volume is
dt s
dh
decreasing) and asked to find when h = 5m.
dt
In order to do that notice that the volume V and the height h are related by
the following equation.
V = πr2 h
Since the radius r = 2m is constant, therefore, the equation above is simplified
as follows.
V = πr2 h
= π22 h
= 4πh
Now, differentiating both sides of the equation with respect to time t above
yields,
dV d
= (4πh)
dt dt
dh
= 4π
dt
dV m3
Since we are given = −3 , therefore,
dt s
dV dh
= 4π
dt dt
dh
−3 = 4π
dt
Copyright © 2024 Md. Shouvik Iqbal. This book is licensed under a Creative Commons Attribution-NonCommercial-ShareAlike 4.0 International License (CC BY-NC-SA 4.0)
Download for free at https://md-shouvik-iqbal.github.io/calculus
Page 260 Related Rates Chapter 4
dh −3
=
dt 4π
m
≈ −0.24
s
m
Therefore, the water level is dropping at the rate of approximately 0.24 .
s
I Example 4.5.3. Sand falls from a conveyor belt at a rate of 1.5 cubic meters
per minute. This forms a conical pile of sand. The radius of the base of this
conical pile is always half the height. How fast is the height of the pile increasing
when the height is 3 meters?
Solution. Let V be the volume of the cone and h be the height of the cone. We
dV m3 h dh
are given = 1.5 and r = , and asked to find when h = 3m.
dt min 2 dt
In order to do that notice that the volume V and the height h are related by
the volume of a right circular cone formula. That is,
1
V = πr2 h
3
h
Since the radius of the pile is always half the height, that is, r = , therefore,
2
1
V = πr2 h
3
2
1 h
= π h
3 2
1
= πh3
12
Now, differentiating both sides of the equation above with respect to time t
yields,
dV d 1 3
= πh
dt dt 12
1 d
= π h3
12 dt
1 dh
= π · 3h2
12
dt
1 2 dh
= πh
4 dt
Copyright © 2024 Md. Shouvik Iqbal. This book is licensed under a Creative Commons Attribution-NonCommercial-ShareAlike 4.0 International License (CC BY-NC-SA 4.0)
Download for free at https://md-shouvik-iqbal.github.io/calculus
Chapter 4 Related Rates Page 261
dV m3
Since = 1.5 and h = 3m,
dt min
dV π dh
= h2
dt 4 dt
π 2 dh
1.5 = (3)
4 dt
dh 6
=
dt 9π
2 m
=
3π min
2 m
Therefore, The height of the sand pile is increasing at a rate of or ap-
3π min
m
proximately 0.212 .
min
I Example 4.5.4. A 10-foot ladder is leaning against a wall. The bottom of the
ladder is being moved away from the wall at a speed of 2 feet per second. At
the moment when the bottom of the ladder is 5 feet away from the wall, how
fast is the top of the ladder sliding down the wall?
Solution. Let x be the distance from the bottom of the ladder to the wall and y
dx ft
be the height of the top of the ladder on the wall. We are given = 2 and
dt s
dy
asked to find when x = 5ft.
dt
In order to do that notice that the ladder forms a right triangle and the variables
are related by the Pythagorean theorem. That is,
x2 + y 2 = 102
Now, differentiating both sides of the equation above with respect to time t
yields,
d 2 d d
x + y 2 = 100
dt dt dt
dx dy
2x + 2y =0
dt dt
dx dy
x +y =0
dt dt
Since x = 5ft, therefore, the value of y can be found using the Pythagorean
theorem as follows.
x2 + y 2 = 100
Copyright © 2024 Md. Shouvik Iqbal. This book is licensed under a Creative Commons Attribution-NonCommercial-ShareAlike 4.0 International License (CC BY-NC-SA 4.0)
Download for free at https://md-shouvik-iqbal.github.io/calculus
Page 262 Linearization and Differentials Chapter 4
y 2 = 100 − x2
p
y = 100 − 52
= 75ft
dx ft
Now, substituting x = 5ft, y = 75ft, and = 2 yields,
dt s
dx dy
x +y =0
dt dt
dy
5 · 2 + 75 = 0
dt
dy 10 ft
=−
dt 75 s
10 ft
Therefore, the top of the ladder is sliding down at a rate of , or approxi-
75 s
ft
mately 0.13 .
s
Having discussed the related rates problems through examples, the following can
be thought of as a general guideline for solving such problems.
1. Identify all given variables and the variables that are to be deter-
mined. Draw a figure if applicable
2. An equation that includes the variables for which the rates of change
are either given or need to be found.
3. Use the Chain Rule and differentiate implicitly both sides of the equa-
tion with respect to time.
4. Substitute all known values into the equation, then solve for the un-
known rate of change. respect to time
Copyright © 2024 Md. Shouvik Iqbal. This book is licensed under a Creative Commons Attribution-NonCommercial-ShareAlike 4.0 International License (CC BY-NC-SA 4.0)
Download for free at https://md-shouvik-iqbal.github.io/calculus
Chapter 4 Linearization and Differentials Page 263
Figure 4.21
Copyright © 2024 Md. Shouvik Iqbal. This book is licensed under a Creative Commons Attribution-NonCommercial-ShareAlike 4.0 International License (CC BY-NC-SA 4.0)
Download for free at https://md-shouvik-iqbal.github.io/calculus
Page 264 Linearization and Differentials Chapter 4
0.01 × m = 4.01 − 4
√ √
0.01m + 4 = 4.01
√
∴ 4.01 = 2 + 0.01m
Copyright © 2024 Md. Shouvik Iqbal. This book is licensed under a Creative Commons Attribution-NonCommercial-ShareAlike 4.0 International License (CC BY-NC-SA 4.0)
Download for free at https://md-shouvik-iqbal.github.io/calculus
Chapter 4 Linearization and Differentials Page 265
f(x)
x
a
Figure 4.22
y − f (a)
The slope of the tangent line is m = . Multiplying both sides by (x − a)
x−a
and solving for y yields,
y − f (a)
m=
x−a
y − f (a)
(x − a) · m = ·
(x−a)
x
− a
(x − a) · m = y − f (a)
(x − a) · m + f (a) = y
∴ y = f (a) + m (x − a)
It is known that for a linear function, the slope remains constant across all
points, therefore, the slope is equivalent to the derivative at any given point.
Thus,
y = f (a) + f 0 (a)(x − a)
This implies that the tangent line to the graph of the function at the point x = a
represents the graph of the following linear function.
Copyright © 2024 Md. Shouvik Iqbal. This book is licensed under a Creative Commons Attribution-NonCommercial-ShareAlike 4.0 International License (CC BY-NC-SA 4.0)
Download for free at https://md-shouvik-iqbal.github.io/calculus
Page 266 Linearization and Differentials Chapter 4
Now, since f (0) = sin(0) = 0 and f 0 (0) = cos(0) = 1, therefore the linearization
at x = 0 yields,
√
I Example 4.6.2. What is the linearization of f (x) = 2 1 + x at x = 3?
√
Solution. The derivative of f (x) = 2 1 + x is,
d √
f 0 (x) = 2 1+x
dx
d 1
= 2 · (1 + x) 2
dx
1 1
= 2 · (1 + x)− 2
2
Copyright © 2024 Md. Shouvik Iqbal. This book is licensed under a Creative Commons Attribution-NonCommercial-ShareAlike 4.0 International License (CC BY-NC-SA 4.0)
Download for free at https://md-shouvik-iqbal.github.io/calculus
Chapter 4 Linearization and Differentials Page 267
1
= 1
(1 + x) 2
1
=√
1+x
√ √ 1 1 1
Since, f (3) = 2 1 + 3 = 2 4 = 4, and f 0 (3) = √ = √ = , therefore
1+3 4 2
the linearization at x = 3 yields,
Having discussed the linearization, we now shift our focus to discuss the differ-
entials that approximates the amount a function changes.
f (x) ≈ L(x)
= f (a) + f 0 (a)(x − a)
∴ f (x) ≈ f (a) + f 0 (a)(x − a)
where, a is a fixed point and x is its nearby points. This implies that
Copyright © 2024 Md. Shouvik Iqbal. This book is licensed under a Creative Commons Attribution-NonCommercial-ShareAlike 4.0 International License (CC BY-NC-SA 4.0)
Download for free at https://md-shouvik-iqbal.github.io/calculus
Page 268 Linearization and Differentials Chapter 4
∆y = f (a + ∆x) − f (a)
Now, sometimes it is more difficult to calculate the exact change in y, that is,
∆y, compared to just approximating it by the use of linearization as discussed
in the previous subsection. That is, for all x near a, f (x) can more easily be
approximated by,
L(x) = f (a) + f 0 (a)(x − a)
∆L = L (a + ∆x) − L (x)
= f (a) + f 0 (a) (a + ∆x − a) − f (a) + f 0 (a) (a − a)
| {z } | {z }
L(a+∆x) L(x)
0
= f (a) + f (a) ∆x − f (a)
= f 0 (a) ∆x
∴ ∆L = f 0 (a) ∆x (1)
Now, based on this approximation ∆L of the actual change ∆y, the concept of
differentials can be defined precisely. Historically, the differential of a function
(denoted as dy) was thought of as an infinitely small (or infinitesimal) change in
y, corresponding to an infinitely small (or infinitesimal) change in x (denoted as
dx), if we were to put it in an oversimplified manner. When these infinitesimally
dy
small changes are compared together by a ratio of , the instantaneous rate of
dx
change (i.e., the derivative) of a function is found. Nevertheless, defining such
infinitesimal quantities dy and dx seemed to be all based on intuition, which
Copyright © 2024 Md. Shouvik Iqbal. This book is licensed under a Creative Commons Attribution-NonCommercial-ShareAlike 4.0 International License (CC BY-NC-SA 4.0)
Download for free at https://md-shouvik-iqbal.github.io/calculus
Chapter 4 Linearization and Differentials Page 269
Now, centuries later, these differentials can indeed be defined rigorously with
the concept of linearization. For that, consider the following figure, where the
change in the function y = f (x) as x changed from x = a to x = a + dx is
approximated by the tangent line drawn at x = a, using linearization.
P
S
Figure 4.23
Now, the differential dy can be defined as the change in this linearization, that
is, ∆L. This implies that
dy = ∆L
= f 0 (a) ∆x from equation (1)
= f 0 (a) (x + dx) − x
| {z }
∆x
0
= f (a) dx since ∆x = dx = real number
dy = f 0 (x) dx
Copyright © 2024 Md. Shouvik Iqbal. This book is licensed under a Creative Commons Attribution-NonCommercial-ShareAlike 4.0 International License (CC BY-NC-SA 4.0)
Download for free at https://md-shouvik-iqbal.github.io/calculus
Page 270 Linearization and Differentials Chapter 4
This definition aligns with the old intuition that when dy ÷ dx, the derivative is
found. That is,
dy = f 0 (x) dx
dy f 0 (x)
dx
=
dx dx
dy
= f 0 (x)
dx
dy
This is perhaps the point where confusions come into play! Previously, was
dx
understood as the derivative of a function, defined as the limit of the quotient
∆y
, as ∆x → 0. In other words,
∆x
dy ∆y
= lim
dx ∆x→0 ∆x
dy
where was merely a notation and not an actual quotient of dy over dx.
dx
However, now that we have defined meanings for dy and dx, the derivative of a
function f 0 (x) can indeed be expressed as a quotient of dy and dx. That is,
dy
dy ÷ dx =
dx
f 0 (x)
dx
=
dx
0
= f (x)
= the derivative of f (x)
Copyright © 2024 Md. Shouvik Iqbal. This book is licensed under a Creative Commons Attribution-NonCommercial-ShareAlike 4.0 International License (CC BY-NC-SA 4.0)
Download for free at https://md-shouvik-iqbal.github.io/calculus
Chapter 4 Newton's Method Page 271
Note that every differential rule summarized before can be written in differential
form. That is,
1
9. d (ln(x)) = d (loge (x)) = dx 20. d cot−1 (x) = −
1
dx
x 1 + x2
1
10. d (loga (x)) = dx 1
x ln (a) 21. d sec−1 (x) =
√ dx
|x| x2 − 1
11. d (sin(x)) = cos(x)dx
1
22. d csc−1 (x) = −
√ dx
12. d (cos(x)) = − sin(x)dx |x| x2 − 1
Copyright © 2024 Md. Shouvik Iqbal. This book is licensed under a Creative Commons Attribution-NonCommercial-ShareAlike 4.0 International License (CC BY-NC-SA 4.0)
Download for free at https://md-shouvik-iqbal.github.io/calculus
Page 272 Newton's Method Chapter 4
Newton’s Method is one of many methods by which the roots of a function can
be approximated. The method can best be described in a graphically. For that,
consider the figure below.
f(x)
r
x
Figure 4.24
In the figure above, the root of the function y = f (x) is r. In order to approxi-
mate the root r, we start with a value x1 such that x1 ≈ r. Since x1 6= r and
f (x1 ) 6= 0, therefore we draw a tangent line to the graph of y = f (x) at the
point (x1 , f (x1 )). Now, observe that the tangent line is close to the graph of
f (x) near the point of tangency, therefore the graph of f (x) and the tangent
line at (x1 , f (x1 )) intercepts the x-axis at approximately the same point. This
Copyright © 2024 Md. Shouvik Iqbal. This book is licensed under a Creative Commons Attribution-NonCommercial-ShareAlike 4.0 International License (CC BY-NC-SA 4.0)
Download for free at https://md-shouvik-iqbal.github.io/calculus
Chapter 4 Newton's Method Page 273
implies that the root r of f (x) is approximately equal to the root of the tangent
line, that is, x2 . Now, since the tangent line is a line and its root x2 can easily
be determined, therefore x2 can be used as a second approximation to r (where,
the first approximation was x1 ) to repeat the process of approximation with a
better precision.
Since the slope of the tangent line at the point x1 is f 0 (x1 ), therefore the equation
of the tangent line yields,
y − f (x1 ) = f 0 (x1 ) (x − x1 )
Now, since y = 0 when x = x2 (as x2 lies on the x-axis) in the equation above,
therefore,
y − f (x1 ) = f 0 (x1 ) (x − x1 )
0 − f (x1 ) = f 0 (x1 ) (x2 − x1 )
f (x1 )
− = (x2 − x1 )
f 0 (x1 )
f (x1 )
x2 = x1 − 0
f (x1 )
where, f 0 (x) 6= 0. Now, since f (x2 ) 6= 0, therefore x2 6= r, and thus x2 ≈ r. A
better approximation x3 can now be derived by repeating the process disussed
above with an initial approximation being x2 to the root r. That is, draw a
tangent line to the graph of f (x) at the point (x2 , f (x2 )), that intercepts the
x-axis at the point x3 . Therefore, the equation of the tangent line is as follows.
y − f (x2 ) = f 0 (x2 ) (x − x2 )
Since the tangent line intercepts the x-axis at the point x = x3 , therefore y = 0
when x = x3 . This implies,
y − f (x2 ) = f 0 (x2 ) (x − x2 )
0 − f (x2 ) = f 0 (x2 ) (x3 − x2 )
f (x2 )
− = x3 − x2
f 0 (x2 )
f (x2 )
x3 = x2 − 0
f (x2 )
where, f 0 (x2 ) 6= 0. If f (x3 ) 6= 0, then x3 6= r, and thus x3 ≈ r. This implies
that the process can be repeated again by considering x3 as the initial approxi-
mation. This yields a sequence of approximation x1 , x2 , x3 , x4 , · · · , as shown in
the following figure.
Copyright © 2024 Md. Shouvik Iqbal. This book is licensed under a Creative Commons Attribution-NonCommercial-ShareAlike 4.0 International License (CC BY-NC-SA 4.0)
Download for free at https://md-shouvik-iqbal.github.io/calculus
Page 274 Newton's Method Chapter 4
f(x)
r
x
Figure 4.25
In general, if the nth approximation is f (xn ) and f 0 (xn ) 6= 0, then the next
approximation yields,
f (xn )
xn+1 = xn − 0
f (xn )
This implies that if xn → r, as n → ∞, then the sequence of successive approx-
imations converges to the actual root r of the function f (x).
Solution. Since f (x) = x4 − π, therefore, f 0 (x) = 4x3 , and thus, by the Newton’s
Method,
f (xn )
xn+1 = xn −
f 0 (xn )
x4 − π
= xn − n 3
4xn
Copyright © 2024 Md. Shouvik Iqbal. This book is licensed under a Creative Commons Attribution-NonCommercial-ShareAlike 4.0 International License (CC BY-NC-SA 4.0)
Download for free at https://md-shouvik-iqbal.github.io/calculus
Chapter 4 Newton's Method Page 275
f (xn )
n xn xn+1 = xn −
f 0 (xn )
1 1 1.5353981634
2 1.5353981634 1.36853218292
3 1.36853218292 1.33282459009
4 1.33282459009 1.33133785791
5 1.33133785791 1.33133536381
6 1.33133536381 1.3313353638
7 1.3313353638 1.3313353638
Solution. Since f (x) = eex − e, therefore, f 0 (x) = eex+1 , and thus, by the New-
ton’s Method,
f (xn )
xn+1 = xn −
f 0 (xn )
eexn − e
= xn − exn +1
e
Calculations of the iterations are presented below for n = 1, 2, · · · , 7.
eexn − e
n xn xn+1 = xn −
eexn +1
1 0.5 0.389001924142
2 0.389001924142 0.368474393475
3 0.368474393475 0.367879922005
4 0.367879922005 0.367879441172
5 0.367879441172 0.367879441171
6 0.367879441171 0.367879441171
7 0.367879441171 0.367879441171
Copyright © 2024 Md. Shouvik Iqbal. This book is licensed under a Creative Commons Attribution-NonCommercial-ShareAlike 4.0 International License (CC BY-NC-SA 4.0)
Download for free at https://md-shouvik-iqbal.github.io/calculus
Page 276 Newton's Method Chapter 4
Although Newton’s Method is a powerful technique to find the root for finding
the roots of a function, it sometimes fails to do so. For example, if the derivative
f (xn )
f 0 (xn ) in the formula xn+1 = xn − 0 turns out to be 0 for any n, then the
f (xn )
formula involves division by zero, making it undefined altogether.
Similarly, Newton’s Method can also fail for other reasons. It might miss the
aimed root and instead converge to a different one, or not converge at all. For
example, consider the following example.
√
I Example 4.7.3. Approximate a root of f (x) = 3 x, using Newton’s Method up
to 7 iterations, where the initial approximation is x1 = 1.
√ 1
Solution. Since f (x) = 3
x, therefore f 0 (x) = 2 . Now, by the Newton’s
3x 3
Method,
f (xn )
xn+1 = xn −
f 0 (xn )
√3 x
n
= xn −
1
2
3xn3
1
xn3
= xn − 2
1 −3
x
3 n
1 1
+ 23
= xn − 1 · xn3
3
3
= xn − 3 · xn3
= xn − 3xn
= −2xn
Copyright © 2024 Md. Shouvik Iqbal. This book is licensed under a Creative Commons Attribution-NonCommercial-ShareAlike 4.0 International License (CC BY-NC-SA 4.0)
Download for free at https://md-shouvik-iqbal.github.io/calculus
Chapter 4 L'Hôpital's Rule Page 277
n xn xn+1 = −2xn
1 1 −2
2 −2 4
3 4 −8
4 −8 16
5 16 −32
6 −32 64
7 64 −128
ax
lim
x→∞ x
=a
ax
lim
x→0 x
Copyright © 2024 Md. Shouvik Iqbal. This book is licensed under a Creative Commons Attribution-NonCommercial-ShareAlike 4.0 International License (CC BY-NC-SA 4.0)
Download for free at https://md-shouvik-iqbal.github.io/calculus
Page 278 L'Hôpital's Rule Chapter 4
=a
∞ 0
In general, limits that take the form of or are indeterminate, and can have
∞ 0
any value. Therefore, these types of limits require a different approach when
determining their values.
The method often used in such situation is known as L’Hôpital’s Rule, and the
idea behind it is quite simple: when both the numerator and denominator of a
fraction approach either 0 or ∞ as x approaches a certain value, directly eval-
uating the limit leads to an indeterminate form. In such cases, examining the
rates at which the numerator and denominator change (i.e., their derivatives)
can provide insight into the function’s behavior. For example, if both the numer-
ator and denominator approaches infinity, we can’t immediately determine the
limit of the fraction. The outcome is ambiguous. However, if we somehow know
that the numerator of the fraction increases at a rapid pace while the denomi-
nator increases slowly, then this information becomes critical, as it shows that
the behavior of the numerator largely dictates the fraction’s limit. Therefore,
instead of directly evaluating the numerator and denominator, we analyze their
respective rates of change (i.e., derivatives) to determine the overall limit of the
fraction. In the following subsections, we go through these cases one by one.
0
Chap. 4 / Sec. 4.8 / Subsec. 4.8.1 : Indeterminate Form
0
f (x)
Consider a function h(x) = . If lim f (x) = 0 as well as lim g(x) = 0,
g(x) x→a x→a
f (x) 0 0
then the limit lim = , which is the indeterminate form of . In order to
x→a g(x) 0 0
determine the actual value of such a limit of indeterminate form, the L’Hôpital’s
Rule is used stating the following.
0
Theorem 4.8.1. L’Hôpital’s Rule for
0
Copyright © 2024 Md. Shouvik Iqbal. This book is licensed under a Creative Commons Attribution-NonCommercial-ShareAlike 4.0 International License (CC BY-NC-SA 4.0)
Download for free at https://md-shouvik-iqbal.github.io/calculus
Chapter 4 L'Hôpital's Rule Page 279
Let f (x) and g(x) be differentiable functions over an open interval I con-
taining a, except possibly at a and suppose g 0 (x) 6= 0. If lim f (x) =
x→a
lim g(x) = 0, then L’Hôpital’s Rule states that
x→a
f (x) f 0 (x)
lim = lim 0
x→a g (x) x→a g (x)
provided that the limit on the right side exists (or is ±∞). The rule is also
applicable for the cases where x → ±∞, x → a+ , or x → a− .
Proof. We prove a special case of this theorem where we assume that f (a) =
g(a) = 0 and f 0 (x) and g 0 (x) are continuous at a, and g 0 (a) 6= 0. Thus, we have,
f 0 (x) f 0 (a)
lim =
x→a g 0 (x) g 0 (a)
f (x) − f (a)
0
f (a) lim
=
x→a x−a
0
g (a) g (x) − g (a)
lim
x→a x−a
f (x) − f (a)
= lim x−a
x→a g (x) − g (a)
x−a
f (x) − f (a) x−a
= lim
·
x→a x
−a g (x) − g (a)
f (x) − f (a)
= lim
x→a g (x) − g (a)
f (x)
= lim
x→a g (x)
ex − 1
I Example 4.8.1. Evaluate the limit lim
x→0 x
Copyright © 2024 Md. Shouvik Iqbal. This book is licensed under a Creative Commons Attribution-NonCommercial-ShareAlike 4.0 International License (CC BY-NC-SA 4.0)
Download for free at https://md-shouvik-iqbal.github.io/calculus
Page 280 L'Hôpital's Rule Chapter 4
= e0
=1
ex − 1
Therefore, lim = 1.
x→0 x
Remark 4.8.1.
ex − 1
lim =1
x→0 x
ax − 1
I Example 4.8.2. Evaluate the limit lim
x→0 x
Solution. Substituting x = 0 directly into the expression yields an indeterminate
0
form of ,
0
ax − 1 a0 − 1
=
x 0
0
=
0
However, by the L’Hôpital’s Rule,
d x
ax − 1 (a − 1)
lim = lim dx
x→0 x x→0 dx
dx
Copyright © 2024 Md. Shouvik Iqbal. This book is licensed under a Creative Commons Attribution-NonCommercial-ShareAlike 4.0 International License (CC BY-NC-SA 4.0)
Download for free at https://md-shouvik-iqbal.github.io/calculus
Chapter 4 L'Hôpital's Rule Page 281
d x d
a − (1)
= lim dx dx
x→0 dx
dx
ax ln(a)
= lim
x→0 1
= a ln(a)
0
= ln(a)
ax − 1
Therefore, lim = ln(a)
x→0 x
Remark 4.8.2.
ax − 1
lim = ln(a)
x→0 x
ln(1 + x)
I Example 4.8.3. Evaluate the limit lim
x→0 x
Solution. Substituting x = 0 directly into the expression yields an indeterminate
0
form of ,
0
ln(1 + x) ln(1 + 0)
=
x 0
0
=
0
However, by the L’Hôpital’s Rule,
d
ln(1 + x) ln(1 + x)
lim = lim dx
x→0 x x→0 dx
dx
1 d
· (1 + x)
= lim 1 + x dx
x→0 dx
dx
1
= lim 1 + x
x→0 1
1
=
1+0
=1
ln(1 + x)
Therefore, lim =1
x→0 x
Copyright © 2024 Md. Shouvik Iqbal. This book is licensed under a Creative Commons Attribution-NonCommercial-ShareAlike 4.0 International License (CC BY-NC-SA 4.0)
Download for free at https://md-shouvik-iqbal.github.io/calculus
Page 282 L'Hôpital's Rule Chapter 4
Remark 4.8.3.
ln(1 + x)
lim =1
x→0 x
∞
Chap. 4 / Sec. 4.8 / Subsec. 4.8.2 : Indeterminate Form
∞
∞
L’Hôpital’s Rule is also applicable to the limits of the indeterminant form .
∞
f (x)
For example, consider a function h(x) = . If lim f (x) = ±∞ as well as
g(x) x→a
f (x) ∞
lim g(x) = ±∞, then the limit lim = , which is the indeterminate form
x→a x→a g(x) ∞
∞
of . Now to solve such a limit, L’Hôpital’s Rule states the following.
∞
±∞
Theorem 4.8.2. L’Hôpital’s Rule for
±∞
Let f (x) and g(x) be differentiable functions over an open interval I con-
taining a, except possibly at a and suppose g 0 (x) 6= 0. If lim f (x) = ±∞
x→a
and lim g(x) = ±∞, then L’Hôpital’s Rule states that
x→a
f (x) f 0 (x)
lim = lim 0
x→a g (x) x→a g (x)
provided that the limit on the right side exists (or is ±∞). The rule is also
applicable for the cases where x → ±∞, x → a+ , or x → a− .
2x2 + 3x + 5
I Example 4.8.4. Evaluate the limit lim
x→∞ 2x2 − 3x − 5
∞
Solution. As x → ∞, the expression yields an indeterminate form of , there-
∞
fore, by L’Hôpital’s Rule,
d 2
2x + 3x + 52 2x + 3x + 5
lim = lim dx
x→∞ 2x2 − 3x − 5 x→∞ d
(2x2 − 3x − 5)
dx
4x + 3
= lim
x→∞ 4x − 3
Copyright © 2024 Md. Shouvik Iqbal. This book is licensed under a Creative Commons Attribution-NonCommercial-ShareAlike 4.0 International License (CC BY-NC-SA 4.0)
Download for free at https://md-shouvik-iqbal.github.io/calculus
Chapter 4 L'Hôpital's Rule Page 283
∞
Now, as x → ∞, the expression again yields an indeterminate form of ,
∞
therefore, we reapply L’Hôpital’s Rule once more,
d
4x + 3 (4x + 3)
lim = lim dx
x→∞ 4x − 3 x→∞ d
(4x − 3)
dx
4
= lim
x→∞ 4
=1
2x2 + 3x + 5
Therefore, lim =1
x→∞ 2x2 − 3x − 5
ln(1 + x)
I Example 4.8.5. Evaluate lim
x→∞ x
∞
Solution. As x → ∞, the expression yields an indeterminate form of , there-
∞
fore, by L’Hôpital’s Rule,
d
ln(1 + x) ln(1 + x)
lim = lim dx
x→∞ x x→∞ dx
dx
1 d
· (1 + x)
= lim 1 + x dx
x→∞ dx
dx
1
= lim 1 + x
x→∞ 1
1
= lim
x→∞ 1 + x
=0
ln(1 + x)
Therefore, lim =0
x→∞ x
Remark 4.8.4.
ln(1 + x)
lim =0
x→∞ x
ln (x + 1)
I Example 4.8.6. Evaluate lim
x→∞ ln (x − 1)
Copyright © 2024 Md. Shouvik Iqbal. This book is licensed under a Creative Commons Attribution-NonCommercial-ShareAlike 4.0 International License (CC BY-NC-SA 4.0)
Download for free at https://md-shouvik-iqbal.github.io/calculus
Page 284 L'Hôpital's Rule Chapter 4
∞
Solution. As x → ∞, the expression yields an indeterminate form of , there-
∞
fore, by L’Hôpital’s Rule,
d
ln (x + 1) ln (x + 1)
lim = lim dx
x→∞ ln (x − 1) x→∞ d
ln (x − 1)
dx
1 d
· (x + 1)
= lim x + 1 dx
x→∞ 1 d
· (x − 1)
x − 1 dx
1
= lim x + 1
x→∞ 1
x−1
x−1
= lim
x→∞ x + 1
∞
Now, as x → ∞, the expression again yields an indeterminate form of ,
∞
therefore, we reapply L’Hôpital’s Rule once more,
d
x−1 (x − 1)
lim = lim dx
x→∞ x + 1 x→∞ d
(x + 1)
dx
1
= lim
x→∞ 1
=1
ln (x + 1)
Therefore, lim =1
x→∞ ln (x − 1)
Copyright © 2024 Md. Shouvik Iqbal. This book is licensed under a Creative Commons Attribution-NonCommercial-ShareAlike 4.0 International License (CC BY-NC-SA 4.0)
Download for free at https://md-shouvik-iqbal.github.io/calculus
Chapter 4 L'Hôpital's Rule Page 285
1
I Example 4.8.7. Evaluate lim x sin 2
x→∞ x
1
Solution. Since the limit lim x sin
2
is of the indeterminate form 0 · ∞,
x→∞ x
therefore L’Hôpital’s Rule cannot be directly applied to it. As a result, this
0 ∞
limit needs some modification to convert the form into either or . This can
0 ∞
1
be done by the use of reciprocals. For example, since the reciprocal of x2 is 2 ,
x
1
therefore x2 = . As a result, the limit can be rewritten as follows,
1
x2
1 1 1
lim x sin
2
= lim sin
x→∞ x x→∞ 1 x
x2
1
sin
x
= lim
x→∞ 1
x2
Now, as x → ∞, both the numerator and denominator approach 0, thus the
0
limit is converted into the indeterminate form such that L’Hôpital’s Rule now
0
can be directly applied to evaluate the limit. That is,
1 d 1
sin sin
x dx x
lim = lim
1
x→∞ x→∞ d 1
x2 dx x2
1 d 1
cos ·
x dx x
= lim
x→∞ 2
− 3
x
1 1
cos · − 2
x x
= lim
x→∞ 2
− 3
x
1
cos
x
−
= lim x2
x→∞ 2
− 3
x
Copyright © 2024 Md. Shouvik Iqbal. This book is licensed under a Creative Commons Attribution-NonCommercial-ShareAlike 4.0 International License (CC BY-NC-SA 4.0)
Download for free at https://md-shouvik-iqbal.github.io/calculus
Page 286 L'Hôpital's Rule Chapter 4
1
cos
x x3
= lim ·
x→∞ x2 2
1
x cos
x
= lim
x→∞ 2
1
z }| {
1
∞ · cos
∞
=
2
=∞
1
This implies that lim x2 sin = ∞.
x→∞ x
Copyright © 2024 Md. Shouvik Iqbal. This book is licensed under a Creative Commons Attribution-NonCommercial-ShareAlike 4.0 International License (CC BY-NC-SA 4.0)
Download for free at https://md-shouvik-iqbal.github.io/calculus
Chapter 4 L'Hôpital's Rule Page 287
−x sin(x) + cos(x)
− cos(x)
= lim
x→0 x cos(x) + sin(x)
x sin(x)
= − lim
x→0 x cos(x) + sin(x)
It is sometimes helpful to change the variable. That is, to evaluate lim f (x),
x→∞
1
it is sometimes helpful to let t = . Now, notice that if x → ∞, then t → 0+ .
x
As a result, we can rewrite the limit in terms of t, as follows,
1
lim f (x) = lim+ f
x→∞ t→0 t
1 1
Note that since we let t = , therefore x = . Thus, if x → +∞, then t → 0
x t
from the positive side.
√
I Example 4.8.9. Evaluate lim 2
x − x + 6x
x→∞
Copyright © 2024 Md. Shouvik Iqbal. This book is licensed under a Creative Commons Attribution-NonCommercial-ShareAlike 4.0 International License (CC BY-NC-SA 4.0)
Download for free at https://md-shouvik-iqbal.github.io/calculus
Page 288 L'Hôpital's Rule Chapter 4
√
Solution. As x → ∞, both the terms x as well as x2 + 6x approach ∞, yielding
an indeterminate form ∞ − ∞. Nevertheless, to evaluate this limit, we do the
following,
s !
6x
lim x − x2 + 6x = lim x − x2 1 + 2
p
x→∞ x→∞ x
!
√
r
6
= lim x − x2 · 1 +
x→∞ x
r !
6
= lim x 1 − 1 +
x→∞ x
1 1
Now, let a new variable t = , then x = . Substituting these variables yields,
x t
r !
6 1 √
lim x 1 − 1 + = lim+ 1 − 1 + 6t
x→∞ x t→0 t
√
1 − 1 + 6t
= lim+
t→0 t
Now, as t → 0+ , both the numerator and denominator yield an indeterminate
0
form , and L’Hôpital’s Rule can now be directly applied. L’Hôpital’s Rule
0
states that,
√ d √
1 − 1 + 6t 1 − 1 + 6t
lim+ = lim+ dt
t→0 t t→0 d
t
dt
1 d
− √ · (1 + 6t)
2 1 + 6t dt
= lim+
t→0 1
1
− √ ·6
2 1 + 6t
= lim+
t→0 1
6
= lim+ − √
t→0 2 1 + 6t
6
=− √
2 1+0
6
=−
2
= −3
√
This implies that lim x − x2 + 6x = −3
x→∞
Copyright © 2024 Md. Shouvik Iqbal. This book is licensed under a Creative Commons Attribution-NonCommercial-ShareAlike 4.0 International License (CC BY-NC-SA 4.0)
Download for free at https://md-shouvik-iqbal.github.io/calculus
Chapter 4 L'Hôpital's Rule Page 289
This implies that we can rewrite the limit lim f (x)g(x) as,
x→a
Copyright © 2024 Md. Shouvik Iqbal. This book is licensed under a Creative Commons Attribution-NonCommercial-ShareAlike 4.0 International License (CC BY-NC-SA 4.0)
Download for free at https://md-shouvik-iqbal.github.io/calculus
Page 290 L'Hôpital's Rule Chapter 4
0
Now, we convert this to a limit of the indeterminate form and apply L’Hôpital’s
0
Rule,
ln (sin(x))
lim+ tan(x) ln (sin(x)) = lim+
x→0 x→0 cot(x)
d
ln (sin(x))
= lim+ dx
d
cot(x)
x→0
dx
1 d
sin(x)
sin(x) dx
= lim+
x→0 −csc2 (x)
cos(x)
sin(x)
= lim+
x→0 1
− 2
sin (x)
cos(x)
= − lim+ · sin2 (x)
x→0 sin(x)
= − cos(0) sin(0)
=0
Solution. As x → 0 from the positive side, xx → 00 . This implies that the limit
has the indeterminate form 00 . To evaluate the limit we let, xx = ex ln(x) = eL
and solve the limit L = lim+ x ln (x). The limit lim+ x ln (x) has an indetermi-
x→0 x→0
nate form 0 · ∞, therefore to use L’Hôpital’s Rule, the indeterminate form 0 · ∞
0 ∞
is modified into the indeterminate form or , as shown below.
0 ∞
1
lim+ x ln (x) = lim+ · ln (x)
x→0 x→0 1
x
ln (x)
= lim+
x→0 1
x
Copyright © 2024 Md. Shouvik Iqbal. This book is licensed under a Creative Commons Attribution-NonCommercial-ShareAlike 4.0 International License (CC BY-NC-SA 4.0)
Download for free at https://md-shouvik-iqbal.github.io/calculus
Chapter 4 L'Hôpital's Rule Page 291
ln (x) 0
Since the limit lim+ has the indeterminate form , therefore L’Hôpital’s
x→0 1 0
x
Rule is used stating that,
d
ln (x) ln (x)
lim = lim+ dx
1
x→0+ x→0 d 1
x dx x
1
= lim+ x
x→0 1
− 2
x
1
= lim+ · −x2
x→0 x
= lim+ −x
x→0
=0
= lim+ e0
x→0
= lim+ 1
x→0
=1
As a result, lim+ xx = 1.
x→0
Copyright © 2024 Md. Shouvik Iqbal. This book is licensed under a Creative Commons Attribution-NonCommercial-ShareAlike 4.0 International License (CC BY-NC-SA 4.0)
Download for free at https://md-shouvik-iqbal.github.io/calculus
Copyright © 2024 Md. Shouvik Iqbal. This book is licensed under a Creative Commons Attribution-NonCommercial-ShareAlike 4.0 International License (CC BY-NC-SA 4.0)
Download for free at https://md-shouvik-iqbal.github.io/calculus
Part III
Integral Calculus
293
Copyright © 2024 Md. Shouvik Iqbal. This book is licensed under a Creative Commons Attribution-NonCommercial-ShareAlike 4.0 International License (CC BY-NC-SA 4.0)
Download for free at https://md-shouvik-iqbal.github.io/calculus
Copyright © 2024 Md. Shouvik Iqbal. This book is licensed under a Creative Commons Attribution-NonCommercial-ShareAlike 4.0 International License (CC BY-NC-SA 4.0)
Download for free at https://md-shouvik-iqbal.github.io/calculus
Chapter 5
An Introduction to Integral
Calculus
Humans have long been familiar with the concept of area, even in ancient times,
and have developed various methods to calculate them. For example, the area
of a rectangle is determined by multiplying its base by its height. Similarly, the
area of a triangle is calculated as half the area of a rectangle since a triangle can
be thought of as one-half of a rectangle when divided by a diagonal. Likewise,
the area of a regular polygon can be found by dividing the polygon into smaller
triangles and summing their areas.
Contrary to the shapes above, the shape of a circle is complex for the fact
that it has no corners or edges, resulting in a smooth, continuous, and round
curve. Finding the area of a circle is thus generally more difficult than the
ones mentioned above. Documented history suggests that it was the Greeks,
particularly Archimedes, who were among the first to solve the problem of finding
the area of a circle through approximation. The method used in that process
is referred to as the “method of exhaustion” for the reason that it involves
295
Copyright © 2024 Md. Shouvik Iqbal. This book is licensed under a Creative Commons Attribution-NonCommercial-ShareAlike 4.0 International License (CC BY-NC-SA 4.0)
Download for free at https://md-shouvik-iqbal.github.io/calculus
Page 296 Chapter 5
inscribing1 (or circumscribing) a regular polygon into a circle and increasing its
sides in a way that the area of the polygon approaches the exact area of the
circle. For example, consider the circle below.
r
h
b
Figure 5.1
In the figure above, a regular polygon is inscribed into the circle. Since each of
1
the isosceles triangles has an area A = hb, therefore the sum of these areas
2
equals the area of the polygon, which is a close approximation of the area of the
circle. That is, the area of the polygon Ap yields,
1 1 1 1
Ap = hb + hb + hb + · · · + hb
2 2 2 2
1
= h (b + b + b + · · · + b)
2
1
= hp
2
where, p is the perimeter of the polygon. Now, if C = 2πr is the circumference
of the circle, then as the number of sides in the polygon increases, the height h in
each polygon approaches the circle’s radius r, that is, h → r; and the perimeter
of the polygon approaches the circumference C, that is, p → C. This implies
that,
1 1
Ap = hp → rC
2 2
1
to fit a smaller shape inside a larger shape in such a way that the smaller shape touches
but does not intersect the larger shape.
Copyright © 2024 Md. Shouvik Iqbal. This book is licensed under a Creative Commons Attribution-NonCommercial-ShareAlike 4.0 International License (CC BY-NC-SA 4.0)
Download for free at https://md-shouvik-iqbal.github.io/calculus
Chapter 5 The Area Under a Curve Page 297
1
= r (2πr)
2
∴ Ap → πr2
A = πr2
The method of exhaustion works effectively for circles due to their regular ge-
ometric properties. However, when applied to irregular shapes lacking such
properties, the method loses its value—it becomes impractical. Therefore, we
need to consider a different approach to calculate the area of irregular shapes.
In modern terms, the problem can be generalized to finding the area of a region
with a curved boundary.
To address this, we begin with a specific case of the general problem: find the
area under the graph of a function y = f (x) and above the x-axis, between two
vertical lines at x = a and x = b. This enclosed region has a curved boundary
only along its upper edge, making it easier to handle. This generalized problem
is commonly referred to as finding the “area under a curve.”
Solving this specific case provides a foundation for addressing more complex
regions, such as those with curved boundaries on both edges. For such regions,
the total area can be determined by subtracting the area under the lower curve
from the area under the upper curve, where both areas belong to the simpler
“area under a curve” category. These principles are discussed further in the
following.
Copyright © 2024 Md. Shouvik Iqbal. This book is licensed under a Creative Commons Attribution-NonCommercial-ShareAlike 4.0 International License (CC BY-NC-SA 4.0)
Download for free at https://md-shouvik-iqbal.github.io/calculus
Page 298 The Area Under a Curve Chapter 5
1 A
1 2 3 4
Figure 5.2
x2
Suppose, we want to determine the exact area A under the curve of f (x) =
4
and above the x-axis, from x = 1 to x = 4. In order to do that, we begin by
breaking down the area into simple, familiar shapes—rectangles.
The idea is straightforward: the area under the curve can be approximated by
adding up the areas of infinitely many narrow rectangles, where each rectangle’s
height is determined by the function’s value at different points. As we increase
the number of rectangles indefinitely, the approximation becomes more accurate,
eventually giving us the exact area.
However, since we’re dealing with infinitely many rectangles, we need a sys-
tematic way to represent their sum. That is, summing these infinitely many
rectangles will be impossible for us to do in the traditional way, so we need
some sort of mathematical “tool” to handle this situation more compactly. And
this is where the summation notation, , comes into play to help us neatly
P
organize and compute the total area. We discuss it in the following subsection
very shortly and then move to calculate the area under the curve.
P
Chap. 5 / Sec. 5.1 / Subsec. 5.1.1 : The Summation ( ) Notation
The summation notation, also known as the sigma notation, is a concise way to
represent a large sum of numbers that follow a specific pattern. For example,
if we ever need to calculate the sum of integers from 1 to 100, how would we
express it? The common approach is to place the ‘+’ sign in between each
Copyright © 2024 Md. Shouvik Iqbal. This book is licensed under a Creative Commons Attribution-NonCommercial-ShareAlike 4.0 International License (CC BY-NC-SA 4.0)
Download for free at https://md-shouvik-iqbal.github.io/calculus
Chapter 5 The Area Under a Curve Page 299
1 + 2 + 3 + 4 + 5 + · · · + 96 + 97 + 98 + 99 + 100
However, that too isn’t systematically concise enough compared to just explicitly
expressing,
As a result, the summation notation was developed with the purpose to address
situations like the one above. In that notation, we write,
100
X
i
i=1
to mean the sum of all i such that i ranges from 1 to 100. In other words, it is
the sum of all integers i from i = 1 to i = 100.
Here, i is called the index of summation. It may be obvious that the choice of
the variable for the index does not matter at all as long as it doesn’t confuse
with the other variables that are in use. There can be any variable in the place
100 100 100
of i to keep track of the terms to be added. For example,
X X X
i= k= j
i=1 k=1 j=1
conveys the same meaning. In fact, the index variable is called a dummy variable
as it has generally no effect upon its choice. It is, however, customary to use the
letters i, j, k for indices.
. Which can be written more compactly using the summation notation as follows,
n
X
ai
i=1
where i is the index of summation, ai is the ith term of the summation, and the
start and end limits of the summation are 1 and n, respectively. Therefore,
Copyright © 2024 Md. Shouvik Iqbal. This book is licensed under a Creative Commons Attribution-NonCommercial-ShareAlike 4.0 International License (CC BY-NC-SA 4.0)
Download for free at https://md-shouvik-iqbal.github.io/calculus
Page 300 The Area Under a Curve Chapter 5
The sum
a1 + a2 + a3 + · · · + an
A few examples are presented below to demonstrate the use of the summation
notation discussed above.
Solution. This is an arithmetic sequence where the first term is 2, and the com-
mon difference is 3. Therefore,
10
X
(2 + 3(i − 1))
i=1
10
Therefore,
X
(2 + 3(i − 1)) = 2 + 5 + 8 + 11 + · · · + 29.
i=1
Solution. This is a geometric sequence with the first term a = 3 and common
ratio r = 2. Therefore,
X4
3 · 2i
i=0
4
Therefore, 3 · 2i = 3 + 6 + 12 + 24 + 48
P
i=0
1 1 1
I Example 5.1.3. Express 1 + + + in terms of the summation notation.
4 9 16
Solution. In order to use the summation notation, notice the denominator of
each term is a square of numbers from 1 to 4. That is,
1 1 1 1
2
+ 2+ 2+ 2
1 2 3 4
Copyright © 2024 Md. Shouvik Iqbal. This book is licensed under a Creative Commons Attribution-NonCommercial-ShareAlike 4.0 International License (CC BY-NC-SA 4.0)
Download for free at https://md-shouvik-iqbal.github.io/calculus
Chapter 5 The Area Under a Curve Page 301
and there are a total of 4 terms. Thus, the sum can be expressed as,
4
X 1
k2
k=1
.
4 1 1 1 1
Therefore, .
P
2
= 1 + + +
k=1 k 4 9 16
A few summation rules are provided below to simplify the process for large and
complex summation.
Rule 5.1.1 (Sigma Notation Rules). The following rules demonstrate the lin-
earity of Sigma notation.
n n
1. Constant Multiple Rule:
X X
cai = c × ai
i=1 i=1
n n n
2. Summation Rule:
X X X
(ai + bi ) = ai + bi
i=1 i=1 i=1
n n n
3. Difference Rule:
X X X
(ai − bi ) = ai − bi
i=1 i=1 i=1
4
I Example 5.1.4. Evaluate
X
(i + 2i)
i=1
Solution. Using the summation rule, we can split the sum into two separate sums,
4
X 4
X 4
X
(i + 2i) = i+ 2i
i=1 i=1 i=1
X4 X 4
= i+2 i
i=1 i=1
4
Now, we calculate the sum
X
i,
i=1
4
X
i=1+2+3+4
i=1
Copyright © 2024 Md. Shouvik Iqbal. This book is licensed under a Creative Commons Attribution-NonCommercial-ShareAlike 4.0 International License (CC BY-NC-SA 4.0)
Download for free at https://md-shouvik-iqbal.github.io/calculus
Page 302 The Area Under a Curve Chapter 5
= 10
= 10 + 2 · 10
= 30
3
I Example 5.1.5. Evaluate
X
2i − i2
i=1
Solution. Using the summation rule, we can split the sum into two separate sums,
3
X 3
X 3
X
2
i2
2i − i = 2i −
i=1 i=1 i=1
= 2 (1 + 2 + 3)
= 12
Similarly,
3
X
i2 = 1 2 + 2 2 + 3 2
i=1
=1+4+9
= 14
= 12 − 14
= −2
Copyright © 2024 Md. Shouvik Iqbal. This book is licensed under a Creative Commons Attribution-NonCommercial-ShareAlike 4.0 International License (CC BY-NC-SA 4.0)
Download for free at https://md-shouvik-iqbal.github.io/calculus
Chapter 5 The Area Under a Curve Page 303
So far, we have calculated the sum manually. For example, to solve the sum
n
i, where n = 3, we would manually calculate it as 1 + 2 + 3 = 6. However,
X
i=1
what happens when the sum is to be calculated for n = 1000, 10000, 100000,
etc.? Clearly, manual calculation falls short in such situations. Therefore, we
have the following formulas.
Copyright © 2024 Md. Shouvik Iqbal. This book is licensed under a Creative Commons Attribution-NonCommercial-ShareAlike 4.0 International License (CC BY-NC-SA 4.0)
Download for free at https://md-shouvik-iqbal.github.io/calculus
Page 304 The Area Under a Curve Chapter 5
2000300010000
=
6
= 333383335000
Therefore, 12 + 22 + 32 + · · · + 100002 = 333383335000
1 A
1 2 3 4
Figure 5.3
There are no simple formulas to calculate the area of this particular shape. How-
ever, its area can be approximated to any desired degree of accuracy by using
the areas of familiar shapes, such as rectangles. The idea here is that the area
A can be bounded between a lower area limit L and an upper area limit U , that
is, L ≤ A ≤ U (see Figure 5.4), in a way that gives a relatively good approxi-
mation of the exact area A in terms of the lower and upper area limit L and U ,
respectively for the reason that L and U are easy to calculate.
Now, to calculate the values of these lower and upper area limits L and U , we
take a visual approach by considering the figure below.
Copyright © 2024 Md. Shouvik Iqbal. This book is licensed under a Creative Commons Attribution-NonCommercial-ShareAlike 4.0 International License (CC BY-NC-SA 4.0)
Download for free at https://md-shouvik-iqbal.github.io/calculus
Chapter 5 The Area Under a Curve Page 305
4 4 4
3 3 3
2 ≤ 2 ≤ 2
1 1 A 1
1 2 3 4 1 2 3 4 1 2 3 4
(a) (b) (c)
Figure 5.4
The lower area limit L of the exact area A is shown in Figure 5.4(a), and the
upper area limit U is in Figure 5.4(c).
The area of the rectangle shown in Figure 5.4(a) is 0.75. This is because the
width of the rectangle is 4 − 1 = 3 and its height is the value of the function
x2 12 1
f (x) = at the point x = 1, that is, f (1) = = , thus, its area yields
4 4 4
1
× 3 = 0.75.
4
Similarly, the area of the rectangle shown in Figure 5.4(c) yields 12. This is be-
42 16
cause the width of the rectangle is 4−1 = 3 and its height is f (4) = = = 4,
4 4
resulting the area to be 4 × 3 = 12.
Notice that the shaded region shown in Figure 5.4(a) is completely contained
within Figure 5.4(b), but not all of the shaded region in Figure 5.4(b) is within
Figure 5.4(a). Therefore, the area of the shaded region in Figure 5.4(a) is strictly
smaller than that of Figure 5.4(b), thus 0.75 < A .
Similarly, notice that the shaded region shown in Figure 5.4(b) is completely
contained within Figure 5.4(c), but not all of the shaded region in Figure 5.4(c)
is within Figure 5.4(b). Therefore, the area of the shaded region in Figure 5.4(b)
is strictly smaller than that of Figure 5.4(c), thus A < 12 .
This implies that the area of the shaded region in Figure 5.4(b) is strictly less
than that of Figure 5.4(c) and greater than that of Figure 5.4(a), therefore, the
exact area A is bounded by the inequality 0.75 < A < 12 .
Copyright © 2024 Md. Shouvik Iqbal. This book is licensed under a Creative Commons Attribution-NonCommercial-ShareAlike 4.0 International License (CC BY-NC-SA 4.0)
Download for free at https://md-shouvik-iqbal.github.io/calculus
Page 306 The Area Under a Curve Chapter 5
4 4
3 3
2 2
1 1
1 2 3 4 1 2 3 4
(d) (e)
Figure 5.5
Now, turning back to our approximation, we see that although the range 0.75 <
A < 12 provides a decent estimation, we can achieve a more accurate approxi-
mation of the exact area A by using a greater number of rectangles. Previously,
we used only one rectangle, but by increasing the number of rectangles, we can
get a better approximation for the reason described below using the following
figure.
4 4 4
3 3 3
2 2 2
1 1 1
1 2 3 4 1 2 3 4 1 2 3 4
Figure 5.6
Copyright © 2024 Md. Shouvik Iqbal. This book is licensed under a Creative Commons Attribution-NonCommercial-ShareAlike 4.0 International License (CC BY-NC-SA 4.0)
Download for free at https://md-shouvik-iqbal.github.io/calculus
Chapter 5 The Area Under a Curve Page 307
For that, we need to first divide the interval [1, 4] into sub-intervals for determin-
ing the left and right endpoints of each sub-intervals, as previously explained.
Let n be the number of rectangles constructed, then if the width of the entire
interval [1, 4] is 4 − 1, then constructing n = 3 rectangles would make the width
4−1 3
of each rectangle equal to = = 1. The height of each rectangle would
3 3
be the function’s value at either endpoint of each sub-interval, as before.
With that, let us first underestimate the area A using the left-endpoint approx-
imation with 3 rectangles as follows.
1 2 3 4
Figure 5.7
12
1. Height: f (1) = = 0.25
4
2. Width: 1
Copyright © 2024 Md. Shouvik Iqbal. This book is licensed under a Creative Commons Attribution-NonCommercial-ShareAlike 4.0 International License (CC BY-NC-SA 4.0)
Download for free at https://md-shouvik-iqbal.github.io/calculus
Page 308 The Area Under a Curve Chapter 5
22 4
1. Height: f (2) = = =1
4 4
2. Width: 1
32 9
1. Height: f (3) = = = 2.25
4 4
2. Width: 1
Total underestimation:
1 2 3 4
Figure 5.8
22 4
1. Height: f (2) = = =1
4 4
Copyright © 2024 Md. Shouvik Iqbal. This book is licensed under a Creative Commons Attribution-NonCommercial-ShareAlike 4.0 International License (CC BY-NC-SA 4.0)
Download for free at https://md-shouvik-iqbal.github.io/calculus
Chapter 5 The Area Under a Curve Page 309
2. Width: 1
32 9
1. Height: f (3) = = = 2.25
4 4
2. Width: 1
42 16
1. Height: f (4) = = =4
4 4
2. Width: 1
Total overestimation:
This implies that using three rectangles to approximate the exact area A gives
3.5 < A < 7.25 , which is clearly a better approximation compared to the pre-
vious one, where the bound for the exact area A was 0.75 < A < 12. Thus,
it has been mathematically shown that using more rectangles provides a closer
approximation of the exact area A.
Since more rectangles cover more of the area A, this means an even better
approximation is always possible as there is no restriction to the number of rect-
angles that can be fitted within a given interval. However, manually calculating
the height, width, and area of each rectangle, and then summing them up for
the total estimation, is going to be quite tedious for large values. For example,
if we fit a billion rectangles within a given interval, is it possible to manually
calculate the area of all those rectangles a billion times? Therefore, let us now
generalize the entire process for any arbitrary function f (x), in the following
subsection.
Copyright © 2024 Md. Shouvik Iqbal. This book is licensed under a Creative Commons Attribution-NonCommercial-ShareAlike 4.0 International License (CC BY-NC-SA 4.0)
Download for free at https://md-shouvik-iqbal.github.io/calculus
Page 310 The Area Under a Curve Chapter 5
a b
Figure 5.9
To determine the exact area A bounded under the curve of f (x) and above the
x-axis, from x = a to x = b, we divide the interval [a, b] into n sub-intervals
of equal width. To do that, we use a set of increasing points a = x0 < x1 <
x2 < x3 < · · · < xn−1 < xn = b that are equally spaced between x = a and x = b.
Since the total width of the interval [a, b] is b − a, therefore dividing it into
b−a
n parts gives each sub-interval a width of . We denote this as ∆x, thus
n
b−a
∆x = . Therefore, ∆x represents the space between each of the points
n
a = x0 < x1 < x2 < x3 < · · · < xn−1 < xn = b, as shown in the figure below.
Δx Δx Δx Δx
Figure 5.10
This implies that the interval [a, b] is divided into the following n sub-intervals:
Copyright © 2024 Md. Shouvik Iqbal. This book is licensed under a Creative Commons Attribution-NonCommercial-ShareAlike 4.0 International License (CC BY-NC-SA 4.0)
Download for free at https://md-shouvik-iqbal.github.io/calculus
Chapter 5 The Area Under a Curve Page 311
where,
x1 = x0 + ∆x
x2 = x0 + 2∆x
x3 = x0 + 3∆x
.. ..
. .
xi−1 = x0 + (i − 1)∆x (1)
xi = x0 + i∆x (2)
.. ..
. .
xn = x0 + n∆x
Now that we have formed the sub-intervals for an arbitrary function f (x), we
now use them to form rectangles that let us approximate the area under the
curve. This can result in either an overestimation or an underestimation of the
exact area, depending on how the rectangles are constructed.
Copyright © 2024 Md. Shouvik Iqbal. This book is licensed under a Creative Commons Attribution-NonCommercial-ShareAlike 4.0 International License (CC BY-NC-SA 4.0)
Download for free at https://md-shouvik-iqbal.github.io/calculus
Page 312 The Area Under a Curve Chapter 5
curve. For that, as we saw before, the left-endpoint of each sub-interval is used
to determine the height of the rectangles. Therefore, we start our discussion
with the left-endpoint approximation and following that, we will discuss the
right-endpoint approximation.
But before that, it is important to note the ith sub-interval, that is,
Therefore, for any arbitrary function f (x), we have the following definition,
Copyright © 2024 Md. Shouvik Iqbal. This book is licensed under a Creative Commons Attribution-NonCommercial-ShareAlike 4.0 International License (CC BY-NC-SA 4.0)
Download for free at https://md-shouvik-iqbal.github.io/calculus
Chapter 5 The Area Under a Curve Page 313
Copyright © 2024 Md. Shouvik Iqbal. This book is licensed under a Creative Commons Attribution-NonCommercial-ShareAlike 4.0 International License (CC BY-NC-SA 4.0)
Download for free at https://md-shouvik-iqbal.github.io/calculus
Page 314 The Area Under a Curve Chapter 5
Therefore, for any arbitrary function f (x), we have the following definition,
Having generalized the left and right endpoint approximations, we can now pro-
ceed from where we left off.
This suggests that for any arbitrary function f (x), the exact bounded area A is
as follows,
n
X n
X
f (xi−1 )∆x < A < f (xi )∆x
i=1 i=1
Copyright © 2024 Md. Shouvik Iqbal. This book is licensed under a Creative Commons Attribution-NonCommercial-ShareAlike 4.0 International License (CC BY-NC-SA 4.0)
Download for free at https://md-shouvik-iqbal.github.io/calculus
Chapter 5 The Area Under a Curve Page 315
Now, if we go back to our previous problem of finding the exact area under the
x2
curve of f (x) = and above the x-axis, from x = 1 to x = 4; we see that the
4
exact area A can be determined by the following inequality,
n
X n
X
f (xi−1 )∆x < A < f (xi )∆x
i=1 i=1
However, in order to use the inequality above, we have to first determine the
value of xi−1 , xi , and ∆x.
Since the entire width of the interval [1, 4] is 3, therefore, dividing this interval
into n sub-interval gives each sub-interval a width of
3
∆x =
n
Now, to determine xi−1 and xi , recall that,
3(i − 1) + n 3
from equation (1), xi−1 = 1 + (i − 1) · =
n n
3 3i + n
from equation (2), xi =1+i· =
n n
n
With that in hand, we now began to calculate the underestimation Ln =
X
f (xi−1 )∆x
i=1
2
x
to the exact area under the curve of the function f (x) = as follows,
4
n
X
Ln = f (xi−1 )∆x
i=1
n
X 3 (i − 1) + n
= f · ∆x
i=1
n
3 (i − 1) + n 2
n
n 3 x2
since, f (x) =
X
= ·
i=1
4 n 4
(3 (i − 1) + n)2
n
X
n2 3
= ·
i=1
4 n
(3 (i − 1))2 + 2 · 3 (i − 1) · n + n2
n
X
n2 3
= ·
i=1
4 n
Copyright © 2024 Md. Shouvik Iqbal. This book is licensed under a Creative Commons Attribution-NonCommercial-ShareAlike 4.0 International License (CC BY-NC-SA 4.0)
Download for free at https://md-shouvik-iqbal.github.io/calculus
Page 316 The Area Under a Curve Chapter 5
n
X 9(i − 1)2 + 6n (i − 1) + n2 3
= ·
i=1
4n2 n
n 3 9(i − 1)2 + 6n (i − 1) + n2
X
=
i=1
4n3
n
3 X 2 2
= · 9(i − 1) + 6n (i − 1) + n
4n3 i=1
n n n
!
3 X 2
X X
= 3 9(i − 1) + 6n (i − 1) + n2
4n i=1 i=1 i=1
n n n
!
3 X X X
= 3 9· (i − 1)2 + 6n · (i − 1) + n2
4n i=1 i=1 i=1
n n n
!
3 X X X
i2 − 2i + 1 + 6n · n2
= 9· (i − 1) +
4n3 i=1
n n n
!i=1 n
i=1
n
! n
!
3 X X X X X X
= 3 9 i2 − 2i + 1 + 6n i− 1 + n2
4n i=1 i=1 i=1 i=1 i=1 i=1
n n n
! n n
! n
!
3 X X X X X X
= 3 9 i2 − 2 i+ 1 + 6n i− 1 + n2
4n
i=1 i=1 i=1 i=1
i=1 i=1
3 n (n + 1) (2n + 1) n (n + 1) n (n + 1) 3
= 3 9 −2 + n + 6n −n +n
4n 6 2 2
3 n (n + 1) (2n + 1) n (n + 1) n (n + 1)
Now, simplifying 3 9 −2 + n + 6n −n +n 3
4n 6 2 2
yields,
3(14n2 − 15n + 3)
Ln =
8n2
n
Similarly, we calculate the overestimation Rn = f (xi )∆x to the exact area
X
i=1
x2
A under the curve of the function f (x) = as follows,
4
n
X
Rn = f (xi )∆x
i=1
n
X 3i + n
= f · ∆x
i=1
n
3i + n 2
n
n 3 x2
since, f (x) =
X
= ·
i=1
4 n 4
Copyright © 2024 Md. Shouvik Iqbal. This book is licensed under a Creative Commons Attribution-NonCommercial-ShareAlike 4.0 International License (CC BY-NC-SA 4.0)
Download for free at https://md-shouvik-iqbal.github.io/calculus
Chapter 5 The Area Under a Curve Page 317
(3i + n)2
n
X
n2 3
= ·
i=1
4 n
(3i)2 + 2 · 3i · n + n2
n
X
n2 3
= ·
i=1
4 n
n
X 9i2 + 6ni + n2 3
= ·
i=1
4n2 n
n
X 3 9i2 + 6ni + n 2
=
i=1
4n3
n
3 X 2
= 3· 9i + 6ni + n2
4n i=1
n n n
!
3 X X X
= 9i2 + 6ni + n2
4n3 i=1 i=1 i=1
n n n
!
3 X X X
= 3 9· i2 + 6n · i+ n2
4n i=1 i=1 i=1
3 n (n + 1) (2n + 1) n (n + 1) 3
= 3 9· + 6n · +n
4n 6 2
3 n (n + 1) (2n + 1) n (n + 1)
Now, simplifying 3 9 · + 6n · + n yields,
3
4n 6 2
3(14n2 + 15n + 3)
Rn =
8n2
This suggests that we have derived two formulas for underestimation Ln and
overestimation Rn to the exact area A, where n represents the number of rect-
x2
angles. That is, the exact area A bounded under the curve of f (x) = and
4
above the x-axis, from x = 1 to x = 4, is given by the following inequality,
3(14n2 − 15n + 3) 3(14n2 + 15n + 3)
< A <
8n2 8n2
To verify these formulas, set n = 1 and n = 3 to see if the resulting inequalities
match with the previously manually calculated inequalities: 0.75 < A < 12 for
n = 1 and 3.5 < A < 7.25 for n = 3.
If n = 1, then
3 14n2 − 15n + 3 3 14n2 + 15n + 3
<A<
8n2 8n2
Copyright © 2024 Md. Shouvik Iqbal. This book is licensed under a Creative Commons Attribution-NonCommercial-ShareAlike 4.0 International License (CC BY-NC-SA 4.0)
Download for free at https://md-shouvik-iqbal.github.io/calculus
Page 318 The Area Under a Curve Chapter 5
3 14 · 12 − 15 · 1 + 3 3 14 · 12 + 15 · 1 + 3
<A<
8 · 12 8 · 12
0.75 < A < 12
Which also matches with the previously manually done inequality of the exact
area A.
For n = 1 rectangles:
For n = 3 rectangles:
For n = 9 rectangles:
Copyright © 2024 Md. Shouvik Iqbal. This book is licensed under a Creative Commons Attribution-NonCommercial-ShareAlike 4.0 International License (CC BY-NC-SA 4.0)
Download for free at https://md-shouvik-iqbal.github.io/calculus
Chapter 5 The Area Under a Curve Page 319
For n = 81 rectangles:
For that, let us first take the limit of the underestimation as n → ∞ as follows.
Copyright © 2024 Md. Shouvik Iqbal. This book is licensed under a Creative Commons Attribution-NonCommercial-ShareAlike 4.0 International License (CC BY-NC-SA 4.0)
Download for free at https://md-shouvik-iqbal.github.io/calculus
Page 320 The Area Under a Curve Chapter 5
Having determined the exact area A in the previously mentioned problem, it’s
important to notice something significant. In finding the exact area, we con-
structed an infinite number of rectangles under the curve and observed that
both the underestimation and overestimation approach the same value. This
implies that to determine the exact area, the use of just one approximation
is sufficient. Moreover, the approximation doesn’t need to be specifically an
overestimation or an underestimation—it can be any approximation when the
number of rectangles is unbounded.
Copyright © 2024 Md. Shouvik Iqbal. This book is licensed under a Creative Commons Attribution-NonCommercial-ShareAlike 4.0 International License (CC BY-NC-SA 4.0)
Download for free at https://md-shouvik-iqbal.github.io/calculus
Chapter 5 The Area Under a Curve Page 321
It is the best moment to introduce the Riemann Sum and define the area under
a curve in terms of that in the following subsection.
Chap. 5 / Sec. 5.1 / Subsec. 5.1.4 : Riemann Sum and the Definition of the
Area Under a Curve
The Riemann sum resolves a few restrictions. For example, we can now choose
any point x∗i ∈ [xi−1 , xi ] to determine the height of the rectangle over the ith
sub-interval, for i = 1, 2, 3, · · · , n, as shown below,
1 2 3 4
Figure 5.11
Doing so would not change the limit of the approximation, if the number of
rectangle n → ∞ is considered. Therefore, let f (x∗i ) represent the height of
the rectangle over the ith sub-interval [xi−1 , xi ], for i = 1, 2, 3, · · · , n, where,
x∗i ∈ [xi−1 , xi ] is any arbitrary point. Then, the area of the rectangle over that
sub-interval yields,
Copyright © 2024 Md. Shouvik Iqbal. This book is licensed under a Creative Commons Attribution-NonCommercial-ShareAlike 4.0 International License (CC BY-NC-SA 4.0)
Download for free at https://md-shouvik-iqbal.github.io/calculus
Page 322 The Area Under a Curve Chapter 5
n
The approximation f (x∗i )∆x is called the Riemann sum, named after Ger-
X
i=1
man mathematician Georg Friedrich Bernhard Riemann. Therefore, we have
the following definition of the approximation above.
where, n is the number of rectangles, x∗i is any point in the ith sub-interval
[xi−1 , xi ], and ∆x is the width of each sub-interval.
represents the exact area A in terms of the Riemann sum. As a result, we can
define the exact area A bounded under the curve of f (x) and above the x-axis,
from x = a to x = b, in terms of the Riemann sum as follows.
Copyright © 2024 Md. Shouvik Iqbal. This book is licensed under a Creative Commons Attribution-NonCommercial-ShareAlike 4.0 International License (CC BY-NC-SA 4.0)
Download for free at https://md-shouvik-iqbal.github.io/calculus
Chapter 5 The Definite Integral Page 323
i=1
exact area A under the curve of f (x) and over the x-axis, from x = a to
x = b, is defined as,
n
A = lim
X
f (x∗i )∆x
n→∞
i=1
Now, notice that the current definition of the “area under a curve” comes with
a great limitation: it only applies when the function f (x) is non-negative. In
other words, for this definition to apply, the curve has to lie entirely above the
x-axis. If any part of the curve lies below the x-axis, the definition above can’t
be applied there.
To resolve the issue, the concept of “Definite Integral” comes into play. Definite
integral expands the definition of the area under a curve to include regions
bounded by curves that can lie below the x-axis by considering both positive
and negative regions bounded by the curve of f (x). To make this distinction, we
introduce the idea of “net signed area,” where the curve lying above the x-axis
contributes positively, and below the x-axis contributes negatively, as discussed
in the following section.
However, that definition required the function f (x) to be non-negative. Since not
all functions satisfy these conditions, we may need to remove these requirements.
As a result, f (x) can now sometimes be negative. But this raises the question:
how do we interpret Riemann sums when f (x) is negative at some or all of its
Copyright © 2024 Md. Shouvik Iqbal. This book is licensed under a Creative Commons Attribution-NonCommercial-ShareAlike 4.0 International License (CC BY-NC-SA 4.0)
Download for free at https://md-shouvik-iqbal.github.io/calculus
Page 324 The Definite Integral Chapter 5
points over the interval [a, b]? The answer is given through the concept of net
signed area, as discussed below.
Chap. 5 / Sec. 5.2 / Subsec. 5.2.1 : Net Signed Area and the Definition of the
Definite Integral
To understand the net signed area, approximate the area bounded by the curve
of f (x) = −x2 from x = 0 to x = 2, using n = 4 rectangles.
Since the interval is [0, 2], therefore after dividing the interval into n sub-intervals
2−0
results in the width of each sub-interval being ∆x = = 0.5. Therefore,
4
the sub-intervals yields,
Now, to use the Riemann sum approximation, we choose the sample points
(which can be any points within each respective sub-interval),
Here, all the values of the height f (x∗i ) are negative. As a result, the Riemann
sum—which approximates the area under the curve—turns out to be also neg-
ative. However, if the Riemann sum is used to approximate the “area” under
the curve, then how can the area—being a physical measurement—be a negative
Copyright © 2024 Md. Shouvik Iqbal. This book is licensed under a Creative Commons Attribution-NonCommercial-ShareAlike 4.0 International License (CC BY-NC-SA 4.0)
Download for free at https://md-shouvik-iqbal.github.io/calculus
Chapter 5 The Definite Integral Page 325
quantity?
The key point to realize here is that the Riemann sum, in this case, does not
approximate the area in the physical sense. Instead, it approximates the “net
signed area,” that is, a difference of areas that accounts for both the positive
and the negative contributions of areas.
Although the positive contribution of area implies the actual positive area, the
negative contribution of area does not literally imply a “negative area,” as if
there were an absence or void of space between the curve and the x-axis. In-
stead, it represents the negative of the actual area bounded above the curve
and below the x-axis (see the red shaded region in Figure 5.12). This happens
because, in the Riemann sum, the height of the ith rectangle is determined by
the the values of the function f (x∗i ), where x∗i ∈ [xi−1 , xi ]. Now, if f (x∗i ) falls
below the x-axis, then the height of the ith rectangle, that is, −f (x∗i ) represents
the negative of the actual height |f (x∗i )|, and thus the product, −f (x∗i )∆x, rep-
resents the negative of the actual product |f (x∗i )∆x|, which is the area of the
ith rectangle when f (x) falls below the x-axis.
Now, times occur when f (x) is positive over some parts of [a, b] and negative
over other parts of [a, b]. In such cases, the Riemann sum approximates the pos-
itive area when f (x) is positive and negative of the area when f (x) is negative.
Therefore, the Riemann sum over the interval [a, b] approximates the sum of the
positive areas that lie above the x-axis and the negative of the areas that lie
beneath the x-axis, which is precisely what we refer to as the net signed area,
as shown below.
Copyright © 2024 Md. Shouvik Iqbal. This book is licensed under a Creative Commons Attribution-NonCommercial-ShareAlike 4.0 International License (CC BY-NC-SA 4.0)
Download for free at https://md-shouvik-iqbal.github.io/calculus
Page 326 The Definite Integral Chapter 5
f(x)
Figure 5.12
In the figure above, the area bounded under the curve of f (x) and above the
x-axis is A1 . Similarly, the area bounded above the curve of f (x) and below the
x-axis is −A2 . Now, the Riemann sum, as the number of rectangles n → ∞,
represents the sum of these areas A1 and −A2 . Therefore, we write,
n
lim
X
f (x∗i )∆x = A1 + (−A2 )
n→∞
i=1
= A1 − A2
The quantity A1 − A2 is what we call the net signed area, which can be positive,
negative, or even zero.
If there’s multiple positive and negative of the areas; for example, suppose,
A1 , A2 , A3 , · · · , An are the areas bounded under the curve of f (x) and above the
x-axis, and −B1 , −B2 , −B3 , · · · , −Bk are the areas bounded above the curve of
f (x) and below the x-axis. Then the net signed area yields,
n
lim
X
f (x∗i )∆x
n→∞
i=1
Copyright © 2024 Md. Shouvik Iqbal. This book is licensed under a Creative Commons Attribution-NonCommercial-ShareAlike 4.0 International License (CC BY-NC-SA 4.0)
Download for free at https://md-shouvik-iqbal.github.io/calculus
Chapter 5 The Definite Integral Page 327
where, R1 is the sum of areas of R that lie above the x-axis and R2 is the
sum of areas of R that lie below the x-axis.
This suggests that the Riemann sum approximates the net signed area bounded
by the curve of f (x) and the x-axis over the interval [a, b]. This approximation
gets closer to the exact net signed area as the number of rectangles n → ∞.
where, max ∆xi is the largest width among all sub-intervals. This idea works
because as max ∆xi → 0, all ∆xi → 0, for xi = 1, 2, 3, · · · , n. As all ∆xi → 0,
n must also increase indefinitely.
With that in mind, it is finally the time that we define the “definite integral” to
expand the previously discussed definition of the area under a curve.
Copyright © 2024 Md. Shouvik Iqbal. This book is licensed under a Creative Commons Attribution-NonCommercial-ShareAlike 4.0 International License (CC BY-NC-SA 4.0)
Download for free at https://md-shouvik-iqbal.github.io/calculus
Page 328 The Definite Integral Chapter 5
Zb
f (x) dx
a
Zb n
lim
X
f (x) dx = f (x∗i )∆xi
max ∆xi →0
a i=1
The definition above suggests that for a definite integral to exist, the limit used
to define that integral must first exist. The precise interpretation of this limit
is that,
Zb n
lim
X
f (x) dx − f (x∗i )∆xi < ε
max ∆xi →0
a i=1
Copyright © 2024 Md. Shouvik Iqbal. This book is licensed under a Creative Commons Attribution-NonCommercial-ShareAlike 4.0 International License (CC BY-NC-SA 4.0)
Download for free at https://md-shouvik-iqbal.github.io/calculus
Chapter 5 The Definite Integral Page 329
Upper limit
of integration
x is the variable
Integrand
Lower limit
of integration
Zb Zb Zb Zb
f (x) dx = f (t) dt = f (u) du = f (s) ds
a a a a
It is important to note that we have defined the definite integral for integrable
functions but not all functions fall into this criteria. A function f (x) is integrable
if f (x) is continuous on the interval [a, b]. Additionally, if f (x) is bounded on
[a, b], meaning that there exists a number M such that |f (x)| < M for all
x ∈ [a, b] and it has only a finite number of discontinuities, then f (x) is also
integrable on [a, b]. This implies that even if a function is not continuous, it can
still be integrable on [a, b]. As a result, we have the following theorem.
The theorem above suggests that if f (x) is continuous on [a, b], then the definite
Zb
integral f (x) dx represents the net signed area bounded by the curve f (x)
a
and the x-axis over that given interval. This means that the integral takes into
account both the areas above the x-axis (which are considered positive) and
those below the x-axis (which are considered negative). As a result, the definite
Zb
integral f (x) dx gives a single numerical value, which corresponds to the total
a
quantity accumulated, considering the signs of the areas.
Copyright © 2024 Md. Shouvik Iqbal. This book is licensed under a Creative Commons Attribution-NonCommercial-ShareAlike 4.0 International License (CC BY-NC-SA 4.0)
Download for free at https://md-shouvik-iqbal.github.io/calculus
Page 330 The Definite Integral Chapter 5
A few examples to evaluate the definite integral using the definition above are
provided below.
Z1
I Example 5.2.1. Evaluate 2xdx
0
Solution. In order to evaluate the definite integral, we divide the interval [0, 1]
into n sub-intervals of equal width. The width of each sub-interval yields,
1−0 1
=
n n
Now, we let the ith sample point xi be the right endpoint of each sub-interval
∗
=1
Z1
Therefore, 2xdx = 1
0
Copyright © 2024 Md. Shouvik Iqbal. This book is licensed under a Creative Commons Attribution-NonCommercial-ShareAlike 4.0 International License (CC BY-NC-SA 4.0)
Download for free at https://md-shouvik-iqbal.github.io/calculus
Chapter 5 The Definite Integral Page 331
Z2
I Example 5.2.2. Evaluate x2 dx
0
Solution. In order to evaluate the definite integral, we divide the interval [0, 2]
into n sub-intervals of equal width. The width of each sub-interval yields,
2−0 2
=
n n
Now, we let the ith sample point x∗i be the right endpoint of each sub-interval
for calculation simplicity. Then, we get,
x∗i = x0 + i∆x
2
=0+i·
n
2i
=
n
Therefore, the Riemann sum for the function f (x) = x2 yields,
n n 2
X X 2i 2
f (x∗i )∆xi = ·
i=1 i=1
n n
n
X 4i2 2
= ·
i=1
n2 n
n
X 8i2
=
i=1
n3
n
8 X 2
= 3 i
n i=1
8 n(n + 1)(2n + 1)
= ·
n3 6
4(n + 1)(2n + 1)
=
3n2
Now, as the number of rectangles n → ∞, then,
n
4(n + 1)(2n + 1)
lim f (x∗i )∆xi = lim
X
n→∞
i=1
n→∞ 3n2
4 (n + 1)(2n + 1)
= · lim
3 n→∞ n2
4 2n2 + 3n + 1
= · lim
3 n→∞ n2
Copyright © 2024 Md. Shouvik Iqbal. This book is licensed under a Creative Commons Attribution-NonCommercial-ShareAlike 4.0 International License (CC BY-NC-SA 4.0)
Download for free at https://md-shouvik-iqbal.github.io/calculus
Page 332 The Definite Integral Chapter 5
2n2 3n 1
4 2
+ 2+ 2
= · lim n n
2
n
3 n→∞ n
n2
3 1
4 2+ + 2
= · lim n n
3 n→∞
1
4 3 1
= · lim 2 + lim + lim 2
3 n→∞ n→∞ n n→∞ n
8
=
3
Z2
8
Therefore, x2 dx = .
3
0
Having discussed the definite integral and its evaluation, we now discuss its
properties in the next subsection.
Chap. 5 / Sec. 5.2 / Subsec. 5.2.2 : The Properties of the Definite Integral
The properties of the definite integral acts as a set of rules that govern how
definite integrals can be manipulated and evaluated. We start our discussion by
introducing the the reversal of limits of integration and the identical limits of
integration, as follows.
Zb n
lim
X
f (x) dx = f (x∗i )∆xi
max ∆xi →0
a i=1
where we indirectly assumed that the lower limit of integration a is less than the
upper limit of integration b, that is, a < b. However, it is certainly possible that
a > b. In such cases, the width of each sub-interval changes from ∆xi to −∆xi ,
for i = 1, 2, 3, · · · , n, in the Riemann sum on the right side of the equation. This
is due to the fact that a − b < 0 is divided into n sub-intervals, rather than
Copyright © 2024 Md. Shouvik Iqbal. This book is licensed under a Creative Commons Attribution-NonCommercial-ShareAlike 4.0 International License (CC BY-NC-SA 4.0)
Download for free at https://md-shouvik-iqbal.github.io/calculus
Chapter 5 The Definite Integral Page 333
Za Zb
f (x) dx = − f (x) dx
b a
Now, if the lower limit of integration a is equal to the upper limit b (i.e., a = b),
then ∆x = 0. This happens because the interval a − a = 0 is divided into n
0
sub-intervals, which is simply = 0. Therefore,
n
Za n
lim
X
f (x) dx = f (x∗i ) · 0
max ∆xi →0
a i=1
=0
Za
∴ f (x) dx = 0
a
Copyright © 2024 Md. Shouvik Iqbal. This book is licensed under a Creative Commons Attribution-NonCommercial-ShareAlike 4.0 International License (CC BY-NC-SA 4.0)
Download for free at https://md-shouvik-iqbal.github.io/calculus
Page 334 The Definite Integral Chapter 5
Copyright © 2024 Md. Shouvik Iqbal. This book is licensed under a Creative Commons Attribution-NonCommercial-ShareAlike 4.0 International License (CC BY-NC-SA 4.0)
Download for free at https://md-shouvik-iqbal.github.io/calculus
Chapter 5 The Definite Integral Page 335
n n
!
lim
X X
= f (x∗i ) ∆xi − g (x∗i ) ∆xi
max ∆xi →0
i=1 i=1
n n
lim lim
X X
= f (x∗i ) ∆xi − g (x∗i ) ∆xi
max ∆xi →0 max ∆xi →0
i=1 i=1
Zb Zb Zb
∴ (f (x) − g(x)) dx = f (x)dx − g(x)dx
a a a
Zb Zb
cf (x)dx = c · f (x)dx
a a
Zb n
lim
X
cf (x)dx = cf (x)∆xi
max ∆xi →0
a i=1
n
!
lim
X
= c· f (x)∆xi
max ∆xi →0
i=1
n
lim lim
X
= c· f (x)∆xi
max ∆xi →0 max ∆xi →0
i=1
n
lim
X
=c· f (x)∆xi
max ∆xi →0
i=1
Zb
=c· f (x)dx
a
Copyright © 2024 Md. Shouvik Iqbal. This book is licensed under a Creative Commons Attribution-NonCommercial-ShareAlike 4.0 International License (CC BY-NC-SA 4.0)
Download for free at https://md-shouvik-iqbal.github.io/calculus
Page 336 The Definite Integral Chapter 5
Zb Zc Zb
f (x)dx = f (x)dx + f (x)dx
a a c
f(x) f(x)
x x
a b a c b
Figure 5.13
Copyright © 2024 Md. Shouvik Iqbal. This book is licensed under a Creative Commons Attribution-NonCommercial-ShareAlike 4.0 International License (CC BY-NC-SA 4.0)
Download for free at https://md-shouvik-iqbal.github.io/calculus
Chapter 5 The Definite Integral Page 337
Zb
As a result, the integral |f (x)| , dx calculates the “total area” between the
a
curve f (x) and the x-axis over the interval [a, b]. This includes the area above
the x-axis where f (x) is positive, as well as the area below the x-axis where f (x)
is negative, since the absolute value converts negative values of f (x) to positive.
Therefore, this integral provides the sum of the areas of both regions, treating
both as positive contributions.
Zb
That is, the definite integral f (x)dx represents the following net signed area.
a
f(x)
x
a b
Figure 5.14
Zb
While the definite integral |f (x)| dx represents the following area under the
a
curve of |f (x)|.
Copyright © 2024 Md. Shouvik Iqbal. This book is licensed under a Creative Commons Attribution-NonCommercial-ShareAlike 4.0 International License (CC BY-NC-SA 4.0)
Download for free at https://md-shouvik-iqbal.github.io/calculus
Page 338 The Definite Integral Chapter 5
f(x)
x
a b
Figure 5.15
represents the sum of areas of the region bounded the function f (x) and
the x-axis, from x = a to x = b.
Having discussed the properties above, they are now summarized below for a
quick reference.
Za Zb
2. f (x) dx = − f (x) dx
b a
Zb Zb
3. k · f (x) dx = k · f (x) dx
a a
Copyright © 2024 Md. Shouvik Iqbal. This book is licensed under a Creative Commons Attribution-NonCommercial-ShareAlike 4.0 International License (CC BY-NC-SA 4.0)
Download for free at https://md-shouvik-iqbal.github.io/calculus
Chapter 5 The Definite Integral Page 339
Zb Zb Zb
4. f (x) + g (x) dx = f (x)dx + g (x)dx
a a a
Zb Zb Zb
5. f (x) − g (x) dx = f (x)dx − g (x)dx
a a a
Zb Zc Zb
6. f (x)dx = f (x)dx + f (x)dx
a a c
Z 1 Z 2
7
I Example 5.2.3. If x dx = − , then what is
2
x2 dx?
2 3 1
Z 1
7
Solution. If x2 dx = − , then by the property above,
2 3
Z 2 Z 1
2
x dx = − x2 dx
1
2
7
=− −
3
7
=
3
Z 2
7
Therefore, x2 dx =
1 3
Z1 Z1 Z1
I Example 5.2.4. If f (x)dx = e and g(x)dx = π, then what is (πf (x) + eg(x)) dx
0 0 0
Z1 Z1
Solution. If f (x)dx = e and g(x)dx = π, then by the properties above,
0 0
Z1 Z1 Z1
(πf (x) + eg(x)) dx = π f (x)dx + e g(x)dx
0 0 0
= πe + eπ
Copyright © 2024 Md. Shouvik Iqbal. This book is licensed under a Creative Commons Attribution-NonCommercial-ShareAlike 4.0 International License (CC BY-NC-SA 4.0)
Download for free at https://md-shouvik-iqbal.github.io/calculus
Page 340 Numerical Integration Chapter 5
= 2eπ
Z1
Therefore, (πf (x) + eg(x)) dx = 2eπ.
0
We then introduced the Riemann sum, where the height of each rectangle could
be based on any point within each sub-interval. This method is valid based on
the fact that all these approximations converge to the same limit as the number
of rectangles approaches infinity.
Copyright © 2024 Md. Shouvik Iqbal. This book is licensed under a Creative Commons Attribution-NonCommercial-ShareAlike 4.0 International License (CC BY-NC-SA 4.0)
Download for free at https://md-shouvik-iqbal.github.io/calculus
Chapter 5 Numerical Integration Page 341
a b
Figure 5.16
Copyright © 2024 Md. Shouvik Iqbal. This book is licensed under a Creative Commons Attribution-NonCommercial-ShareAlike 4.0 International License (CC BY-NC-SA 4.0)
Download for free at https://md-shouvik-iqbal.github.io/calculus
Page 342 Numerical Integration Chapter 5
n
for i = 1, 2, · · · , n
X
= ∆x · f (mi )
i=1
n
X
= f (mi ) ∆x
i=1
n
X xi−1 + xi
= f ∆x
i=1
2
The Midpoint rule approximation gets better and better as the number of rect-
angles n increases. This forms the following definition.
n
X
Mn = f (mi ) ∆x
i=1
xi−1 + xi
where, mi = is the midpoint of the ith interval, [xi−1 , xi ], and
2
∆x is the width of each sub-interval.
Zπ
I Example 5.3.1. Use the Midpoint Rule with 6 sub-intervals to estimate sin(x)dx
0
Copyright © 2024 Md. Shouvik Iqbal. This book is licensed under a Creative Commons Attribution-NonCommercial-ShareAlike 4.0 International License (CC BY-NC-SA 4.0)
Download for free at https://md-shouvik-iqbal.github.io/calculus
Chapter 5 Numerical Integration Page 343
π π
+ π
m2 = 6 3 =
2 4
π π
+ 5π
m3 = 3 2 =
2 12
π 2π
+
m4 = 2 3 = 7π
2 12
2π 5π
+
m5 = 3 6 = 3π
2 4
5π
+π 11π
m6 = 6 =
2 12
Now, we approximate the integral as follows,
Zπ 6
sin(x)dx ≈
X
f (mi )∆x
0 i=1
= f (m1 )∆x + f (m2 )∆x + f (m3 )∆x + f (m4 )∆x + f (m5 )∆x + f (m6 )∆x
π π 5π 7π 3π 11π
= ∆x f +f +f +f +f +f
12 4 12 12 4 12
π π π 5π 7π 3π 11π
= · sin + sin + sin + sin + sin + sin
6 12 4 12 12 4 12
≈ 2.02303
Rπ
Therefore, sin(x)dx ≈ 2.02303.
0
Copyright © 2024 Md. Shouvik Iqbal. This book is licensed under a Creative Commons Attribution-NonCommercial-ShareAlike 4.0 International License (CC BY-NC-SA 4.0)
Download for free at https://md-shouvik-iqbal.github.io/calculus
Page 344 Numerical Integration Chapter 5
a b
Figure 5.17
Now, in order to use the Trapezoidal rule approximation to approximate the net
area bounded by the curve of f (x) and the x-axis on the interval [a, b], we divide
b−a
the interval [a, b] into n sub-intervals of equal width ∆x = . This forms
n
the following sub-intervals:
a b
Figure 5.18
Copyright © 2024 Md. Shouvik Iqbal. This book is licensed under a Creative Commons Attribution-NonCommercial-ShareAlike 4.0 International License (CC BY-NC-SA 4.0)
Download for free at https://md-shouvik-iqbal.github.io/calculus
Chapter 5 Numerical Integration Page 345
The height of each trapezoid is equal to the width of each sub-interval, that is,
b−a
h = ∆x = and the lengths of each trapezoid’s sides a and b are f (xi−1 ) and
n
f (xi ), respectively, for i = 1, 2, 3, · · · . Therefore, the area of the ith trapezoid
yields,
f (xi−1 ) + f (xi )
area = ∆x
2
Now, summing these quantities for i = 1, 2, 3, · · · , n, we get the following sum
denoted by Tn .
f (x0 ) + f (x1 ) f (x1 ) + f (x2 ) f (xn−1 ) + f (xn )
Tn = ∆x + ∆x + · · · + ∆x
2 2 2
| {z } | {z } | {z }
1st trapezoid 2nd trapezoid nth trapezoid
f (x0 ) + f (x1 ) f (x1 ) + f (x2 ) f (xn−1 ) + f (xn )
= + + ··· + ∆x
2 2 2
f (x0 ) f (x1 ) f (x1 ) f (x2 ) f (xn−1 ) f (xn )
= + + + + ··· + + ∆x
2 2 2 2 2 2
f (x0 ) f (x1 ) f (x1 ) f (xn−1 ) f (xn−1 ) f (xn )
=
2 + + + · · · + + + ∆x
| 2 {z 2 } | 2 {z 2 } 2
f (x1 ) f (xn−1 )
f (x0 ) f (x n )
=
2 + f (x 1 ) + f (x 2 ) + · · · + f (xn−1 ) + ∆x
| {z } 2
n−1 P
f (xi )
i=1
n−1
!
f (x0 ) X f (xn )
= + f (xi ) + ∆x
2 i=1
2
The Trapezoidal rule approximation gets better and better as the number of
rectangles n increases. This leads to the following definition.
∆x
Tn = f (x0 ) + 2f (x1 ) + 2f (x2 ) + · · · + 2f (xn−1 ) + f (xn )
2
Zb
Where, lim Tn = f (x) dx
n→∞
a
Copyright © 2024 Md. Shouvik Iqbal. This book is licensed under a Creative Commons Attribution-NonCommercial-ShareAlike 4.0 International License (CC BY-NC-SA 4.0)
Download for free at https://md-shouvik-iqbal.github.io/calculus
Page 346 Numerical Integration Chapter 5
x0 = 0 + 0 · 0.25 = 0
x1 = 0 + 1 · 0.25 = 0.25
x2 = 0 + 2 · 0.25 = 0.5
x3 = 0 + 3 · 0.25 = 0.75
x4 = 0 + 4 · 0.25 = 1
f (0) = 02 = 0
f (0.25) = (0.25)2 = 0.0625
f (0.5) = (0.5)2 = 0.25
f (0.75) = (0.75)2 = 0.5625
f (1) = (1)2 = 1
Copyright © 2024 Md. Shouvik Iqbal. This book is licensed under a Creative Commons Attribution-NonCommercial-ShareAlike 4.0 International License (CC BY-NC-SA 4.0)
Download for free at https://md-shouvik-iqbal.github.io/calculus
Chapter 5 Numerical Integration Page 347
Copyright © 2024 Md. Shouvik Iqbal. This book is licensed under a Creative Commons Attribution-NonCommercial-ShareAlike 4.0 International License (CC BY-NC-SA 4.0)
Download for free at https://md-shouvik-iqbal.github.io/calculus
Copyright © 2024 Md. Shouvik Iqbal. This book is licensed under a Creative Commons Attribution-NonCommercial-ShareAlike 4.0 International License (CC BY-NC-SA 4.0)
Download for free at https://md-shouvik-iqbal.github.io/calculus
Chapter 6
These two concepts may seem unrelated—how could the slope of a curve be
possibly connected to the area under it? However, the relation between these
counter-intuitive ideas can ultimately be shown by the Fundamental Theorem of
Calculus, which we aim to study shortly. But, before we dive into that, though, it
is essential to understand the concepts of antiderivatives and indefinite integrals.
Therefore, we begin our discussion with these foundational concepts.
§ 6.1 Antidifferentiation
The concept of an inverse operation is a key concept in mathematics. For ex-
ample, in arithmetic, addition and subtraction are inverse of one another as one
operation can reverse the effect of the other. Similarly, multiplication and di-
vision are inverses of one another as applying one operation can be undone by
applying the other.
349
Copyright © 2024 Md. Shouvik Iqbal. This book is licensed under a Creative Commons Attribution-NonCommercial-ShareAlike 4.0 International License (CC BY-NC-SA 4.0)
Download for free at https://md-shouvik-iqbal.github.io/calculus
Page 350 Antidifferentiation Chapter 6
This concept may seem complicated and, quite frankly, somewhat confusing.
Nevertheless, to properly understand the concept, consider the following.
derivative
Now, suppose another function F (x). If F (x) is differentiable, then its derivative
is denoted as F 0 (x). That is,
derivative
equ
al
Copyright © 2024 Md. Shouvik Iqbal. This book is licensed under a Creative Commons Attribution-NonCommercial-ShareAlike 4.0 International License (CC BY-NC-SA 4.0)
Download for free at https://md-shouvik-iqbal.github.io/calculus
Chapter 6 Antidifferentiation Page 351
antiderivative
That is, F (x) is an antiderivative of f (x). Based on that, we have the following
definition of an antiderivative of a function.
F 0 (x) = f (x)
for all x ∈ I
Solution. If F (x) = x2 , then the derivative is F 0 (x) = 2x. Now, if f (x) = 2x,
then an antiderivative of f (x) is F (x) = x2 , for the reason that F 0 (x) = f (x).
x3
Now, if f (x) = x , then an antiderivative of f (x) is F (x) = , for the reason
2
3
that F 0 (x) = f (x).
Copyright © 2024 Md. Shouvik Iqbal. This book is licensed under a Creative Commons Attribution-NonCommercial-ShareAlike 4.0 International License (CC BY-NC-SA 4.0)
Download for free at https://md-shouvik-iqbal.github.io/calculus
Page 352 Antidifferentiation Chapter 6
x3
Now, notice that although is an antiderivative of x2 , it is not—however—the
3
x3
antiderivative of x2 . This is because the function F (x) = is not the only
3
function whose derivative is x2 . Actually, every function of the form F (x) =
x3
+ C, where C ∈ R a constant, has a derivative of x2 . This is because,
3
d x3
d
F (x) = +C
dx dx 3
0
d x3
d >
= + (C)
dx 3 dx
= x2 + 0
= x2
x3
This implies that the most general antiderivative of x is + C, where C ∈ R, 2
3
and thus we have the following theorem.
F (x) + C
This implies that all the antiderivatives of a function differ only by a constant C
and therefore, they belong to one general family of functions, that is, functions
of the form F (x) + C.
It’s fascinating how each differentiation formula, when reversed, transforms into
an antidifferentiation formula. For example, as we’ve just seen, an antiderivative
x3 xn+1
of x2 is , suggesting an antiderivative of xn to be , where n 6= −1,
3 n+1
because
n+1
d x 1 d
= · xn+1
dx n + 1 n + 1 dx
1
· (n + 1) · xn+1−1
=
n+1
Copyright © 2024 Md. Shouvik Iqbal. This book is licensed under a Creative Commons Attribution-NonCommercial-ShareAlike 4.0 International License (CC BY-NC-SA 4.0)
Download for free at https://md-shouvik-iqbal.github.io/calculus
Chapter 6 Antidifferentiation Page 353
1
= ·
(n+1)
· xn
n+1
= xn where n 6= −1
d
Similarly, an antiderivative of sin(x) is − cos(x), because (− cos(x)) = −(− sin(x)) =
dx
d x
sin(x). An antiderivative of e is e , because
x x
(e ) = ex , and so on. In the
dx
table below, a list of functions along with their particular antiderivatives are
presented for a quick reference, where each of the particular antiderivatives can
be verified by differentiating and matching with the original function.
Table 6.1
Particular
Function
antiderivative
0 C∈R
1 x
n nx
xn+1
xn , n 6= 1
n+1
nxn−1 xn
1 √
√ x
2 x
ex ex
ax · ln(a) ax
1
ln(x) = loge (x)
x
1
loga (x)
x ln (a)
cos(x) sin(x)
sin(x) − cos(x)
Copyright © 2024 Md. Shouvik Iqbal. This book is licensed under a Creative Commons Attribution-NonCommercial-ShareAlike 4.0 International License (CC BY-NC-SA 4.0)
Download for free at https://md-shouvik-iqbal.github.io/calculus
Page 354 Antidifferentiation Chapter 6
1
I Example 6.1.3. What is the general antiderivative of f (x) = √
2 x
1
Solution. The antiderivative of f (x) = √ is,
2 x
√
F (x) = x+C
Because,
d√ d
F 0 (x) = x+ C
dx dx
1
= √
2 x
∴ F 0 (x) = f (x)
1 √
Therefore, the antiderivative of f (x) = √ is F (x) = x+C, where C ∈ R.
2 x
Copyright © 2024 Md. Shouvik Iqbal. This book is licensed under a Creative Commons Attribution-NonCommercial-ShareAlike 4.0 International License (CC BY-NC-SA 4.0)
Download for free at https://md-shouvik-iqbal.github.io/calculus
Chapter 6 Antidifferentiation Page 355
F (x) = − cos(x) + C
Because,
d d
F 0 (x) = (− cos(x)) + C
dx dx
= sin(x)
∴ F 0 (x) = f (x)
1
I Example 6.1.5. What is the general antiderivative of f (x) = √
1 − x2
1
Solution. The general antiderivative of f (x) = √ is,
1 − x2
Because,
d d
F 0 (x) = sin−1 (x) + C
dx dx
1
=√
1 − x2
∴ F 0 (x) = f (x)
1
Therefore, the antiderivative of f (x) = √ is F (x) = sin−1 (x) + C, where
1−x2
C ∈ R.
Copyright © 2024 Md. Shouvik Iqbal. This book is licensed under a Creative Commons Attribution-NonCommercial-ShareAlike 4.0 International License (CC BY-NC-SA 4.0)
Download for free at https://md-shouvik-iqbal.github.io/calculus
Page 356 Antidifferentiation Chapter 6
The notation above is called the indefinite integral and read as “the integral of
f (x) dee x.”
Now, observe carefully that the notation for a collection of antiderivatives is the
Zb
same as the notation for a definite integral f (x)dx, except for the core differ-
a
ence that a definite integral is a numerical value that represents the net signed
area of the region bounded by the curve of a function f (x) and the x-axis, from
x = a to x = b. On the other hand, an indefinite integral is a family of functions
that represents the collection of all antiderivatives of the function f (x). Clearly,
a numerical value and a family of functions cannot be the same thing and there-
fore the definite and indefinite integral belongs to two distinct concepts. This
distinction is also reflected in their notation for the limits of integration. That
Zb
is, a definite integral includes limits of integration in its integral sign , while
Z a
The reason for using such similar notation for two distinct concepts and causing
temporary confusion will become clear once we understand the Fundamental
Theorem of Calculus. Without giving too much away, the Fundamental The-
orem of Calculus essentially shows that a definite integral can be calculated
without using any limit of approximation by the use of an indefinite integral.
And, therefore such similar notation for an indefinite integral is strategically
chosen for later convenience.
Z
Similar to the notation of a definite integral, the symbol is also followed by
Copyright © 2024 Md. Shouvik Iqbal. This book is licensed under a Creative Commons Attribution-NonCommercial-ShareAlike 4.0 International License (CC BY-NC-SA 4.0)
Download for free at https://md-shouvik-iqbal.github.io/calculus
Chapter 6 Antidifferentiation Page 357
the function f (x) called the integrad and the differential dx called the variable
of integration in the notation of an indefinite integral. The differential dx is
simply there to remind the fact that x is the independent variable, similar to
d
the role in the Leibniz notation for derivative, f (x).
dx
This implies that given a function f (x), the indefinite integral of f (x), that is,
Z
f (x)dx
represents all the antiderivative of the function f (x) that are of the form F (x) +
C. This suggests that
Remark 6.1.1. Z
f (x)dx = F (x) + C
Having discussed the concept of antiderivatives and its notation, we now aim to
show how Differentiation and Antidifferentiation are indeed inverse operations
of each other. For that, recall that for a given function f (x), the family of all
antiderivatives of f (x) is represented as,
Z
f (x)dx = F (x) + C
Copyright © 2024 Md. Shouvik Iqbal. This book is licensed under a Creative Commons Attribution-NonCommercial-ShareAlike 4.0 International License (CC BY-NC-SA 4.0)
Download for free at https://md-shouvik-iqbal.github.io/calculus
Page 358 Antidifferentiation Chapter 6
The result above suggests that when a function f (x) is first antidifferentiated
and then differentiated, it results in the same function f (x) or the function f (x)
is unchanged.
Similarly, we now show that when a function is first differentiated and then
antidifferentiated, it also results in the same function or does not change at all.
In order to do that, recall that for a given function F (x), if its antiderivative is
f (x) and is differentiated, then we get,
d
f (x) = f 0 (x)
dx
Now, antidifferentiating both sides of the equation above (that is, finding the
family of all antiderivatives of the function f 0 (x)) yields,
Z Z
d
f (x)dx = f 0 (x)dx
dx
Z
F (x)dx = f (x) + C
Copyright © 2024 Md. Shouvik Iqbal. This book is licensed under a Creative Commons Attribution-NonCommercial-ShareAlike 4.0 International License (CC BY-NC-SA 4.0)
Download for free at https://md-shouvik-iqbal.github.io/calculus
Chapter 6 Antidifferentiation Page 359
Theorem 6.1.2.
Z Z
d d
f (x)dx = f (x) and f (x)dx = f (x) + C
dx dx
The theorem above suggests that differentiation and antidifferentiation are in-
deed inverse operations of each others. Since they are inverses of each other,
therefore a few indefinite integrals (i.e., the collection/family of all antideriva-
tives) can easily be obtained by reversing the derivative of various functions, as
listed below.
Note that each indefinite integral listed below can be verified by differentiat-
ing the function on the right-hand side of the equation and confirming that it
matches the integrand of the corresponding indefinite integral.
This intuition suggests that if the “antiderivative” on the right side of the equa-
tion is differentiated, it equals the “derivative” in the integrand on the left side.
Z Z
1. dx = x + C 6. ax ln(a)dx = ax + C
xn+1
Z
dx
Z
2. n
x dx = +C 7. = ln(x) + C
n+1 x
√
Z Z
1 dx
3. √ dx = x + C 8. = loga (x) + C
2 x x ln(a)
Z
1 x
Z
4. x
a dx = a +C 9. sin(x)dx = − cos(x) + C
ln(a)
Z Z
5. x
e dx = e + C x
10. cos(x)dx = sin(x) + C
Copyright © 2024 Md. Shouvik Iqbal. This book is licensed under a Creative Commons Attribution-NonCommercial-ShareAlike 4.0 International License (CC BY-NC-SA 4.0)
Download for free at https://md-shouvik-iqbal.github.io/calculus
Page 360 Antidifferentiation Chapter 6
Z Z
dx
11. sec (x)dx = tan(x) + C
2
15. √ = sin−1 (x) + C
1−x 2
Z
12. csc2 (x)dx = − cot(x) + C Z
dx
16. = tan−1 (x) + C
1 + x2
Z
13. sec(x) tan(x)dx = sec(x) + C
Z
Z dx
14. csc(x) cot(x)dx = − csc(x)+C 17. √ = sec−1 (x) + C
2
|x| x − 1
Z
I Example 6.1.6. Evaluate 7xdx
Z
0
I Example 6.1.7. Evaluate dx
x
Solution. Using the indefinite integrals, we write,
Z Z
0
dx = 0dx
x
Copyright © 2024 Md. Shouvik Iqbal. This book is licensed under a Creative Commons Attribution-NonCommercial-ShareAlike 4.0 International License (CC BY-NC-SA 4.0)
Download for free at https://md-shouvik-iqbal.github.io/calculus
Chapter 6 Antidifferentiation Page 361
Z
=0· dx
=0·x+C
=0+C
Z
0
Therefore, dx = 0 + C
x
Z
I Example 6.1.8. Evaluate x2 + x3 + x5 dx
x3 x4 x6
= + + +C
3 4 6
x3 x4 x6
Z
Therefore, 2 3 5
x +x +x dx = + + +C
3 4 6
Z
I Example 6.1.9. Evaluate sin(5x)dx
Therefore,
Z Z
d
−5 sin(5x)dx = cos(5x)dx
Z dx
−5 · sin(5x)dx = cos(5x) + C
Z
1
sin(5x)dx = − cos(5x) + C
5
Z
1
Therefore, sin(5x)dx = − cos(5x) + C
5
For the cases illustrated in the example above, let us now generalize the indefinite
integral for any constant a 6= 0, as follows.
Copyright © 2024 Md. Shouvik Iqbal. This book is licensed under a Creative Commons Attribution-NonCommercial-ShareAlike 4.0 International License (CC BY-NC-SA 4.0)
Download for free at https://md-shouvik-iqbal.github.io/calculus
Page 362 Antidifferentiation Chapter 6
Z
dx
I Example 6.1.10. Evaluate
2x
Z
dx 1
Solution. If we let a = 2, then the integral matches the form = ln |x|+C.
ax a
Therefore,
Z
1 1
dx = ln |x| + C
2x 2
Z
1 1
Therefore, dx = ln |x| + C.
2x 2
Copyright © 2024 Md. Shouvik Iqbal. This book is licensed under a Creative Commons Attribution-NonCommercial-ShareAlike 4.0 International License (CC BY-NC-SA 4.0)
Download for free at https://md-shouvik-iqbal.github.io/calculus
Chapter 6 Antidifferentiation Page 363
Z
3
I Example 6.1.11. Evaluate √ dx
25 − x2
Z
dx
Solution. If we let a = 5, then the integral matches the form √ =
x a2 − x2
sin−1 + C. Therefore,
a
Z Z
3 1
√ dx = 3 · √ dx
25 − x2 52 − x2
−1 x
= 3sin +C
5
Z
3 −1 x
Therefore, √ dx = 3sin +C
25 − x2 5
√
x2 + x x2 − 25 + 9
Z
I Example 6.1.12. Evaluate √ √ dx
9x x2 − 25 + x3 x2 − 25
√
x2 + x x2 − 25 + 9
Z
Solution. In order to evaluate the integral √ √ dx, we
9x x2 − 25 + x3 x2 − 25
simplify the integrand as follows,
√ √
x2 + x x2 − 25 + 9 x2 + x x2 − 25 + 9
√ √ = √
9x x2 − 25 + x3 x2 − 25 x x2 − 25 (9 + x2 )
√
x x2 − 25 + x2 + 9
= √
x x2 − 25 (x2 + 9)
√
x x2− 25 x2+9
= √
+ √ 2
x x2 − 25 (x2 + 9) x x2 − 25x
+9
1 1
= 2 + √
x + 9 x x2 − 25
This implies that,
√
x2 + x x2 − 25 + 9
Z Z
1 1
√ √ dx = + √ dx
9x x2 − 25 + x3 x2 − 25 x2 + 9 x x2 − 25
Z Z
1 1
= dx + √ dx
x2 + 9 x x2 − 25
Z Z
1 1
= 2 2
dx + √ dx
x +3 x x − 52
2
Z Z
dx 1 −1 x
dx
The integrals above matches the form = tan +C and √ =
a2 + x2 a a x x2 − a2
1 −1 x
sec + C, for a = 3 and a = 5, respectively. Therefore, we write,
a a
√
x2 + x x2 − 25 + 9
Z Z Z
1 1
√ √ dx = 2 2
dx + √ dx
9x x2 − 25 + x3 x2 − 25 3 +x x x2 − 52
Copyright © 2024 Md. Shouvik Iqbal. This book is licensed under a Creative Commons Attribution-NonCommercial-ShareAlike 4.0 International License (CC BY-NC-SA 4.0)
Download for free at https://md-shouvik-iqbal.github.io/calculus
Page 364 The Fundamental Theorem of Calculus (FTC) Chapter 6
1 x 1 x
= tan−1 + sec−1 +C
3 3 5 5
√
x2 + x x2 − 25 + 9
Z
1 x 1 x
Therefore, √ √ dx = tan−1 + sec−1 +C
9x x2 − 25 + x3 x2 − 25 3 3 5 5
Now, up to this point, these two concepts may have seemed unrelated. How-
ever, the Fundamental Theorem of Calculus now states that differentiation and
integration are actually opposite processes. This might seem contradictory, as
we have discussed, in the previous section that, it is antidifferentiation—not
integration—that serves as the inverse of differentiation.
However, the key point to realize here is that the Fundamental Theorem of Cal-
culus now allows us to perform integration in terms of antidifferentiation. That
is, we can now evaluate definite integrals using antiderivatives, rather than re-
lying on the limit of a Riemann Sum.
It is for this reason that we can say—by the Fundamental Theorem of Calculus—
that “integration” and “antidifferentiation” are essentially the same thing when
evaluating a definite integral.
Copyright © 2024 Md. Shouvik Iqbal. This book is licensed under a Creative Commons Attribution-NonCommercial-ShareAlike 4.0 International License (CC BY-NC-SA 4.0)
Download for free at https://md-shouvik-iqbal.github.io/calculus
Chapter 6 The Fundamental Theorem of Calculus (FTC) Page 365
Having introduced the Fundamental Theorem of Calculus and its core idea that
integration can be performed through antidifferentiation, we will now explore
how this is possible. Specifically, we will derive the first part of the theorem,
which then leads to the derivation of the second part. This, in turn, will enable
us to evaluate definite integrals using antiderivatives.
In order to do that, consider a function y = f (t) defined over an interval [a, b]. If
the function f (t) is integrable over [a, b], then the exact area A under the curve
f (t) and above the t-axis, over the interval [a, b], that is, from t = a to t = b, is
given by the definite integral A provided below,
Zb
A= f (t) dt
a
moveable
fixed bound bound
a x
Figure 6.1
Notice here that the definite integral above is not able to produce any value
as an area unless the upper limit of integration is specified. That is, since the
upper limit of integration is variable x, therefore for each input in the upper limit
x, there would be a corresponding output as a numerical value of the definite
Copyright © 2024 Md. Shouvik Iqbal. This book is licensed under a Creative Commons Attribution-NonCommercial-ShareAlike 4.0 International License (CC BY-NC-SA 4.0)
Download for free at https://md-shouvik-iqbal.github.io/calculus
Page 366 The Fundamental Theorem of Calculus (FTC) Chapter 6
This is the exact area from a to x. Since x is a variable that can take on values
greater than a, therefore, an increment h > 0 in x would yield,
x+h
Z
A(x + h) = f (t) dt
a
This implies that A(x + h) is the exact area from a to x + h as shown as the
shaded region in the figure below,
a x x+h
Figure 6.2
Recall that A(x) denotes the exact area from a to x. Therefore, subtracting
A(x + h) from A(x) would yield the exact area from x to x + h as shown below,
h
subtracted area
a x x+h
Figure 6.3
Copyright © 2024 Md. Shouvik Iqbal. This book is licensed under a Creative Commons Attribution-NonCommercial-ShareAlike 4.0 International License (CC BY-NC-SA 4.0)
Download for free at https://md-shouvik-iqbal.github.io/calculus
Chapter 6 The Fundamental Theorem of Calculus (FTC) Page 367
Now, if the increment h > 0 is very small then the exact area of A(x + h) − A(x)
is,
as shown below,
a x+h
x
Figure 6.4
Now, dividing both sides by h and then taking the limit as h → 0 yields,
A(x + h) − A(x)
≈ f (x)
h
A(x + h) − A(x)
lim = lim f (x)
h→0 h h→0
0
A (x) = f (x) (2)
A(x + h) − A(x)
Here, lim = A0 (x) by the definition of a derivative. Since
h→0 h
A0 (x) = f (x) in the equation above, therefore A(x) is an antiderivative of f (x),
by the definition of antiderivatives. As a result, we have the following theorem.
Copyright © 2024 Md. Shouvik Iqbal. This book is licensed under a Creative Commons Attribution-NonCommercial-ShareAlike 4.0 International License (CC BY-NC-SA 4.0)
Download for free at https://md-shouvik-iqbal.github.io/calculus
Page 368 The Fundamental Theorem of Calculus (FTC) Chapter 6
A0 (x) = f (x)
That is, the area function A(x) is an antiderivative of f (x), by the definition
of antiderivatives.
For that, notice that the area function A(x) above is an antiderivative of f (x).
It is not, however, the only antiderivative of f (x). From previous discussions, we
know that all antiderivatives of a function differ by a constant. So, we let F (x)
be another antiderivative of f (x), such that F (x) = A(x)−C or A(x) = F (x)+C.
Now, to derive the second part of the theorem, we need to determine the value
of C in the equation above. In order to do that, notice that when x = a, the
area function yields,
Za
A(x) = f (t) dt
a
=0
A(a) = 0
Copyright © 2024 Md. Shouvik Iqbal. This book is licensed under a Creative Commons Attribution-NonCommercial-ShareAlike 4.0 International License (CC BY-NC-SA 4.0)
Download for free at https://md-shouvik-iqbal.github.io/calculus
Chapter 6 The Fundamental Theorem of Calculus (FTC) Page 369
F (a) + C = 0
C = −F (a)
Now, substituting the value of C in A(x) = F (x) + C yields,
A(x) = F (x) + (−F (a))
= F (x) − F (a)
This implies that the definite integral A(x) from t = a to t = x (where x is a
variable point) can be calculated by evaluating and subtracting the antideriva-
tive F (a) from F (x), without using the limit of an approximation.
This further implies that to calculate the definite integral A(x) from t = a to
t = x, that is, over the interval [a, b]; we need to evaluate the antiderivative F (x)
at both endpoints of the interval [a, b] and subtract F (a) from F (b). Therefore,
we have the following theorem.
Zb
f (x) dx = F (b) − F (a)
a
The part two of the Fundamental Theorem of Calculus above suggests that
Zb
the definite integral f (x) dx can be evaluated in terms of antiderivatives by
a
evaluating F (x) at both endpoints of [a, b], that is, F (a) and F (b), and then
subtracting F (a) from F (b).
Notice that if it’s shown that the integral in the theorem above is being evaluated,
then the following notation is conveniently used.
Zb ib
f (x) dx = F (x)
a
a
= F (b) − F (a)
That is, the difference F (b) − F (a) is denoted as follows,
ib h ib
F (x) = F (b) − F (a) or F (x) = F (b) − F (a)
a a
Copyright © 2024 Md. Shouvik Iqbal. This book is licensed under a Creative Commons Attribution-NonCommercial-ShareAlike 4.0 International License (CC BY-NC-SA 4.0)
Download for free at https://md-shouvik-iqbal.github.io/calculus
Page 370 The Fundamental Theorem of Calculus (FTC) Chapter 6
Chap. 6 / Sec. 6.2 / Subsec. 6.2.1 : The Inverse Relationship between Differ-
entiation and Integration
Previously, we have seen, in antidifferentiation, that when a function f (x) is first
antidifferentiated and then differentiated, it results in the same function f (x).
That is, Z
d
f (x)dx = f (x)
dx
Similarly, we see in the first part of the Fundamental Theorem of Calculus that
when a function f (t) is first integrated and then differentiated, it results in the
same. That is, from the equation (3), we get,
Zx
d
f (t)dx = f (x)
dx
a
Similarly, we see in the second part of the Fundamental Theorem of Calculus that
when a function f (x) is first differentiated and then integrated over the interval
[a, b], it results in the same function f (x), evaluated at both the endpoints a and
b. That is,
Zb
d
f (x)dx = f (b) − f (a)
dx
a
d
where, f (x) is an antiderivative of f (x) = f 0 (x) (i.e., simply, F (x))
dx
Copyright © 2024 Md. Shouvik Iqbal. This book is licensed under a Creative Commons Attribution-NonCommercial-ShareAlike 4.0 International License (CC BY-NC-SA 4.0)
Download for free at https://md-shouvik-iqbal.github.io/calculus
Chapter 6 The Fundamental Theorem of Calculus (FTC) Page 371
This is the reason why the terms “antidifferentiation” and “integration” are of-
ten used interchangeably, and both “integration” and “antidifferentiation” are
considered the inverse operations of “differentiation” when evaluating a definite
integral using the Fundamental Theorem of Calculus.
Z1
I Example 6.2.1. Evaluate x2 dx
0
Z1
Solution. In order to evaluate the definite integral x2 dx, we only need to find
0
the antiderivative F (x) of the function f (x) = x , so that F 0 (x) yields f (x). We 2
Now, notice carefully that since definite integrals yield a specific numerical value,
the constant of integration (C) is not relevant as it cancels out during the eval-
uation process. That is,
Zb h ib
f (x)dx = F (x) + C
a
a
Copyright © 2024 Md. Shouvik Iqbal. This book is licensed under a Creative Commons Attribution-NonCommercial-ShareAlike 4.0 International License (CC BY-NC-SA 4.0)
Download for free at https://md-shouvik-iqbal.github.io/calculus
Page 372 The Fundamental Theorem of Calculus (FTC) Chapter 6
= (F (b) + C) − (F (a) + C)
= F (b) + C − F (a) − C
= F (b) − F (a)
Notice also that, since theZ difference F (b) − F (a) can be written shortly as
ib
F (x) , and since F (x) = f (x)dx, therefore, we can write,
a
Zb
f (x)dx = F (b) − F (a)
a
ib
= F (x)
a
Z b
= f (x)dx
a
Remark 6.2.1.
Zb Z b
f (x)dx = f (x)dx
a
a
Zπ
I Example 6.2.2. Evaluate sin(x)dx
0
= − (cos(π) − cos(0))
= − (−1 − 1)
=2
Zπ
Therefore, sin(x)dx = 2
0
Copyright © 2024 Md. Shouvik Iqbal. This book is licensed under a Creative Commons Attribution-NonCommercial-ShareAlike 4.0 International License (CC BY-NC-SA 4.0)
Download for free at https://md-shouvik-iqbal.github.io/calculus
Chapter 6 The Fundamental Theorem of Calculus (FTC) Page 373
Zπ
I Example 6.2.3. Evaluate e2x dx
e
Z+e
edx
I Example 6.2.4. Evaluate
x
−e
Z+e Z+e
edx dx
=e
x x
−e −e
= e · [ln |x|]+e
−e
= e · (ln |e| − ln |−e|)
= e · (1 − 1)
=0
Z+e
edx
Therefore, =0
x
−e
Copyright © 2024 Md. Shouvik Iqbal. This book is licensed under a Creative Commons Attribution-NonCommercial-ShareAlike 4.0 International License (CC BY-NC-SA 4.0)
Download for free at https://md-shouvik-iqbal.github.io/calculus
Page 374 The Fundamental Theorem of Calculus (FTC) Chapter 6
Z1
dx
I Example 6.2.5. Evaluate
1 − sin(x)
0
Copyright © 2024 Md. Shouvik Iqbal. This book is licensed under a Creative Commons Attribution-NonCommercial-ShareAlike 4.0 International License (CC BY-NC-SA 4.0)
Download for free at https://md-shouvik-iqbal.github.io/calculus
Chapter 6 The Fundamental Theorem of Calculus (FTC) Page 375
Copyright © 2024 Md. Shouvik Iqbal. This book is licensed under a Creative Commons Attribution-NonCommercial-ShareAlike 4.0 International License (CC BY-NC-SA 4.0)
Download for free at https://md-shouvik-iqbal.github.io/calculus
Copyright © 2024 Md. Shouvik Iqbal. This book is licensed under a Creative Commons Attribution-NonCommercial-ShareAlike 4.0 International License (CC BY-NC-SA 4.0)
Download for free at https://md-shouvik-iqbal.github.io/calculus
Chapter 7
Methods of Integration
However, this approach has its own limitations. So far, we only know the an-
tiderivatives for a limited set of functions, specifically those whose derivatives
we’ve encountered before. This is far from sufficient. For example, what is the
antiderivative of f (x) = sec(x)? In other words, what is that function F (x)
such that when it is differentiated, it equals f (x)? As of now, we don’t have an
answer, which means we are unable to evaluate any definite integral involving
f (x) = sec(x). This presents a fundamental challenge.
377
Copyright © 2024 Md. Shouvik Iqbal. This book is licensed under a Creative Commons Attribution-NonCommercial-ShareAlike 4.0 International License (CC BY-NC-SA 4.0)
Download for free at https://md-shouvik-iqbal.github.io/calculus
Page 378 Integration by Substitution Chapter 7
Conceptually, this method can be thought of as a “backward rule” for the Chain
Rule in differentiation. In differentiation, the Chain Rule allows us to differen-
tiate a composite function by breaking it down into its constituent parts. For
example, if y = f (g(x)), then the Chain rule states that the derivative of y
with respect to x is f 0 (g(x)) · g 0 (x). Now, if we were to integrate the function
f 0 (g(x)) · g 0 (x), the substitution rule allows us to simplify the integral by chang-
ing its variables.
du
Now, since u = g(x), therefore = g 0 (x) or in the differential form:
dx
du = g 0 (x)dx
Z
This suggests that f (g (x)) g 0 (x) dx = F (g (x))+C can be expressed in terms
of the variable u by substituting u = g(x) and du = g(x)dx, as follows:
Z
f (g (x)) g 0 (x) dx = F (g (x)) +C
| {z } | {z } | {z }
u du u
Copyright © 2024 Md. Shouvik Iqbal. This book is licensed under a Creative Commons Attribution-NonCommercial-ShareAlike 4.0 International License (CC BY-NC-SA 4.0)
Download for free at https://md-shouvik-iqbal.github.io/calculus
Chapter 7 Integration by Substitution Page 379
Z
f (u) du = F (u) + C
Z
By doing so, we can transform a complex integral f (g(x))g 0 (x) dx = F (g(x))+
Z
C with respect to x, into a simpler integral f (u) du = F (u) + C with respect
to a new variable u. This technique is useful because the new integral with
respect to u is often easier to solve than the original integral with respect to x.
Therefore, we have the following theorem.
= F (u) + C
= F (g (x)) + C
The bottom line is that finding the correct substitution often requires some trial
and error. It’s common to make an incorrect guess, so if the first choice doesn’t
work, simply try a different one.
Copyright © 2024 Md. Shouvik Iqbal. This book is licensed under a Creative Commons Attribution-NonCommercial-ShareAlike 4.0 International License (CC BY-NC-SA 4.0)
Download for free at https://md-shouvik-iqbal.github.io/calculus
Page 380 Integration by Substitution Chapter 7
Z
I Example 7.1.1. Evaluate (3x + 4)4 dx
u5
Z
(3x + 4)4 dx = +C
15
(3x + 4)5
= +C
15
(3x + 4)5
Z
Therefore, (3x + 4)4 dx = +C
15
Z
I Example 7.1.2. Evaluate
p
x x2 + 2dx
√
Solution. In order to solve this integral, let the variable u = x2 + 2, so that,
du d
x2 + 2
=
dx dx
= 2x
du = 2xdx
1
∴ dx = du
2x
This implies that,
√ 1
Z p Z
x x2 + 2dx = x u du
2x
Copyright © 2024 Md. Shouvik Iqbal. This book is licensed under a Creative Commons Attribution-NonCommercial-ShareAlike 4.0 International License (CC BY-NC-SA 4.0)
Download for free at https://md-shouvik-iqbal.github.io/calculus
Chapter 7 Integration by Substitution Page 381
Z
1 1
= x · u 2 du
2x
3
1 u2
= · +C
2 3
2
3
2u 2
= +C
6
3
u2
= +C
3
3
x2 + 2 2
= +C
3
3
x2 + 2
Z 2
Therefore,
p
x x2 + 2dx = +C
3
Z
1
I Example 7.1.3. Evaluate dx
x ln(x)
Solution. In order to solve this integral, let the variable u = ln(x), so that,
du d
= ln(x)
dx dx
1
=
x
1
∴ du = dx
x
This implies that,
Z Z
1 1 1
dx = · dx
x ln(x) ln(x) x
| {z } |{z}
u du
Z
1
= du
u
= ln |u| + C
= ln |ln(x)| + C
Z
1
Therefore, dx = ln |ln(x)| + C
x ln(x)
Sometimes the choice of u works for more than one substitution. For example,
consider the following example.
Copyright © 2024 Md. Shouvik Iqbal. This book is licensed under a Creative Commons Attribution-NonCommercial-ShareAlike 4.0 International License (CC BY-NC-SA 4.0)
Download for free at https://md-shouvik-iqbal.github.io/calculus
Page 382 Integration by Substitution Chapter 7
Z
x
I Example 7.1.4. Evaluate √ dx
x+1
Solution. The integral above can be evaluated in multiple ways using the substi-
tution rule. This is because the choice of u is not apparently visible. Whether
√
we choose u = x + 1 or u = x + 1, both yield the same answer.
du
=1
dx
∴ du = dx
3 3 1
= u 2 − 2u 2 + C
2
3 3 1
= (x + 1) 2 − 2(x + 1) 2 + C
2
√ √
Choosing u = x + 1: if u = x + 1, then u2 = x + 1 or x = u2 − 1, and
d d 2 d
x= u − 1
du du du
dx
= 2u
du
∴ dx = 2udu
u2 − 1
Z Z
x
√ dx = 2udu
x+1 u
Copyright © 2024 Md. Shouvik Iqbal. This book is licensed under a Creative Commons Attribution-NonCommercial-ShareAlike 4.0 International License (CC BY-NC-SA 4.0)
Download for free at https://md-shouvik-iqbal.github.io/calculus
Chapter 7 Integration by Substitution Page 383
u2 − 1
Z
2udu =
Z u
u2 − 1 du
=2·
Z Z
2
=2· u du − du
3
u
=2· −u +C
3
2u3
= − 2u + C
3
√ 3
2 x+1 √
= −2 x+1+C
3
2 3 √
= (x + 1) 2 − 2 x + 1 + C
3
√
Z
x 2
Which yields the same result. Therefore,
3
√ dx = (x + 1) 2 −2 x + 1+
x+1 3
C
Z
I Example 7.1.5. Evaluate (ax + b)n dx
Copyright © 2024 Md. Shouvik Iqbal. This book is licensed under a Creative Commons Attribution-NonCommercial-ShareAlike 4.0 International License (CC BY-NC-SA 4.0)
Download for free at https://md-shouvik-iqbal.github.io/calculus
Page 384 Integration by Substitution Chapter 7
(ax + b)n+1
Z
Therefore, (ax + b)n dx = +C
a (n + 1)
Remark 7.1.1.
(ax + b)n+1
Z
(ax + b)n dx = +C n 6= 1
a (n + 1)
Z
I Example 7.1.6. Evaluate sin (ax + b)dx
u = ax + b =⇒ du = adx
du
This implies that dx = . Thus, we write,
a
Z Z
du
sin (ax + b) dx = sin (u)
a
Z
1
= · sin (u) du
a
1
= · (− cos (u))
a
1
= − cos (u)
a
Since u = ax + b, therefore
Z
1
sin (ax + b)dx = − cos (ax + b)
a
Z
1
Therefore, sin (ax + b)dx = − cos (ax + b) + C
a
Remark 7.1.2.
Z
1
sin (ax + b)dx = − cos (ax + b) + C
a
Z
I Example 7.1.7. Evaluate cos (ax + b) dx
u = ax + b =⇒ du = adx
Copyright © 2024 Md. Shouvik Iqbal. This book is licensed under a Creative Commons Attribution-NonCommercial-ShareAlike 4.0 International License (CC BY-NC-SA 4.0)
Download for free at https://md-shouvik-iqbal.github.io/calculus
Chapter 7 Integration by Substitution Page 385
du
This implies that dx = . Thus, we write,
a
Z Z
du
cos (ax + b) dx = cos (u)
a
Z
1
= · cos (u) du
a
1
= · sin (u) + C
a
Since u = ax + b, therefore
Z
1
cos (ax + b) dx = sin (ax + b) + C
a
Z
1
Therefore, cos (ax + b) dx = sin (ax + b) + C
a
Remark 7.1.3.
Z
1
cos (ax + b) dx = sin (ax + b) + C
a
Z
I Example 7.1.8. Evaluate tan (ax + b) dx
u = ax + b =⇒ du = adx
du
This implies that dx = . Thus, we write,
a
Z Z
du
tan (ax + b) dx = tan (u)
a
Z
1
= · tan (u) du
a
sin (u)
Z
1
= · du
a cos (u)
sin (u)
Z
Now, to solve the integral du, we apply the substitution rule once
cos (u)
again. That is, we let
v = cos(u) =⇒ dv = − sin(u)du
Copyright © 2024 Md. Shouvik Iqbal. This book is licensed under a Creative Commons Attribution-NonCommercial-ShareAlike 4.0 International License (CC BY-NC-SA 4.0)
Download for free at https://md-shouvik-iqbal.github.io/calculus
Page 386 Integration by Substitution Chapter 7
Thus, we write,
sin (u)
Z Z
1
du = (−dv)
cos (u) v
Z
1
=− dv
v
= − ln |v| + C
Remark 7.1.4.
Z
1 1
tan (ax + b) dx = − ln |cos (ax + b)| + C = ln |sec (ax + b)| + C
a a
Z
I Example 7.1.9. Evaluate cot (ax + b) dx
u = ax + b =⇒ du = adx
du
This implies that dx = . Thus, we write,
a
Z Z
du
cot (ax + b) dx = cot (u)
a
Copyright © 2024 Md. Shouvik Iqbal. This book is licensed under a Creative Commons Attribution-NonCommercial-ShareAlike 4.0 International License (CC BY-NC-SA 4.0)
Download for free at https://md-shouvik-iqbal.github.io/calculus
Chapter 7 Integration by Substitution Page 387
Z
1
= · cot (u) du
a
cos (u)
Z
1
= · du
a sin (u)
cos (u)
Z
Now, to solve the integral du, we apply the substitution rule once
sin (u)
more. That is, we let,
v = sin(u) =⇒ dv = cos(u)du
Thus, we write,
cos (u)
Z Z
1
du = dv
sin (u) v
= ln |v| + C
Remark 7.1.5.
Z
1 1
cot (ax + b) dx = ln |sin (ax + b)| + C = − ln |csc (ax + b)| + C
a a
Z
I Example 7.1.10. Evaluate sec (ax + b) dx
Copyright © 2024 Md. Shouvik Iqbal. This book is licensed under a Creative Commons Attribution-NonCommercial-ShareAlike 4.0 International License (CC BY-NC-SA 4.0)
Download for free at https://md-shouvik-iqbal.github.io/calculus
Page 388 Integration by Substitution Chapter 7
u = ax + b =⇒ du = adx
du
This implies that dx = . Thus, we write,
a
Z Z
du
sec (ax + b) dx = sec (u)
a
Z
1
= · sec (u) du
a
tan (u) + sec (u)
Z
1
= · sec (u) · du
a tan (u) + sec (u)
sec (u) tan (u) + sec2 (u)
Z
1
= · du
a tan (u) + sec (u)
Thus, we write,
sec (u) tan (u) + sec2 (u)
Z Z
1
du = dv
tan (u) + sec (u) v
= ln |v| + C
Remark 7.1.6.
Z
1
sec (ax + b) dx = ln |tan (ax + b) + sec (ax + b)| + C
a
Copyright © 2024 Md. Shouvik Iqbal. This book is licensed under a Creative Commons Attribution-NonCommercial-ShareAlike 4.0 International License (CC BY-NC-SA 4.0)
Download for free at https://md-shouvik-iqbal.github.io/calculus
Chapter 7 Integration by Substitution Page 389
Z
I Example 7.1.11. Evaluate csc (ax + b) dx
u = ax + b =⇒ du = adx
du
This implies that dx = . Thus, we write,
a
Z Z
du
csc (ax + b) dx = csc (u)
a
Z
1
= · csc (u) du
a
csc (u) + cot (u)
Z
1
= · csc (u) · du
a csc (u) + cot (u)
csc2 (u) + cot (u) csc (u)
Z
1
= · du
a csc (u) + cot (u)
csc2 (u) + cot (u) csc (u)
Z
Now, to solve the integral du, we apply the substitu-
csc (u) + cot (u)
tion rule once more. That is, we let,
Thus, we write,
csc2 (u) + cot (u) csc (u)
Z Z
1
du = (−dv)
csc (u) + cot (u) v
Z
1
=− dv
v
= − ln |v| + C
Copyright © 2024 Md. Shouvik Iqbal. This book is licensed under a Creative Commons Attribution-NonCommercial-ShareAlike 4.0 International License (CC BY-NC-SA 4.0)
Download for free at https://md-shouvik-iqbal.github.io/calculus
Page 390 Integration by Substitution Chapter 7
Remark 7.1.7.
Z
1
csc (ax + b) dx = − ln |csc(ax + b) + cot(ax + b)| + C
a
f 0 (x)
Z
I Example 7.1.12. Evaluate dx, where f (x) 6= 0 is any function.
f (x)
Solution. In order to evaluate the integral, we let,
u = f (x) =⇒ du = f 0 (x)dx
Thus, we write,
f 0 (x)
Z Z
1
dx = du
f (x) u
= ln |f (x)| + C
f 0 (x)
Z
Therefore, dx = ln |f (x)| + C
f (x)
Remark 7.1.8.
f 0 (x)
Z
dx = ln |f (x)| + C
f (x)
A few common integrals above are summarized below for a quick reference.
(ax + b)n+1
Z
1. (ax + b)n dx = +C
a (n + 1)
Z
1
2. sin (ax + b)dx = − cos (ax + b) + C
a
Z
1
3. cos (ax + b) dx = sin (ax + b) + C
a
Z
1
4. tan (ax + b) dx = − ln |sec (ax + b)| + C
a
Z
1
5. cot (ax + b) dx = ln |sin (ax + b)| + C
a
Z
1
6. sec (ax + b) dx = ln |tan (ax + b) + sec (ax + b)| + C
a
Copyright © 2024 Md. Shouvik Iqbal. This book is licensed under a Creative Commons Attribution-NonCommercial-ShareAlike 4.0 International License (CC BY-NC-SA 4.0)
Download for free at https://md-shouvik-iqbal.github.io/calculus
Chapter 7 Integration by Substitution Page 391
Z
1
7. csc (ax + b) dx = − ln |csc (ax + b) + cot (ax + b)| + C
a
f 0 (x)
Z
8. dx = ln |f (x)| + C
f (x)
Z
I Example 7.1.13. Evaluate (5x + 5)5 dx
cos(x)
Z
I Example 7.1.14. Evaluate dx
sin(x)
Solution. We know that,
f 0 (x)
Z
dx = ln |f (x)| + C
f (x)
This implies that
cos(x)
Z
dx = ln |sin(x)| + C
sin(x)
cos(x)
Z
Therefore, dx = ln |sin(x)| + C
sin(x)
Chap. 7 / Sec. 7.1 / Subsec. 7.1.1 : Substitution Rule for Definite Integrals
The substitution rule can also be applied to definite integrals. For that, when
changing variables from x to u, the limits of integration are also changed accord-
ingly. That is, if u = g(x), the original lower limit x = a becomes u = g(a), and
Copyright © 2024 Md. Shouvik Iqbal. This book is licensed under a Creative Commons Attribution-NonCommercial-ShareAlike 4.0 International License (CC BY-NC-SA 4.0)
Download for free at https://md-shouvik-iqbal.github.io/calculus
Page 392 Integration by Substitution Chapter 7
the upper limit x = b becomes u = g(b). After substituting both the variable
and the limits, we can complete the integration with respect to u. As a result,
we have the following theorem.
Zb Zg(b)
f (g(x))g 0 (x)dx = f (u)du
a g(a)
Z2
2x
I Example 7.1.15. Evaluate dx
2 + x2
0
Z2 Z2
2x 1
dx = 2 2xdx
2 + x2 2| + x | {z }
0 0 {z } du
u
Z6
1
= du
u
2
i6
= ln |u|
2
= ln |6| − ln |2|
≈ 1.0986
Copyright © 2024 Md. Shouvik Iqbal. This book is licensed under a Creative Commons Attribution-NonCommercial-ShareAlike 4.0 International License (CC BY-NC-SA 4.0)
Download for free at https://md-shouvik-iqbal.github.io/calculus
Chapter 7 Integration by Substitution Page 393
Z2
2x
dx ≈ 1.0986
2 + x2
0
Ze2
ex − e−x
I Example 7.1.16. Evaluate dx
ex + e−x
e
Solution. In order to solve the integral, let the variable u = ex + e−x , then
du d x
e + e−x
=
dx dx
du d x d
= e + e−x
dx dx dx
du
= ex − e−x
dx
du = ex − e−x dx differential form
= ln e + e−e − ln ee + e−e
e2 2
≈ 4.666
Ze2
ex − e−x
Therefore, dx ≈ 4.666
ex + e−x
e
Copyright © 2024 Md. Shouvik Iqbal. This book is licensed under a Creative Commons Attribution-NonCommercial-ShareAlike 4.0 International License (CC BY-NC-SA 4.0)
Download for free at https://md-shouvik-iqbal.github.io/calculus
Page 394 Integration by Parts Chapter 7
Suppose, we’re given h(x) = f (x) · g(x), then the product rule in differentiation
states that,
d d
h(x) = f (x) · g(x)
dx dx
d d
= f (x) · g(x) + g(x) · f (x)
dx dx
0 0 0
∴ h (x) = f (x) g (x) + g(x)f (x) (1)
Z Z
f (x) g (x) dx = f (x) g (x) − g(x)f 0 (x) dx
0
Copyright © 2024 Md. Shouvik Iqbal. This book is licensed under a Creative Commons Attribution-NonCommercial-ShareAlike 4.0 International License (CC BY-NC-SA 4.0)
Download for free at https://md-shouvik-iqbal.github.io/calculus
Chapter 7 Integration by Parts Page 395
Z
I Example 7.2.1. Evaluate x sin(x)dx
Z
Solution. Notice that the integral sin(x)dx is easier to solve, however, the
Z
integral x sin(x)dx is not. In order to solve such an integral, we use integration
by parts, stating the following.
Z Z
u dv = uv − v du
Z Z
This necessitates us to write the integral x sin(x)dx in the form udv. That
Z
is to identify which part of the integral x sin(x)dx is u and which part is dv.
Now to choose u and dv, notice that if we choose u = x, then dv = sin(x)dx fits
a basic integration formula. Therefore, we let the following,
u=x =⇒ du = dx
Z Z
dv = sin(x)dx =⇒ v = dv = sin(x)dx = − cos(x)
Z
I Example 7.2.2. Evaluate x cos(x)dx
Copyright © 2024 Md. Shouvik Iqbal. This book is licensed under a Creative Commons Attribution-NonCommercial-ShareAlike 4.0 International License (CC BY-NC-SA 4.0)
Download for free at https://md-shouvik-iqbal.github.io/calculus
Page 396 Integration by Parts Chapter 7
Solution. In order to solve the integral above, we will use integration by parts,
stating the following. Z Z
u dv = uv − v du
Z Z
This necessitates us to write the integral x sin(x)dx in the form udv. That
Z
is, to identify which part of the integral is x cos(x)dx is u and which part is dv.
u=x =⇒ du = dx
Z Z
dv = cos(x)dx =⇒ v = dv = cos(x)dx = sin(x)
Z
I Example 7.2.3. Evaluate ln(x)dx
= x ln(x) − x + C
Copyright © 2024 Md. Shouvik Iqbal. This book is licensed under a Creative Commons Attribution-NonCommercial-ShareAlike 4.0 International License (CC BY-NC-SA 4.0)
Download for free at https://md-shouvik-iqbal.github.io/calculus
Chapter 7 Integration by Parts Page 397
Z
Therefore, ln(x)dx = x ln(x) − x + C
Remark 7.2.1. Z
ln(x)dx = x ln(x) − x + C
It’s important to note that while this method isn’t foolproof, it works well enough
to be considered useful. Below are a few examples where this rule applies per-
fectly.
Copyright © 2024 Md. Shouvik Iqbal. This book is licensed under a Creative Commons Attribution-NonCommercial-ShareAlike 4.0 International License (CC BY-NC-SA 4.0)
Download for free at https://md-shouvik-iqbal.github.io/calculus
Page 398 Integration by Parts Chapter 7
Z
I Example 7.2.4. Evaluate xex dx
u=x =⇒ du = dx
Z Z
dv = ex dx =⇒ v = dv = ex dx = ex
= xex − ex + C
= (x − 1) ex + C
Z
Therefore, xex dx = (x − 1) ex + C
Z
I Example 7.2.5. Evaluate x2 ex dx
u = x2 =⇒ du = 2xdx
Z Z
dv = ex dx =⇒ v = dv = ex dx = ex
Copyright © 2024 Md. Shouvik Iqbal. This book is licensed under a Creative Commons Attribution-NonCommercial-ShareAlike 4.0 International License (CC BY-NC-SA 4.0)
Download for free at https://md-shouvik-iqbal.github.io/calculus
Chapter 7 Integration by Parts Page 399
Z
= uv − v du
Z
= x2 ex − ex 2x dx
Z Z
Now, notice that the integral 2xe dx is of the form
x
f (x)g(x)dx such that
it can be further integrated by integration by parts. In order to do that, we
choose, by the LIATE rule, the following.
u = 2x =⇒ du = 2dx
Z Z
dv = ex dx =⇒ v = dv = ex dx = ex
Z
= 2xe − 2 ex dx
x
= 2xex − 2ex
= (x2 − 2x − 2)ex + C
Z
Therefore, x2 ex dx = (x2 − 2x − 2)ex + C
Z
I Example 7.2.6. Evaluate x2 sin(x)dx
u = x2 =⇒ du = 2x dx
Z Z
dv = sin(x)dx =⇒ v = dv = sin(x) dx = − cos(x)
Copyright © 2024 Md. Shouvik Iqbal. This book is licensed under a Creative Commons Attribution-NonCommercial-ShareAlike 4.0 International License (CC BY-NC-SA 4.0)
Download for free at https://md-shouvik-iqbal.github.io/calculus
Page 400 Integration by Parts Chapter 7
Z
= uv − v du
Z
= x (− cos(x)) −
2
(− cos(x))2x dx
Z
= −x cos(x) + 2
2
x cos(x) dx
Z Z
Now, notice that the integral x cos(x) dx is of the form f (x)g(x)dx such
that it can be further integrated by integration by parts. In order to do that,
we choose, by the LIATE rule, the following.
u=x =⇒ du = dx
Z Z
dv = cos(x)dx =⇒ v = dv = cos(x)dx = sin(x)
= x sin(x) − (− cos(x))
= x sin(x) + cos(x)
Chap. 7 / Sec. 7.2 / Subsec. 7.2.2 : Integration by Parts for Definite Integrals
Integration by parts can also be applied to definite integrals as stated by the
following theorem.
Copyright © 2024 Md. Shouvik Iqbal. This book is licensed under a Creative Commons Attribution-NonCommercial-ShareAlike 4.0 International License (CC BY-NC-SA 4.0)
Download for free at https://md-shouvik-iqbal.github.io/calculus
Chapter 7 Integration by Parts Page 401
Zb ib Zb
u(x)v 0 (x)dx = u(x)v(x) − v(x)u0 (x)dx
a
a a
Z2
I Example 7.2.7. Evaluate ln(x)dx
1
Z2 Z2
ln(x)dx = u(x) v 0 (x)dx
|{z} | {z }
1 1 u dv
2
Z2
= u(x) v(x) − v(x) u0 (x)dx
|{z} |{z} |{z} | {z }
u v 1 1 v du
Z2
i2 1
= ln(x) · x − x dx
1 x
1
i2 Z2
= x · ln(x) − dx
1
1
= 2 · ln(2) − 1 · ln(1) − (2 − 1)
= 2 ln(2) − 1
≈ 0.386
Z2
Therefore, ln(x)dx ≈ 0.386
1
Copyright © 2024 Md. Shouvik Iqbal. This book is licensed under a Creative Commons Attribution-NonCommercial-ShareAlike 4.0 International License (CC BY-NC-SA 4.0)
Download for free at https://md-shouvik-iqbal.github.io/calculus
Page 402 Integration by Parts Chapter 7
Now, if we let
Copyright © 2024 Md. Shouvik Iqbal. This book is licensed under a Creative Commons Attribution-NonCommercial-ShareAlike 4.0 International License (CC BY-NC-SA 4.0)
Download for free at https://md-shouvik-iqbal.github.io/calculus
Chapter 7 Trigonometric Integrals Page 403
Z Z
+ (n − 1) sin n−2
(x) dx − (n − 1) sinn (x) dx
Z Z
sin (x) dx + (n − 1)
n
sinn (x) dx = − sinn−1 (x) cos (x)
Z
+ (n − 1) sinn−2 (x) dx
Z Z
n sin (x) dx = − sin
n
(x) cos (x) + (n − 1) sinn−2 (x) dx
n−1
n−1
Z Z
1 n−1
1. sin (x) dx = − sin
n
(x) cos (x) + sinn−2 (x) dx
n n
n−1
Z Z
1 n−1
2. cos (x) dx = cos
n
(x) sin (x) + cosn−2 (x) dx
n n
Z Z
1
3. tann (x) dx = tann−1 (x) − tann−2 (x) dx, n 6= 1
n−1
Z Z
1
4. cot (x) dx = −
n
cotn−1
(x) − cotn−2 (x) dx, n 6= 1
n−1
n−2
Z Z
1
5. sec (x) dx =
n
secn−2
(x) tan (x) − secn−2 (x) dx,
n−1 n−1
n 6= 1
n−2
Z Z
1
6. csc (x) dx =
n
csc n−2
(x) cot (x) − cscn−2 (x) dx,
n−1 n−1
n 6= 1
Copyright © 2024 Md. Shouvik Iqbal. This book is licensed under a Creative Commons Attribution-NonCommercial-ShareAlike 4.0 International License (CC BY-NC-SA 4.0)
Download for free at https://md-shouvik-iqbal.github.io/calculus
Page 404 Trigonometric Integrals Chapter 7
The same, however, cannot be said for integrating the following types of inte-
grals, Z Z
sin (x)dx
2
or sin(x) cos(x)dx
This is because neither the substitution rule nor integration by parts is immedi-
ately useful in this case, as the trigonometric expressions don’t lend themselves
to obvious substitutions or algebraic simplifications. This limitation becomes
even more apparent when considering even complex examples, such as:
Z
sin4 (x) cos3 (x)dx
These types of integrals fall under the category of trigonometric integrals, which
require different strategies than the ones we have discussed so far. In this section,
we aim to develop techniques for integrating such integrals. We will do so by
manipulating trigonometric expressions using identities, reducing powers, and
transforming products into more manageable forms.
Chap. 7 / Sec. 7.3 / Subsec. 7.3.1 : Integrals Involving Powers of sin(x) and
cos(x)
In order to integrate integrals of the form,
Z Z
sinm (x)dx and cosn (x)dx
two strategies are primarily used based on whether the power is odd or even. For
example, with even powers of sin(x) or cos(x), we use the half-angle formulas,
1 − cos(2x) 1 + cos(2x)
sin2 (x) = and cos2 (x) =
2 2
to reduce the powers of sin(x) or cos(x) in the integrand.
Similarly, with odd powers of sin(x) or cos(x), the best idea is to split off a single
factor of sin(x) or cos(x) and to use the Pythagorean identities in a way that
Copyright © 2024 Md. Shouvik Iqbal. This book is licensed under a Creative Commons Attribution-NonCommercial-ShareAlike 4.0 International License (CC BY-NC-SA 4.0)
Download for free at https://md-shouvik-iqbal.github.io/calculus
Chapter 7 Trigonometric Integrals Page 405
Z
I Example 7.3.1. Evaluate cos3 (x)dx
Solution. For odd powers, we use substitution by expressing the cosine function
in terms of sine. For that, we rewrite cos3 (x) as the following,
Now, since cos2 (x) = 1 − sin2 (x), therefore, the integral becomes,
Z Z
cos (x)dx = cos2 (x) cos(x)dx
3
Z
1 − sin2 (x) cos(x)dx
=
Z
I Example 7.3.2. Evaluate sin4 (x)dx
Solution. For even powers, we express sin4 (x) as (sin2 (x))2 , and then use the
half-angle formula for sin2 (x). That is, since,
1 − cos(2x)
sin2 (x) =
2
therefore, we write the following,
Z Z
sin (x)dx = sin2 (x) dx
4
2
Copyright © 2024 Md. Shouvik Iqbal. This book is licensed under a Creative Commons Attribution-NonCommercial-ShareAlike 4.0 International License (CC BY-NC-SA 4.0)
Download for free at https://md-shouvik-iqbal.github.io/calculus
Page 406 Trigonometric Integrals Chapter 7
1 − cos(2x)
Z 2
= dx
2
(1 − cos(2x))2
Z
dx =
4
1 − 2 cos(2x) + cos2 (2x)
Z 2
= dx
Z 4
1
1 − 2 cos(2x) + cos2 (2x) dx
= ·
4
Z Z Z
1
= · dx − 2 cos(2x)dx + cos2 (2x)dx
4
Z
Now, to evaluate the integral cos2 (2x)dx, we use the following half-angle
formula for cos2 (2x), that is,
1 + cos(4x)
cos2 (2x) =
2
This implies that
Z Z Z Z
1
sin (x)dx = ·
4
dx − 2 cos(2x)dx + cos (2x)dx 2
4
1 + cos(4x)
Z Z Z
1
= · dx − 2 cos(2x)dx + dx
4 2
Z Z Z
1 1
= · dx − 2 cos(2x)dx + · (1 + cos(4x)) dx
4 2
Z Z Z Z
1 1
= · dx − 2 cos(2x)dx + · dx + cos(4x)dx
4 2
sin(2x) 1 sin(4x)
1
= · x−2· + · x+ +C
4 2 2 4
1 sin(4x)
1 1
= · x − sin(2x) + x + +C
4 2 2 4
x sin(2x) x sin(4x)
= − + + +C
4 4 8 32
x sin(2x) x sin(4x)
Z
Therefore, sin4 (x)dx = − + + +C
4 4 8 32
Copyright © 2024 Md. Shouvik Iqbal. This book is licensed under a Creative Commons Attribution-NonCommercial-ShareAlike 4.0 International License (CC BY-NC-SA 4.0)
Download for free at https://md-shouvik-iqbal.github.io/calculus
Chapter 7 Trigonometric Integrals Page 407
Z
I Example 7.3.3. Evaluate sin4 (x) cos3 (x) dx
Z
= sin4 (x) 1 − sin2 (x) cos (x) dx
Copyright © 2024 Md. Shouvik Iqbal. This book is licensed under a Creative Commons Attribution-NonCommercial-ShareAlike 4.0 International License (CC BY-NC-SA 4.0)
Download for free at https://md-shouvik-iqbal.github.io/calculus
Page 408 Trigonometric Integrals Chapter 7
u5 u7
= − +C
5 7
sin6 (x) sin8 (x)
= − +C
6 8
sin6 (x) sin8 (x)
Z
Therefore, sin4 (x) cos3 (x) dx = − +C
6 8
Z
I Example 7.3.4. Evaluate sin3 (x) cos4 (x) dx
Z
= cos4 (x) 1 − cos2 (x) sin (x) dx
u5 u7
=− + +C
5 7
cos7 (x) cos5 (x)
= − +C
7 5
cos7 (x) cos5 (x)
Z
Therefore, sin3 (x) cos4 (x) dx = − +C
7 5
Copyright © 2024 Md. Shouvik Iqbal. This book is licensed under a Creative Commons Attribution-NonCommercial-ShareAlike 4.0 International License (CC BY-NC-SA 4.0)
Download for free at https://md-shouvik-iqbal.github.io/calculus
Chapter 7 Trigonometric Integrals Page 409
Chap. 7 / Sec. 7.3 / Subsec. 7.3.3 : Integrals Involving Powers of tan(x) and
sec(x)
The idea behind integrating integrals involving powers of tan(x) and sec(x) is
similar to that of integrals involving powers of sin(x) and cos(x) as discussed
before. For example, consider integrating the following integral.
Z
I Example 7.3.5. Evaluate sec3 (x)dx
Solution. For odd powers, we use integration by parts by rewriting sec3 (x) as
sec(x) sec2 (x) and letting,
Therefore,
Z Z
sec (x)dx =
3
sec(x)sec2 (x)dx
Z
= udv
Z
= uv − vdu
Z
= sec(x) tan(x) − tan(x) · sec(x) tan(x)dx
Z
= sec(x) tan(x) − tan2 (x) sec(x)dx
Z Z
sec (x)dx = sec(x) tan(x) −
3
sec2 (x) − 1 sec(x)dx
Z Z Z
sec (x)dx = sec(x) tan(x) − sec (x)dx + sec(x)dx
3 3
Z Z Z
sec (x)dx + sec (x)dx = sec(x) tan(x) + sec(x)dx
3 3
Z
2 · sec3 (x)dx = sec(x) tan(x) + ln |sec(x) + tan(x)| + C
sec(x) tan(x) + ln |sec(x) + tan(x)| + C
Z
sec3 (x)dx =
2
Copyright © 2024 Md. Shouvik Iqbal. This book is licensed under a Creative Commons Attribution-NonCommercial-ShareAlike 4.0 International License (CC BY-NC-SA 4.0)
Download for free at https://md-shouvik-iqbal.github.io/calculus
Page 410 Trigonometric Integrals Chapter 7
Z
1 1
sec3 (x)dx = sec(x) tan(x) + ln |sec(x) + tan(x)| + C
2 2
Z
1 1
Therefore, sec3 (x)dx = sec(x) tan(x) + ln |sec(x) + tan(x)| + C
2 2
Z
I Example 7.3.6. Evaluate tan4 (x)dx
Solution. For even powers, we rewrite tan4 (x) as tan2 (x) · tan2 (x) and use the
following identity,
tan2 (x) + sec2 (x) = 1
Z Z
tan (x)sec (x)dx −
2 2
sec2 (x) − 1 dx
=
Z Z Z
= tan (x)sec (x)dx − sec (x)dx + dx
2 2 2
Now, let u = tan(x), then du = sec2 (x)dx in the first integral. Therefore,
Z Z Z Z
tan (x)dx = tan (x)sec (x)dx − sec (x)dx + dx
4 2 2 2
Z Z Z
= u du − sec (x)dx + dx
2 2
u3
Z Z
= − sec (x)dx + dx
2
3
tan3 (x)
= − tan(x) + x + C
3
tan3 (x)
Z
Therefore, tan (x)dx =
4
− tan(x) + x + C
3
Copyright © 2024 Md. Shouvik Iqbal. This book is licensed under a Creative Commons Attribution-NonCommercial-ShareAlike 4.0 International License (CC BY-NC-SA 4.0)
Download for free at https://md-shouvik-iqbal.github.io/calculus
Chapter 7 Trigonometric Integrals Page 411
Z
I Example 7.3.7. Evaluate tan3 (x) sec4 (x) dx
Z
= tan3 (x) tan2 (x) + 1 sec2 (x) dx
Now, let u = tan(x), then du = sec2 (x)dx. Thus, we write the following,
Z Z
tan (x) sec (x) dx = tan3 (x) sec2 (x) sec2 (x) dx
3 4
Copyright © 2024 Md. Shouvik Iqbal. This book is licensed under a Creative Commons Attribution-NonCommercial-ShareAlike 4.0 International License (CC BY-NC-SA 4.0)
Download for free at https://md-shouvik-iqbal.github.io/calculus
Page 412 Trigonometric Substitution Chapter 7
Z
tan3 (x) tan2 (x) + 1 sec2 (x) dx
=
Z Z
tan (x) tan (x) + 1 sec (x) dx =
3 2 2
u3 u2 + 1 du
Z
u5 + u3 du
=
Z Z
= u du + u3 du
5
u6 u4
= + +C
6 4
tan6 (x) tan4 (x)
= + +C
6 4
tan6 (x) tan4 (x)
Z
Therefore, tan3 (x) sec4 (x) dx = + +C
6 4
√
is quite difficult to solve. Nevertheless, if we can replace 1 + x2 with a product
of squares, then it becomes easier to solve.
The
p goal of this√section is to transform the sum or difference of squares,
p that is,
a ± x , and x − a , into products of squares. It turns out that a2 ± x2 ,
2 2 2 2
√
and x2 − a2 can be transformed into products of squares by seemingly unex-
pected substitution: trigonometric substitution. Really, what do trigonometric
functions have to do with the sum or difference of squares? This is what we see
in the following subsections.
Copyright © 2024 Md. Shouvik Iqbal. This book is licensed under a Creative Commons Attribution-NonCommercial-ShareAlike 4.0 International License (CC BY-NC-SA 4.0)
Download for free at https://md-shouvik-iqbal.github.io/calculus
Chapter 7 Trigonometric Substitution Page 413
√
Chap. 7 / Sec. 7.4 / Subsec. 7.4.1 : Integrals Involving a2 − x2
√
Suppose, we’re given an integral whose integrand involves the term a2 − x2 ,
where a is a positive constant. Now, notice what happens when we substitute
√
x = a sin(θ) in the expression a2 − x2 , as shown below.
q
a − x = a2 − (a sin (θ))2
p
2 2
q
= a2 − a2 sin2 (θ)
q
= a2 1 − sin2 (θ)
It is known from the Pythagorean identity that sin2 (θ) + cos2 (θ) = 1, or,
cos2 (θ) = 1 − sin2 (θ). Therefore,
= a2 1 − sin2 (θ)
s
| {z }
cos2 (θ)
= a2 · cos2 (θ)
p
= |a · cos (θ)|
= a cos (θ) (1)
√
This implies that the substitution x = a sin(θ) transform the difficult a2 − x 2
into an easier product a cos(θ). That is,
a2 − x2 = a cos(θ)
p
if we let x = a sin(θ).
√
The substitution x = a sin(θ) for a2 − x2 works because, if we consider a right-
angled triangle whose one side is x and the hypotenuse is a, the length of the
√
other side becomes a2 − x2 from the Pythagorean theorem, as shown below.
a
x
Figure 7.1
Copyright © 2024 Md. Shouvik Iqbal. This book is licensed under a Creative Commons Attribution-NonCommercial-ShareAlike 4.0 International License (CC BY-NC-SA 4.0)
Download for free at https://md-shouvik-iqbal.github.io/calculus
Page 414 Trigonometric Substitution Chapter 7
Z
dx
I Example 7.4.1. Evaluate √
4 − x2
Solution. Since a = 2 in the integral above, therefore to solve the integral, let
x = 2 sin(θ), then dx = 2 cos(θ)dθ. This implies that,
2 cos(θ)dθ
Z Z
dx
√ = q
4 − x2 4 − (2 sin(θ))2
2 cos(θ)
Z
= q dθ
4 − (2 sin(θ)) 2
2 cos(θ)
Z
= p dθ
4 − 4sin2 (θ)
2 cos(θ)
Z
= q dθ
4 1 − sin2 (θ)
2 cos(θ)
Z
= √ p dθ
4 · cos2 (θ)
2 cos(θ)
Z
= dθ
2 cos(θ)
Z
= dθ
=θ+C
−1 x
Since x = 2 sin(θ), therefore, θ = sin . As a result,
2
Z
dx
√ =θ+C
4 − x2 x
= sin−1 +C
2
Z
dx x
Therefore, √ = sin−1 +C
4 − x2 2
Z p
I Example 7.4.2. Evaluate 9 − x2 dx
Copyright © 2024 Md. Shouvik Iqbal. This book is licensed under a Creative Commons Attribution-NonCommercial-ShareAlike 4.0 International License (CC BY-NC-SA 4.0)
Download for free at https://md-shouvik-iqbal.github.io/calculus
Chapter 7 Trigonometric Substitution Page 415
1 + cos(2θ)
Since it is known, from the double-angle identity, that cos2 (θ) = ,
2
therefore,
1 + cos(2θ)
Z Z
9 cos (θ)dθ = 9
2
dθ
Z 2
9
= (1 + cos(2θ)) dθ
2
Z Z
9
= dθ + cos(2θ)dθ
2
sin(2θ)
9
= θ+ +C
2 2
Since it is known, from the double-angle identity, that sin(2θ) = 2 sin(θ) cos(θ),
therefore,
sin(2θ)
Z
9
9 cos (θ)dθ =
2
θ+ +C
2 2
2 sin(θ) cos(θ)
9
= θ+ +C
2 2
−1 x
Since x = 3 sin(θ), therefore θ = sin . Thus,
3
2 sin(θ) cos(θ)
Z p
2
9
9 − x dx = θ+ +C
2 2
Copyright © 2024 Md. Shouvik Iqbal. This book is licensed under a Creative Commons Attribution-NonCommercial-ShareAlike 4.0 International License (CC BY-NC-SA 4.0)
Download for free at https://md-shouvik-iqbal.github.io/calculus
Page 416 Trigonometric Substitution Chapter 7
−1 x −1 x
sin cos sin 2 sin
9 x
= sin−1 + 3 3 + C
2 3 2
9 −1 x x x
= sin + sin sin−1 cos sin−1 +C
2 3 3 3
√
Since sin(sin−1 (x)) = x and cos(sin−1 (x)) = 1 − x2 , therefore,
Z p
9 −1 x
−1 x
−1 x
2
9 − x dx = sin + sin sin cos sin +C
2 3 3 3
!
9 x x r x 2
= sin−1 + · 1− +C
2 3 3 3
r
9 −1 x 3x x 2
= sin + 1− +C
2 3 2 3
Z p
9 x 3x r x 2
Therefore, 9 − x dx = sin
2 −1
+ 1− +C
2 3 2 3
Z p
I Example 7.4.3. Evaluate a2 − x2 dx
Z
= a cos (θ) cos (θ) dθ
Z
=a 2
cos2 (θ) dθ (E1)
To deal with the integrand of the above integral, we make use of the half-angle
identity, stating that,
1 + cos(2θ)
cos2 (θ) =
2
Copyright © 2024 Md. Shouvik Iqbal. This book is licensed under a Creative Commons Attribution-NonCommercial-ShareAlike 4.0 International License (CC BY-NC-SA 4.0)
Download for free at https://md-shouvik-iqbal.github.io/calculus
Chapter 7 Trigonometric Substitution Page 417
x = a sin(θ)
x
= sin(θ)
a x
∴ θ = sin−1
a
x
Substituting θ = sin −1
into equation (E4) yields the following,
a
a2
q
= θ + sin (θ) 1 − sin (θ) + C
2
2
a2 x r
x x
= sin−1 + sin sin−1 1 − sin2 sin−1 +C
2 a a a
!
a2 x x r x 2
= sin−1 + · 1− +C
2 a a a
Copyright © 2024 Md. Shouvik Iqbal. This book is licensed under a Creative Commons Attribution-NonCommercial-ShareAlike 4.0 International License (CC BY-NC-SA 4.0)
Download for free at https://md-shouvik-iqbal.github.io/calculus
Page 418 Trigonometric Substitution Chapter 7
r !
a2 x x x 2
= sin−1 + · 1− 2 +C
2 a a a
r
a2 −1 x a2 x a2 − x2
= sin + +C
2 a 2 ar a2
a2 −1 x x 1 p 2
= sin + a · a − x2 + C
2 a 2 a2
a2 −1 x x a p 2
= sin + √ · a − x2 + C
2 a 2 a2
a2 −1 x x p 2
= sin + a − x2 + C
2 a 2
This suggests the following,
a2
Z p x xp
a2 − x2 dx = sin−1 + a2 − x 2 + C
2 a 2
Remark 7.4.1.
a2
Z p x xp
a2 − x2 dx = sin−1 + a2 − x 2 + C
2 a 2
Z
dx
I Example 7.4.4. Evaluate √
a2 − x 2
Solution. To evaluate the integral, we let x = a sin(θ). Then, the differential
dx = a cos(θ)dθ. Therefore,
a cos (θ) dθ
Z Z
dx
√ = q
a2 − x 2 a2 − (a sin (θ))2
a cos (θ) dθ
Z
=
a2 − a2 sin2 (θ)
p
a cos (θ) dθ
Z
= q
a2 1 − sin2 (θ)
a cos (θ) dθ
Z
= √ q
1 − sin2 (θ)
a2 ·
a cos (θ) dθ
Z
= q
a 1 − sin2 (θ)
Copyright © 2024 Md. Shouvik Iqbal. This book is licensed under a Creative Commons Attribution-NonCommercial-ShareAlike 4.0 International License (CC BY-NC-SA 4.0)
Download for free at https://md-shouvik-iqbal.github.io/calculus
Chapter 7 Trigonometric Substitution Page 419
a cos (θ) dθ
Z
=
a cos2 (θ)
p
a cos (θ) dθ
Z
=
a cos (θ)
Z
= dθ
=θ+C (E1)
x = a sin(θ)
x
= sin(θ)
a x
∴ θ = sin−1
a
x
Now, substituting θ = sin−1 into equation (E1), we have the following,
a
−1 x
θ + C = sin +C
a
This suggests the following,
Z
dx x
√ = sin−1 +C
a2 − x2 a
Remark 7.4.2. Z
dx −1 x
√ = sin +C
a2 − x2 a
√
Chap. 7 / Sec. 7.4 / Subsec. 7.4.2 : Integrals Involving a2 + x2
√
Suppose, we’re given an integral whose integrand involves the term a2 + x2 ,
where a is a positive constant. If we substitute x = a tan(θ), then
q
a + x = a2 + (a tan(θ))2
p
2 2
q
= a2 + a2 tan2 (θ)
q
= a2 (1 + tan2 (θ))
Copyright © 2024 Md. Shouvik Iqbal. This book is licensed under a Creative Commons Attribution-NonCommercial-ShareAlike 4.0 International License (CC BY-NC-SA 4.0)
Download for free at https://md-shouvik-iqbal.github.io/calculus
Page 420 Trigonometric Substitution Chapter 7
It is known from the Pythagorean identity that sec2 (x) = 1+tan2 (x). Therefore,
= a2 1 + tan2 (θ)
s
| {z }
sec2 (θ)
a2 · sec2 (θ)
p
=
= |a · sec (θ)|
= a sec (θ) (2)
√
This implies that the substitution x = a tan(θ) transform the difficult a2 + x 2
into an easier product a sec(θ). That is,
a2 + x2 = a sec (θ)
p
if we let x = a tan(θ).
√
The substitution x = a tan(θ) for a2 + x2 works because, if we consider a
right-angled triangle whose opposite side is x and adjacent side is a, then the
hypotenuse is given by the Pythagorean theorem, as follows.
Figure 7.2
Copyright © 2024 Md. Shouvik Iqbal. This book is licensed under a Creative Commons Attribution-NonCommercial-ShareAlike 4.0 International License (CC BY-NC-SA 4.0)
Download for free at https://md-shouvik-iqbal.github.io/calculus
Chapter 7 Trigonometric Substitution Page 421
∴ x = a tan(θ)
Z
dx
I Example 7.4.5. Evaluate √
x2 + 9
√
Solution. Notice that the integral above involves the term x2 + a2 , where a = 3.
Therefore, to solve the integral, we let x = 3 tan(θ), then dx = 3 sec2 (θ)dθ.
Therefore,
3sec2 (θ)dθ
Z Z
dx
√ = q
x2 + 9 (3 tan(θ))2 + 9
3sec2 (θ)dθ
Z
= p
9tan2 (θ) + 9
3sec2 (θ)dθ
Z
= p
9 (tan2 (θ) + 1)
3sec2 (θ)dθ
Z
= √ p
9 · sec2 (θ)
3sec2 (θ)dθ
Z
=
3 sec(θ)
Z
= sec(θ)dθ
Z
Since sec(θ)dθ = ln |sec(θ) + tan(θ)| + C and if x = 3 tan(θ), then θ =
x
tan−1 , therefore,
3
Z
dx
√ = ln |sec(θ) + tan(θ)| + C
x2 + 9
−1 x −1 x
= ln sec tan + tan tan +C
3 3
√
Since tan(tan−1 (x)) = x and sec(tan−1 (x)) = 1 + x2 , therefore,
Z
dx
−1 x
−1 x
√ = ln sec tan + tan tan +C
x2 + 9 3 3
r x 2 x
= ln 1+ + +C
3 3
r
x2 x
= ln 1+ + +C
9 3
Copyright © 2024 Md. Shouvik Iqbal. This book is licensed under a Creative Commons Attribution-NonCommercial-ShareAlike 4.0 International License (CC BY-NC-SA 4.0)
Download for free at https://md-shouvik-iqbal.github.io/calculus
Page 422 Trigonometric Substitution Chapter 7
r
9 + x2 x
= ln + +C
9 3
√
x2 + 9 x
= ln √ + +C
9 3
√
x x2 + 9
= ln + +C
3 3
√
x2 + 9
Z
dx x
Therefore, √ = ln + +C
x2 + 9 3 3
Z
dx
I Example 7.4.6. Evaluate
a2 + x2
Solution. To evaluate the integral, we let x = a tan(θ). Then, the differential of
x becomes dx = a sec2 (θ)dθ. Therefore,
Z Z
dx dx
=
2
a +x 2
a + (a tan (x))2
2
asec2 (θ) dθ
Z
=
a2 + a2 tan2 (θ)
asec2 (θ) dθ
Z
=
a2 sec2 (θ)
Z
1
= dθ
a
θ
= +C (E1)
a
Now, since x = a tan(θ), therefore,
x = a tan(θ)
x
= tan(θ)
a
x
∴ θ = tan−1 ( )
a
x
Substituting θ = tan−1 into equation (E1) yields the following,
a
x
θ tan−1
+C = a +C
a a
Copyright © 2024 Md. Shouvik Iqbal. This book is licensed under a Creative Commons Attribution-NonCommercial-ShareAlike 4.0 International License (CC BY-NC-SA 4.0)
Download for free at https://md-shouvik-iqbal.github.io/calculus
Chapter 7 Trigonometric Substitution Page 423
Remark 7.4.3. Z
dx 1 −1 x
= tan +C
a2 + x2 a a
Z p
I Example 7.4.7. Evaluate a2 + x2 dx
Copyright © 2024 Md. Shouvik Iqbal. This book is licensed under a Creative Commons Attribution-NonCommercial-ShareAlike 4.0 International License (CC BY-NC-SA 4.0)
Download for free at https://md-shouvik-iqbal.github.io/calculus
Page 424 Trigonometric Substitution Chapter 7
a2
= (sec (θ) tan (θ) + ln |tan (θ) + sec (θ)|) + C
2
a2
q q
= 1 + tan (θ) tan (θ) + ln tan (θ) + 1 + tan (θ) + C
2 2 (E4)
2
Now, to express the integral in terms of x, notice that x = a tan(θ), therefore,
x = a tan(θ)
x
= tan(θ)
a
−1 x
∴ θ = tan
a
x
Substituting θ = tan −1
into equation (E4), we get,
a
a2
q q
= 1 + tan (θ) tan (θ) + ln tan (θ) + 1 + tan (θ) + C
2 2
2
a2
r x x
= 1 + tan2 tan−1 tan tan−1
2 a a
x r x
+ ln tan tan−1 + 1 + tan2 tan−1 +C
a a
r r !
a2 x 2 x x x 2
= 1+ · + ln + 1 + +C
2 a a a a
Copyright © 2024 Md. Shouvik Iqbal. This book is licensed under a Creative Commons Attribution-NonCommercial-ShareAlike 4.0 International License (CC BY-NC-SA 4.0)
Download for free at https://md-shouvik-iqbal.github.io/calculus
Chapter 7 Trigonometric Substitution Page 425
r r !
a2 x2 x x x2
= 1 + 2 · + ln + 1 + 2 +C
2 a a a a
r r !
2 2 2 2
1 a + x x x a + x
= a2 · + a2 ln + +C
2 a2 a a a2
r r !
1 1 x x 1
· a2 + x2 · + a2 ln +
p p
= a2 2
· a2 + x2 + C
2 a a a a2
√ !
1 x a 2 + x2
x a2 + x2 + a2 ln +
p
= +C
2 a a
√ !
1 x + a 2 + x2
x a2 + x2 + a2 ln
p
= +C
2 a
Remark 7.4.4.
√ !
a2 x2
Z p
1 x+ +
x a2 + x2 + a2 ln
p
a2 + x2 dx = +C
2 a
√
Chap. 7 / Sec. 7.4 / Subsec. 7.4.3 : Integrals Involving x2 − a2
√
Suppose, we’re given an integral whose integrand involves the term x2 − a2 ,
where a is a positive constant. If we substitute x = a sec(θ), then
q
x − a = (a sec (θ))2 − a2
p
2 2
= a2 sec2 (θ) − a2
p
p
= a2 (sec2 (θ) − 1)
It is known from the Pythagorean identity that sec2 (x) = 1+tan2 (x) or, sec2 (x)−
1 = tan2 (x). Therefore,
= a2 sec2 (θ) − 1
s
| {z }
tan2 (θ)
Copyright © 2024 Md. Shouvik Iqbal. This book is licensed under a Creative Commons Attribution-NonCommercial-ShareAlike 4.0 International License (CC BY-NC-SA 4.0)
Download for free at https://md-shouvik-iqbal.github.io/calculus
Page 426 Trigonometric Substitution Chapter 7
q
= a2 · tan2 (θ)
= |a · tan (θ)|
= a tan (θ)
√
This implies that the substitution x = a sec(θ) transforms the difficult x 2 − a2
into an easier product a tan(θ). That is,
x2 − a2 = a tan (θ)
p
if we let x = a tan(θ).
Figure 7.3
Z
xdx
I Example 7.4.8. Evaluate √
x2 − 16
Copyright © 2024 Md. Shouvik Iqbal. This book is licensed under a Creative Commons Attribution-NonCommercial-ShareAlike 4.0 International License (CC BY-NC-SA 4.0)
Download for free at https://md-shouvik-iqbal.github.io/calculus
Chapter 7 Trigonometric Substitution Page 427
√
Solution. Notice that the integral above involves the term x2 − a2 , where a = 4.
Therefore, to solve the integral, we let x = 4 sec(θ), then dx = 4 sec(θ) tan(θ)dθ.
Thus,
= 4tan2 (θ) + C
x
Since x = 4 sec(θ), therefore θ = sec −1
. Thus,
4
Z
xdx
√ = 4tan2 (θ) + C
2
x − 16 x
= 4tan2 sec−1 +C
4
√
Since tan(sec−1 ) = x1 − 1, therefore,
Z
xdx
−1 x
√ = 4tan sec
2
+C
x2 − 16 4
r
x 2
=4· −1+C
4
r
x2 − 16
=4· +C
p 16
= x2 − 16 + C
Copyright © 2024 Md. Shouvik Iqbal. This book is licensed under a Creative Commons Attribution-NonCommercial-ShareAlike 4.0 International License (CC BY-NC-SA 4.0)
Download for free at https://md-shouvik-iqbal.github.io/calculus
Page 428 Trigonometric Substitution Chapter 7
Z
xdx
Therefore,
p
√ = x2 − 16 + C
x2 − 16
Z p
I Example 7.4.9. Evaluate x2 − a2 dx
Copyright © 2024 Md. Shouvik Iqbal. This book is licensed under a Creative Commons Attribution-NonCommercial-ShareAlike 4.0 International License (CC BY-NC-SA 4.0)
Download for free at https://md-shouvik-iqbal.github.io/calculus
Chapter 7 Trigonometric Substitution Page 429
a2
Z p
x2 − a2 dx = (sec (θ) tan (θ) − ln | sec(θ) + tan(θ)|) + C (E2)
2
Now, from the Pythagorean theorem, it is known that,
a2
= (sec (θ) tan (θ) − ln | sec(θ) + tan(θ)|) + C
2
a2
sec (θ) sec (θ) − 1 − ln | sec(θ) + sec (θ) − 1| + C (E4)
p p
= 2 2
2
Now, to express the integral in terms of x, notice that x = a sec(θ), therefore,
x = a sec(θ)
x
= sec(θ)
a x
∴ θ = sec−1
a
Copyright © 2024 Md. Shouvik Iqbal. This book is licensed under a Creative Commons Attribution-NonCommercial-ShareAlike 4.0 International License (CC BY-NC-SA 4.0)
Download for free at https://md-shouvik-iqbal.github.io/calculus
Page 430 Trigonometric Substitution Chapter 7
x
Substituting θ = sec−1 into equation (E4), we obtain the following,
a
a2
sec (θ) sec2 (θ) − 1 − ln sec(θ) + sec2 (θ) − 1 + C
p p
=
2
a2 x r x
= sec sec−1 sec2 sec−1 −1
2 a a
x r x
− ln sec sec −1
+ sec sec 2 −1 −1 +C
a a
r r !
a2 x x 2 x x 2
= − 1 − ln + −1 +C
2 a a a a
r r !
2 2 2
a x x x x
= 2
− 1 − ln + −1 +C
2 a a a a2
r r !
a2 x x 2 − a2 x x 2 − a2
= − ln + +C
2 a a2 a a2
r r !
2 2 2 2
1 x x − a x x − a
= a2 · − a2 · ln + +C
2 a a2 a a2
√ √ !
1 x 2
x −a 2 x 2
x −a 2
= a2 · · − a2 · ln + +C
2 a a a a
√ !
1 x + x 2 − a2
x x2 − a2 − a2 ln
p
= +C
2 a
Remark 7.4.5.
√ !
x2 − a2
Z p
1 x+
x x2 − a2 − a2 ln
p
x2 − a2 dx = +C
2 a
Copyright © 2024 Md. Shouvik Iqbal. This book is licensed under a Creative Commons Attribution-NonCommercial-ShareAlike 4.0 International License (CC BY-NC-SA 4.0)
Download for free at https://md-shouvik-iqbal.github.io/calculus
Chapter 7 Partial Fractions Page 431
P (x)
f (x) =
Q(x)
where, P (x) and Q(x) 6= 0 are both polynomials. In that case, f (x) is said to
be a rational function and is expressed as the ratio of polynomial functions. A
polynomial is a function of the form,
Copyright © 2024 Md. Shouvik Iqbal. This book is licensed under a Creative Commons Attribution-NonCommercial-ShareAlike 4.0 International License (CC BY-NC-SA 4.0)
Download for free at https://md-shouvik-iqbal.github.io/calculus
Page 432 Partial Fractions Chapter 7
Now, notice that the integral on the right side of the equation is straightforward
to solve, whereas the integral on the left side is not. That is,
Z Z Z
2 4 2 4
+ dx = dx + dx
x−1 x+2 x−1 x+2
Z Z
1 1
=2 dx + 4 dx
x−1 x+2
= 2 ln |x − 1| + 4 |x + 2| + C
Z
2x
∴ dx = 2 ln |x − 1| + 4 |x + 2| + C
x2 +x−2
While the method is efficient, its practicality relies on the ability to factor
6x 2 4
2
into fractions and , referred to as partial fractions. For
x +x−2 x−1 x+2
2 4
example, if and are given, then it is of no effort that we may deter-
x−1 x+2
6x
mine 2 , however, the converse is not true. Therefore, in this section,
x +x−2
various techniques are discussed regarding that issue.
However, that comes with a few considerations. First of all, we can only decom-
P (x)
pose a rational function if the degree of P (x) is less than that of Q(x),
Q(x)
symbolically, deg P (x) < deg Q(x) . Such a rational function is called a
proper rational function. In contrast, if we ever face an improper rational func-
tion, where
the degree
of P (x) is greater than Q(x) or equal to that, that is,
deg P (x) ≥ deg Q(x) , then we’ll first need to perform long polynomial
P (x) R(x)
division to express as S(x) + , where R(x) is the remainder and
Q(x) Q(x)
deg R(x) < deg Q(x) .
Copyright © 2024 Md. Shouvik Iqbal. This book is licensed under a Creative Commons Attribution-NonCommercial-ShareAlike 4.0 International License (CC BY-NC-SA 4.0)
Download for free at https://md-shouvik-iqbal.github.io/calculus
Chapter 7 Partial Fractions Page 433
Chap. 7 / Sec. 7.5 / Subsec. 7.5.1 : The denominator Q(x) is a product of lin-
ear factors
P (x)
Remark 7.5.1. If Q(x) in the rational function can be factored into
Q(x)
a product of linear factors, where each linear factor is distinct, that is,
factors of the form,
P (x) P (x)
=
Q(x) (a1 x + b1 ) (a2 x + b2 ) · · · (an x + bn )
A1 A2 An
= + + ··· +
a1 x + b1 a2 x + b2 an x + bn
x2 + 2x + 1
Z
I Example 7.5.1. Evaluate dx
x3 + 2x2 + x2 + 2x
Solution. In order to integrate the integral above, notice first that x3 + 2x2 + x2 + 2x
can be factored as x(x + 1)(x + 2). As a result,
x2 + 2x + 1 x2 + 2x + 1
=
x3 + 2x2 + x2 + 2x x(x + 1)(x + 2)
A B C
= + +
x (x + 1) (x + 2)
Now, the goal is to determine the values of A, B, and C. For that multiply
x(x + 1)(x + 2) in the both sides of the equation below,
x2 + 2x + 1 A B C
= + +
x(x + 1)(x + 2) x (x + 1) (x + 2)
A B C
x2 + 2x + 1 = ·
x(x + 1)(x + 2) + · x
(x+1)(x
+ 2) + · x(x + 1)
(x+2)
x
(x+ 1)
(x+ 2)
Copyright © 2024 Md. Shouvik Iqbal. This book is licensed under a Creative Commons Attribution-NonCommercial-ShareAlike 4.0 International License (CC BY-NC-SA 4.0)
Download for free at https://md-shouvik-iqbal.github.io/calculus
Page 434 Partial Fractions Chapter 7
A+B+C =1 (1)
3A + 2B + C = 2 (2)
2A = 1 (3)
1
Since 2A = 1, therefore, A = . Substituting the value of A into equation (1)
2
yields the following,
1
+B+C =1
2
1
B =1−C − (4)
2
Substituting the value of A and B into equation (2) yields the following,
1 1
3 +2 1−C − +C =2
2 2
1 1
C =2−2 1−C − −3
2 2
3
= 2 − 2 + 2C + 1 −
2
3
= 2C + 1 −
2
1
= 2C −
2
1
C − 2C = −
2
1
−C = −
2
1
∴C=
2
Substituting the value of A and C into equation (4) yields the following,
1
B =1−C −
2
=0
1 1
Therefore, we have found the value of A, B, and C to be , 0, and , respectively.
2 2
Thus, we write,
x2 + 2x + 1 A B C
= + +
x(x + 1)(x + 2) x (x + 1) (x + 2)
Copyright © 2024 Md. Shouvik Iqbal. This book is licensed under a Creative Commons Attribution-NonCommercial-ShareAlike 4.0 International License (CC BY-NC-SA 4.0)
Download for free at https://md-shouvik-iqbal.github.io/calculus
Chapter 7 Partial Fractions Page 435
1 1
0
= 2 + + 2
x (x + 1) (x + 2)
1 1
= +
2x 2(x + 2)
As a result,
x2 + 2x + 1
Z Z
1 1
dx = + dx
x3 + 2x2 + x2 + 2x 2x 2(x + 2)
Z Z
1 1
= dx + dx
2x 2(x + 2)
Z Z
1 1 1 1
= · dx + · dx
2 x 2 x+2
1 1
= ln |x| + ln |x + 2| + C
2 2
x2 + 2x + 1
Z
1 1
Therefore, 3 2 2
dx = ln |x| + ln |x + 2| + C
x + 2x + x + 2x 2 2
Chap. 7 / Sec. 7.5 / Subsec. 7.5.2 : The denominator Q(x) contains repeated
product of linear factors
P (x)
Remark 7.5.2. If Q(x) in the rational function has at least one repeated
Q(x)
liner factor, that is, factor of the form,
P (x) P (x)
=
Q(x) (ax + b)n
A1 A2 An
= + + ··· +
ax + b (ax + b)2 (ax + b)n
x2 + x + 1
Z
I Example 7.5.2. Evaluate dx
(x + 1)2 (x − 1)
Copyright © 2024 Md. Shouvik Iqbal. This book is licensed under a Creative Commons Attribution-NonCommercial-ShareAlike 4.0 International License (CC BY-NC-SA 4.0)
Download for free at https://md-shouvik-iqbal.github.io/calculus
Page 436 Partial Fractions Chapter 7
Solution. In order to integrate the integral above, we express the given rational
function as a sum of partial fractions as follows.
x2 + x + 1 A B C
2 = + +
(x + 1) (x − 1) (x − 1) (x + 1) (x + 1)2
Now, we compare the coefficients of x2 , x, and the constant terms on both sides
of the equations, as follows.
A+B =1 (1)
2A + C = 1 (2)
A−B−C =1 (3)
A−B−C =1
A − (1 − A) − C = 1
A−1+A−C =1
2A − C = 2 (4)
4A = 3
3
A=
4
3
Substituting A = into equation (2), we get,
4
3
2 +C =1
4
Copyright © 2024 Md. Shouvik Iqbal. This book is licensed under a Creative Commons Attribution-NonCommercial-ShareAlike 4.0 International License (CC BY-NC-SA 4.0)
Download for free at https://md-shouvik-iqbal.github.io/calculus
Chapter 7 Partial Fractions Page 437
3
+C =1
2
3
C =1−
2
1
=−
2
1
This implies that C = − . Substituting the value of A into equation (1) yields
2
the following,
A+B =1
3
B =1−
4
1
=
4
1
This implies that B = . Thus, we have the partial fraction decomposition as
4
follows,
3 1 1
x2 + x + 1 4 4 2
2 = + −
(x + 1) (x − 1) (x − 1) (x + 1) (x + 1)2
This implies that
3 1 1
2
x +x+1 4 4 2
= + −
(x + 1)2 (x − 1) (x − 1) (x + 1) (x + 1)2
3 1 1
x2 + x + 1
Z Z
4 4 2
dx = + − dx
(x + 1)2 (x − 1) (x − 1) (x + 1) (x + 1) 2
3 Z 1Z Z 1
= 4 dx + 4 dx − 2 dx
(x − 1) (x + 1) (x + 1)2
Z Z Z
3 1 1 1 1 1
= · dx + · dx − · dx
4 (x − 1) 4 (x + 1) 2 (x + 1)2
Z
3 1 1 1
= ln |x − 1| + C1 + ln |x + 1| + C2 − · dx
4 4 2 (x + 1)2
(5)
Z
1
Now, to solve the integral dx, we use the u-substitution rule. There-
(x + 1)2
Copyright © 2024 Md. Shouvik Iqbal. This book is licensed under a Creative Commons Attribution-NonCommercial-ShareAlike 4.0 International License (CC BY-NC-SA 4.0)
Download for free at https://md-shouvik-iqbal.github.io/calculus
Page 438 Partial Fractions Chapter 7
Chap. 7 / Sec. 7.5 / Subsec. 7.5.3 : The denominator Q(x) contains irreducible
quadratic factors
P (x)
Remark 7.5.3. If Q(x) in the rational function has at least one irre-
Q(x)
ducible quadratic factor, that s, factor of the form,
ax2 + bx + c where, b2 − 4ac < 0
P (x)
then, there exist constants A and B such that the rational function
Q(x)
Copyright © 2024 Md. Shouvik Iqbal. This book is licensed under a Creative Commons Attribution-NonCommercial-ShareAlike 4.0 International License (CC BY-NC-SA 4.0)
Download for free at https://md-shouvik-iqbal.github.io/calculus
Chapter 7 Partial Fractions Page 439
P (x) P (x)
= 2
Q(x) ax + bx + c
Ax + B
= 2
ax + bx + c
x2 + x + 5
Z
I Example 7.5.3. Evaluate dx
x3 + 5x
Solution. Notice that x3 + 5x can be factored as x(x2 + 5). Since x2 + 5 cannot
be factored any further as b2 − 4ac < 0, therefore we write the integrand as,
x2 + x + 5 x2 + x + 5
=
x3 + 5x x(x2 + 5)
A Bx + C
= + 2
x x +5
Now, we multiply both sides by the denominator x3 + 5x to eliminate the de-
nominators. That is,
Now, we compare the coefficients of x2 , x, and the constant term both sides of
the equation,
A+B =1 (1)
C=1 (2)
5A = 5 (3)
A+B =1
B=0
Copyright © 2024 Md. Shouvik Iqbal. This book is licensed under a Creative Commons Attribution-NonCommercial-ShareAlike 4.0 International License (CC BY-NC-SA 4.0)
Download for free at https://md-shouvik-iqbal.github.io/calculus
Page 440 Partial Fractions Chapter 7
1 0·x+1
= + 2
x x +5
1 1
= + 2
x x +5
As a result,
x2 + x + 5
Z Z
1 1
dx = + dx
x3 + 5x x x2 + 5
Z Z
1 1
= dx + 2
dx
x x +5
√
Z
1 x
Now, to solve the integral dx, we let u = √ , then x = 5u and
x2 + 5 5
1
du = √ dx. Thus, we get,
5
Z Z √
1 5
dx = 2√ du
x2 + 5 5u + 5
Z √
5
= du
5 (u2 + 1)
√ Z
5 1
= · du
5 u2 + 1
Z
1
Now, to solve the integral du, we let u = tan(z), then du = sec2 (z)dz.
u2 +1
Thus,
sec2 (z)
Z Z
1
du = dz
u2 + 1 tan2 (z) + 1
sec2 (z)
Z
= dz
sec2 (z)
Z
= dz
=z+C
Copyright © 2024 Md. Shouvik Iqbal. This book is licensed under a Creative Commons Attribution-NonCommercial-ShareAlike 4.0 International License (CC BY-NC-SA 4.0)
Download for free at https://md-shouvik-iqbal.github.io/calculus
Chapter 7 Partial Fractions Page 441
1 x
= √ tan−1 √ +C
5 5
As a result,
x2 + x + 5
Z Z Z
1 1
dx + dx = dx
x x3 + 5x 2
x +5
1 x
= ln |x| + √ tan −1
√ +C
5 5
Z 2
x +x+5 1 x
Therefore, dx = ln |x| + √ tan −1
√ +C
x3 + 5x 5 5
Chap. 7 / Sec. 7.5 / Subsec. 7.5.4 : The denominator Q(x) contains repeated
irreducible quadratic factors
P (x)
Remark 7.5.4. If Q(x) in the rational function has at least one irre-
Q(x)
ducible quadratic factor, that is, factor of the form,
P (x) P (x)
=
Q(x) (ax + bx + c)n
2
A1 x + B1 A2 x + B2 An x + Bn
= 2 + + ··· +
ax + bx + c (ax2 + bx + c)2 (ax2 + bx + c)n
x3 + x2 + x + 1
Z
I Example 7.5.4. Evaluate dx
x(x2 + 1)2
Solution. Since x2 + 1 cannot be factored any further as b2 − 4ac < 0, therefore
we write the integral as,
x3 + x2 + x + 1 A Bx + C Dx + E
2 = + 2 +
x(x2 + 1) x x +1 (x2 + 1)2
Now, we multiply both sides by the denominator x(x2 + 1) to eliminate the
2
Copyright © 2024 Md. Shouvik Iqbal. This book is licensed under a Creative Commons Attribution-NonCommercial-ShareAlike 4.0 International License (CC BY-NC-SA 4.0)
Download for free at https://md-shouvik-iqbal.github.io/calculus
Page 442 Partial Fractions Chapter 7
A+B =0
C=1
2A + B + D = 1
C +E =1
A=1
x3 + x2 + x + 1 A Bx + C Dx + E
= + +
x(x2 + 1)2 x x2 + 1 (x2 + 1)2
1 −x + 1 0·x+0
= + 2 +
x x +1 (x2 + 1)2
1 1−x
= + 2
x x +1
1 1 x
= + 2 − 2
x x +1 x +1
As a result,
x3 + x2 + x + 1
Z Z
1 1 x
dx =
+ − dx
x(x2 + 1)2 x x2 + 1 x2 + 1
Z Z Z
1 1 x
= dx + 2
dx − 2
dx
x x +1 x +1
Z Z
1 1
Here, dx = ln |x| + C1 and 2
dx = tan−1 (x) + C2 . However, to
xZ x +1
x
integrate 2
dx, we use the substitution rule and let u = x2 + 1, then
x +1
du
du = 2xdx or = xdx. Thus, we write,
2
Z Z
x 1 du
dx =
x2 + 1 u 2
Copyright © 2024 Md. Shouvik Iqbal. This book is licensed under a Creative Commons Attribution-NonCommercial-ShareAlike 4.0 International License (CC BY-NC-SA 4.0)
Download for free at https://md-shouvik-iqbal.github.io/calculus
Chapter 7 Improper Integrals Page 443
Z
1 1
= · du
2 u
1
= · ln |u| + C3
2
1
= ln x2 + 1 + C3
2
Therefore, we write,
Z 3
x + x2 + x + 1
Z Z Z
1 1 x
2 dx = dx + 2
dx − 2
dx
2
x(x + 1) x x +1 x +1
1
= ln |x| + tan−1 (x) − ln x2 + 1 + C
2
Z 3
x + x2 + x + 1 1
Therefore, 2 dx = ln |x| − ln x2 + 1 + tan−1 (x) + C
x(x2 + 1) 2
Copyright © 2024 Md. Shouvik Iqbal. This book is licensed under a Creative Commons Attribution-NonCommercial-ShareAlike 4.0 International License (CC BY-NC-SA 4.0)
Download for free at https://md-shouvik-iqbal.github.io/calculus
Page 444 Improper Integrals Chapter 7
x
1
Figure 7.4
Copyright © 2024 Md. Shouvik Iqbal. This book is licensed under a Creative Commons Attribution-NonCommercial-ShareAlike 4.0 International License (CC BY-NC-SA 4.0)
Download for free at https://md-shouvik-iqbal.github.io/calculus
Chapter 7 Improper Integrals Page 445
Z∞ Zb
1 1
dx = lim dx
x2 b→∞ x2
1 1
=1
Z∞ Zb
f (x) dx = lim f (x) dx
b→∞
a a
Zb Zb
f (x) dx = lim f (x) dx
a→−∞
−∞ a
Z∞ Zc Zb
f (x) dx = lim f (x) dx + lim f (x) dx
a→−∞ b→∞
−∞ a c
In each case, if the limit exists, then the improper integral is called
convergent, if not, it is called divergent.
Z∞
1
I Example 7.6.1. Is the integral dx convergent or divergent?
x
1
Copyright © 2024 Md. Shouvik Iqbal. This book is licensed under a Creative Commons Attribution-NonCommercial-ShareAlike 4.0 International License (CC BY-NC-SA 4.0)
Download for free at https://md-shouvik-iqbal.github.io/calculus
Page 446 Improper Integrals Chapter 7
= lim ln(b)
b→∞
=∞
Since the limit does not yield a finite value, therefore the limit does not exist.
Z∞
1
As a result, the integral dx is divergent.
x
1
Z∞
1
I Example 7.6.2. Evaluate dx
1 + x2
0
Copyright © 2024 Md. Shouvik Iqbal. This book is licensed under a Creative Commons Attribution-NonCommercial-ShareAlike 4.0 International License (CC BY-NC-SA 4.0)
Download for free at https://md-shouvik-iqbal.github.io/calculus
Chapter 7 Improper Integrals Page 447
subsection, we will examine how the area of the region extends infinitely in the
1
vertical direction. To illustrate, consider the area A under the curve y = √
x
and above the x-axis, from x = 0 to x = 1, as shown below.
f(x)
x
1
Figure 7.5
1
Notice that y = √ increases without a bound as x → 0+ , and is not defined at
x
Z1
1
the exact point x = 0. However, we can still determine the area A = √ dx,
x
0
Z1
1
if we first determine the integral √ dx and take the limit as a → 0+ . That
x
a
is,
Z1
1 √ i1
√ dx = 2 x
x a
a
√ √
=2 1−2 a
√
=2−2 a
√
=2 1− a
√
Now, take the limit of 2 (1 − a) as a → 0+ .
√
lim+ 2 1 − a = 2 (1 − 0)
a→0
=2
Copyright © 2024 Md. Shouvik Iqbal. This book is licensed under a Creative Commons Attribution-NonCommercial-ShareAlike 4.0 International License (CC BY-NC-SA 4.0)
Download for free at https://md-shouvik-iqbal.github.io/calculus
Page 448 Improper Integrals Chapter 7
Observe how the area of the bounded region is finite, even though the length of
the boundary curve is infinite!
1. If f (x) is continuous over (a, b], where lim+ f (x) = ±∞. Then
x→a
Zb Zb
f (x) dx = lim+ f (x) dx
c→a
a c
2. If f (x) is continuous over [a, b), where lim− f (x) = ±∞. Then
x→b
Zb Zc
f (x) dx = lim− f (x) dx
c→b
a a
3. If f (x) is continuous over [a, b], except at a point c such that a < c <
b. Then
Zb Zc Zb
f (x) dx = f (x) dx + f (x) dx
a a c
Zc Zb
given that improper integrals f (x) dx and f (x) dx exist.
a c
In every case, if the limits exist, the improper integral is called con-
vergent, if not, it is called divergent.
Z1
I Example 7.6.3. Evaluate ln(x)dx
0
Copyright © 2024 Md. Shouvik Iqbal. This book is licensed under a Creative Commons Attribution-NonCommercial-ShareAlike 4.0 International License (CC BY-NC-SA 4.0)
Download for free at https://md-shouvik-iqbal.github.io/calculus
Chapter 7 Improper Integrals Page 449
Z1 h i1
lim ln(x)dx = lim+ x ln(x) − x
a→0+ a→0 a
a
= lim+ (a − a ln(a) − 1)
a→0
= 0 − lim+ a ln(a) − 1
a→0
Now, to find the limit lim+ a ln(a), we apply the L’Hôpital’s Rule as follows,
a→0
ln(a)
lim+ a ln(a) = lim+
a→0 a→0 1
a
d
ln(a)
= lim+ da
a→0 d 1
da a
1
= lim+ a
a→0 1
− 2
a
= lim+ (−a)
a→0
=0
As a result, we get,
Z1 Z1
ln(x)dx = lim+ ln(x)dx
a→0
0 a
Copyright © 2024 Md. Shouvik Iqbal. This book is licensed under a Creative Commons Attribution-NonCommercial-ShareAlike 4.0 International License (CC BY-NC-SA 4.0)
Download for free at https://md-shouvik-iqbal.github.io/calculus
Page 450 Table of Integrals Chapter 7
= 0 − lim+ a ln(a) − 1
a→0
=0−0−1
= −1
Z1
Therefore, ln(x)dx = −1
0
In integration, finding the antiderivative does not follow a set of simple, uni-
versal rules. Integrating complex functions directly is often challenging and less
intuitive. For this reason, mathematicians have compiled tables of integrals.
These tables list the integrals of many common functions. By referencing these
tables, we can quickly find the antiderivative of a function that matches one of
the known forms, without having to derive each one from scratch.
Author's Interruption. While most textbook authors place the table of inte-
grals in the appendices to serve as a reference and avoid distracting the
reader, I have chosen to include them in the main text. This is due to the
reason I felt that the integrals listed in the table are often overlooked by
readers. Although memorizing every single integral listed in the table is not
necessary at a beginner to intermediate level, I believe that a thoughtful
review of them can be valuable for those interested in advanced calculus
or mathematical research. For beginners, the elementary integrals should
be of top priority.
Copyright © 2024 Md. Shouvik Iqbal. This book is licensed under a Creative Commons Attribution-NonCommercial-ShareAlike 4.0 International License (CC BY-NC-SA 4.0)
Download for free at https://md-shouvik-iqbal.github.io/calculus
Chapter 7 Table of Integrals Page 451
Copyright © 2024 Md. Shouvik Iqbal. This book is licensed under a Creative Commons Attribution-NonCommercial-ShareAlike 4.0 International License (CC BY-NC-SA 4.0)
Download for free at https://md-shouvik-iqbal.github.io/calculus
Page 452 Table of Integrals Chapter 7
Copyright © 2024 Md. Shouvik Iqbal. This book is licensed under a Creative Commons Attribution-NonCommercial-ShareAlike 4.0 International License (CC BY-NC-SA 4.0)
Download for free at https://md-shouvik-iqbal.github.io/calculus
Chapter 7 Table of Integrals Page 453
Z
1
10. sec−1 (ax) dx = xsec−1 (ax) −
ln ax + a2 x2 − 1 + C
p
a
Z
11. csc (x) dx = xcsc (x) + ln x + x − 1 + C
p
−1 −1 2
Z
1
12. csc−1 (ax) dx = xcsc−1 (ax) + ln ax + a2 x2 − 1 + C
p
a
Chap. 7 / Sec. 7.7 / Subsec. 7.7.4 : Reduction Formula for Trigonometric Func-
tions
sinn−1 (x) cos (x) n − 1
Z Z
1. sin (x) dx = −
n
+ sinn−2 (x) dx
n n
cosn−1 (x) sin (x) n − 1
Z Z
2. cos (x) dx =
n
+ cosn−2 (x) dx
n n
Copyright © 2024 Md. Shouvik Iqbal. This book is licensed under a Creative Commons Attribution-NonCommercial-ShareAlike 4.0 International License (CC BY-NC-SA 4.0)
Download for free at https://md-shouvik-iqbal.github.io/calculus
Page 454 Table of Integrals Chapter 7
tann−1 (x)
Z Z
3. tan (x) dx =
n
− tann−2 (x) dx, n 6= 1
n−1
cotn−1 (x)
Z Z
4. cot (x) dx = −
n
− cotn−2 (x) dx, n 6= 1
n−1
secn−2 (x) tan (x) n − 2
Z Z
5. sec (x) dx =
n
− secn−2 (x) dx, n 6= 1
n−1 n−1
cscn−2 (x) cot (x) n − 2
Z Z
6. csc (x) dx =
n
− cscn−2 (x) dx, n 6= 1
n−1 n−1
sinm+1 (x) cosn−1 (x) n − 1
Z Z
7. sin (x) cos (x) dx =
m n
+ sinm (x) cosn−2 (x) dx,
m+n m+n
m 6= −n
xn cos (ax) n
Z Z
8. x sin (ax) dx = −
n
+ xn−1 cos (ax) dx, a 6= 0
a a
xn sin (ax) n
Z Z
9. x cos (ax) dx =
n
− xn−1 sin (ax) dx, a 6= 0
a a
Copyright © 2024 Md. Shouvik Iqbal. This book is licensed under a Creative Commons Attribution-NonCommercial-ShareAlike 4.0 International License (CC BY-NC-SA 4.0)
Download for free at https://md-shouvik-iqbal.github.io/calculus
Chapter 7 Table of Integrals Page 455
Copyright © 2024 Md. Shouvik Iqbal. This book is licensed under a Creative Commons Attribution-NonCommercial-ShareAlike 4.0 International License (CC BY-NC-SA 4.0)
Download for free at https://md-shouvik-iqbal.github.io/calculus
Page 456 Table of Integrals Chapter 7
Z √ √
a2 + x2 a+ a2 + x 2
7. dx = a2 + x2 − a ln
p
+C
x x
Z √ √
a2 + x2 a2 + x2
8. ln
p
dx = x + a2 + x2 − +C
x2 x
Z
dx x
9. q = √ +C
3 a 2 a2 + x2
2 2
(a + x )
x2
Z
dx 1
10. = 2 ln +C
x (a2 + x2 ) 2a a2 + x2
x b ln |ax + b|
Z
x
5. dx = − +C
ax + b a a2
x2 (ax + b)2 − 4b (ax + b) + 2b2 ln |ax + b|
Z
6. dx = +C
ax + b 2a3
Z
dx 1 x
7. = ln +C
x (ax + b) b ax + b
Z
dx 1 a ax + b
8. = − + 2 ln +C
x2 (ax
+ b) bx b x
√
2 (ax − 2b) ax + b
Z
x
9. √ dx = +C
ax + b 3a2
Z √
2
q
10. x ax + bdx = 2
(3ax − 2b) (ax + b)3 + C
15a
Copyright © 2024 Md. Shouvik Iqbal. This book is licensed under a Creative Commons Attribution-NonCommercial-ShareAlike 4.0 International License (CC BY-NC-SA 4.0)
Download for free at https://md-shouvik-iqbal.github.io/calculus
Chapter 7 Table of Integrals Page 457
(ax + b)n+1
Z
ax + b b
11. x(ax + b) dx = n
− + C, n 6= −1, −2
a2 n+2 n+1
Chap. 7 / Sec. 7.7 / Subsec. 7.7.9 : Integrals with Exponential and Trigono-
metric Functions
eax (a sin (bx) − b cos (bx))
Z
1. eax sin (bx) dx = +C
a2 + b2
eax (a cos (bx) − b sin (bx))
Z
2. eax cos (bx) dx = +C
a2 + b2
Chap. 7 / Sec. 7.7 / Subsec. 7.7.10 : Integrals with Exponential and Logarith-
mic Functions
Z
dx
1. = ln |ln (x)| + C
x ln (x)
1
Z x n+1
ln (x) −
n+1
2. x ln (x) dx =
n
+ C, n 6= −1
n+1
Z
3. xex dx = xex − ex + C
xn eax n
Z Z
4. n ax
x e dx = − xn−1 eax dx, a 6= 0
a a
Z Z
5. ln (x) dx = xln (x) − n
n n
lnn−1 (x) dx
Copyright © 2024 Md. Shouvik Iqbal. This book is licensed under a Creative Commons Attribution-NonCommercial-ShareAlike 4.0 International License (CC BY-NC-SA 4.0)
Download for free at https://md-shouvik-iqbal.github.io/calculus
Page 458 Table of Integrals Chapter 7
xn+1
Z
Z x n+1
tan −1
(x) − dx
x2 + 1
3. xn tan−1 (x) dx = , n 6= −1
n+1
x−a
√
2
a2 sin−1
− −
Z p
(x a) 2ax x a
4. 2ax − x2 dx = + + C, a > 0
2 2
−1 x − a
Z
dx
5. √ = sin + C, a > 0
2ax − x2 a
Copyright © 2024 Md. Shouvik Iqbal. This book is licensed under a Creative Commons Attribution-NonCommercial-ShareAlike 4.0 International License (CC BY-NC-SA 4.0)
Download for free at https://md-shouvik-iqbal.github.io/calculus
Chapter 8
Applications of Integration
In the previous chapters, definite integrals were discussed in terms of the Rie-
mann Sum, and it was shown how they can be evaluated using antiderivatives
through the Fundamental Theorem of Calculus. This chapter will now focus on
the definite integral as a computational tool and demonstrate its importance in
solving and understanding a wide range of mathematical problems.
i = 1, 2, 3, · · · , n. Then,
459
Copyright © 2024 Md. Shouvik Iqbal. This book is licensed under a Creative Commons Attribution-NonCommercial-ShareAlike 4.0 International License (CC BY-NC-SA 4.0)
Download for free at https://md-shouvik-iqbal.github.io/calculus
Page 460 Average Value of a Function Chapter 8
is the total value of the function. Now, to take its average, we divide it by the
number of terms, that is, divide it by n,
Copyright © 2024 Md. Shouvik Iqbal. This book is licensed under a Creative Commons Attribution-NonCommercial-ShareAlike 4.0 International License (CC BY-NC-SA 4.0)
Download for free at https://md-shouvik-iqbal.github.io/calculus
Chapter 8 Average Value of a Function Page 461
Zb
1
fave = · f (x) dx
b−a
a
I Example 8.1.1. Find the average value of the function f (x) = x2 over the
interval [0, 2].
x3
Z
Solution. Since f (x) = x , therefore x2 dx =
2
+ C. Thus, we write,
3
Z2
1
fave = · x2 dx
2−0
0
2
1 x3
= ·
2 3 0
1 23
= ·
2 3
4
=
3
Therefore, the average value of the function f (x) = x2 over the interval [0, 2] is
4
.
3
I Example 8.1.2. Find the average value of the function f (x) = ex over the
interval [0, e].
Z
Solution. Since f (x) = e , therefore ex dx = ex + C. Thus, we write,
x
Ze
1
fave = · ex dx
e−0
0
1 i e
= · ex
e 0
1
= · ee − e0
e
Copyright © 2024 Md. Shouvik Iqbal. This book is licensed under a Creative Commons Attribution-NonCommercial-ShareAlike 4.0 International License (CC BY-NC-SA 4.0)
Download for free at https://md-shouvik-iqbal.github.io/calculus
Page 462 Average Value of a Function Chapter 8
ee − 1
=
e
≈ 5.207
Therefore, the average value of the function f (x) = ex over the interval [0, e] is
approximately 5.207.
Chap. 8 / Sec. 8.1 / Subsec. 8.1.1 : The Mean Value Theorem for Integrals
When we were discussing the average value of a function over a specific interval,
we might have wondered if there was a particular point within that interval
where the function’s value is exactly equal to its average value over the entire
interval, as the function can take on infinitely many values. That is, if there
exists a specific point, say c, within the interval such that f (c) is the same as
the average values of the function fave within that range. This observation brings
us to the mean value theorem for integrals.
Theorem 8.1.1. (The Mean Value Theorem For Integrals) If f (x) is a con-
tinuous function over an interval [a, b], then there exists a point c ∈ [a, b],
such that
f (c) = fave
Zb
1
= · f (x) dx
b−a
a
Equivalently,
Zb
f (x) dx = f (c) · (b − a)
a
I Example 8.1.3. Find a number c in the interval [0, 2] such that the average
value of f (x) = x + 2 equals f (c).
Copyright © 2024 Md. Shouvik Iqbal. This book is licensed under a Creative Commons Attribution-NonCommercial-ShareAlike 4.0 International License (CC BY-NC-SA 4.0)
Download for free at https://md-shouvik-iqbal.github.io/calculus
Chapter 8 Average Value of a Function Page 463
Solution. First, we will find the average value of f (x) = x + 2 on [0, 2] as follows,
Z2
1
fave = · (x + 2)dx
2−0
0
2 2
1 x
= · + 2x
2 2 0
2
1 2
= · +2·2
2 2
=3
Now, according to the mean value theorem for integrals, there must be a point
c ∈ [0, 2] such that f (c) = 3. That is,
f (c) = c + 2 = 3
c=3−2
=1
I Example 8.1.4. Find a number c in the interval [0, 1] such that the average
value of f (x) = ex equals f (c).
Solution. First, we will find the average value of f (x) = ex on [0, 1] as follows,
Z1
1
fave = · ex dx
1−0
0
i1
x
=e
0
= e − e01
=e−1
Now, according to the mean value theorem for integrals, there must be a point
c ∈ [0, 1] such that f (c) = e − 1. That is,
f (c) = ec = e − 1
ln(ec ) = ln(e − 1)
c · ln(e) = ln(e − 1)
| {z }
1
Copyright © 2024 Md. Shouvik Iqbal. This book is licensed under a Creative Commons Attribution-NonCommercial-ShareAlike 4.0 International License (CC BY-NC-SA 4.0)
Download for free at https://md-shouvik-iqbal.github.io/calculus
Page 464 Area of a Region Between Two Curves Chapter 8
c = ln(e − 1)
upper curve
x
a b
lower curve
Figure 8.1
Now, in order to find the area of the region that is enclosed by the curves over
the interval [a, b], we partition the interval [a, b] into n sub-interval of equal
b−a
width ∆x = . On each sub-interval, we construct a rectangle spanning
n
from the lower curve to the upper curve. Thus, the height of the ith rectangle
yields f (x∗i )−g(x∗i ), where, x∗i is a sample point chosen from the ith sub-interval.
Therefore, the area of the ith rectangle yields (f (x∗i ) − g(x∗i )) · |{z}
∆x . Summing
width
| {z }
height
the areas of the n rectangles provides an estimate for the exact area enclosed by
the curves, that is,
Xn
A≈ (f (x∗i ) − g(x∗i )) · ∆x
i=1
Copyright © 2024 Md. Shouvik Iqbal. This book is licensed under a Creative Commons Attribution-NonCommercial-ShareAlike 4.0 International License (CC BY-NC-SA 4.0)
Download for free at https://md-shouvik-iqbal.github.io/calculus
Chapter 8 Area of a Region Between Two Curves Page 465
upper curve
x
a b
lower curve
Figure 8.2
This forms the Riemann sum. Now, letting n → ∞, we get the exact area A
bounded by the curves and the vertical lines x = a and x = b. That is,
n
A = lim
X
(f (x∗i ) − g(x∗i )) · ∆x
x→∞
i=1
As a result, we have the following definition of the area of a region between two
curves.
Definition 8.2.1. (Area of a Region Between Two Curves) Let f (x) and g(x)
be continuous functions such that f (x) ≥ g(x) over the interval [a, b]. The
area of the region bounded by the graphs of f (x) and g(x) over [a, b] is
Zb
A= (f (x) − g(x))dx
a
I Example 8.2.1. Find the area of the region bounded by f (x) = sin(x) and
g(x) = cos(x), over the interval [1, 3].
Solution. The graph of f (x) = sin(x) is the upper curve and the graph of g(x) =
cos(x) is the lower curve on the interval [1, 3]. Therefore, the area of the region
between these curves is,
Z3 Z3 Z3
(sin(x) − cos(x)) dx = sin(x)dx − cos(x)dx
1 1 1
Copyright © 2024 Md. Shouvik Iqbal. This book is licensed under a Creative Commons Attribution-NonCommercial-ShareAlike 4.0 International License (CC BY-NC-SA 4.0)
Download for free at https://md-shouvik-iqbal.github.io/calculus
Page 466 Area of a Region Between Two Curves Chapter 8
Z3 Z3
= sin(x)dx − cos(x)dx
1 1
i3 i3
= − cos(x) − sin(x)
1 1
= − cos(3) + cos(1) − sin(3) + sin(1)
≈ 2.23
Therefore, the area of the region bounded by f (x) = sin(x) and g(x) = cos(x),
over the interval [1, 3] is approximately 2.23.
I Example 8.2.2. Find the area of the region bounded by f (x) = x and g(x) = x2 ,
over the interval [0, 1].
Solution. The graph of f (x) = x is the upper curve and the graph of g(x) = x2 is
the lower curve on the interval [0, 1]. Therefore, the area of the region between
these curves is,
Z1 Z1 Z1
(x − x2 )dx = xdx − x2 dx
0 0 0
2 1 3 1
x x
= −
2 0 3 0
1 1
= −
2 3
≈ 0.167
Therefore, the area of the region bounded by f (x) = x and g(x) = x2 , over the
interval [0, 1] is approximately 0.167.
Chap. 8 / Sec. 8.2 / Subsec. 8.2.1 : Area of a Region between Two Curves with
respect to y
There are times when it is easier to integrate the area with respect to y instead
of x. For example, consider integrating the area of a region enclosed by the
graph of x = f (y) and x = g(y) over the interval [c, d] lying on the y-axis, where
f (y) ≥ g(y), implying that the graph of f (y) lies to the right of the graph of g(y)
Copyright © 2024 Md. Shouvik Iqbal. This book is licensed under a Creative Commons Attribution-NonCommercial-ShareAlike 4.0 International License (CC BY-NC-SA 4.0)
Download for free at https://md-shouvik-iqbal.github.io/calculus
Chapter 8 Area of a Region Between Two Curves Page 467
and the lower and upper bound of the region are y = c and y = d, respectively,
as shown below.
f(x)
Figure 8.3
Definition 8.2.2 (Area of a Region Between Two Curves with respect to y). Let f (y)
and g(y) be continuous functions such that f (y) ≥ g(y) over the interval
[c, d]. The area of the region bounded by the graphs of f (y) and g(y) over
Copyright © 2024 Md. Shouvik Iqbal. This book is licensed under a Creative Commons Attribution-NonCommercial-ShareAlike 4.0 International License (CC BY-NC-SA 4.0)
Download for free at https://md-shouvik-iqbal.github.io/calculus
Page 468 Area of a Region Between Two Curves Chapter 8
[c, d] is
Zd
A= (f (y) − g(y))dy
c
I Example 8.2.3. Find the area of the region bounded by x = y 2 and x = 4y.
y 2 = 4y
y 2 − 4y = 0
y(y − 4) = 0
I Example 8.2.4. Find the area of the region bounded by the graph of y = x2
and y = x + 2.
Copyright © 2024 Md. Shouvik Iqbal. This book is licensed under a Creative Commons Attribution-NonCommercial-ShareAlike 4.0 International License (CC BY-NC-SA 4.0)
Download for free at https://md-shouvik-iqbal.github.io/calculus
Chapter 8 Area of a Region Between Two Curves Page 469
Solution. We will integrate the area of the bounded region with respect to y.
Therefore, we first need to solve for x in terms of y, as shown below.
√
y = x2 =⇒ x= y
y =x+2 =⇒ x=y−2
x2 = x + 2
x2 − x − 2 = 0
(x + 1)(x − 2) = 0
x=2 =⇒ y = 22 = 4
x = −1 =⇒ y = (−1) + 2 = 1
√
Now, notice that since x = y − 2 is to the right of x = y, therefore, we have
the following,
Z4 Z4 Z4
√ √
y − (y − 2) dy = ydy − (y − 2)dy
1 1 1
4
Z4 Z4
√
Z
= ydy − ydy − 2dy
1 1 1
Z4 Z4 Z4
1
= y 2 dy − ydy + 2 dy
1 1 1
4
3 4
y2 y2
= − + 2 y]41
3 2 1
2 1
3 3 2
12
42 12 4
= − − − + 2 (4 − 1)
3 3 2 2
2 2
8 1 1
= − − 8− +6
3 3 2
2 2
Copyright © 2024 Md. Shouvik Iqbal. This book is licensed under a Creative Commons Attribution-NonCommercial-ShareAlike 4.0 International License (CC BY-NC-SA 4.0)
Download for free at https://md-shouvik-iqbal.github.io/calculus
Page 470 Volume: The Slicing Method Chapter 8
19
=
6
Therefore, the area of the region bounded by the graph of y = x2 and y = x + 2
19
is .
6
Although intuitive, these ideas are too vague to serve as a formal definition,
as they lack the necessary level of mathematical rigor. Nevertheless, having
previously defined the concept of area rigorously in terms of the definite inte-
gral, we can now use that definition to define the concept of volume and discuss
methods for calculating a specific type of volume: the volume of a solid of rev-
olution. In order to do that, three primary techniques will be discussed in this
chapter—slicing, disks, and washers—each appropriate for solids with particular
characteristics.
Having this on mind, we now begin our discussion of defining the volume of a
solid.
Chap. 8 / Sec. 8.3 / Subsec. 8.3.1 : The Definition of the Volume of a Solid
To precisely define the volume of a solid, we make use of the slicing method.
This is a technique that determines the volume by integrating the cross-sectional
areas of the solid. Put simply: this method works by “slicing” the solid into thin
cross-sectional pieces, calculating the area of each slice, and then summing up
Copyright © 2024 Md. Shouvik Iqbal. This book is licensed under a Creative Commons Attribution-NonCommercial-ShareAlike 4.0 International License (CC BY-NC-SA 4.0)
Download for free at https://md-shouvik-iqbal.github.io/calculus
Chapter 8 Volume: The Slicing Method Page 471
Figure 8.4
Copyright © 2024 Md. Shouvik Iqbal. This book is licensed under a Creative Commons Attribution-NonCommercial-ShareAlike 4.0 International License (CC BY-NC-SA 4.0)
Download for free at https://md-shouvik-iqbal.github.io/calculus
Page 472 Volume: The Slicing Method Chapter 8
P
S
Figure 8.5
Here, the cross-section turns out to be a circle, because the solid S is a right
circular cylinder. The area of this cross-section is what we refer to as the “cross-
sectional area” (the region highlighted in blue, in this case).
We can use this cross-sectional area to define the volume of the solid S. The
idea is that since the area of the cross-section can exactly be determined using
definite integrals, the volume of S can also be determined by summing up all
the area slices of the cross-section.
With that idea in mind, now consider calculating the volume of the solid shown
below, from x = a to x = b.
Copyright © 2024 Md. Shouvik Iqbal. This book is licensed under a Creative Commons Attribution-NonCommercial-ShareAlike 4.0 International License (CC BY-NC-SA 4.0)
Download for free at https://md-shouvik-iqbal.github.io/calculus
Chapter 8 Volume: The Slicing Method Page 473
a b
Figure 8.6
To calculate the volume of the solid within the interval [a, b], the solid is in-
tersected with a plane to form a cross-section. The area of this cross-section
is represented by a known, integrable function A(x). Next, the interval [a, b]
b−a
is divided into n sub-intervals, each with an equal thickness of ∆x = .
n ∗
The ith sub-interval is denoted by [xi−1 , xi ]. Within each sub-interval, we let xi
represent an arbitrary point, as shown below.
a b
Figure 8.7
Now, let the area of the cross-section (i.e., the cross-sectional area) at the point
x∗i is A(x∗i ) and thus, the volume Vi at the point x∗i can be approximated as
Vi ≈ A(x∗i ) · ∆x
Copyright © 2024 Md. Shouvik Iqbal. This book is licensed under a Creative Commons Attribution-NonCommercial-ShareAlike 4.0 International License (CC BY-NC-SA 4.0)
Download for free at https://md-shouvik-iqbal.github.io/calculus
Page 474 Volume: The Slicing Method Chapter 8
This forms the Riemann sum. Now, as n increases indefinitely, the approxima-
tion gets better and better to the actual volume V over the interval [a, b]. Thus,
n
we take the limit of the sum A(x∗i )∆x as n → ∞ as follows,
X
i=1
n
V = lim
X
A(x∗i )∆x
n→∞
i=1
Zb
= A(x∗i )dx
a
Zb
V = A(x)dx
a
Having defined the definition of the volume of a solid in terms of the definite
integral, we now turn our attention to discuss a specific type of solids known as
solids of revolution. These are three-dimensional shapes formed by rotating a
two-dimensional region around a specified axis. Our goal in the following sub-
section will be to discuss the methods used to calculate the volumes of such
solids systematically.
Copyright © 2024 Md. Shouvik Iqbal. This book is licensed under a Creative Commons Attribution-NonCommercial-ShareAlike 4.0 International License (CC BY-NC-SA 4.0)
Download for free at https://md-shouvik-iqbal.github.io/calculus
Chapter 8 Volume: The Slicing Method Page 475
R represent the region enclosed by the graph of f (x) and the x-axis over the
interval [a, b], as shown below.
x
a b
Figure 8.8
Figure 8.9
Now, in order to determine the volume of this solid of revolution over the interval
b−a
[a, b], we divide the interval into n sub-interval of equal thickness ∆x = .
n
Copyright © 2024 Md. Shouvik Iqbal. This book is licensed under a Creative Commons Attribution-NonCommercial-ShareAlike 4.0 International License (CC BY-NC-SA 4.0)
Download for free at https://md-shouvik-iqbal.github.io/calculus
Page 476 Volume: The Slicing Method Chapter 8
Then [xi−1 , xi ] is the ith sub-interval. Let x∗i be a point in the ith sub-interval
[xi−1 , xi ], as shown below.
Figure 8.10
The area of the cross-section at the point x∗i is A(x∗i ). Notice that A(x∗i ) has
a specific form. That is, all the cross-sections perpendicular to the x-axis form
a circular disk whose radius is the function’s value f (x). For this reason, the
cross-section area yields,
A(x∗i ) = π · (radius)2
= πf (x∗i )2
Therefore, by the previously discussed slicing method, the volume of the solid
yields,
Zb
V = A (x) dx
a
Zb
= πf (x)2 dx
a
Copyright © 2024 Md. Shouvik Iqbal. This book is licensed under a Creative Commons Attribution-NonCommercial-ShareAlike 4.0 International License (CC BY-NC-SA 4.0)
Download for free at https://md-shouvik-iqbal.github.io/calculus
Chapter 8 Volume: The Slicing Method Page 477
interval [a, b]; then the volume V of the solid of revolution that is obtained
by rotating R about the x-axis is the following,
Z b
V = πf (x)2 dx
a
I Example 8.3.1. Let R be the region bounded by the graph of f (x) = x2 and
the x-axis, over the interval [0, 5]. What is the volume of the solid of revolution
obtained by rotating R about the x-axis?
Solution. We know that the volume of a solid of revolution is,
Z b
V = πf (x)2 dx
a
Z 5
V = π(x2 )2 dx
0
Z 5
=π x4 dx
0
5 5
x
=π
5 0
= 625π
√
I Example 8.3.2. Let R be the region bounded by the graph of f (x) = x
and the x-axis, over the interval [0, 100]. What is the volume of the solid of
revolution obtained by rotating R about the x-axis?
Solution. We know that the volume of a solid of revolution is,
Z b
V = πf (x)2 dx
a
Z 100
√
V = π xdx
0
Copyright © 2024 Md. Shouvik Iqbal. This book is licensed under a Creative Commons Attribution-NonCommercial-ShareAlike 4.0 International License (CC BY-NC-SA 4.0)
Download for free at https://md-shouvik-iqbal.github.io/calculus
Page 478 Volume: The Slicing Method Chapter 8
Z 100
1
=π x 2 dx
0
" 3
#100
2x 2
=π
3
0
2000π
=
3
Therefore, the volume of the solid of revolution obtained by rotating R about
2000π
the x-axis is .
3
x
a
b
Figure 8.11
If we revolve R about the x-axis, the solid that forms has a hole through it. In
other words, we get a hollowed-out solid of revolution, as shown below.
Copyright © 2024 Md. Shouvik Iqbal. This book is licensed under a Creative Commons Attribution-NonCommercial-ShareAlike 4.0 International License (CC BY-NC-SA 4.0)
Download for free at https://md-shouvik-iqbal.github.io/calculus
Chapter 8 Volume: The Slicing Method Page 479
x
a
b
Figure 8.12
Now, in order to determine the volume of this solid of revolution over the interval
b−a
[a, b], we divide the interval into n sub-interval of equal width ∆x = . Then
n
[xi−1 , xi ] is the ith sub-interval. Let xi be a point in the ith sub-interval [xi−1 , xi ],
∗
as shown below.
y
x
a
b
Figure 8.13
Notice that, this time, the cross-section through the solid perpendicular to the
x-axis forms a circular washer (a ring-shaped solid). This circular washer has
an outer radius R = f (x) and an inner radius r = g(x), where a ≤ x ≤ b. This
is shown in more detail in the following,
Copyright © 2024 Md. Shouvik Iqbal. This book is licensed under a Creative Commons Attribution-NonCommercial-ShareAlike 4.0 International License (CC BY-NC-SA 4.0)
Download for free at https://md-shouvik-iqbal.github.io/calculus
Page 480 Volume: The Slicing Method Chapter 8
a
b
Figure 8.14
For this reason, the cross-sectional area of the circular washer is the area of the
entire disk (that is, πf (x)2 ) minus the area of the hole (that is, πg(x)2 ). This
implies that the area of the circular washer is as follows,
Since we’ve found the area of the cross-section, we can apply the slicing method
to determine the volume of the solid as follows.
Zb
π f (x)2 − g(x)2 dx
V =
a
√
I Example 8.3.3. Suppose R is a region bounded by the graph of f (x) = x and
g(x) = x2 on the interval [0, 1]. What is the volume of the solid that is obtained
by rotating the region R about the x-axis?
Copyright © 2024 Md. Shouvik Iqbal. This book is licensed under a Creative Commons Attribution-NonCommercial-ShareAlike 4.0 International License (CC BY-NC-SA 4.0)
Download for free at https://md-shouvik-iqbal.github.io/calculus
Chapter 8 Volume: The Method of Cylindrical Shells Page 481
Solution. Since f (x) ≥ g(x) on the interval [0, 1], we’ll use the washer method.
The area of the cross-section at the point x is,
A (x) = π f (x)2 − g(x)2
√ 2
2
=π x − x2
= π x − x4
Z1
π x − x4 dx
V =
0
Z1
x − x4 dx
=π
0
1
x2 x5
=π −
2 5 0
3π
=
10
Copyright © 2024 Md. Shouvik Iqbal. This book is licensed under a Creative Commons Attribution-NonCommercial-ShareAlike 4.0 International License (CC BY-NC-SA 4.0)
Download for free at https://md-shouvik-iqbal.github.io/calculus
Page 482 Volume: The Method of Cylindrical Shells Chapter 8
x
a b
Figure 8.15
It is evident that when this region R revolves around the y-axis, a solid of
revolution is generated. Our objective is to determine the volume of this solid.
y
Figure 8.16
Copyright © 2024 Md. Shouvik Iqbal. This book is licensed under a Creative Commons Attribution-NonCommercial-ShareAlike 4.0 International License (CC BY-NC-SA 4.0)
Download for free at https://md-shouvik-iqbal.github.io/calculus
Chapter 8 Volume: The Method of Cylindrical Shells Page 483
x
a b
Figure 8.17
Notice that as the region R revolves around the y-axis, the rectangle built on
the ith sub-interval generates a thin cylindrical shell, as shown below.
y
Figure 8.18
Copyright © 2024 Md. Shouvik Iqbal. This book is licensed under a Creative Commons Attribution-NonCommercial-ShareAlike 4.0 International License (CC BY-NC-SA 4.0)
Download for free at https://md-shouvik-iqbal.github.io/calculus
Page 484 Volume: The Method of Cylindrical Shells Chapter 8
The formulation above can be generalized for a revolving region bounded by two
curves f (x) and g(x), where f (x) ≥ g(x), instead of a region bounded by the
only function f (x) and the x-axis on the interval [a, b].
In order to do this, only the height of the rectangle built on an arbitrary point
x∗i needs to be adjusted in the previous formulation. That is, instead of height
being f (x∗i ) on the ith interval, the height now is the difference between two
functions, that is, (f (x∗i ) − g(x∗i )). So, the generalized formulation yields,
n
V = lim
X
2πx∗i (f (x∗i ) − g(x∗i )) ∆x
n→∞
i=1
Zb
= 2πx (f (x) − g(x)) dx
a
Zb
V = 2πx f (x) − g(x) dx
a
Copyright © 2024 Md. Shouvik Iqbal. This book is licensed under a Creative Commons Attribution-NonCommercial-ShareAlike 4.0 International License (CC BY-NC-SA 4.0)
Download for free at https://md-shouvik-iqbal.github.io/calculus
Chapter 8 Volume: The Method of Cylindrical Shells Page 485
√
I Example 8.4.1. Let R be the region bounded by the graph of f (x) = x and
the x-axis on the interval [0, 4]. What is the volume of the solid of revolution
generated when the region R is rotated about the y-axis?
Solution. The volume of the solid of revolution can be determined by the shell
√
method, where x is the radius and f (x) = x is the height of the cylindrical
shell. Therefore, the volume of the solid by the shell method on the interval
[0, 4] yields,
Zb
V = 2πxf (x)dx
a
Z4
√
= 2πx xdx
0
Z4
1
= 2π x1 x 2 dx
0
Z4
3
= 2π x 2 dx
0
" 5
#4
x2
= 2π 5
2 0
" 5
#4
2x 2
= 2π
5
0
" 5
#4
2x 2
= 2π
5
0
64π
= 2π ·
5
128π
=
5
128π
Therefore, the volume of the solid of revolution is .
5
I Example 8.4.2. Let R be hthe region bounded by the graph of f (x) = sin(x) and
πi
the x-axis on the interval 0, . What is the volume of the solid of revolution
2
generated when the region R is rotated about the y-axis?
Copyright © 2024 Md. Shouvik Iqbal. This book is licensed under a Creative Commons Attribution-NonCommercial-ShareAlike 4.0 International License (CC BY-NC-SA 4.0)
Download for free at https://md-shouvik-iqbal.github.io/calculus
Page 486 Volume: The Method of Cylindrical Shells Chapter 8
Solution. The volume of the solid of revolution can be determined by the shell
method, where x is the radius and f (x) = sin(x) is the height of the cylindrical
shell. Therefore, the volume of the solid by the shell method on the interval
π
[0, ] is,
2
Zb
V = 2πxf (x)dx
a
π
Z2
= 2πx sin(x)dx
0
π
Z2
= 2π x sin(x)dx
0
π
Z2
Now, to solve the integral x sin(x)dx, integration by parts is used as follows.
0
u=x =⇒ du = dx
Z Z
dv = sin(x)dx =⇒ v = dv = sin(x)dx = − cos(x)
0
0 0
h iπ h iπ
= −x cos(x) + sin(x)
2 2
0 0
=0+1
=1
= 2π · 1
= 2π
Copyright © 2024 Md. Shouvik Iqbal. This book is licensed under a Creative Commons Attribution-NonCommercial-ShareAlike 4.0 International License (CC BY-NC-SA 4.0)
Download for free at https://md-shouvik-iqbal.github.io/calculus
Chapter 8 Arc Length Page 487
Figure 8.19
b−a
We partition the interval [a, b] into n sub-intervals of equal width ∆x = us-
n
ing points a = x0 , x1 , x2 , · · · , xn−1 , xn = b, and connect the corresponding points
(x0 , f (x0 )), (x1 , f (x1 )), (x2 , f (x2 )), · · · , (xn−1 , f (xn−1 )), (xn , f (xn )) on the graph
of f (x) with line segments, resulting in a polygonal line with n line segments.
When n is sufficiently large (and hence ∆x is small), the length of this polygonal
line is a good approximation to the actual curve.
Copyright © 2024 Md. Shouvik Iqbal. This book is licensed under a Creative Commons Attribution-NonCommercial-ShareAlike 4.0 International License (CC BY-NC-SA 4.0)
Download for free at https://md-shouvik-iqbal.github.io/calculus
Page 488 Arc Length Chapter 8
Since we’ve used a regular partition, the change in the horizontal distance on
every interval is fixed, that is, ∆x. However, the change in the vertical distance
varies from interval to interval. In general, the change in the vertical distance
on the ith interval is ∆yi = f (xi ) − f (xi−1 ), for i = 1, 2, 3, · · · , n. This implies
that the line segment over the ith sub-interval represents the hypotenuse of a
right triangle with sides of length ∆x and |∆yi | = |f (xi ) − f (xi−1 )|, as shown
below,
Figure 8.20
Since, the hypotenuse is the ith line segment for i = 1, 2, 3, · · · , n, therefore the
length of each line segment is,
for, i = 1, 2, 3, · · · , n
p
(∆x)2 + |∆yi |2
s
(∆yi )2
q
(∆x)2 + |∆yi |2 = (∆x)2 + (∆x)2 ·
(∆x)2
v
u 2 !
u ∆yi
= t(∆x)2 1+
∆x
Copyright © 2024 Md. Shouvik Iqbal. This book is licensed under a Creative Commons Attribution-NonCommercial-ShareAlike 4.0 International License (CC BY-NC-SA 4.0)
Download for free at https://md-shouvik-iqbal.github.io/calculus
Chapter 8 Arc Length Page 489
s
∆yi 2
q
2
= (∆x) · 1 +
∆x
s
∆yi 2
= ∆x · 1 +
∆x
Now, summing these lengths together for i = 1, 2, 3, · · · , n, we get the length of
the polygonal lines that approximates the actual length L of the curve. that is,
s
n 2
X ∆yi
L≈ ∆x · 1+
i=1
∆x
∆yi
Notice that represents the slope of the line segment over the ith interval.
∆x
Now, by the Mean Value Theorem, this slope equals the derivative f 0 (x∗i ) for
some point x∗i ∈ [xi−1 , xi ]. Therefore,
s
n
∆yi 2
X
L≈ ∆x · 1 +
i=1
∆x
s
n
∆yi 2
X
= 1+ · ∆x
i=1
∆x
X n q
= 1 + f 0 (x∗i )2 ∆x
i=1
This forms the Riemann sum. Now, letting n → ∞, the sum approaches a
definite integral which is the exact length L of the curve. That is,
n q
L = lim
X
1 + f 0 (x∗i )2 ∆x
n→∞
i=1
Zb q
= 1 + f 0 (x)2 dx
a
As a result, we have the following definition for the arc length of y = f (x).
Zb q
L= 1 + f 0 (x)2 dx
a
Copyright © 2024 Md. Shouvik Iqbal. This book is licensed under a Creative Commons Attribution-NonCommercial-ShareAlike 4.0 International License (CC BY-NC-SA 4.0)
Download for free at https://md-shouvik-iqbal.github.io/calculus
Page 490 Arc Length Chapter 8
Solution. Since f (x) = 5x+3, therefore, f 0 (x) = 5. Thus, we write the following,
Z5 p
L= 1 + 52 dx
0
5
√ Z
= 26 · dx
0
√ i5
= 26 · x
0
√
= 26 · 5
≈ 25.495
I Example 8.5.2. Find the length of the graph of f (x) = ln(cos(x)) from x = 0
to x = 1.
sin(x)
Solution. Since f (x) = ln(cos(x)), therefore, f 0 (x) = − . Thus, we write
cos(x)
the following,
Z1
s
sin(x)
2
L= 1+ − dx
cos(x)
0
Z1 q
= 1 + (− tan(x))2 dx
0
Z1 q
= 1 + tan2 (x)dx
0
Z1 p
= sec2 (x)dx
0
Copyright © 2024 Md. Shouvik Iqbal. This book is licensed under a Creative Commons Attribution-NonCommercial-ShareAlike 4.0 International License (CC BY-NC-SA 4.0)
Download for free at https://md-shouvik-iqbal.github.io/calculus
Chapter 8 The Area of a Surface of Revolution Page 491
Z1
= sec(x)dx
0
i1
= ln |sec(x) + tan(x)|
0
= ln |sec(1) + tan(1)| − ln |sec(0) + tan(0)|
≈ 1.23
It is known from geometry that the lateral surface area A of the following right
circular cone
Figure 8.21
Copyright © 2024 Md. Shouvik Iqbal. This book is licensed under a Creative Commons Attribution-NonCommercial-ShareAlike 4.0 International License (CC BY-NC-SA 4.0)
Download for free at https://md-shouvik-iqbal.github.io/calculus
Page 492 The Area of a Surface of Revolution Chapter 8
A = πr`
where, r is the radius of the base of the cone and ` is the slant height of the
√
cone, which is equal to r2 + h2 by the Pythagorean theorem. As a result, we
have the formula for the lateral surface area A of a right circular cone as,
A = πr`
p
= πr r2 + h2
Now, using the formula for the lateral surface area of a right circular cone, we
will derive the formula for the lateral surface area of a frustum of a right circular
cone. A frustum of a right circular cone is a part of a cone that remains after
the top portion of the cone is cut off by a plane parallel to the base.
In order to derive the formula for the frustum, consider a linear function f (x) =
nx over an interval [a, b], where 0 < a < b and n > 0, as shown below.
f(x)
x
a b
Figure 8.22
Now, rotate this line segment about the x-axis resulting in a frustum of a cone
as shown below.
Copyright © 2024 Md. Shouvik Iqbal. This book is licensed under a Creative Commons Attribution-NonCommercial-ShareAlike 4.0 International License (CC BY-NC-SA 4.0)
Download for free at https://md-shouvik-iqbal.github.io/calculus
Chapter 8 The Area of a Surface of Revolution Page 493
f(x)
x
a b
Figure 8.23
We need to determine the surface area S of this frustum. The idea is to subtract
the small surface area of the frustum over [0, a] from the larger surface area of
the frustum over [0, b]. This allows us to calculate the surface area of the frustum
over [a, b].
Now, notice carefully that the radius r of the cone over [0, b] is f (b) = nb and
the height h over [0, b] is b. Thus, the surface area of the cone over [0, b] yields,
p
S[0,b] = πr r2 + h2
q
= π (nb) (nb)2 + b2
p
= π (nb) b2 (n2 + 1)
√ p
= π (nb) b2 · (n2 + 1)
p
= π (nb) b · (n2 + 1)
p
= πb2 n n2 + 1
Similarly, the radius r of the cone over [0, a] is f (a) = na and the height h over
[0, a] is a. Thus, the surface area of the cone [0, a] yields,
p
S[0,a] = πr r2 + h2
q
= π (na) (na)2 + a2
p
= π (na) a2 (n2 + 1)
√ p
= π (na) a2 · (n2 + 1)
p
= π (na) a · (n2 + 1)
Copyright © 2024 Md. Shouvik Iqbal. This book is licensed under a Creative Commons Attribution-NonCommercial-ShareAlike 4.0 International License (CC BY-NC-SA 4.0)
Download for free at https://md-shouvik-iqbal.github.io/calculus
Page 494 The Area of a Surface of Revolution Chapter 8
p
= πa2 n n2 + 1
Now, the surface area S of the frustum over [a, b] yields the following,
S = S[0,b] − S[0,a]
p p
= πb2 n n2 + 1 − πa2 n n2 + 1
p
= πn b2 − a2 n2 + 1
Now, we calculate the line segment ` between (a, f (a)) and (b, f (b)) for fur-
ther simplification in the equation above. For that, note that the line segment
between the point (a, f (a)) = (a, an) and (b, f (b)) = (b, bn) yields the following.
q
` = (b − a)2 + (bn − an)2
q
= b2 − 2ab + a2 + (bn)2 − 2anbn + (an)2
p
= b2 − 2ab + a2 + b2 n2 − 2abn2 + a2 n2
p
= b2 − 2ab + a2 + n2 (b2 − 2ab + a2 )
p
= (b2 − 2ab + a2 ) (1 + n2 )
p p
= (b2 − 2ab + a2 ) · (1 + n2 )
q
= (b − a)2 · (1 + n2 )
p
p
= (b − a) (n2 + 1)
√
Now, since the surface area S of the frustum over [a, b] is πn b2 − a2 n2 + 1.
Therefore,
p
S = πn b2 − a2 n2 + 1
p
= πn (b + a) (b − a) n2 + 1
| {z }
(b2 −a2 )
p
= πn (b + a) (b − a) n2 + 1
| {z }
`
= πn (b + a) `
= π |{z}
bn + |{z}
an `
f (b) f (a)
= π f (b) + f (a) `
Copyright © 2024 Md. Shouvik Iqbal. This book is licensed under a Creative Commons Attribution-NonCommercial-ShareAlike 4.0 International License (CC BY-NC-SA 4.0)
Download for free at https://md-shouvik-iqbal.github.io/calculus
Chapter 8 The Area of a Surface of Revolution Page 495
Thus, we have derived the formula for the lateral surface area S of a frustum of
a right circular cone that was obtained by rotating the function f (x) over the
interval [a, b]. With this result, we are now equipped to derive a general formula
for the surface area of a surface of revolution.
Chap. 8 / Sec. 8.6 / Subsec. 8.6.1 : The Generalized Area Formula for a Sur-
face of Revolution
The formula for the surface area S of a frustum of a cone in the previous sub-
section was found to be the following,
S = π f (b) + f (a) `
Based on this result, we now derive the general formula for the surface area of
a surface of revolution. In order to do that, consider a continuous function f (x)
over an interval [a, b], as shown below.
f(x)
x
a b
Figure 8.24
Now, rotate this curve of f (x) about the x-axis, resulting in the following solid
of revolution,
Copyright © 2024 Md. Shouvik Iqbal. This book is licensed under a Creative Commons Attribution-NonCommercial-ShareAlike 4.0 International License (CC BY-NC-SA 4.0)
Download for free at https://md-shouvik-iqbal.github.io/calculus
Page 496 The Area of a Surface of Revolution Chapter 8
f(x)
x
b
a
Figure 8.25
Now, we need to determine the surface area S of this solid of revolution. For
b−a
that, we divide the interval [a, b] into n sub-interval of equal width ∆x = .
n
Then [xi−1 , xi ] becomes the ith sub-interval, as shown below.
f(x)
x
b
a
Figure 8.26
Now, notice that the line segment ` over the ith sub-interval, that is, the line
segment between the points (xi , f (xi−1 )) and (xi , f (xi )) is calculated as follows,
p
` = (xi − xi−1 )2 + (f (xi ) − f (xi−1 ))2
p
= (∆x)2 + (∆yi )2
Copyright © 2024 Md. Shouvik Iqbal. This book is licensed under a Creative Commons Attribution-NonCommercial-ShareAlike 4.0 International License (CC BY-NC-SA 4.0)
Download for free at https://md-shouvik-iqbal.github.io/calculus
Chapter 8 The Area of a Surface of Revolution Page 497
Here, since all the sub-intervals have equal width, therefore width ∆x is the
same for all i and thus ∆x doesn’t have any subscripts. Notice, however, that
this is not the case for the change in y, that is, ∆yi = f (xi ) − f (xi−1 ).
As a result, the surface area of this surface of revolution over [xi−1 , xi ] is given
by the formula of the surface area of a frustrum of a cone over [xi−1 , xi ], that is,
Si = π (f (xi ) + f (xi−1 )) `
q
= π (f (xi ) + f (xi−1 )) (∆x)2 + (∆yi )2
f (xi ) − f (xi−1 )
= f 0 (x∗i )
∆x
f (xi ) − f (xi−1 ) = f 0 (x∗i ) ∆x (2)
Copyright © 2024 Md. Shouvik Iqbal. This book is licensed under a Creative Commons Attribution-NonCommercial-ShareAlike 4.0 International License (CC BY-NC-SA 4.0)
Download for free at https://md-shouvik-iqbal.github.io/calculus
Page 498 The Area of a Surface of Revolution Chapter 8
Now, when n (the number of sub-intervals within the interval [a, b]) is sufficiently
large, then xi−1 ≈ xi ≈ x∗i . This implies that f (xi−1 ) ≈ f (xi ) ≈ f (x∗i ), for
i = 1, 2, 3, · · · , n, given that the function f (x) is continuous. Therefore,
Xn r
S≈ π (f (x∗i ) + f (x∗i )) 1 + f 0 (x∗i )2 ∆x
i=1
n
X
r
= 2πf (x∗i ) 1 + f 0 (x∗i )2 ∆x
i=1
This forms the Riemann sum. Now if n → ∞, then ∆x → 0, and we have the
following,
n r
S = lim
X
∗
2πf (xi ) 1 + f 0 (x∗i )2 ∆x
n→∞
i=1
Zb r
= 2πf (x) 1+ f 0 (x)2 dx
a
Zb r
S= 2πf (x) 1 + f 0 (x)2 dx
a
I Example 8.6.1. Find the surface area generated by rotating the function f (x) =
2 about the x-axis over the interval [0, 3].
Copyright © 2024 Md. Shouvik Iqbal. This book is licensed under a Creative Commons Attribution-NonCommercial-ShareAlike 4.0 International License (CC BY-NC-SA 4.0)
Download for free at https://md-shouvik-iqbal.github.io/calculus
Chapter 8 The Area of a Surface of Revolution Page 499
Z3
= 4πdx
0
Z3
= 4π · dx
0
i3
= 4π · x
0
= 4π · 3
= 12π
√
I Example 8.6.2. If the graph of f (x) = x is rotated about the x-axis over the
interval [0, 2], then what is the area of the surface of this revolution?
√ 1
Solution. Since f (x) = x, therefore f 0 (x) = √ . As a result, we write the
2 x
following,
Z2
s 2
√
1
S= 2π x 1+ √ dx
2 x
0
Z2
s 2
√
1
= 2π x· 1+ √ dx
2 x
0
Z2 r
√ 1
= 2π x· 1+ dx
4x
0
Z2 r
√ 4x + 1
= 2π x· dx
4x
0
Z2 √
√ 4x + 1
= 2π x · √ dx
2x
0
Z2 √
4x + 1
= 2π dx
2
0
Z2
1 √
= 2π 4x + 1dx
2
0
Copyright © 2024 Md. Shouvik Iqbal. This book is licensed under a Creative Commons Attribution-NonCommercial-ShareAlike 4.0 International License (CC BY-NC-SA 4.0)
Download for free at https://md-shouvik-iqbal.github.io/calculus
Page 500 The Area of a Surface of Revolution Chapter 8
Z2
(1)
1
=π· (4x + 1) 2 dx
0
Z2
Now, in order to solve the integral π · (4x + 1) 2 dx, we use the substitution
1
0
rule (that is, the u-substitution). For that, let u = 4x + 1, then
du d
= (4x + 1)
dx dx
=4
du
∴ dx =
4
Now, for the limit of integration, we do the following,
For x=0 =⇒ u=4·0+1=1
For x=2 =⇒ u=4·2+1=9
du
Now, substituting u = 4x + 1, dx = , and the limits of integration into the
4
equation (1), we get the following,
Z2 Z9
1 1 du
π· (4x + 1) dx = π ·2 u2
4
0 1
Z9
π 1
= · u 2 du
4
1
9
3
π u 2
= ·
4 3
2 1
3 3
π 9 1 2 2
= · −
4 3 3
2 2
π 52
= ·
4 3
13π
=
3
13π
Therefore, the surface area is .
3
Copyright © 2024 Md. Shouvik Iqbal. This book is licensed under a Creative Commons Attribution-NonCommercial-ShareAlike 4.0 International License (CC BY-NC-SA 4.0)
Download for free at https://md-shouvik-iqbal.github.io/calculus
Chapter 8 Work Page 501
§ 8.7 Work
In everyday language, we often use the word “work” to mean any kind of task
or activity we do, whether it involves any sort of movement. For example, when
we say, “I did a lot of work today” we typically emphasize the fact that a lot of
energy or effort has been used to get something done.
However, this common understanding of work doesn’t align with the scientific
definition of work. For example, simply holding a heavy object in place requires
energy and effort, but in the scientific sense, no work is done if there is no
movement. Mathematically, work is defined as a measurable quantity that only
occurs when a force F moves an object in the direction of that force. That is,
work is defined as follows.
W =F ·d (1)
In defining work, the equation above only works when the force remains constant
throughout its entire application. But in reality, the application of force often
changes. For example, imagine we’re pushing an object along a path, and the
force we exert changes from point to point as we move along. Let’s say we’re
moving the object from x = a to x = b along the x-axis by a continuously
varying force F (x), as shown in the figure below.
f(x)
x
a b
Figure 8.27
Now the question is how much work is done in moving that object based on
the definition of work we have seen so far? Equation (1) cannot be directly
Copyright © 2024 Md. Shouvik Iqbal. This book is licensed under a Creative Commons Attribution-NonCommercial-ShareAlike 4.0 International License (CC BY-NC-SA 4.0)
Download for free at https://md-shouvik-iqbal.github.io/calculus
Page 502 Work Chapter 8
f(x)
x
a b
Figure 8.28
Now, if we calculate this area under the graph, we can determine the work done
by the force F (x) over that interval. To calculate the area under the graph, we
divide the interval [a, b] into n sub-intervals that are sufficiently small such that
the force does not change much, that is, remains nearly constant within each
sub-divided sub-interval. This approach allows us to treat the force as constant
over each sub-interval, making it easier to approximate the work done on each
sub-interval using the formula for work. By adding together these small approx-
imations for each sub-interval, we create a Riemann sum. This Riemann sum
gives us an approximation of the total work W done over the entire interval [a, b].
Finally, we take the limit of these Riemann sums as the number of sub-intervals
increases without a bound, and thus obtain the actual work W done in moving
the object.
In order to do this, let us divide the interval [a, b] into n sub-interval of equal
b−a
width ∆x = by using points a = x0 , x1 , x2 , · · · , xn−1 , xn = b between
n
x = a and x = b. Let x∗i be any point in the ith sub-interval, for i = 1, 2, 3, · · · , n.
Since, the interval [a, b] is divided into sub-intervals in a way that the force
within each sub-interval remains approximately constant, therefore the force in
the ith sub-interval is roughly constant with a value F (x∗i ). Thus, the work done
in moving the object across the ith sub-interval is approximately F (x∗i ) · ∆x,
Copyright © 2024 Md. Shouvik Iqbal. This book is licensed under a Creative Commons Attribution-NonCommercial-ShareAlike 4.0 International License (CC BY-NC-SA 4.0)
Download for free at https://md-shouvik-iqbal.github.io/calculus
Chapter 8 Work Page 503
according to Equation (1). Similarly, summing the work done over every sub-
interval, we get an approximation of the total work W done over the interval
[a, b], that is,
Xn
W ≈ F (x∗i )∆x
i=1
n Zb
Now, if we let the number of sub-interval n → ∞, then
X
F (x∗i )∆x → F (x)dx =
i=1 a
W . Thus, taking the limit as n → ∞, we get
n
W = lim
X
F (x∗i )∆x
n→∞
i=1
Zb
= F (x)dx
a
I Example 8.7.1. Find the work done by a force F (x) = ex applied over the
distance from x = 0 to x = 1.
= e1 − e0
≈ 1.718
Copyright © 2024 Md. Shouvik Iqbal. This book is licensed under a Creative Commons Attribution-NonCommercial-ShareAlike 4.0 International License (CC BY-NC-SA 4.0)
Download for free at https://md-shouvik-iqbal.github.io/calculus
Page 504 Work Chapter 8
Therefore, the work done by a force F (x) = ex applied over the distance from
x = 0 to x = 1 is approximately 1.718 units.
N
I Example 8.7.2. A spring has a spring constant k = 200 m . How much work
is required to stretch the spring from its natural length to a displacement of
x = 0.1m?
Solution. To calculate the work done in stretching the spring, we integrate the
force F (x) over the displacement of 0.1m. That is,
Z0.1
W = F (x)dx
0
F (x) = kx
N
Since, k = 200 m , therefore,
F (x) = kx
= 200x
Copyright © 2024 Md. Shouvik Iqbal. This book is licensed under a Creative Commons Attribution-NonCommercial-ShareAlike 4.0 International License (CC BY-NC-SA 4.0)
Download for free at https://md-shouvik-iqbal.github.io/calculus
Chapter 8 Work Page 505
Copyright © 2024 Md. Shouvik Iqbal. This book is licensed under a Creative Commons Attribution-NonCommercial-ShareAlike 4.0 International License (CC BY-NC-SA 4.0)
Download for free at https://md-shouvik-iqbal.github.io/calculus
Copyright © 2024 Md. Shouvik Iqbal. This book is licensed under a Creative Commons Attribution-NonCommercial-ShareAlike 4.0 International License (CC BY-NC-SA 4.0)
Download for free at https://md-shouvik-iqbal.github.io/calculus
Index
507
Copyright © 2024 Md. Shouvik Iqbal. This book is licensed under a Creative Commons Attribution-NonCommercial-ShareAlike 4.0 International License (CC BY-NC-SA 4.0)
Download for free at https://md-shouvik-iqbal.github.io/calculus
of Differentiability, 137 of Vertical Asymptote, 71
of Differentials, 270 of Work Done by a Variable
of Discontinuous Integrands, 448 Force, 503
of Indefinite Integral, 357 Definition of the Area Under a
of Infinite Intervals, 445 Curve, 321
of Infinite Limits, 69 Definition of the Volume of a Solid,
of Infinite Limits at Infinity, 76 470
of Linearization, 266 Derivation of Derivatives, 143
of One-sided Infinite Limits, 69 Difference Quotient, 116
of One-sided Limits, 31 Difference Rule, 152
of the Critical Point, 231 Differentiability, 136
of the Definite Integral, 328 Differential Calculus, 115
of the Definition of the Volume Differentials, 267
of a Solid, 474 Differentiation
of the Derivative of a Function, of Composite Functions, 172
128 of the Cosecant Function, 171
of the Epsilon - Delta Definition of the Cosine Function, 164
of Limit, 108 of the Cotangent Function, 167
of the Formal Definition of of the Exponential Function
Limit, 108 with Base a, 192
of the Exponential Function
of the Horizontal Asymptote, 74
with Base e, 188
of the Instantaneous Rate of
of the Natural Logarithm, 195
Change, 124, 125
of the Secant Function, 169
of the Left-Endpoint
of the Sine Function, 161
Approximation, 313
of the Tangent Function, 165
of the Limit at Infinity, 74
of the Common Logarithm, 198
of the Midpoint Rule, 342
Differentiation Rules
of the Net Signed Area, 327
for Exponential Functions, 188
of the Relative Extreme Values,
for Logarithmic Functions, 195
228
for Trigonometric Functions, 161
of the Riemann Sum, 322
Direct Substitution, 37
of the Right-Endpoint
Discontinuous Integrands, 446
Approximation, 314
Disk Method, 474
of the Slope of a Tangent Line,
Dividing Out Technique, 46
125
of the Tangent Line, 126 Elementary Rules of Differentiation,
of the Trapezoidal Rule, 345 143
508
Copyright © 2024 Md. Shouvik Iqbal. This book is licensed under a Creative Commons Attribution-NonCommercial-ShareAlike 4.0 International License (CC BY-NC-SA 4.0)
Download for free at https://md-shouvik-iqbal.github.io/calculus
End Behaviors, 75 involving a Sum of Functions,
Epsilon - Delta Definition of Limit, 334
104 involving Absolute Values, 336
Existence of a Limit, 32 involving Powers of Sine and
Extreme Value Theorem, 226 Cosine Function, 404
Extreme Values of a Function, 223 involving Powers of Tangent and
Secant Function, 409
First Derivative Test, 241
involving Product of Powers of
Formal Definition of Limit, 104
Sine and Cosine Function,
Fundamental Theorem of Calculus,
407
364
involving Product of Powers of
General Properties of Limit of a Tangent and Secant
Function, 34 Function, 411
Generalized Area Approximation, over Multiple Sub-intervals, 336
310 Integration
Generalized Area Formula for a by Parts, 394
Surface of Revolution, 495 by Parts for Definite Integrals,
Higher-Order Derivatives, 139 400
by Reduction Formulas, 402
Identical Limits of Integration, 332
by Substitution, 378
Implicit Differentiation, 182
Intermediate Value Theorem, 102
Improper Integrals, 443
Inverse Relationship between
Increasing and Decreasing
Differentiation and
Functions, 236
Integration, 370
Test, 239
Increasing, Decreasing, and L’Hôpital’s Rule, 277
Constant Functions, 237 Left-Endpoint Approximation, 312
Indefinite integral, 356 Leibniz Notation, 134
Infinite LIATE Rule, 397
Intervals, 443 Limit, 24
Limits, 69 at Infinity, 73
Limits at Infinity, 75 involving Infty, 68
Inflection Points, 248 of a Composite Function, 42
Informal Definition of Limit, 29 of a Function, 29
Integrals of a Reciprocal, 39
involving a Constant, 335 of Functions Involving Radicals
involving a Difference of at Infinity, 88
Functions, 334 of Polynomial Functions, 37
509
Copyright © 2024 Md. Shouvik Iqbal. This book is licensed under a Creative Commons Attribution-NonCommercial-ShareAlike 4.0 International License (CC BY-NC-SA 4.0)
Download for free at https://md-shouvik-iqbal.github.io/calculus
of Polynomial Functions at Relative (Local) Extreme Values,
Infinity, 78 227
of Power functions at Infinity, 77 Reversal of Limits of Integration,
of Rational Functions, 37 332
of Rational Functions at Riemann Sum, 321
Infinity, 80 Right-Endpoint Approximation, 313
of Transcendental Functions, 40 Rolle’s Theorem, 250
of Transcendental Functions at Rolle’s Theorem and The Mean
Infinity, 91 Value Theorem, 250
Linearization, 263
Linearization and Differentials, 262 Sandwich Theorem, 52
Location of Absolute (Global) Second Derivative Test, 250
Extreme Values, 233 Slicing Method, 470
Logarithmic Differentiation, 200 Squeeze Theorem, 52
Substitution Rule
Mean Value Theorem, 253 for Definite Integrals, 391
for Integrals, 462 Summation Notation, 298
Midpoint Rule, 340 Summation Rule, 150
510
Copyright © 2024 Md. Shouvik Iqbal. This book is licensed under a Creative Commons Attribution-NonCommercial-ShareAlike 4.0 International License (CC BY-NC-SA 4.0)
Download for free at https://md-shouvik-iqbal.github.io/calculus
Functions, 334 of the Limit of Polynomial
of Integrals Involving Absolute Functions, 38
Values, 338 of the Limit of Transcendental
of Integrals over Multiple Functions, 41
Sub-intervals, 336 of the Power Rule, 148
of Integration by Parts, 395 of the Product Rule, 156
of Integration by Substitution, of the Quotient Rule, 159
379 of the Second Derivative Test,
of Not Continuous Implies Not 250
Differentiable, 138 of the Summation Rule, 150
of Reversal of Limits of of the Washer Method, 480
Integration, 333 Extreme Value Theorem, 226
of the Constant Multiple Rule, Fundamental Theorem of
146 Calculus, Part I, 367
of the Derivative of the Fundamental Theorem of
Cosecant Function, 171 Calculus, Part II, 369
of the Derivative of the Cosine Intermediate Value Theorem,
Function, 164 103
of the Derivative of the Mean Value Theorem, 254
Cotangent Function, 168 Mean Value Theorem For
of the Derivative of the Integrals, 462
Exponential Function with of Continuity of Combined
Base a, 193 Functions, 101
of the Derivative of the of Limit Laws, 34
Exponential Function with of the Chain Rule, 175
Base e, 191 of the Constant Rule, 144
of the Derivative of the Natural of the Derivative of Identity
Logarithm, 196 Function, 147
of the Derivative of the Sine of the Derivative of the
Function, 162 Common Logarithm, 198
of the Difference Rule, 152 of the Derivative of the Secant
of the Disk Method, 476 Function, 169
of the First Derivative Test, 242 of the Derivative of the Tangent
of the Inflection Points Test, 249 Function, 166
of the Limit of a Composite of the Limit of Rational
Function, 42 Functions, 38
of the Limit of a Reciprocal, 40 Relative/Local Extreme Value
511
Copyright © 2024 Md. Shouvik Iqbal. This book is licensed under a Creative Commons Attribution-NonCommercial-ShareAlike 4.0 International License (CC BY-NC-SA 4.0)
Download for free at https://md-shouvik-iqbal.github.io/calculus
Theorem, 229 Integrals, 403
Rolle’s Theorem, 251 Manipulation, 60
Squeeze Theorem, 54 Substitution, 412
Trapezoidal Rule, 343
Travel, 23 Washer Method, 478
Trigonometric Work, 501
512
Copyright © 2024 Md. Shouvik Iqbal. This book is licensed under a Creative Commons Attribution-NonCommercial-ShareAlike 4.0 International License (CC BY-NC-SA 4.0)
Download for free at https://md-shouvik-iqbal.github.io/calculus
License
By exercising the Licensed Rights (defined below), You accept and agree to
be bound by the terms and conditions of this Creative Commons Attribution-
NonCommercial-ShareAlike 4.0 International Public License (”Public License”).
To the extent this Public License may be interpreted as a contract, You are
granted the Licensed Rights in consideration of Your acceptance of these terms
and conditions, and the Licensor grants You such rights in consideration of ben-
efits the Licensor receives from making the Licensed Material available under
these terms and conditions.
Section 1 – Definitions.
b. Adapter’s License means the license You apply to Your Copyright and
Similar Rights in Your contributions to Adapted Material in accordance
with the terms and conditions of this Public License.
513
Copyright © 2024 Md. Shouvik Iqbal. This book is licensed under a Creative Commons Attribution-NonCommercial-ShareAlike 4.0 International License (CC BY-NC-SA 4.0)
Download for free at https://md-shouvik-iqbal.github.io/calculus
c. BY-NC-SA Compatible License means a license listed at creativecom-
mons.org/compatiblelicenses, approved by Creative Commons as essen-
tially the equivalent of this Public License.
f. Exceptions and Limitations means fair use, fair dealing, and/or any
other exception or limitation to Copyright and Similar Rights that applies
to Your use of the Licensed Material.
i. Licensed Rights means the rights granted to You subject to the terms
and conditions of this Public License, which are limited to all Copyright
and Similar Rights that apply to Your use of the Licensed Material and
that the Licensor has authority to license.
514
Copyright © 2024 Md. Shouvik Iqbal. This book is licensed under a Creative Commons Attribution-NonCommercial-ShareAlike 4.0 International License (CC BY-NC-SA 4.0)
Download for free at https://md-shouvik-iqbal.github.io/calculus
l. Share means to provide material to the public by any means or process
that requires permission under the Licensed Rights, such as reproduction,
public display, public performance, distribution, dissemination, communi-
cation, or importation, and to make material available to the public in-
cluding in ways that members of the public may access the material from
a place and at a time individually chosen by them.
m. Sui Generis Database Rights means rights other than copyright result-
ing from Directive 96/9/EC of the European Parliament and of the Council
of 11 March 1996 on the legal protection of databases, as amended and/or
succeeded, as well as other essentially equivalent rights anywhere in the
world.
n. You means the individual or entity exercising the Licensed Rights under
this Public License. Your has a corresponding meaning.
Section 2 – Scope.
a. License grant.
1. Subject to the terms and conditions of this Public License, the Licen-
sor hereby grants You a worldwide, royalty-free, non-sublicensable,
non-exclusive, irrevocable license to exercise the Licensed Rights in
the Licensed Material to:
A. reproduce and Share the Licensed Material, in whole or in part,
for NonCommercial purposes only; and
B. produce, reproduce, and Share Adapted Material for NonCom-
mercial purposes only.
2. Exceptions and Limitations. For the avoidance of doubt, where
Exceptions and Limitations apply to Your use, this Public License
does not apply, and You do not need to comply with its terms and
conditions.
3. Term. The term of this Public License is specified in Section 6(a).
4. Media and formats; technical modifications allowed. The Li-
censor authorizes You to exercise the Licensed Rights in all media and
formats whether now known or hereafter created, and to make tech-
nical modifications necessary to do so. The Licensor waives and/or
agrees not to assert any right or authority to forbid You from mak-
ing technical modifications necessary to exercise the Licensed Rights,
515
Copyright © 2024 Md. Shouvik Iqbal. This book is licensed under a Creative Commons Attribution-NonCommercial-ShareAlike 4.0 International License (CC BY-NC-SA 4.0)
Download for free at https://md-shouvik-iqbal.github.io/calculus
including technical modifications necessary to circumvent Effective
Technological Measures. For purposes of this Public License, simply
making modifications authorized by this Section 2(a)(4) never pro-
duces Adapted Material.
5. Downstream recipients.
A. Offer from the Licensor – Licensed Material. Every recipi-
ent of the Licensed Material automatically receives an offer from
the Licensor to exercise the Licensed Rights under the terms and
conditions of this Public License.
B. Additional offer from the Licensor – Adapted Material.
Every recipient of Adapted Material from You automatically re-
ceives an offer from the Licensor to exercise the Licensed Rights
in the Adapted Material under the conditions of the Adapter’s
License You apply.
C. No downstream restrictions. You may not offer or impose
any additional or different terms or conditions on, or apply any
Effective Technological Measures to, the Licensed Material if do-
ing so restricts exercise of the Licensed Rights by any recipient of
the Licensed Material.
6. No endorsement. Nothing in this Public License constitutes or
may be construed as permission to assert or imply that You are, or
that Your use of the Licensed Material is, connected with, or spon-
sored, endorsed, or granted official status by, the Licensor or others
designated to receive attribution as provided in Section 3(a)(1)(A)(i).
b. Other rights.
1. Moral rights, such as the right of integrity, are not licensed under
this Public License, nor are publicity, privacy, and/or other similar
personality rights; however, to the extent possible, the Licensor waives
and/or agrees not to assert any such rights held by the Licensor to the
limited extent necessary to allow You to exercise the Licensed Rights,
but not otherwise.
2. Patent and trademark rights are not licensed under this Public Li-
cense.
3. To the extent possible, the Licensor waives any right to collect royal-
ties from You for the exercise of the Licensed Rights, whether directly
516
Copyright © 2024 Md. Shouvik Iqbal. This book is licensed under a Creative Commons Attribution-NonCommercial-ShareAlike 4.0 International License (CC BY-NC-SA 4.0)
Download for free at https://md-shouvik-iqbal.github.io/calculus
or through a collecting society under any voluntary or waivable statu-
tory or compulsory licensing scheme. In all other cases the Licensor
expressly reserves any right to collect such royalties, including when
the Licensed Material is used other than for NonCommercial pur-
poses.
a. Attribution.
b. ShareAlike.
In addition to the conditions in Section 3(a), if You Share Adapted Material
You produce, the following conditions also apply.
517
Copyright © 2024 Md. Shouvik Iqbal. This book is licensed under a Creative Commons Attribution-NonCommercial-ShareAlike 4.0 International License (CC BY-NC-SA 4.0)
Download for free at https://md-shouvik-iqbal.github.io/calculus
1. The Adapter’s License You apply must be a Creative Commons license
with the same License Elements, this version or later, or a BY-NC-SA
Compatible License.
2. You must include the text of, or the URI or hyperlink to, the Adapter’s
License You apply. You may satisfy this condition in any reasonable
manner based on the medium, means, and context in which You Share
Adapted Material.
3. You may not offer or impose any additional or different terms or con-
ditions on, or apply any Effective Technological Measures to, Adapted
Material that restrict exercise of the rights granted under the Adapter’s
License You apply.
a. for the avoidance of doubt, Section 2(a)(1) grants You the right to extract,
reuse, reproduce, and Share all or a substantial portion of the contents of
the database for NonCommercial purposes only;
c. You must comply with the conditions in Section 3(a) if You Share all or a
substantial portion of the contents of the database.
For the avoidance of doubt, this Section 4 supplements and does not replace
Your obligations under this Public License where the Licensed Rights include
other Copyright and Similar Rights.
Section 5 – Disclaimer of Warranties and Limitation of Liability.
518
Copyright © 2024 Md. Shouvik Iqbal. This book is licensed under a Creative Commons Attribution-NonCommercial-ShareAlike 4.0 International License (CC BY-NC-SA 4.0)
Download for free at https://md-shouvik-iqbal.github.io/calculus
non-infringement, absence of latent or other defects, accuracy,
or the presence or absence of errors, whether or not known or
discoverable. Where disclaimers of warranties are not allowed in
full or in part, this disclaimer may not apply to You.
a. This Public License applies for the term of the Copyright and Similar
Rights licensed here. However, if You fail to comply with this Public Li-
cense, then Your rights under this Public License terminate automatically.
b. Where Your right to use the Licensed Material has terminated under Sec-
tion 6(a), it reinstates:
For the avoidance of doubt, this Section 6(b) does not affect any right
the Licensor may have to seek remedies for Your violations of this Public
License.
c. For the avoidance of doubt, the Licensor may also offer the Licensed Ma-
terial under separate terms or conditions or stop distributing the Licensed
Material at any time; however, doing so will not terminate this Public
License.
519
Copyright © 2024 Md. Shouvik Iqbal. This book is licensed under a Creative Commons Attribution-NonCommercial-ShareAlike 4.0 International License (CC BY-NC-SA 4.0)
Download for free at https://md-shouvik-iqbal.github.io/calculus
d. Sections 1 , 5 , 6 , 7 , and 8 survive termination of this Public License.
Section 8 – Interpretation.
a. For the avoidance of doubt, this Public License does not, and shall not
be interpreted to, reduce, limit, restrict, or impose conditions on any use
of the Licensed Material that could lawfully be made without permission
under this Public License.
520
Copyright © 2024 Md. Shouvik Iqbal. This book is licensed under a Creative Commons Attribution-NonCommercial-ShareAlike 4.0 International License (CC BY-NC-SA 4.0)
Download for free at https://md-shouvik-iqbal.github.io/calculus